International Economics: Theory and Policy, 12e (Krugman) Chapter 1 Introduction 1.1
What Is International Economics About?
1) Historians of economic thought often describe ________ written by ________ and published in ________ as the first real exposition of an economic model. A) "Of the Balance of Trade"; David Hume; 1776 B) "Wealth of Nations"; David Hume; 1758 C) "Wealth of Nations"; Adam Smith; 1758 D) "Wealth of Nations"; Adam Smith; 1776 E) "Of the Balance of Trade"; David Hume; 1758 Answer: E Difficulty: Easy AACSB: Application of knowledge 2) From 1960 to 2019 A) the U.S. economy roughly tripled in size. B) U.S. imports roughly tripled in size. C) the share of U.S. trade in the global economy roughly tripled in size compared with the economy as a whole. D) U.S. imports roughly tripled as compared to U.S. exports. E) U.S. exports roughly tripled in size. Answer: C Difficulty: Easy AACSB: Application of knowledge 3) From 1960 to 2019, both U.S. imports and exports have increased, ________ have grown more, leading to a large excess of ________. A) imports; imports over exports B) exports; exports over imports C) imports; exports over imports D) exports; imports over exports Answer: A Difficulty: Easy AACSB: Application of knowledge 4) The facts that U.S. imports and exports plunged in 2009 during the global economic crisis and in 2020 during the Covid-19 pandemic demonstrated the close links between A) world trade and the overall state of the world economy. B) crisis and unemployment. C) the pandemic and shortage of goods and services. D) the business cycle and trade policies. Answer: A Difficulty: Easy AACSB: Application of knowledge 1 Copyright © 2022 Pearson Education, Inc.
5) The United States is less dependent on trade than most other countries because A) the United States is a relatively large country with diverse resources. B) the United States is a "Superpower." C) the military power of the United States makes it less dependent on anything. D) the United States invests in many other countries. E) many countries invest in the United States. Answer: A Difficulty: Easy AACSB: Application of knowledge 6) Theories of international economics from the 18th and 19th centuries are A) not relevant to current policy analysis. B) only of moderate relevance in today's modern international economy. C) highly relevant in today's modern international economy. D) the only theories that are actually relevant to modern international economy. E) not well understood by modern mathematically oriented theorists. Answer: C Difficulty: Easy AACSB: Application of knowledge 7) International economics ________ use the same fundamental methods of analysis as other branches of economics, because ________. A) does not; the level of complexity of international issues is unique B) does not; the interactions associated with international economic relations is highly mathematical C) does not; international economics takes a different perspective on economic issues D) does; international economists must reach an agreement with other economists on every economic issue E) does; the motives and behavior of individuals are the same in international trade as they are in domestic transactions Answer: E Difficulty: Easy AACSB: Application of knowledge 8) Because the Constitution forbids restraints on interstate trade A) the U.S. may not impose tariffs on imports from NAFTA countries. B) the U.S. may not affect the international value of the $ U.S. C) the U.S. may not put restraints on foreign investments in California if it involves a financial intermediary in New York State. D) the U.S. may not impose export duties. E) the U.S. may not disrupt commerce between Florida and Hawaii. Answer: E Difficulty: Easy AACSB: Application of knowledge
2 Copyright © 2022 Pearson Education, Inc.
9) An important insight of international economics is that when countries sell goods and services to each other, A) the exchange is almost always mutually beneficial. B) one country always benefits at the expense of the other. C) it only benefits the low wage country. D) it only benefits the high wage country. E) it is almost never beneficial to both countries. Answer: A Difficulty: Easy AACSB: Application of knowledge 10) "Trade is generally harmful if there are large disparities between countries in wages." A) This is generally true. B) This is generally false. C) Trade theory has nothing to say about this issue. D) This is true if the trade partner ignores child labor laws. E) This is true if the trade partner uses prison labor. Answer: B Difficulty: Easy AACSB: Application of knowledge 11) If there are large disparities in wage levels between countries, then A) trade is likely to be harmful to both countries. B) trade is likely to be harmful to the country with the high wages. C) trade is likely to be harmful to the country with the low wages. D) trade is likely to be harmful to neither country. E) trade is likely to have no effect on either country. Answer: D Difficulty: Easy AACSB: Application of knowledge 12) If there are large disparities in productivity between countries, then A) trade is likely to be harmful to neither country. B) trade is likely to be harmful to the country with the high wages. C) trade is likely to be harmful to the country with the low wages. D) trade is likely to be harmful to both countries. E) trade is likely to have no effect on either country. Answer: A Difficulty: Easy AACSB: Application of knowledge
3 Copyright © 2022 Pearson Education, Inc.
13) Which of the following statements is TRUE? A) Trade is mutually beneficial when countries exchange goods and services. B) Trade is harmful if large disparities exist between countries in productivity. C) Trade is harmful if large disparities exist between countries in wages. D) Trading with less-advanced, lower-wage countries will drag down a country's standard of living. Answer: A Difficulty: Easy AACSB: Application of knowledge 14) The benefits of international trade are derived from trade in A) tangible goods only. B) intangible goods only. C) goods but not services. D) services but not goods. E) anything of value. Answer: E Difficulty: Easy AACSB: Application of knowledge 15) Which of the following statements is NOT TRUE? A) International trade will have no effects on income distribution. B) International trade may hurt particular groups within nations. C) International trade can adversely impact owners of resources that are specific to industries that compete with imports. D) International trade can alter the distribution of income between broad groups, such as workers and the owners of capital. Answer: A Difficulty: Moderate AACSB: Application of knowledge 16) Who sells what to whom A) has been a major preoccupation of international economics. B) is not a valid concern of international economics. C) is not considered important for government foreign trade policy since such decisions are made in the private competitive market. D) is determined by political rather than economic factors. E) is less important than international economic theory. Answer: A Difficulty: Easy AACSB: Application of knowledge
4 Copyright © 2022 Pearson Education, Inc.
17) The insight that patterns of trade are primarily determined by international differences in labor productivity was first proposed by A) David Ricardo. B) David Hume. C) Adam Smith. D) Eli Heckscher. E) Lerner and Samuelson. Answer: A Difficulty: Easy AACSB: Application of knowledge 18) International economists cannot discuss the effects of international trade or recommend changes in government policies toward trade with any confidence unless they know A) their theory is the best available. B) their theory is internally consistent. C) their theory passes the "reasonable person" legal criteria. D) their theory is good enough to explain the international trade that is actually observed. E) their theory accounts for China's unique position in international trade. Answer: D Difficulty: Easy AACSB: Application of knowledge 19) Which of the following is NOT a major concern of international economic theory? A) protectionism B) the balance of payments C) exchange rate determination D) bilateral trade relations with China E) the international capital market Answer: D Difficulty: Easy AACSB: Application of knowledge 20) Which of the following does NOT belong? A) NAFTA B) Uruguay Round C) World Trade Organization D) non-tariff barriers E) major free trade agreements of the 1990s Answer: D Difficulty: Easy AACSB: Application of knowledge
5 Copyright © 2022 Pearson Education, Inc.
21) In 1998 an economic and financial crisis in South Korea caused it to experience A) a surplus in their balance of payments. B) a deficit in their balance of payments. C) a balanced balance of payments. D) an unbalanced balance of payments. E) a lull in international trade. Answer: A Difficulty: Easy AACSB: Application of knowledge 22) In 1998, countries that ran large trade surpluses include A) China and South Korea. B) U.S. and Japan. C) China and U.S. D) U.S. and South Korea. Answer: A Difficulty: Easy AACSB: Application of knowledge 23) After World War II, the United States has pursued a broad policy of A) strengthening "Fortress America" protectionism. B) removing barriers to international trade. C) isolating Iran and other members of the "axis of evil." D) protecting the U.S. from the economic impact of oil producers. E) restricting trade of manufactured goods. Answer: B Difficulty: Easy AACSB: Application of knowledge 24) Cost-benefit analysis of international trade A) is basically useless. B) is empirically intractable. C) focuses attention primarily on conflicts of interest within nations. D) focuses attention on conflicts of interest between nations. E) never leads to government intervention in international trade. Answer: C Difficulty: Moderate AACSB: Application of knowledge
6 Copyright © 2022 Pearson Education, Inc.
25) An improvement in a country's balance of payments means a decrease in its balance of payments deficit, or an increase in its surplus. In fact, we know that a surplus in a balance of payments A) is always beneficial. B) is usually beneficial. C) is never harmful. D) is sometimes harmful. E) is always harmful. Answer: D Difficulty: Moderate AACSB: Application of knowledge 26) The GATT is A) an international agreement. B) an international U.N. agency. C) an international IMF agency. D) a U.S. government agency. E) a collection of tariffs. Answer: A Difficulty: Easy AACSB: Application of knowledge 27) The balance of payments has become a central issue for the United States because A) when the balance of payments is not balanced, society is unbalanced. B) the U.S. economy cannot grow when the balance of payments is in deficit. C) the U.S. has run huge trade deficits in every year since 1982. D) the U.S. never experienced a surplus in its balance of payments. E) the U.S. once ran a large trade surplus of about $40 billion. Answer: C Difficulty: Easy AACSB: Application of knowledge 28) In September 2010, the finance minister of ________ declared that the world was "in the midst of an international currency war" because of rapid appreciation in the value of the country's currency, the ________. A) England; pound sterling B) Germany; euro C) Japan; yen D) China; renminbi E) Brazil; Real Answer: E Difficulty: Easy AACSB: Application of knowledge
7 Copyright © 2022 Pearson Education, Inc.
29) The study of exchange rate determination is a relatively new part of international economics, since A) for much of the past century, exchange rates were fixed by government action. B) the calculations required for this were not possible before modern computers became available. C) economic theory developed by David Hume demonstrated that real exchange rates remain fixed over time. D) dynamic overshooting asset pricing models are a recent theoretical development. E) the exchange rate never fluctuates. Answer: A Difficulty: Easy AACSB: Application of knowledge 30) A fundamental problem in international economics is how to produce A) a perfect degree of monetary harmony. B) an acceptable degree of harmony among the international trade policies of different countries. C) a world government that can harmonize trade and monetary policies. D) a counter-cyclical monetary policy so that all countries will not be adversely affected by a financial crisis in one country. E) a worldwide form of currency. Answer: B Difficulty: Easy AACSB: Application of knowledge 31) For almost 70 years international trade policies have been governed A) by the World Trade Organization. B) by the International Monetary Fund. C) by the World Bank. D) by an international treaty known as the General Agreement on Tariffs and Trade (GATT). E) by the North American Free Trade Agreement (NAFTA). Answer: D Difficulty: Easy AACSB: Application of knowledge 32) International capital markets experience a kind of risk not faced in domestic capital markets, namely A) "economic meltdown" risk. B) Flood and hurricane crisis risk. C) the risk of unexpected downgrading of assets by Standard and Poor. D) the risk of currency fluctuations. E) the risk of political upheaval. Answer: D Difficulty: Moderate AACSB: Application of knowledge
8 Copyright © 2022 Pearson Education, Inc.
33) Since 1994, trade rules have been enforced by A) the WTO. B) the G10. C) the GATT. D) The U.S. Congress. E) the European Union. Answer: A Difficulty: Easy AACSB: Application of knowledge 34) The international capital market is A) the place where you can rent earth moving equipment anywhere in the world. B) a set of arrangements by which individuals and firms exchange money now for promises to pay in the future. C) the arrangement where banks build up their capital by borrowing from the Central Bank. D) the place where emerging economies accept capital invested by banks. E) exclusively concerned with the debt crisis that ended in the 1990s. Answer: B Difficulty: Easy AACSB: Application of knowledge 35) The international financial crisis of 2007 was the result of A) failure of the Euro currency. B) runaway inflation in the U.S. C) a deep global recession. D) the collapse of global currency markets. E) defaults on U.S. mortgage-backed securities. Answer: E Difficulty: Easy AACSB: Application of knowledge 36) It is argued that global trade tends to be more important to countries with smaller economies than the U.S. Is this empirically verified? Answer: Yes. Figure 1-2 shows exports and imports as a percentage of national income in the U.S. and five other countries and notes that "International trade is even more important to most other countries than it is to the U.S." Difficulty: Moderate AACSB: Application of knowledge 37) It is argued that if a rich high wage country such as the United States were to expand trade with a relatively poor and low wage country such as Mexico, then U.S. industry would migrate south, and U.S. wages would fall to the level of Mexico's. What do you think about this argument? Answer: The student may think anything. The purpose of the question is to set up a discussion, which will lead to the models in the following chapters. Difficulty: Moderate AACSB: Reflective thinking 9 Copyright © 2022 Pearson Education, Inc.
38) How are the patterns of international trade, that is the pattern of what different countries export and import, explained? Answer: Climate explains why Brazil exports coffee. Natural resources explains why Saudi Arabia exports oil. More generally, differences in labor productivity and in the availability of land, labor, and capital within different countries explain patterns of trade. More recent research suggests that there is a significant random component involved, as well. Difficulty: Moderate AACSB: Application of knowledge 39) International trade theory implies that international trade is beneficial to all trading countries. However, casual observation leads to the conclusion that official obstruction of international trade flows is widespread. How might you reconcile these two facts? Answer: This question is meant to allow students to offer preliminary discussions of issues, which will be explored in depth later in the book. Difficulty: Moderate AACSB: Application of knowledge 1.2
International Economics: Trade and Money
1) International economics can be divided into two broad sub-fields A) macro and micro. B) developed and less developed. C) monetary and barter. D) international trade and international money. E) static and dynamic. Answer: D Difficulty: Easy AACSB: Application of knowledge 2) International trade analysis focuses on A) the real transactions that involve a physical movement of goods or a tangible commitment of economic resources in the international economy. B) the monetary side of the international economy. C) the financial transactions such as foreign purchases of U.S. dollars. D) the devaluation of currencies of the international economy. Answer: A Difficulty: Easy AACSB: Application of knowledge
10 Copyright © 2022 Pearson Education, Inc.
3) International monetary analysis focuses on A) the real side of the international economy. B) the international trade side of the international economy. C) the international investment side of the international economy. D) the issues of international cooperation between Central Banks. E) the monetary side of the international economy, such as currency exchange. Answer: E Difficulty: Easy AACSB: Application of knowledge 4) The distinction between international trade and international money is NOT entirely clear because A) real developments in the trade accounts do not have monetary implications. B) the balance of payments includes only real measures. C) developments caused by purely monetary changes have no real effects. D) trade models focus on real, or barter relationships. E) most international trade involves monetary transactions. Answer: E Difficulty: Easy AACSB: Application of knowledge 5) The distinction between international trade and international money is NOT entirely clear because A) many monetary events have important consequences for trade. B) real developments in the trade accounts do not have monetary implications. C) the balance of payments includes only real measures. D) developments caused by purely monetary changes have no real effects. E) trade models focus on real, or barter relationships. Answer: A Difficulty: Easy AACSB: Application of knowledge
11 Copyright © 2022 Pearson Education, Inc.
International Economics: Theory and Policy, 12e (Krugman) Chapter 2 World Trade: An Overview 2.1
Who Trades with Whom?
1) Approximately what percent of all world production of goods and services is exported to other countries? A) 10% B) 30% C) 50% D) 100% E) 90% Answer: B Difficulty: Easy AACSB: Application of knowledge 2) In 2019, United States top 15 trading partners do not include A) Australia. B) Mexico. C) Canada. D) China. E) Vietnam. Answer: A Difficulty: Easy AACSB: Application of knowledge 3) In 2019, United States top 15 trading partners account for ________ of the value of U.S. trade that year. A) 75 percent B) 25 percent C) 50 percent D) 85 percent E) 90 percent Answer: A Difficulty: Easy AACSB: Application of knowledge
1 Copyright © 2022 Pearson Education, Inc.
4) Annual Gross Domestic Product (GDP) measures A) the total value of all goods and services produced domestically in a country's economy in one year. B) the total value of all goods and services produced domestically and abroad by a country's citizens in one year. C) the total value of all goods and services attributed to each individual citizen of a country in one year. D) the total value of all goods and services imported and exported by a country in one year. Answer: A Difficulty: Easy AACSB: Application of knowledge 5) The gravity model offers a logical explanation for the fact that A) trade between Asia and the U.S. has grown faster than NAFTA trade. B) trade in services has grown faster than trade in goods. C) trade in manufactures has grown faster than in agricultural products. D) Intra-European Union trade exceeds international trade by the European Union. E) the U.S. trades more with Western Europe than it does with Canada. Answer: D Difficulty: Moderate AACSB: Application of knowledge 6) The gravity model suggests that over time A) trade between neighboring countries will increase. B) trade between all countries will increase. C) world trade will eventually be swallowed by a black hole. D) the value of trade between two countries will be proportional to the product of the two countries' GDP. Answer: D Difficulty: Easy AACSB: Application of knowledge 7) The gravity model explains why A) trade between Sweden and Germany exceeds that between Sweden and Spain. B) countries with oil reserves tend to export oil. C) capital rich countries export capital intensive products. D) intra-industry trade is relatively more important than other forms of trade between neighboring countries. E) European countries rely most often on natural resources. Answer: A Difficulty: Moderate AACSB: Application of knowledge
2 Copyright © 2022 Pearson Education, Inc.
8) The gravity model explains why A) trade between the U.S. and Germany exceeds that between the U.S. and Sweden. B) countries with oil reserves tend to export oil. C) capital rich countries export capital intensive products. D) intra-industry trade is relatively more important than other forms of trade between neighboring countries. E) European countries rely most often on natural resources. Answer: A Difficulty: Moderate AACSB: Application of knowledge 9) According to the Gravity Model, the reason why the United States trades more heavily with Germany, the United Kingdom, and France than with other countries in Europe is because of A) the size of their economies. B) their ties to the European Union. C) mutual rights and responsibilities. D) their currencies and trade policies. Answer: A Difficulty: Easy AACSB: Application of knowledge 10) According to the gravity model, a characteristic that tends to affect the probability of trade existing between any two countries is A) their cultural affinity. B) the average weight/value of their traded goods. C) their colonial-historical ties. D) the distance between them. E) the number of different product varieties produced by their industries. Answer: D Difficulty: Easy AACSB: Application of knowledge 11) In general, which of the following do NOT tend to increase trade between two countries? A) linguistic and/or cultural affinity B) historical ties C) larger economies D) mutual membership in preferential trade agreements E) the existence of well controlled borders between countries Answer: E Difficulty: Moderate AACSB: Application of knowledge
3 Copyright © 2022 Pearson Education, Inc.
12) Why does the gravity model work? A) Large economies became large because they were engaged in international trade. B) Large economies have relatively large incomes, and hence spend more on government promotion of trade and investment. C) Large economies have relatively larger areas, which raises the probability that a productive activity will take place within the borders of that country. D) Large economies tend to have large incomes and tend to spend more on imports. E) Large economies tend to avoid trading with small economies. Answer: D Difficulty: Easy AACSB: Application of knowledge 13) We see that the Netherlands, Belgium, and Ireland trade considerably more with the United States than with many other countries. A) This is explained by the gravity model, since these are all large countries. B) This is explained by the gravity model, since these are all small countries. C) This fails to be consistent with the gravity model, since these are small countries. D) This fails to be consistent with the gravity model, since these are large countries. E) This is explained by the gravity model, since they do not share borders. Answer: C Difficulty: Easy AACSB: Application of knowledge 14) The two neighbors of the United States do a lot more trade with the United States than European economies of equal size. A) This contradicts predictions from gravity models. B) This is consistent with predictions from gravity models. C) This is irrelevant to any inferences that may be drawn from gravity models. D) This is because these neighboring countries have exceptionally large GDPs. E) This relates to Belgium's trade record with the U.S. Answer: B Difficulty: Moderate AACSB: Application of knowledge 15) Trade between British Columbia and a Canada province is likely much larger than trade with an equally distant U.S. state because of A) negative effects of the borders. B) size of local economy. C) population. D) religion. Answer: A Difficulty: Easy AACSB: Application of knowledge
4 Copyright © 2022 Pearson Education, Inc.
16) Which of the following does NOT explain the extent of trade between Ireland and the U.S.? A) historical ties B) cultural linguistic ties C) gravity model D) multinational corporations E) large numbers of Irish-Americans Answer: C Difficulty: Moderate AACSB: Application of knowledge 17) Canadian and Mexican economies combined are only as big as the economy of France, but they do 12 times as much trade of the United States. Using the gravity model, explain. Answer: Based on gravity model, student may discuss how Canada and Mexico are closer to the United States than France. In addition, they are also part of US-Mexico-Canada agreement (USMCA) (previously known as NAFTA) and France is not. Difficulty: Moderate AACSB: Reflective thinking 2.2
The Changing Pattern of World Trade
1) Since the early 1970s, world's trade as a share of world production has A) remained constant. B) increased. C) decreased. D) fluctuated widely with no clear trend. E) increased slightly before dropping off. Answer: B Difficulty: Easy AACSB: Application of knowledge 2) What caused the ratio of world exports to world GDP to fall sharply before World War I and not return to 1913 level until 1970s? A) Wars and protectionism. B) Natural disasters. C) Loss of interest among trading nations. D) Invention of railroad and steamship. E) Jets and Internet. Answer: A Difficulty: Easy AACSB: Application of knowledge
5 Copyright © 2022 Pearson Education, Inc.
3) A situation where a final product goes through many production stages by different companies in different countries instead of being produced by one company in one country is called A) vertical disintegration. B) vertical integration. C) horizontal integration. D) horizontal disintegration. Answer: A Difficulty: Easy AACSB: Application of knowledge 4) In 2017, the biggest share of world trade is in A) manufactured goods. B) fuels and mining products. C) agricultural products. D) services. Answer: A Difficulty: Easy AACSB: Application of knowledge 5) In the early 20th century, the United Kingdom exported mainly A) manufactured goods. B) services. C) primary products including agricultural. D) technology intensive products. E) livestock. Answer: A Difficulty: Easy AACSB: Application of knowledge 6) In the early 20th century, the United Kingdom imported mainly A) manufactured goods. B) services. C) primary products including agricultural. D) technology intensive products. E) from the United States. Answer: C Difficulty: Easy AACSB: Application of knowledge
6 Copyright © 2022 Pearson Education, Inc.
7) In the early 20th century, the United States imported and exported mainly A) primary products. B) services. C) manufactured goods. D) technology intensive products. E) livestock. Answer: A Difficulty: Easy AACSB: Application of knowledge 8) In 2015, the United States imported and exported mainly A) manufactured goods. B) primary products. C) services. D) technology intensive products. E) livestock. Answer: A Difficulty: Easy AACSB: Application of knowledge 9) Over the past 50 years, developing countries' exports have seen a(n) ________ in manufactured goods and a(n)_______ in agricultural products. A) increase; decrease B) decrease; increase C) increase; increase D) decrease; decrease Answer: A Difficulty: Easy AACSB: Application of knowledge 10) In the present, most of the exports from China are A) manufactured goods. B) services. C) primary products including agricultural. D) technology intensive products. E) overpriced by world market standards. Answer: A Difficulty: Easy AACSB: Application of knowledge
7 Copyright © 2022 Pearson Education, Inc.
11) In today's modern economy, international trade in services A) dominates world trade. B) does not exist. C) is an increasingly important component of global trade. D) is relatively stagnant. E) far surpasses the predictions of economist Alan Blinder. Answer: C Difficulty: Easy AACSB: Application of knowledge 12) When a service previously done within a country is shifted to a foreign location, the change is known as A) service outsourcing. B) location shifting. C) service outreaching. D) global transferring. Answer: A Difficulty: Easy AACSB: Application of knowledge 13) Jobs that must be done close to the customer or non-tradable consist of ________ of total U.S. employment. A) 60 percent B) 50 percent C) 40 percent D) 30 percent E) 70 percent Answer: A Difficulty: Easy AACSB: Application of knowledge 14) World trade grew rapidly in the decade leading up to World War I but then fell significantly and did not return to pre-World War I levels until the late 1970s. Explain the fall and rise of world trade. Answer: Two subsequent world wars, the Great Depression of the 1930s and the widespread protectionism depressed world trade between 1913 and 1970s. Since 1970s, world trade has risen, partly reflecting the so-called "vertical disintegration" of production where a product often goes through many production stages in different countries before reaching the hands of consumers. Difficulty: Moderate AACSB: Application of knowledge
8 Copyright © 2022 Pearson Education, Inc.
15) When comparing the composition of world trade in the early 20th century to the early 21st century, we find major compositional changes. These include a relative decline in trade in agricultural and primary-products (including raw materials). How would you explain this in terms of broad historical developments during this period? Answer: The typical composition of world production during this period experienced major changes. Focusing on today's Industrialized Countries (primarily members of the OECD), the industrial-employment composition was focused primarily on agriculture. Most value was in land. The predominant single consumption category was food. Since then, the economies shifted from the agricultural to the manufacturing sectors (continuing trends begun over a century earlier in the industrial revolution). Incomes rose, and consumption shifted in favor of (increasingly affordable) manufactures. Both income and price elasticities were greater in manufactures than in agricultural products. At the same time there was a steady tendency for synthetic (manufactured) inputs to replace agricultural based raw materials and industrial inputs. Hence, trade and, of course, international trade conformed to overall changes in patterns of world production and consumption. Difficulty: Moderate AACSB: Application of knowledge 16) Third world or developing countries mainly exported primary products in the 1970s. How have things changed since then? Explain. Answer: The developing countries have experienced major compositional shifts from exports of primary products (including agricultural and raw materials) to exports of manufactures. Depending on how much is covered during the lecture, students can explain further or do more research on their own to answer this question. Difficulty: Moderate AACSB: Application of knowledge 17) Explain how important service trade may become in the years ahead. Answer: With the advancement of telecommunication and Internet, new types of service trading such as service outsourcing will become more popular and play an important role. One example is the rise of overseas call and help center, where the person answering a customer's request for information may be thousands of miles away. Given 40% of total US employment that is in tradable activities includes more service than manufacturing jobs, trade in services may eventually become bigger than trade in manufactures. Difficulty: Moderate AACSB: Reflective thinking
9 Copyright © 2022 Pearson Education, Inc.
2.3
Do Old Rules Still Apply?
1) How have the roles of natural resources and human resources changed over the past century? Answer: A century ago, each country's exports were shaped largely by its climate and natural resources. Tropical countries exported tropical products such as coffee and cotton; land-rich countries such as the United States and Australia exported food to densely populated European nations. In modern trade, human resources and human-created resources (in the form of machinery, other types of capital, technology) are more important than natural resources. Difficulty: Moderate AACSB: Application of knowledge 2) The Services sector has been steadily rising in relative importance in GDP of the United States, as well as elsewhere around the world. Since "services" have been identified as "nontradable" (e.g., it is difficult to export haircuts), it may be argued that this trend will likely slow the rapid growth in international trade. Discuss. Answer: This argument stands on questionable logical foundations. The past half century has seen a steady growth in the absolute and relative importance of international trade. This trend has been reversed only by global conflicts, i.e. the two World Wars. This trend has remained steady and robust despite major compositional shifts (e.g., from primary to manufacturing), and location shifts (e.g., the sudden rise of NICs as a significant group of exporters). The trend will probably continue into the reasonable future, fueled by both super-regional preferential trade regions and a growing impact of the multilateral forces, represented institutionally by the World Trade Organization (WTO)—as illustrated by the recent abolishment of the epitome cartelized trade, the world trade in textiles. Driven by technology—especially in the areas of communication and transportation—a reversal of the growing trade trend is not likely in the near future. In any case, many "services" are in fact quite tradable. Examples would be financial services, long-distance teaching, "help-desk" outsourcing, consulting and management services and others. In fact, when a tourist gets a haircut, we see that even haircuts become a "tradable" service. Difficulty: Moderate AACSB: Application of knowledge
10 Copyright © 2022 Pearson Education, Inc.
International Economics: Theory and Policy, 12e (Krugman) Chapter 3 Labor Productivity and Comparative Advantage: The Ricardian Model 3.1
The Concept of Comparative Advantage
1) In order to produce winter roses, the U.S. economy must produce fewer of other things, such as computers. Such a trade-off is an example of A) opportunity cost. B) trade cost. C) sunk cost. D) production cost. Answer: A Difficulty: Easy AACSB: Application of knowledge 2) A country has a ________ in producing a good if the opportunity cost of producing that good in terms of other goods is lower in that country than it is in other countries. A) comparative advantage B) absolute advantage C) trading advantage D) cost advantage Answer: A Difficulty: Easy AACSB: Application of knowledge 3) When a country can produce a good or service at a lower opportunity cost than another country, the country is said to have a(n) A) comparative advantage B) absolute advantage C) trading advantage D) cost advantage Answer: A Difficulty: Easy AACSB: Application of knowledge 4) Trade between two countries can benefit both countries if A) each country exports that good in which it has a comparative advantage. B) each country enjoys superior terms of trade. C) each country has a more elastic demand for the imported goods. D) each country has a more elastic supply for the exported goods. E) each country produces a wide range of goods for export. Answer: A Difficulty: Easy AACSB: Application of knowledge
1 Copyright © 2022 Pearson Education, Inc.
5) A country engaging in trade according to the principles of comparative advantage gains from trade because it A) is producing exports indirectly more efficiently than it could alternatively. B) is producing imports indirectly more efficiently than it could domestically. C) is producing exports using fewer labor units. D) is producing imports indirectly using fewer labor units. E) is producing exports while outsourcing services. Answer: B Difficulty: Easy AACSB: Application of knowledge 6) The earliest statement of the principle of comparative advantage is associated with A) David Hume. B) David Ricardo. C) Adam Smith. D) Eli Heckscher. E) Bertil Ohlin. Answer: B Difficulty: Easy AACSB: Application of knowledge 7) The Ricardian model attributes the gains from trade associated with the principle of comparative advantage result to A) differences in technology. B) differences in preferences. C) differences in labor productivity. D) differences in resources. E) gravity relationships among countries. Answer: C Difficulty: Easy AACSB: Application of knowledge 8) The Ricardian model demonstrates that A) trade between two countries will benefit both countries. B) trade between two countries may benefit both regardless of which good each exports. C) trade between two countries may benefit both if each exports the product in which it has a comparative advantage. D) trade between two countries may benefit one but harm the other. E) trade between two countries always benefits the country with a larger labor force. Answer: C Difficulty: Easy AACSB: Application of knowledge
2 Copyright © 2022 Pearson Education, Inc.
9) In the Ricardian model, comparative advantage is likely to be due to A) scale economies. B) home product taste bias. C) greater capital availability per worker. D) labor productivity differences. E) political pressure. Answer: D Difficulty: Easy AACSB: Application of knowledge 10) In a two-country, two-product world, the statement "Germany enjoys a comparative advantage over France in autos relative to ships" is equivalent to A) France having a comparative advantage over Germany in ships. B) France having a comparative disadvantage compared to Germany in autos and ships. C) Germany having a comparative advantage over France in autos and ships. D) France having no comparative advantage over Germany. E) France should produce autos. Answer: A Difficulty: Moderate AACSB: Application of knowledge 3.2
A One-Factor Economy
1) Unit labor requirement is defined as A) the number of hours of labor required to produce one unit of output. B) the quantity of output that can be produced with one hour of labor. C) the amount of input other than labor required to produce one unit of output. D) the quantity of output that can be produced without using labor input. Answer: A Difficulty: Easy AACSB: Application of knowledge 2) The production possibility frontier shows A) the maximum amount of goods and services that can be produced by an economy during a given time period using all its limited resources. B) the minimum amount of goods and services that can be produced by an economy during a given time period using all its limited resources. C) the potential amount of goods and services that can be produced by an economy in the future with current limited resources. D) the increasing amount of goods and services that can be produced by an economy during a given time period without sacrificing some production of another good. E) the decreasing amount of goods and services that can be produced by an economy during a given time period with sacrificing some production of another good. Answer: A Difficulty: Easy AACSB: Application of knowledge 3 Copyright © 2022 Pearson Education, Inc.
3) If a production possibilities frontier is a straight line, then production occurs under conditions of A) increasing opportunity costs. B) constant opportunity costs. C) decreasing opportunity costs. D) infinite opportunity costs. E) uncertain opportunity costs. Answer: B Difficulty: Easy AACSB: Application of knowledge 4) The number of gallons of wine the economy would have to give up in order to produce an extra pound of cheese is called A) the opportunity cost of a pound of cheese in terms of wine. B) the opportunity cost of a gallon of wine in terms of cheese. C) the opportunity cost of a pound of cheese and a gallon of wine. D) the opportunity cost of either a pound of cheese or a gallon of wine. Answer: A Difficulty: Easy AACSB: Application of knowledge 5) Assume aLW and aLC are the unit labor requirements in wine and cheese production, respectively. The opportunity cost of cheese in terms of wine is A) aLC/aLW B) aLW/aLC C) 1/aLW D) 1/aLC Answer: A Difficulty: Easy AACSB: Application of knowledge 6) The economy will specialize in the production of ________ if the relative price of cheese exceeds its opportunity cost in terms of wine; it will specialize in the production of ________ if the relative price of cheese is less than its opportunity in terms of wine. A) cheese; wine B) wine; cheese C) cheese; cheese D) wine; wine Answer: A Difficulty: Moderate AACSB: Application of knowledge
4 Copyright © 2022 Pearson Education, Inc.
7) Assume PC and PW are the prices of cheese and wine, and aLW and aLC are the unit labor requirements in wine and cheese production, respectively. The economy will specialize in the production of cheese and will not produce wine if A) PC/PW > aLC/aLW B) PC/PW < aLC/aLW C) PC/PW = aLC/aLW D) PC/aLC = PW/aLW Answer: A Difficulty: Difficult AACSB: Application of knowledge 8) Use the information in the table below to answer the following questions.
(a) Does either country have an absolute advantage in the production of wheat or beef? Explain. (b) What is the opportunity cost of wheat in each country? (c) What is the opportunity cost of beef in each country? (d) Analyze comparative advantage and opportunities for trade between the U.S. and Argentina. Answer: (a) The U.S. has an absolute advantage in the production of both wheat and beef because labor productivity in the U.S. exceeds labor productivity in Argentina for both products. (b) In the U.S., the opportunity cost of wheat is 100/300 or 0.33 units of beef. In Argentina, the opportunity cost of wheat is 20/20 or 1.0 unit of beef. (c) In the U.S., the opportunity cost of beef is 300/100 or 3.0 units of wheat. In Argentina, the opportunity cost of beef is 20/20 or 1.0 unit of wheat. (d) The U.S. has a comparative advantage in wheat production and Argentina has a comparative advantage in beef production. If the U.S. can trade wheat to Argentina at a rate of more than 0.33 units of beef per unit of wheat, then the U.S. will benefit. If Argentina can trade beef to the U.S. at a rate of more than one unit of wheat per unit of beef, then Argentina will benefit. Difficulty: Moderate AACSB: Application of knowledge
5 Copyright © 2022 Pearson Education, Inc.
9) Use the information in the table below to answer the following questions.
(a) Does either country have an absolute advantage in the production of wheat or beef? Explain. (b) What is the opportunity cost of wheat in each country? (c) What is the opportunity cost of beef in each country? (d) Analyze comparative advantage and opportunities for trade between the U.S. and Argentina. Answer: (a) The U.S. has an absolute advantage in the production of both wheat and beef because labor productivity in the U.S. exceeds labor productivity in Argentina for both products. (b) In the U.S., the opportunity cost of wheat is 100/200 or 0.5 units of beef. In Argentina, the opportunity cost of wheat is 80/20 or 4.0 units of beef. (c) In the U.S., the opportunity cost of beef is 200/100 or 2.0 units of wheat. In Argentina, the opportunity cost of beef is 20/80 or 0.25 units of wheat. (d) The U.S. has a comparative advantage in wheat production and Argentina has a comparative advantage in beef production. If the U.S. can trade wheat to Argentina at a rate of more than 0.5 units of beef per unit of wheat, then the U.S. will benefit. If Argentina can trade beef to the U.S. at a rate of more than 0.25 unit of wheat per unit of beef, then Argentina will benefit. Difficulty: Moderate AACSB: Application of knowledge 10) Use the information in the table below to answer the following questions.
(a) Does either country have an absolute advantage in the production of wheat or beef? Explain. (b) What is the opportunity cost of wheat in each country? (c) What is the opportunity cost of beef in each country? (d) Analyze comparative advantage and opportunities for trade between the U.S. and Argentina. Answer: (a) Argentina has an absolute advantage in the production of both wheat and beef because labor productivity in Argentina exceeds labor productivity in the U.S. for both products. (b) In the U.S., the opportunity cost of wheat is 200/100 or 2.0 units of beef. In Argentina, the opportunity cost of wheat is 400/200 or 2.0 units of beef. (c) In the U.S., the opportunity cost of beef is 100/200 or 0.5 units of wheat. In Argentina, the opportunity cost of beef is 400/200 or 0.5 units of wheat. (d) Neither country has a comparative advantage and there is, therefore, no opportunity for beneficial trade. Difficulty: Moderate AACSB: Application of knowledge 6 Copyright © 2022 Pearson Education, Inc.
3.3
Trade in a One-Factor World
1) Assume Home's unit labor requirements in cheese and wine production are aLW and aLC; and Foreign's unit labor requirements in cheese and wine production are a*LW and a*LC. Home has a comparative advantage in cheese when A) aLC/aLW < a*LC/a*LW B) aLC/aLW > a*LC/a*LW C) aLC/aLW = a*LC/a*LW D) aLC/ a*LC = aLW /a*LW Answer: A Difficulty: Moderate AACSB: Analytical thinking 2) When one country can produce a unit of a good with less labor than another country, the first country has a(n) ________ in producing that good. A) absolute advantage B) comparative advantage C) advanced advantage D) comparable advantage Answer: A Difficulty: Easy AACSB: Application of knowledge 3) In order to know whether a country has a comparative advantage in the production of one particular product we need information on at least ________ unit labor requirements. A) four B) one C) two D) three E) five Answer: A Difficulty: Easy AACSB: Application of knowledge
7 Copyright © 2022 Pearson Education, Inc.
4) Given the information in the table above A) Neither country has a comparative advantage in cloth. B) Foreign has a comparative advantage in cloth. C) Home has a comparative advantage in both cloth and widgets. D) Home has a comparative advantage in cloth. E) Neither country has a comparative advantage in widgets. Answer: D Difficulty: Easy AACSB: Application of knowledge 5) Given the information in the table above, Home should A) export cloth. B) export widgets. C) export both and import nothing. D) export and import nothing. E) export widgets and import cloth. Answer: A Difficulty: Moderate AACSB: Application of knowledge 6) Given the information in the table above, if the Home economy suffered a meltdown, and the Unit Labor Requirements doubled to 20 for cloth and 40 for widgets then home should A) export cloth. B) export widgets. C) export both and import nothing. D) export and import nothing. E) export widgets and import cloth. Answer: A Difficulty: Moderate AACSB: Application of knowledge 7) Given the information in the table above, if wages were to double in Home, then Home should A) export cloth. B) export widgets. C) export both and import nothing. D) export and import nothing. E) export widgets and import cloth. Answer: A Difficulty: Moderate AACSB: Application of knowledge
8 Copyright © 2022 Pearson Education, Inc.
8) Given the information in the table above A) neither country has a comparative advantage in cloth. B) Home has a comparative advantage in widgets. C) Foreign has a comparative advantage in widgets. D) Home has a comparative advantage in both cloth and widgets. E) neither country has a comparative advantage in widgets. Answer: C Difficulty: Moderate AACSB: Application of knowledge 9) Given the information in the table above, Home's opportunity cost of cloth is A) 0.5. B) 2.0. C) 6.0. D) 1.5. E) 3.0. Answer: A Difficulty: Moderate AACSB: Analytical thinking 10) Given the information in the table above, Home's opportunity cost of widgets is A) 0.5. B) 2.0. C) 6.0. D) 1.5. E) 3.0. Answer: B Difficulty: Moderate AACSB: Analytical thinking 11) Given the information in the table above, Foreign's opportunity cost of cloth is A) 0.5. B) 2.0. C) 6.0. D) 1.5. E) 3.0. Answer: B Difficulty: Moderate AACSB: Analytical thinking
9 Copyright © 2022 Pearson Education, Inc.
12) Given the information in the table above, Foreign's opportunity cost of widgets is A) 0.5. B) 2.0. C) 6.0. D) 1.5. E) 3.0. Answer: A Difficulty: Moderate AACSB: Analytical thinking 13) Given the information in the table above, if the world equilibrium price of widgets were 4 cloth, then A) both countries could benefit from trade with each other. B) neither country could benefit from trade with each other. C) each country will want to export the good in which it enjoys comparative advantage. D) neither country will want to export the good in which it enjoys comparative advantage. E) both countries will want to specialize in cloth. Answer: A Difficulty: Moderate AACSB: Application of knowledge 14) Given the information in the table above, if the world equilibrium price of widgets were 40 cloths, then A) both countries could benefit from trade with each other. B) neither country could benefit from trade with each other. C) each country will want to export the good in which it enjoys comparative advantage. D) neither country will want to export the good in which it enjoys comparative advantage. E) both countries will want to specialize in cloth. Answer: A Difficulty: Moderate AACSB: Application of knowledge 15) In a two-product, two-country world, international trade can lead to increases in A) consumer welfare only if output of both products is increased. B) output of both products and consumer welfare in both countries. C) total production of both products but not consumer welfare in both countries. D) consumer welfare in both countries but not total production of both products. E) prices of both goods in both countries. Answer: B Difficulty: Moderate AACSB: Application of knowledge
10 Copyright © 2022 Pearson Education, Inc.
16) A nation engaging in trade according to the Ricardian model will find its consumption bundle A) inside its production possibilities frontier. B) on its production possibilities frontier. C) outside its production possibilities frontier. D) inside its trade-partner's production possibilities frontier. E) on its trade-partner's production possibilities frontier. Answer: C Difficulty: Moderate AACSB: Application of knowledge 17) According to Ricardo, a country will have a comparative advantage in the product in which its A) labor productivity is relatively low. B) labor productivity is relatively high. C) labor mobility is relatively low. D) labor mobility is relatively high. E) labor is outsourced to neighboring countries. Answer: B Difficulty: Moderate AACSB: Application of knowledge 18) Assume that labor is the only factor of production and that wages in the United States equal $20 per hour while wages in Japan are $10 per hour. Production costs would be lower in the United States as compared to Japan if A) U.S. labor productivity equaled 40 units per hour and Japan's 15 units per hour. B) U.S. labor productivity equaled 30 units per hour and Japan's 20 units per hour. C) U.S. labor productivity equaled 20 units per hour and Japan's 30 units per hour. D) U.S. labor productivity equaled 15 units per hour and Japan's 25 units per hour. E) U.S. labor productivity equaled 15 units per hour and Japan's 40 units per hour. Answer: A Difficulty: Moderate AACSB: Application of knowledge
11 Copyright © 2022 Pearson Education, Inc.
19) An examination of the Ricardian model of comparative advantage yields the clear result that trade is (potentially) beneficial for each of the two trading partners since it allows for an expanded consumption choice for each. However, for the world as a whole the expansion of production of one product must involve a decrease in the availability of the other, so that it is not clear that trade is better for the world as a whole as compared to an initial situation of non-trade (but efficient production in each country). Are there in fact gains from trade for the world as a whole? Explain. Answer: If we were to combine the production possibility frontiers of the two countries to create a single world production possibility frontier, then it is true that any change in production points (from autarky to specialization with trade) would involve a tradeoff of one good for another from the world's perspective. In other words, the new solution cannot possibly involve the production of more of both goods. However, since we know that each country is better off at the new solution, it must be true that the original points were not on the trade contract curve between the two countries, and it was in fact possible to make some people better off without making others worse off, so that the new solution does indeed represent a welfare improvement from the world's perspective. Difficulty: Difficult AACSB: Application of knowledge
20) Given the information in the table above. What is the opportunity cost of cloth in terms of widgets in Home? Answer: One half a widget. Difficulty: Moderate AACSB: Analytical thinking 21) Given the information in the table above. What is the opportunity cost of cloth in terms of widgets in Foreign? Answer: 2 widgets. Difficulty: Moderate AACSB: Analytical thinking
12 Copyright © 2022 Pearson Education, Inc.
3.4
Misconceptions About Comparative Advantage
1) Which of the following statements is TRUE? A) The competitive advantage of an industry depends not only on its productivity relative to the foreign industry, but also on the domestic wage rate relative to the foreign wage rate. B) The competitive advantage of an industry only depends on its productivity relative to the foreign industry. C) The competitive advantage of an industry only depends on the domestic wage rate relative to the foreign wage rate. D) The competitive advantage of an industry depends on a country's absolute productivity advantage over other countries. Answer: A Difficulty: Moderate AACSB: Application of knowledge 2) Mahatma Gandhi exhorted his followers in India to promote economic welfare by decreasing imports. This approach A) makes no sense. B) makes no economic sense. C) is consistent with the Ricardian model of comparative advantage. D) is not consistent with the Ricardian model of comparative advantage. E) guarantees benefits for Indian workers. Answer: D Difficulty: Moderate AACSB: Application of knowledge 3) The Country of Rhozundia is blessed with rich copper deposits. The cost of copper produced (relative to the cost of widgets produced) is therefore very low. From this information we know that A) Rhozundia has a comparative advantage in copper. B) Rhozundia should import copper and export widgets. C) Rhozundia should export both widgets and copper. D) Rhozundia should invest in more widget production. E) Rhozundia may or may not have a comparative advantage in copper. Answer: E Difficulty: Moderate AACSB: Application of knowledge
13 Copyright © 2022 Pearson Education, Inc.
4) We know that in antiquity, China exported silk because no one in any other country knew how to produce this product. From this information we know that A) China had a comparative advantage in silk. B) China had an absolute advantage, but not a comparative advantage in silk. C) no comparative advantage could exist because the technology was not diffused. D) China exported silk for political reasons even though it had no comparative advantage. E) China was unable to profit by exporting silk because it was unknown in the rest of the world. Answer: A Difficulty: Moderate AACSB: Application of knowledge 5) The pauper labor and the exploitation arguments A) are theoretical weaknesses that limit the applicability of the Ricardian concept of comparative advantage. B) are theoretically irrelevant to the Ricardian model, and do not limit its logical relevance. C) are not relevant because the Ricardian model is based on the labor theory of value. D) are not relevant because the Ricardian model allows for different technologies in different countries. E) invalidate the Ricardian model. Answer: B Difficulty: Easy AACSB: Application of knowledge 6) If labor productivities were exactly proportional to wage levels internationally, this would A) not negate the logical basis for trade in the Ricardian model. B) render the Ricardian model theoretically correct but practically useless. C) negate the logical basis for trade in the Ricardian model. D) negate the applicability of the Ricardian model if the number of products were greater than the number of trading partners. E) demonstrate the validity of the Ricardian model. Answer: A Difficulty: Moderate AACSB: Application of knowledge 7) Many countries in sub-Saharan Africa have very low labor productivities in many sectors, for example in manufacturing and agriculture. They often despair of even trying to attempt to build their industries unless it is done in an autarkic context, behind protectionist walls because they do not believe they can compete with more productive industries abroad. Discuss this issue in the context of the Ricardian model of comparative advantage. Answer: The Ricardian model of comparative advantage argues that every country must have a comparative advantage in some product (assuming there are more products than countries). However, the Ricardian model is not a growth model, and cannot be used to identify growth modes or linkages. Difficulty: Difficult AACSB: Application of knowledge
14 Copyright © 2022 Pearson Education, Inc.
8) In 1975, wage levels in South Korea were roughly 5% of those in the United States. It is obvious that if the United States had allowed Korean goods to be freely imported into the United States at that time, this would have caused devastation to the standard of living in the United States, because no producer in this country could possibly compete with such low wages. Discuss this assertion in the context of the Ricardian model of comparative advantage. Answer: According to the Ricardian Model, regardless of relative wage levels, the United States would be able to provide its populace with a higher standard of living than would be possible without trade. Also, low wages tend to be associated with low productivities. Difficulty: Moderate AACSB: Application of knowledge 3.5
Comparative Advantage with Many Goods
1) Assume w and w* are wage rates per hour in Home and in Foreign; aLi and a*Li are the unit labor requirements for Home and Foreign in production of good i. Any good for which a*Li/aLi > w/w* will be produced A) at Home. B) in Foreign. C) both at Home and in Foreign. D) neither at Home nor in Foreign. Answer: A Difficulty: Moderate AACSB: Application of knowledge 2) The two-country, multi-product model differs from the two-country, two-product model in that, in the former A) the relative wage ratio will determine the pattern of trade (which good is exported by which country). B) which country will export which product is determined entirely by labor productivity data. C) full specialization is likely to hold in equilibrium. D) none of the goods are potentially nontraded. E) domestic relative prices are not relevant. Answer: A Difficulty: Moderate AACSB: Application of knowledge 3) In a many-good Ricardian model, equilibrium relative wage is determined by A) the intersection of the derived relative demand curve for labor with the relative supply. B) the intersection of the demand curve for labor with the supply curve for labor. C) the intersection of the production possibility frontier and the horizontal axis. D) the intersection of the production possibility frontier and the vertical axis. Answer: A Difficulty: Moderate AACSB: Application of knowledge
15 Copyright © 2022 Pearson Education, Inc.
4) In a two-country, multi-product Ricardian model, the relative demand for labor has a ________ shape. A) stepped B) round C) straight line D) zigzag Answer: A Difficulty: Easy AACSB: Application of knowledge 5) How does the two-good, two-country version of the Ricardian model differ from the twocountry, many-good model in terms of the determination which goods are produced and exported by each country? Answer: In the two-good-two-country version of the Ricardian model, comparative advantage is totally determined by physical productivity ratios. Changes in wage rates in either country do not change physically determined comparative advantages, and therefore cannot affect which product will be exported by which country. However, when there are more than two goods in the two-country model, changes in wage rates in one or the other country can in fact determine which good or goods each of the countries will export. The physical productivity definition of comparative advantage employed in the two-good model becomes ambiguous. Instead, changes in relative wage rates will alter international competitiveness along the "chain of comparative advantage." Difficulty: Moderate AACSB: Application of knowledge 6) Explain the "stepped" shape of the relative demand for labor in a two-country, multiple-good Ricardian model. Answer: When we increase the wage rate of Home workers relative to Foreign workers, the relative demand for good produced in Home will decline and the demand for Home labor will decline with it. In addition, the relative demand for Home labor will drop off abruptly whenever an increase in the relative Home wage makes a good cheaper to produce in Foreign. So the curve alternates between smooth downward sloping sections where the pattern of specialization does not change and "flats" where the relative demand shifts abruptly because of shifts in the pattern of specialization. These "flats" correspond to the relative wages that equal the ratio of Home to Foreign productivity for each of the goods. Difficulty: Moderate AACSB: Application of knowledge 7) Explain the vertical shape of the relative supply of labor in a two-country, multiple-good Ricardian model. Answer: The relative supply of labor is determined by the relative sizes of Home's and Foreign's labor forces. Assuming the number of person-hours available does not vary with the wage, the relative wage has no effect on relative labor supply and RS is a vertical line. Difficulty: Moderate AACSB: Application of knowledge 16 Copyright © 2022 Pearson Education, Inc.
3.6
Adding Transport Costs and Nontraded Goods
1) Assume that transportation costs are especially high for Widgets in the two-country, twoproduct Ricardian model, and Country A enjoys a comparative advantage in Widgets, then A) country B must also enjoy a comparative advantage in Widgets. B) country B may end up exporting Widgets. C) country A may switch to having a comparative advantage in the other good. D) country A will still export Widgets. E) trade may be impossible between the two countries. Answer: E Difficulty: Moderate AACSB: Application of knowledge 2) Which of the following is most likely to be a nontraded good in a Ricardian two-country, multi-good model? A) steel B) textiles C) haircuts D) petroleum E) telemarketer services Answer: C Difficulty: Easy AACSB: Application of knowledge 3) Which of the following is most likely to be a traded good in a Ricardian two-country, multigood model? A) textiles B) cement C) haircut D) auto repair Answer: A Difficulty: Easy AACSB: Application of knowledge 4) Which of the following statements is TRUE? A) Free trade is beneficial only if your country is strong enough to stand up to foreign competition. B) Free trade is beneficial only if your competitor does not pay unreasonably low wages. C) Free trade is beneficial only if both countries have access to the same technology. D) Free trade is never beneficial for developing countries. E) Free trade can be beneficial to the economic welfare of all countries involved. Answer: E Difficulty: Moderate AACSB: Application of knowledge
17 Copyright © 2022 Pearson Education, Inc.
3.7
Empirical Evidence on the Ricardian Model
1) Which of the following has been confirmed by empirical tests of the Ricardian model? A) All predictions of the model for a multi-product, multi-country world are highly unrealistic. B) The existence of nontraded goods results in a high degree of specialization among countries. C) International trade has no impact on income distribution. D) The unimportance of economies of scale as a cause of trade. E) Companies tend to export goods in which they have a relatively high level of productivity. Answer: E Difficulty: Moderate AACSB: Application of knowledge 2) When compared with China, the growth of clothing exports originating in Bangladesh is the result of A) the comparative advantage that Bangladesh has in the production of clothing for export. B) the absolute advantage that China has in the production of clothing for export. C) the absolute advantage that Bangladesh has in the production of clothing for export. D) the comparative and absolute advantage that China has in the production of clothing for export. E) the comparative and absolute advantage that Bangladesh has in the production of clothing for export. Answer: A Difficulty: Easy AACSB: Application of knowledge 3) The growth of clothing exports originating in Bangladesh is the result of the A) high productivity of workers in Bangladesh. B) low wages in Bangladesh. C) low productivity of workers in other countries. D) low productivity of workers in Bangladesh in industries other than those that produce clothing for export. E) high wages in other countries. Answer: D Difficulty: Easy AACSB: Application of knowledge 4) Which of the following statements is TRUE? A) A country's exports are determined by its relative high productivity in the industry compared with relative productivity in other sectors. B) A country's exports are determined by its high productivity in the industry compared with that of foreigners. C) A country's exports are determined by its low productivity in the industry compared with that of foreigners. D) A country's exports are determined by its ability to match other countries' productivity. Answer: A Difficulty: Moderate AACSB: Application of knowledge 18 Copyright © 2022 Pearson Education, Inc.
5) When compared with China, the growth of clothing exports originating in Bangladesh clearly illustrates the difference between absolute and comparative advantage. Discuss and explain. Answer: While Bangladesh has an absolute disadvantage in clothing (and, on average, everything else), the country has a comparative advantage in clothing manufacture and export. Exports of clothing from Bangladesh have consequently surpassed those from China. Difficulty: Moderate AACSB: Application of knowledge 6) When compared with China, the growth of clothing exports originating in Bangladesh clearly illustrates the Ricardian model of comparative advantage. Discuss and explain. Answer: While Bangladesh has an absolute disadvantage in clothing (and, on average, everything else), the country has a comparative advantage in clothing manufacture and export. Exports of clothing from Bangladesh have consequently surpassed those from China. Difficulty: Moderate AACSB: Application of knowledge
19 Copyright © 2022 Pearson Education, Inc.
International Economics: Theory and Policy, 12e (Krugman) Chapter 4 Specific Factors and Income Distribution 4.1
The Specific Factors Model
1) The Ricardian model of international trade demonstrates that trade can be mutually beneficial. Why, then, do governments restrict imports of some goods? A) The Ricardian model is often incorrect in its prediction that trade can be mutually beneficial. B) Import restrictions are the result of trade wars between hostile countries. C) Trade can have substantial effects on a country's distribution of income. D) Imports are only restricted when foreign-made goods do not meet domestic standards of quality. E) Restrictions on imports are intended to benefit domestic consumers. Answer: C Difficulty: Easy AACSB: Application of knowledge 2) The Ricardian two-country, two-good model predicts that there are potential benefits from trade, but NOT A) when both countries have the same types of technology available. B) when one country has significantly lower wages than the other country. C) the mechanism that determines which country will specialize in which good. D) the effect of trade on income distribution. E) when one country has an absolute advantage in the production of both goods. Answer: D Difficulty: Easy AACSB: Application of knowledge 3) International trade can have important effects on the distribution of income because A) the more powerful country dictates the terms of trade. B) some resources are immobile in the short run. C) different countries use different currencies. D) of government corruption. E) rich countries take advantage of poor countries. Answer: B Difficulty: Moderate AACSB: Application of knowledge
1 Copyright © 2022 Pearson Education, Inc.
4) The Ricardian model of international trade demonstrates that trade can be mutually beneficial. Why, then, do governments restrict imports of some goods? A) Import restrictions are the result of trade wars between hostile countries. B) Trade can have significant harmful effects on some segments of a country's economy. C) Restrictions on imports can have significant beneficial effects on domestic consumers. D) The Ricardian model is often incorrect in its prediction that trade can be mutually beneficial. E) Imports are only restricted when foreign-made goods do not meet domestic standards of quality. Answer: B Difficulty: Moderate AACSB: Application of knowledge 5) The effect of trade on income distribution A) implies that there are no real gains from trade. B) is insignificant in the short run. C) can be significant in the short run. D) refutes the model of comparative advantage. E) is positive for all segments of an economy. Answer: C Difficulty: Easy AACSB: Application of knowledge 6) International trade can have important effects on the distribution of income because A) different industries employ different factors of production. B) rich countries take advantage of poor countries. C) of government corruption. D) different countries use different currencies. E) the more powerful country dictates the terms of trade. Answer: A Difficulty: Easy AACSB: Application of knowledge 7) U.S.'s trade policies with regard to sugar import restrictions reflect the fact that A) U.S. sugar cane farmers have significant political power. B) U.S. imports most of the sugar consumed in the country. C) U.S. has a comparative advantage in sugar production and therefore exports most of its sugar crop. D) there would be no gains from trade available to U.S. if it engaged in free trade in sugar. E) there are gains from trade that U.S. captures by engaging in free trade in sugar. Answer: A Difficulty: Easy AACSB: Application of knowledge
2 Copyright © 2022 Pearson Education, Inc.
8) The specific factors model was developed by A) Adam Smith and David Ricardo. B) C.B. deMille and Gordon Willis. C) Paul Samuelson and Ronald Jones. D) Bill Clinton and Monica Lewinsky. E) Richard Nixon and Robert Kennedy. Answer: C Difficulty: Easy AACSB: Application of knowledge 9) In the specific factors model, labor is defined as a(an) A) intensive factor. B) mobile factor. C) variable factor. D) fixed factor. E) specific factor. Answer: B Difficulty: Easy AACSB: Application of knowledge 10) In the specific factors model, which of the following is treated as a specific factor? A) food B) land C) labor D) technology E) cloth Answer: B Difficulty: Easy AACSB: Application of knowledge 11) In the specific factors model, which of the following is treated as a specific factor? A) capital B) technology C) cloth D) food E) labor Answer: A Difficulty: Easy AACSB: Application of knowledge
3 Copyright © 2022 Pearson Education, Inc.
12) The specific factors model assumes that there are ________ goods and ________ factor(s) of production. A) two; two B) two; one C) three; two D) four; three E) two; three Answer: E Difficulty: Easy AACSB: Application of knowledge 13) A factor of production that cannot be used outside of a particular sector of an economy is a(an) A) variable factor. B) export-competing factor. C) mobile factor. D) import-competing factor. E) specific factor. Answer: E Difficulty: Easy AACSB: Application of knowledge 14) A factor of production that can be used only in the production of one good is a(an) A) import-competing factor. B) export-competing factor. C) mobile factor. D) variable factor. E) specific factor. Answer: C Difficulty: Easy AACSB: Application of knowledge 15) The degree of a factor's specificity is directly related to A) the cost of the factor as a proportion of the long-run total cost of production. B) technology differences between two countries, with a more advanced technology resulting in more factor specificity. C) the amount of time required to redeploy the factor to a different industry. D) factor quality, with higher quality factors having a higher level of specificity. E) the mobility of the factor, with more mobile factors having more specificity. Answer: C Difficulty: Easy AACSB: Application of knowledge
4 Copyright © 2022 Pearson Education, Inc.
16) The degree of a factor's specificity is inversely related to A) the amount of time required to redeploy the factor to a different industry. B) factor quality, with lower quality factors having a lower level of specificity. C) the cost of the factor as a proportion of the long-run total cost of production. D) the mobility of the factor, with more mobile factors having less specificity. E) technology differences between two countries, with a less advanced technology resulting in less factor specificity. Answer: D Difficulty: Easy AACSB: Application of knowledge 17) A worker who has invested in ________ skills will be ________ mobile than would otherwise be the case. A) occupation-specific; less B) occupation-nominal; less C) occupation-specific; more D) ethical; less E) ethical; more Answer: A Difficulty: Easy AACSB: Application of knowledge 18) Marginal product of labor is A) the addition to output generated by adding one more person-hour. B) the output produced by an average worker per hour. C) the change in average output when there is a change in the number of workers. D) the maximum total output generated given other fixed inputs of production. Answer: A Difficulty: Easy AACSB: Application of knowledge 19) Because of ________, adding a worker means that each worker has less capital to work with, each successive increment of labor will add less to production than the last. A) diminishing returns B) increasing returns C) constant returns D) total returns Answer: A Difficulty: Easy AACSB: Application of knowledge
5 Copyright © 2022 Pearson Education, Inc.
20) In the specific factors model, a country's production possibility frontier is ________ because of ________. A) a straight line; diminishing marginal returns B) a curved line; diminishing marginal returns C) a straight line; constant marginal returns D) a curved line; constant marginal returns E) a curved line; a limited supply of labor Answer: B Difficulty: Easy AACSB: Application of knowledge 21) In the four-quadrant diagram of the specific factors model, the graph in the upper right quadrant is a country's A) production possibility frontier. B) production function for food. C) labor allocation constraint. D) labor supply curve. E) production function for cloth. Answer: A Difficulty: Easy AACSB: Application of knowledge 22) In the four-quadrant diagram of the specific factors model, the graph in the lower right quadrant is a country's A) production function for food. B) production possibility frontier. C) labor supply curve. D) labor allocation constraint. E) production function for cloth. Answer: E Difficulty: Easy AACSB: Application of knowledge 23) In the four-quadrant diagram of the specific factors model, the graph in the upper left quadrant is a country's A) production possibility frontier. B) labor allocation constraint. C) production function for food. D) production function for cloth. E) labor supply curve. Answer: C Difficulty: Easy AACSB: Application of knowledge
6 Copyright © 2022 Pearson Education, Inc.
24) The slope of a country's production possibility frontier with cloth measured on the horizontal and food measured on the vertical axis in the specific factors model is equal to ________ and it ________ as more cloth is produced. A) -MPLC/MPLF; becomes steeper B) -MPLF/MPLC; becomes steeper C) -MPLC/MPLF; is constant D) -MPLF/MPLC; becomes flatter E) -MPLF/MPLC; is constant Answer: B Difficulty: Easy AACSB: Application of knowledge 25) The slope of a country's production possibility frontier with cloth measured on the horizontal and food measured on the vertical axis in the Ricardian model is equal to ________ and it ________ as more cloth is produced. A) -MPLF/MPLC; becomes steeper B) -MPLF/MPLC; becomes flatter C) -MPLC/MPLF; is constant D) -MPLF/MPLC; is constant E) -MPLC/MPLF; becomes steeper Answer: D Difficulty: Easy AACSB: Application of knowledge 26) Under perfect competition, the equilibrium price of labor used to produce cloth will be equal to A) the marginal product of labor in the production of cloth times the price of cloth. B) the ratio of the marginal product of labor in the production of cloth to the marginal product of labor in the production of food times the ratio of the price of cloth to the price of food. C) the slope of the production possibility frontier. D) the average product of labor in the production of cloth times the price of cloth. E) the price of cloth divided by the marginal product of labor in the production of cloth. Answer: A Difficulty: Easy AACSB: Application of knowledge 27) When a country's labor market is in equilibrium in the specific factors model, the wage rate A) will be higher in the sector where product price is lower. B) will be the same in both sectors. C) will be higher in the sector where product price is higher. D) will be higher in the export-competing sector. E) will be higher in the import-competing sector. Answer: B Difficulty: Easy AACSB: Application of knowledge 7 Copyright © 2022 Pearson Education, Inc.
28) Which of the following statements is TRUE? A) At the production point, the production possibility frontier must be tangent to a line whose slope is minus the price of cloth divided by that of food. B) At the production point, the production possibility frontier is a straight line. C) At the production point, the production possibility frontier is sloping upwards. D) At the production point, the production possibility frontier is sloping downwards. Answer: A Difficulty: Easy AACSB: Application of knowledge 29) In the specific factors model, which of the following will increase the quantity of labor used in food production? A) an increase in the price of cloth relative to that of food B) an increase in the price of food relative to that of cloth C) a decrease in the price of labor D) an equal percentage increase in the price of food and cloth E) an equal percentage decrease in the price of food and cloth Answer: B Difficulty: Moderate AACSB: Application of knowledge 30) In the specific factors model, which of the following will increase the quantity of labor used in cloth production? A) a decrease in the price of labor B) an increase in the price of cloth relative to that of food C) an equal percentage decrease in the price of food and cloth D) an equal percentage increase in the price of food and cloth E) an increase in the price of food relative to that of cloth Answer: B Difficulty: Moderate AACSB: Application of knowledge 31) The slope of a country's production possibility frontier is equal to ________ and the optimal production point is located where the slope is equal to ________. Assume that output of good Y is measured on the vertical axis, output of good X is measured on the horizontal axis, MPL is the marginal product of labor with a subscript indicating which good, P is the price of a good, and w is the wage rate. A) -PX/PY; -MPLY/MPLX B) -MPLY/MPLX; -PX/PY C) -MPLX/MPLY; -PX/PY D) -MPLY/w; -MPLF/w E) -PX/w; -PY/w Answer: B Difficulty: Moderate AACSB: Application of knowledge 8 Copyright © 2022 Pearson Education, Inc.
32) In the specific factors model, a 5% increase in the price of food accompanied by a 5% increase in the price of cloth will cause wages to ________, the production of cloth to ________, and the production of food to ________. A) increase by more then 5%; increase; remain unchanged B) increase by 5%; remain unchanged; remain unchanged C) remain constant; decrease; decrease D) remain constant; increase; increase E) increase by less then 5%; decrease; increase Answer: B Difficulty: Moderate AACSB: Application of knowledge 33) In the specific factors model, a 5% increase in the price of food accompanied by a 5% increase in the price of cloth will cause ________ in the welfare of labor, ________ in the welfare of the fixed factor in the production of food, and ________ in the welfare of the fixed factor in the production of cloth. A) a decrease; an increase; an increase B) a decrease; a decrease; a decrease C) no change; no change; no change D) an increase; a decrease; a decrease E) an increase; an increase; an increase Answer: C Difficulty: Moderate AACSB: Application of knowledge 34) In the specific factors model, a 5% decrease in the price of food accompanied by a 5% decrease in the price of cloth will cause ________ in the welfare of labor, ________ in the welfare of the fixed factor in the production of food, and ________ in the welfare of the fixed factor in the production of cloth. A) an increase; an increase; an increase B) a decrease; a decrease; a decrease C) no change; no change; no change D) a decrease; an increase; an increase E) an increase; a decrease; a decrease Answer: C Difficulty: Moderate AACSB: Application of knowledge
9 Copyright © 2022 Pearson Education, Inc.
35) In the specific factors model, a 5% increase in the price of food accompanied by a 0% increase in the price of cloth will cause wages to ________, the production of cloth to ________, and the production of food to ________. A) remain constant; decrease; decrease B) increase by less then 5%; decrease; increase C) remain constant; increase; increase D) increase by more then 5%; increase; remain unchanged E) increase by 5%; remain unchanged; remain unchanged Answer: B Difficulty: Moderate AACSB: Application of knowledge 36) In the specific factors model, a 0% increase in the price of food accompanied by a 5% increase in the price of cloth will cause wages to ________, the production of cloth to ________, and the production of food to ________. A) increase by more then 5%; increase; remain unchanged B) increase by less then 5%; increase; decrease C) remain constant; decrease; decrease D) increase by 5%; remain unchanged; remain unchanged E) remain constant; increase; increase Answer: B Difficulty: Moderate AACSB: Application of knowledge 37) In the specific factors model, a 5% increase in the price of food accompanied by a 10% increase in the price of cloth will cause ________ in the welfare of labor, ________ in the welfare of the fixed factor in the production of food, and ________ in the welfare of the fixed factor in the production of cloth. A) an ambiguous change; an increase; a decrease B) a decrease; an ambiguous change; an ambiguous change C) an ambiguous change; an ambiguous change; an ambiguous change D) an ambiguous change; a decrease; an increase E) an increase; a decrease; an increase Answer: D Difficulty: Moderate AACSB: Application of knowledge
10 Copyright © 2022 Pearson Education, Inc.
38) In the specific factors model, a 5% increase in the price of food accompanied by a 1% increase in the price of cloth will cause ________ in the welfare of labor, ________ in the welfare of the fixed factor in the production of food, and ________ in the welfare of the fixed factor in the production of cloth. A) an increase; a decrease; an increase B) an ambiguous change; an increase; a decrease C) an ambiguous change; an ambiguous change; an ambiguous change D) a decrease; an ambiguous change; an ambiguous change E) an ambiguous change; a decrease; an increase Answer: B Difficulty: Moderate AACSB: Application of knowledge 39) The effect of a relative price change on the distribution of income can be stated as followed EXCEPT A) The change in welfare for the mobile factor is certain. B) The factor specific to the sector whose relative price increases is definitely better off. C) The factor specific to the sector whose relative price decreases is definitely worse off. D) The change in welfare for the mobile factor is ambiguous. Answer: A Difficulty: Easy AACSB: Application of knowledge 40) Why does trade have strong effects on the distribution of income? Answer: First, resources cannot move immediately or without cost from one industry to another–a short-run consequence of trade. Second, industries differ in the factors of production they demand. A shift in the mix of goods a country produces will ordinarily reduce the demand for some factors of production while raising the demand for others–a long-run consequence of trade. While trade may benefit a nation as a whole, it often hurts significant groups within the country in the short run and potentially, but to a lesser extent, in the long run. Difficulty: Easy AACSB: Application of knowledge 41) Compare the possibilities possibility frontiers in the Ricardian model and the specific factors model? Answer: In the Ricardian model, where labor is the only factor of production, the production possibility frontier is a straight line because the opportunity cost of cloth in terms of food is constant. In the specific factors model, however, the addition of other factors of production changes the shape of the production possibility frontier PP to a curve. The curvature of PP reflects diminishing returns to labor in each sector; these diminishing returns are the crucial difference between the specific factors and the Ricardian models. Difficulty: Easy AACSB: Application of knowledge
11 Copyright © 2022 Pearson Education, Inc.
42) How does a relative price change affect the distribution of income? Answer: The effect of a relative price change on the distribution of income can be summarized as follows: (1) The factor specific to the sector whose relative price increases is definitely better off. (2) The factor specific to the sector whose relative price decreases is definitely worse off. (3) The change in welfare for the mobile factor is ambiguous. Difficulty: Easy AACSB: Application of knowledge 4.2
International Trade in the Specific Factors Model
1) For trade to take place, A) a country must face a world relative price that differs from the relative price that would prevail in the absence of trade. B) a country must face a world relative price that is the same as the relative price that would prevail in the absence of trade C) a country must adjust its domestic relative price to match the world relative price. D) a country must influence world relative price to match the relative price that would prevail in the absence of trade. Answer: A Difficulty: Easy AACSB: Application of knowledge 2) The relative price of a unit of cloth in the small isolated country of Moribundia is 5 units of food. When the central city, Mudhole, puts in an airstrip, the country is able to engage in trade. If the relative price of cloth in the outside world is 3 units of food, then Moribundia will export ________ and ________ factors used in the production of ________ will benefit. A) cloth; mobile; cloth B) food; mobile; food C) food; immobile; cloth D) food; immobile; food E) cloth; immobile; cloth Answer: D Difficulty: Moderate AACSB: Application of knowledge 3) The relative price of a unit of cloth in the small isolated country of Moribundia is 5 units of food. When the central city, Mudhole, puts in an airstrip, the country is able to engage in trade. If the relative price of cloth in the outside world is 8 units of food, then Moribundia will export ________ and ________ factors used in the production of ________ will benefit. A) cloth; immobile; food B) food; mobile; food C) food; immobile; food D) cloth; immobile; cloth E) cloth; mobile; cloth Answer: D Difficulty: Moderate AACSB: Application of knowledge 12 Copyright © 2022 Pearson Education, Inc.
4) Which of the following statement is TRUE? A) When opening up to trade, an economy exports the good whose relative price has increased and imports the good whose relative price has decreased. B) When opening up to trade, an economy exports the good whose relative price has decreased and imports the good whose relative price has increased. C) When opening up to trade, an economy exports the good whose relative price has decreased and imports the good whose relative price has stayed the same. D) When opening up to trade, an economy exports the good whose relative price has stayed the same and imports the good whose relative price has increased. Answer: A Difficulty: Easy AACSB: Application of knowledge 5) Why might the relative supply curve for the world be different from that for the specific factors economy? What is it important for trade to take place? Answer: The other countries in the world could have different technologies, as in the Ricardian model. When the specific factors model has more than one factor of production, the other countries could also differ in their resources: the total amounts of land, capital, and labor available. What is important here is that the economy faces a different relative price when it is open to international trade. Difficulty: Easy AACSB: Application of knowledge
13 Copyright © 2022 Pearson Education, Inc.
4.3
Income Distribution and the Gains from Trade
1) Which of the following statements is TRUE? A) Trade benefits the factor specific to the export sector of each country but hurts the factor specific to the import-competing sectors, with ambiguous effects on mobile factors. B) Trade hurts the factor specific to the export sector of each country but benefits the factor specific to the import-competing sectors, with ambiguous effects on mobile factors. C) Trade benefits both the factor specific to the export sector of each country but the factor specific to the import-competing sectors, with ambiguous effects on mobile factors. D) Trade hurts both the factor specific to the export sector of each country but hurts the factor specific to the import-competing sectors, with ambiguous effects on mobile factors. Answer: A Difficulty: Easy AACSB: Application of knowledge 2) In the specific factors model, the effects of trade on welfare are ________ for mobile factors, ________ for fixed factors used to produce the exported good, and ________ for fixed factors used to produce the imported good. A) ambiguous; positive; negative B) positive; positive; positive C) ambiguous; negative; positive D) positive; ambiguous; ambiguous E) negative; ambiguous; ambiguous Answer: A Difficulty: Easy AACSB: Application of knowledge 3) In the specific factors model, the effects of trade on welfare overall are ________ and for fixed factors used to produce the exported good they are ________. A) positive; positive B) positive; ambiguous C) negative; positive D) positive; negative E) ambiguous; positive Answer: A Difficulty: Easy AACSB: Application of knowledge
14 Copyright © 2022 Pearson Education, Inc.
4) In the specific factors model, the effects of trade on welfare overall are ________ and for fixed factors used to produce the imported good they are ________. A) ambiguous; positive B) positive; negative C) positive; positive D) positive; ambiguous E) negative; positive Answer: B Difficulty: Easy AACSB: Application of knowledge 5) The overall welfare effects of trade are ________ if ________. A) positive; the domestic economy grows faster than do foreign economies B) positive; more people gain from trade than lose from it C) negative; some people are made worse off by trade D) negative; those who lose can compel those who gain to compensate them for their losses E) positive; those who gain can compensate those who lose and still be better off Answer: E Difficulty: Easy AACSB: Application of knowledge 6) Suppose a specific factors economy produces two goods: F and C. Given that the economy is open to trade, and assuming that D is consumption, Q is production, and P is price, the budget constraint can be defined as A) =( / )*( - ). B) C)
-
=( +
/
)*( =
+
). .
D) + =( / )*( + ). Answer: A Difficulty: Moderate AACSB: Application of knowledge 7) A country's budget constraint states that A) real income in the exporting country must be equal to real income in the importing country. B) the value of exports must be equal to the value of imports. C) a country will engage in trade only if the value of imports exceed the value of exports. D) unless a country engages in trade, the value of exports cannot exceed the value of goods produced. E) a country will engage in trade only if the value of exports exceeds the value of imports. Answer: B Difficulty: Easy AACSB: Application of knowledge
15 Copyright © 2022 Pearson Education, Inc.
8) A country's budget constraint states that A) real income in the exporting country must be equal to real income in the importing country. B) unless a country engages in trade, the value of goods consumed cannot exceed the value of goods produced. C) a country will engage in trade only if the value of goods consumed exceeds the value of goods produced. D) a country will engage in trade only if the value of goods produced exceeds the value of goods consumed. E) whether or not a country engages in trade, the value of goods consumed must be equal to the value of goods produced. Answer: E Difficulty: Easy AACSB: Application of knowledge 9) The fundamental reason why trade potentially benefits a country is that it A) expands the economy's choices. B) guarantees that everyone is better off. C) promotes restoration of natural resources. D) increases dependence on foreign countries. Answer: A Difficulty: Easy AACSB: Application of knowledge 10) In the specific factors model, who gains and who loses from trade as relative price changes when a country is open to trade? Answer: Trade benefits the factor specific to the export sector of each country but hurts the factor specific to the import-competing sectors, with ambiguous effects on mobile factors. Difficulty: Easy AACSB: Application of knowledge 11) What are the two important features of the budget constraint for a trading economy? Answer: Suppose the economy is producing two goods: F and C. Given that the economy is open to trade and P is the price. First, the slope of the budget constraint is minus / , the relative price of cloth. The reason is that consuming one less unit of cloth saves the economy this is enough to purchase / extra units of food. In other words, one unit of cloth can be
;
exchanged on world markets for / units of food. Second, the budget constraint is tangent to the production possibility frontier at the chosen production point. Difficulty: Easy AACSB: Application of knowledge
16 Copyright © 2022 Pearson Education, Inc.
4.4
The Political Economy of Trade: A Preliminary View
1) Those who will lose from free trade are ________ factors in sectors that produce goods that are ________. A) mobile; also imported B) mobile; exported C) immobile; exported D) mobile; untraded E) immobile; also imported Answer: E Difficulty: Easy AACSB: Application of knowledge 2) The effect of trade on specialized employees of import-competing industries will be ________ jobs and ________ pay because they are relatively ________. A) more; lower; immobile B) more; higher; immobile C) fewer; lower; immobile D) more; higher; mobile E) fewer; lower; mobile Answer: C Difficulty: Moderate AACSB: Application of knowledge 3) Economists consider the effects of free trade on income distribution to be ________ important than the effects on overall welfare because ________. A) more; the effects on income distribution are major and consequential B) less; the effects on income distribution are minor and inconsequential C) more; those who are harmed are not compensated by those who gain D) less; the wealthy benefit and only the poor lose E) less; those who are harmed can be compensated by those who gain Answer: E Difficulty: Moderate AACSB: Application of knowledge 4) Economists consider the effects of free trade on income distribution to be ________ important than the effects on overall welfare because ________. A) more; the effects on income distribution are major and consequential B) less; the wealthy benefit and only the poor lose C) less; many factors besides trade affect income distribution D) more; those who are harmed are not compensated by those who gain E) less; the effects on income distribution are minor and inconsequential Answer: C Difficulty: Moderate AACSB: Application of knowledge
17 Copyright © 2022 Pearson Education, Inc.
5) There is a bias in the political process against free trade because A) those who gain from free trade can't compensate those who lose. B) the gains from free trade cannot be measured. C) those who lose from free trade are better organized than those who gain. D) there is a high correlation between the volume of imports and the unemployment rate. E) foreign governments make large donations to U.S. political campaigns. Answer: C Difficulty: Moderate AACSB: Application of knowledge 6) Economists believe A) it is more important to stress the overall potential gains from trade than the possible losses to some groups in a country. B) it is more important to stress the overall potential losses from trade than the possible gains to some groups in a country. C) it is important to focus only on the overall potential gains from trade. D) it is important to focus only on the losses from trade. Answer: A Difficulty: Easy AACSB: Application of knowledge 7) When conflicting interests arise during open trade, A) economists do not often have the deciding voice in economic policy. B) economists often have the deciding voice in economic policy. C) economists are only concerned about their own interests in economic policy. D) economists often stand up for the more organized groups in economic policy. Answer: A Difficulty: Easy AACSB: Application of knowledge 8) U.S. imports of sugar are limited by an import quota that, according to a study updated in 2015, imposed annual losses on American consumers of A) $3.5 billion. B) $1.5 billion. C) $1 billion. D) $3.7 billion. E) $2 billion. Answer: A Difficulty: Easy AACSB: Application of knowledge
18 Copyright © 2022 Pearson Education, Inc.
9) U.S. imports of sugar are limited by an import quota that, according to a study updated in 2015, imposed a total cost on American consumers close to $________, or an average cost of ________ per year for every U.S. household. A) $2 billion; $110 B) $3 billion; $2,000 C) $3.5 billion; $30 D) $105 million; $3 E) $370 million; $2,000 Answer: C Difficulty: Easy AACSB: Application of knowledge 10) U.S. imports of sugar are limited by an import quota that, according to a study updated in 2015, imposed a total cost on American consumers close to $________, or an average consumer cost of over ________ per year for every job saved in the U.S sugar industry. A) $3 billion; $10 B) $105 million; $3 C) $2 billion; $110 D) $3.5 billion; $3,000,000 E) $370 million; $20 Answer: D Difficulty: Easy AACSB: Application of knowledge 11) The ________ provides extended unemployment coverage for an additional year and tuition reimbursement for new job skill acquisitions to some of those affected workers who lost jobs in the import-competing sectors. A) U.S. Trade Adjustment Assistance program B) U.S. Unemployment Assistance program C) U.S. Civilian Conservation Corps program D) U.S. Works Progress program Answer: A Difficulty: Easy AACSB: Application of knowledge 12) Over the past 60 years in the United States, A) there is no evidence of a positive correlation between the unemployment rate and imports. B) there is evidence of a positive correlation between the unemployment rate and imports. C) there is evidence of a positive correlation between the unemployment rate and overall economic conditions. D) there is evidence of a positive correlation between the unemployment rate and exports. Answer: A Difficulty: Easy AACSB: Application of knowledge
19 Copyright © 2022 Pearson Education, Inc.
13) Those U.S. industries that compete most heavily with Chinese imports tend to A) suffer the most severe employment losses. B) gain the most employment by trade. C) be unaffected by trade. D) switch to all non-manufacturing services. Answer: A Difficulty: Easy AACSB: Application of knowledge 14) Many workers in US import-competing industries with China A) suffer long unemployment spells and face large drops in wages when they do find another employment. B) got laid off and then found another employment with higher wages. C) are not affected by competition and trade. D) get retrained to find a better job. Answer: A Difficulty: Easy AACSB: Application of knowledge 15) It is hard to make the case that the decline in the U.S. manufacturing share down to ________ in 2010 was driven by the growth in Chinese imports. A) 8.8 percent B) 7.8 percent C) 6.8 percent D) 5.8 percent Answer: A Difficulty: Easy AACSB: Application of knowledge 16) Trump administration's trade war with China resulted in A) the sharp decrease in US imports for 2019. B) the sharp increase in imports for 2019. C) the sharp increase in exports for 2019. D) the sharp decrease in exports for 2019. Answer: A Difficulty: Easy AACSB: Application of knowledge
20 Copyright © 2022 Pearson Education, Inc.
17) Protecting the import-competing sectors limits the number of imported intermediate goods, therefore ________ product prices and has a ________ impact on employment by U.S. producers who rely on those inputs. A) raises; negative B) lowers; positive C) lowers; unknown D) raises; positive Answer: A Difficulty: Easy AACSB: Application of knowledge 18) As a result of Trump administration's tariffs on selected imports to protect manufacturing jobs, governments in China and other countries that export those affected goods to the United States retaliated with A) higher tariffs of their own imposed on U.S. exporters. B) stopped trading with the U.S. C) lowered tariffs of their own imposed on U.S. exporters. D) increased government subsidies for those affected export goods. Answer: A Difficulty: Easy AACSB: Application of knowledge 19) In many of the protected sectors contained a high proportion of intermediate goods imported by U.S. producers, A) the jobs affected in sectors that use imports as intermediate goods vastly outnumber the jobs in the protected sectors. B) the jobs affected in sectors that use imports as intermediate goods vastly was the same number as the jobs in the protected sectors. C) the jobs in the protected sectors vastly outnumber the jobs affected in sectors that use imports as intermediate goods. D) the jobs in sectors that use imports as intermediate goods and the jobs in the protected sectors were both preserved. Answer: A Difficulty: Easy AACSB: Application of knowledge 20) Why don't economists generally stress the income distribution effects of trade? Answer: (1) Income distribution effects are not specific to international trade. (2) It is always better to allow trade and compensate those who are hurt by it than to prohibit the trade. (3) Those who stand to lose from increased trade are typically better organized than those who stand to gain (because the former are more concentrated within regions and industries). Difficulty: Easy AACSB: Application of knowledge
21 Copyright © 2022 Pearson Education, Inc.
21) Describe the gains and losses from U.S. sugar import quota. Why has it been going on for many years and continuing to be supported under 2014 U.S. Farm Bill? Answer: Under the US sugar import quota, the average price of sugar in the US market has been more than twice the average price on world market. Although the gains for American sugar producers are substantially smaller compared to the loss incurred by American consumers every year because the import restrictions, this has been going on for many years because those sugar producers are organized in trade associations that actively lobby on their members' behalf and make large campaign contributions. On the other hand, most consumers are unaware that the import quota even exists, let alone that it reduces their standard of living. Most of the cost is hidden because most sugar is consumed as an ingredient in other foods rather than purchased directly. Even if they were aware, $30 is not a large enough sum to provoke people into organizing protests and writing letters to their congressional representatives. Difficulty: Easy AACSB: Application of knowledge 22) The job losses generated by trade receive much more public attention than job gains from trade. Discuss. Answer: In the U.S. economy, the job losses from import competition are much more visible– and much more directly connected with trade policy–than the jobs gains from trade in other sectors. Some of those new export sector jobs are outside of manufacturing (such as firms providing business services to foreign customers). Because many imported goods are used as intermediate inputs, new jobs are also created at firms that benefit from cheaper inputs in their production process. Therefore, it is much harder to trace all those new jobs created by trade and job losses attract much more public attention. Difficulty: Easy AACSB: Application of knowledge 23) Explain the high costs of employment losses in U.S. industries that compete heavily with China's imports. Answer: U.S. industries that compete most heavily with Chinese imports tend to suffer the most severe employment losses, especially for workers with relatively lower education levels whose skills are closely tied to a sector in decline in the United States. Many of those workers suffer long unemployment spells and face large drops in wages when they do find another employment. And because manufacturing employment in those hard-hit sectors is geographically concentrated, this impact of import competition also translates into prolonged negative shocks for some of the affected regions. Difficulty: Easy AACSB: Application of knowledge 24) Why is it not surprising that Trump tariffs have led to job losses in manufacturing? Answer: (1) Those tariffs generated job losses in downstream U.S. manufacturing sectors that were double the number of jobs gained in the sectors protected by the tariffs. (2) Countries retaliated with tariffs of their own on US exporters, and job losses caused by the retaliatory tariffs were more than triple that number of jobs gained. Difficulty: Easy AACSB: Application of knowledge 22 Copyright © 2022 Pearson Education, Inc.
4.5
International Labor Mobility
1) In modern economies, A) restrictions on international labor mobility are common. B) labor is far more mobile internationally than it is intra-nationally. C) outsourcing increases international labor mobility. D) restrictions on international labor mobility are rare. E) labor is far more mobile internationally than capital. Answer: A Difficulty: Easy AACSB: Application of knowledge 2) Whenever international migration is possible, A) workers will want to move from the low-wage to the high-wage country. B) workers will want to move from the high-wage to the low-wage country. C) workers will want to stay in the low-wage country without the desire to move. D) workers will want to stay closer to where their families are. Answer: A Difficulty: Easy AACSB: Application of knowledge
23 Copyright © 2022 Pearson Education, Inc.
3) Refer to the graph above. Points A, B, and C represent ________, ________, and ________, respectively. A) the global wage rate before migration; the wage rate in foreign after migration; the wage rate in home after migration B) the global wage rate before migration; the wage rate in home after migration; the wage rate in foreign after migration C) equilibrium wage rate after migration from foreign to home has occurred; the wage rate in home before migration; the wage rate in foreign before migration D) equilibrium wage rate after migration from home to foreign has occurred; the wage rate in foreign before migration; the wage rate in home before migration E) the wage rate in home before migration; the wage rate in home after migration; the wage rate in foreign after migration Answer: D Difficulty: Easy AACSB: Application of knowledge
24 Copyright © 2022 Pearson Education, Inc.
4) In the two-country model of international labor mobility A) the effect of migration is to cause real wages in the two countries to diverge. B) the long-run equilibrium global real wage is equal to the greater of the pre-migration wages in the two countries. C) labor has only limited international mobility. D) the long-run equilibrium global real wage is equal to the lesser of the pre-migration wages in the two countries. E) the effect of migration is to cause real wages in the two countries to converge. Answer: E Difficulty: Easy AACSB: Application of knowledge 5) In the two-country model of international labor mobility A) the long-run equilibrium assumes countries' policies place significant restrictions on migration. B) the long-run equilibrium assumes that desired migration exceeds actual migration. C) the long-run equilibrium assumes that actual migration exceeds desired migration. D) the long-run equilibrium is the result of a divergence of the real wages in the two countries. E) the long-run equilibrium assumes that desired and actual migration are equal. Answer: E Difficulty: Moderate AACSB: Application of knowledge 6) In the two-country model of international labor mobility A) migration has no effect on global output, although some groups are made better off. B) migration has no effect on global output, although some groups are made worse off. C) migration results in increased global output, and all groups are made better off. D) migration may reduce global output, although some groups are made better off. E) migration results in increased global output, although some groups are made worse off. Answer: E Difficulty: Easy AACSB: Application of knowledge 7) Which of the following statements is MOST accurate? A) International labor mobility allows everyone to be made better off in principle, but leaves some groups worse off in practice. B) International labor mobility makes everyone better off. C) International labor mobility makes everyone worse off. D) International labor mobility allows countries to increase their overall welfare. Answer: A Difficulty: Easy AACSB: Application of knowledge
25 Copyright © 2022 Pearson Education, Inc.
8) In the past decade, with the European Union expansion to the east, A) there has been large immigration flows within this newly expanded union. B) there has been insignificant immigration flows within EU. C) there has been decreasing immigration flows within this newly expanded union. D) there has been no change in immigration flows within EU. Answer: A Difficulty: Easy AACSB: Application of knowledge 9) Following Germany's reunification in 1990, A) wage convergence was observed between East and West Germany as well as between Eastern and Western Europe. B) wage divergence was observed between East and West Germany as well as between Eastern and Western Europe. C) wage gaps and inequality were getting bigger between East and West Germany as well as between Eastern and Western Europe. D) wage differences stayed the same between East and West Germany as well as between Eastern and Western Europe Answer: A Difficulty: Easy AACSB: Application of knowledge 10) Which of the following statements is TRUE? A) Migrating workers often have very different characteristics than native workers in the receiving country. B) Migrating workers often have very similar characteristics as native workers in the receiving country. C) Migrating workers often underperform native workers in the receiving country. D) Migrating workers often outperform native workers in the receiving country. Answer: A Difficulty: Easy AACSB: Application of knowledge 11) The average wage differentials across countries reflect A) different distribution of worker characteristics as well as different types of jobs. B) different labor market structures. C) different culture and history. D) different government's regulations on immigration. Answer: A Difficulty: Easy AACSB: Application of knowledge
26 Copyright © 2022 Pearson Education, Inc.
12) Immigration into the U.S. over the past century has caused the percentage of immigrants in the U.S. population to A) fall steadily over the entire century. B) rise steadily until the 1970s and fall thereafter. C) rise steadily over the entire century. D) fall steadily until the 1970s and increase thereafter. E) remain relatively constant over the time period. Answer: D Difficulty: Easy AACSB: Application of knowledge 13) At the turn of the 20th century, the number of foreign-born U.S. residents increased dramatically due to A) vast immigration from Eastern and Southern Europe. B) vast immigration from Latin America and Asia. C) vast immigration from Australia and New Zealand. D) vast immigration from Africa. Answer: A Difficulty: Easy AACSB: Application of knowledge 14) What was the reason why US immigration decreased sharply in the mid-20th century? A) Tighter restrictions on immigration imposed in the 1920s. B) Fewer restrictions on immigration imposed after the 1920s. C) No more restrictions on immigration were imposed after the 1920s. D) US average wages were lower than the rest of the world. Answer: A Difficulty: Easy AACSB: Application of knowledge 15) A new wave of immigration to the US began around ________ with most immigrants coming from Latin America and Asia. A) 1970 B) 1960 C) 1945 D) 1920 Answer: A Difficulty: Easy AACSB: Application of knowledge
27 Copyright © 2022 Pearson Education, Inc.
16) As of 2019, foreign-born workers make up ________ of the U.S. labor force. A) 17.4 percent B) 12.2 percent C) 50 percent D) 3 percent Answer: A Difficulty: Easy AACSB: Application of knowledge 17) Relative to native-born workers, foreign-born US workers are concentrated A) in both the highest and lowest educational groups. B) in only the lowest educational groups. C) in only the highest educational groups. D) in the medium income group of the US population. Answer: A Difficulty: Easy AACSB: Application of knowledge 18) Among US workers with doctorate degrees in the STEM fields, ________ percent were foreign born in 2013, and so were ________ percent of software engineers in Silicon Valley in the same year. A) 55; 70 B) 24.3; 9.9 C) 20; 30 D) 1; 8 Answer: A Difficulty: Easy AACSB: Application of knowledge 19) In 2019, ________ percent of immigrant labor force had not completed high school or its equivalent, compared with only ________ percent of US native-born workers. A) 24.3; 9.9 B) 55; 70 C) 20; 10 D) 15; 5 Answer: A Difficulty: Easy AACSB: Application of knowledge
28 Copyright © 2022 Pearson Education, Inc.
20) Because workers with different education levels represent different inputs into production and cannot easily be substituted for one another, most estimates suggest that immigration has actually A) raised the wages of the vast majority of native-born Americans. B) lowered the wages of the vast majority of native-born Americans. C) no impact on the wages of the vast majority of native-born Americans. D) converged the wages of the vast majority of native-born Americans. Answer: A Difficulty: Easy AACSB: Application of knowledge 21) America's gross domestic product–the total value of all goods and services produced here–is A) larger because of immigrant workers. B) smaller because of immigrant workers. C) unchanged under the impact of immigrant workers. D) volatile because of immigrant workers. Answer: A Difficulty: Easy AACSB: Application of knowledge 22) The difference between the gain in GDP and the cost in wages paid to immigrants is called A) immigration surplus. B) immigration deficit. C) immigration balance. D) immigration overdue. Answer: A Difficulty: Easy AACSB: Application of knowledge 23) Which of the following statements is TRUE? A) Immigration is an extremely contentious political issue. B) Immigration is never a contentious political issue. C) Immigration is a non-debatable political issue. D) Immigration is a popular political issue that everyone can mutually agree upon. Answer: A Difficulty: Easy AACSB: Application of knowledge 24) Describe the migration flows in the European Union in the past decade. Answer: With the expansion of the European Union (EU) to the east with new members including the Czech Republic, Estonia, Cyprus, Latvia, Lithuania, Hungary, Malta, and Poland in 2004, Bulgaria and Romania in 2007, and Croatia in 2013, there has been large migration flows within this newly expanded union. Given large wage differences in favor of Western Europe, relative to its new EU partners in Eastern Europe, those migration flows have predominantly gone from east to west. Difficulty: Easy AACSB: Application of knowledge 29 Copyright © 2022 Pearson Education, Inc.
25) What does average wage differentials across countries reflect? Answer: They reflect a different distribution of worker characteristics as well as different types of jobs. Migrating workers often have very different characteristics than native workers in the receiving country. Difficulty: Easy AACSB: Application of knowledge 26) How has the wage converged in the EU over the past decades? Answer: In 1997, the average compensation of a manufacturing worker in Eastern Europe is 14 percent of their counterparts in Western Europe, but this number doubles over the ensuing decade, increasing to 27 percent in 2015. Clearly, large compensation differentials still persist, but the trend is toward convergence. A decade earlier, a similar process of wage convergence was observed between East and West Germany following reunification in 1990. That year, the compensation differential (in favor of West Germany) increased from 7 to 37 percent. Five years later, East Germans were earning 72 percent of the compensation of West Germans (though this convergence growth has markedly slowed down since). Difficulty: Easy AACSB: Application of knowledge 27) What are the overall effects of foreign-born immigration workers on America's income? Answer: America's gross domestic product is clearly larger because of immigrant workers. However, much of this increase in the value of production is used to pay wages to the immigrants themselves. Estimates of the "immigration surplus"–the difference between the gain in GDP and the cost in wages paid to immigrants–are generally small, on the order of 0.1 percent of GDP. Difficulty: Easy AACSB: Application of knowledge 28) Describe briefly the shares of immigrants in the US population over the past two centuries. Answer: The share of immigrants in the U.S. population has varied greatly over the past two centuries. At the turn of the 20th century, the number of foreign-born U.S. residents increased dramatically due to vast immigration from Eastern and Southern Europe. Tight restrictions on immigration imposed in the 1920s brought an end to this era, and by the 1960s immigrants were a minor factor on the American scene. A new wave of immigration began around 1970, this time with most immigrants coming from Latin America and Asia. Although the share of immigrants has been steadily increasing ever since, it is still below the levels reached during the first wave of immigration. Difficulty: Easy AACSB: Application of knowledge 29) How has the new wave of immigration after 1970 affected the U.S. economy? Answer: The most direct effect is that immigration has expanded the workforce. As of 2019, foreign-born workers make up 17.4 percent of the U.S. labor force–that is, without immigrants the United States would have 17.4 percent fewer workers. Difficulty: Easy AACSB: Application of knowledge 30 Copyright © 2022 Pearson Education, Inc.
30) Other things equal, one would expect the increase in the US foreign-born workforce to reduce wages. One widely cited estimate is that average wages in the United States are 3 percent lower than they would be in the absence of immigration. Why can this estimate be misleading? Answer: Comparisons of average wages can be misleading because immigrants to the United States have a very different education profile relative to the overall U.S. population. Relative to native-born workers, foreign-born workers in the US are concentrated in both the lowest and highest educational groups, which bring different advantages and disadvantages to the US economy. Difficulty: Moderate AACSB: Application of knowledge 31) What are the economic advantages and disadvantages of immigration to the US economy? Answer: On the one hand, foreign-born workers with professional and doctorate degrees provide the U.S. economy with much needed skills, especially in fields such as science, technology, engineering, and mathematics (STEM). In 2016, then President Barack Obama congratulated the six U.S.-based scientists who had just received the Nobel prize–and noted that all of them were immigrants. On the other hand, foreign-born workers are also concentrated in the lowest educational groups who earn low wages and pay low taxes, which may have a negative effect on average US wages and impose high costs on the government. Difficulty: Easy AACSB: Application of knowledge 32) Why is it complicated to assess the economic effects of immigration? Answer: On one side, immigrants pay taxes, helping cover the cost of government. On the other side, they impose costs on the government because their cars need roads to drive on, their children need schools to study in, and so on. Because many immigrants earn low wages and hence pay low taxes, some estimates suggest that immigrants cost more in additional spending than they pay in. Difficulty: Moderate AACSB: Application of knowledge
31 Copyright © 2022 Pearson Education, Inc.
International Economics: Theory and Policy, 12e (Krugman) Chapter 5 Resources and Trade: The Heckscher-Ohlin Model 5.1
Model of a Two-Factor Economy
1) In the Heckscher-Ohlin model, comparative advantage is influenced by the interaction between nations' resources, called ________, and the technology of production, called ________, used in the production of different goods. A) factor abundance; factor intensity B) factor resources; factor technology C) relative abundance; relative intensity D) interactive resource; interactive technology Answer: A Difficulty: Easy AACSB: Application of knowledge 2) The theory that emphasizes the interplay between the proportions in which different factors of production are available in different countries and the proportions in which they are used in producing different goods, is referred to as the A) factor-proportions theory. B) relative abundance theory. C) resource theory. D) endowment theory. Answer: A Difficulty: Easy AACSB: Application of knowledge 3) The Heckscher-Ohlin theory is also referred to as the A) factor-proportions theory. B) relative abundance theory. C) resource theory. D) endowment theory. Answer: A Difficulty: Easy AACSB: Application of knowledge 4) According to the Heckscher-Ohlin model, the source of comparative advantage is a country's A) factor endowments. B) technology. C) advertising. D) human capital. E) political system. Answer: A Difficulty: Moderate AACSB: Application of knowledge
1 Copyright © 2022 Pearson Education, Inc.
5) In the Heckscher-Ohlin model, countries are assumed to differ only in terms of their A) factor endowments. B) tastes and preferences. C) available technologies. D) factor productivities. E) physical size. Answer: A Difficulty: Moderate AACSB: Application of knowledge 6) In the 2-factor, 2-good Heckscher-Ohlin model, the two countries differ in A) tastes and preferences. B) military capabilities. C) the size of their economies. D) relative abundance of factors of production. E) labor productivities. Answer: D Difficulty: Easy AACSB: Application of knowledge 7) The Heckscher-Ohlin model differs from the Ricardian model of Comparative Advantage in that the former A) has only two countries. B) has only two products. C) has two factors of production. D) has two production possibility frontiers (one for each country). E) has varying wage rates. Answer: C Difficulty: Easy AACSB: Application of knowledge 8) If a country produces good Y (measured on the vertical axis) and good X (measured on the horizontal axis), then the absolute value of the slope of its production possibility frontier is equal to A) the opportunity cost of good X. B) the price of good X divided by the price of good Y. C) the price of good Y divided by the price of good X. D) the opportunity cost of good Y. E) the cost of capital (assuming that good Y is capital intensive) divided by the cost of labor. Answer: A Difficulty: Easy AACSB: Application of knowledge
2 Copyright © 2022 Pearson Education, Inc.
9) In the 2-factor, 2-good Heckscher-Ohlin model, the production possibility frontier is kinked when A) there is no factor substitution in production. B) the opportunity cost of production is constant. C) there are unemployed factor resources. D) a country does not engage in trade. E) transportation costs are very high. Answer: A Difficulty: Moderate AACSB: Application of knowledge 10) In the Heckscher-Ohlin model, if capital can be substituted for labor and vice versa, the production possibility frontier A) no longer has a kink. B) still has a kink. C) is a solid line with a constant slope. D) shifts outward. E) shifts inward. Answer: A Difficulty: Easy AACSB: Application of knowledge 11) Which of the following statements is NOT true? In the Heckscher-Ohlin model, the economy produces at the point A) that minimizes the value of production given the prices it faces. B) that maximizes the value of production given the prices it faces. C) which is on the highest possible isovalue line. D) at which the opportunity cost of cloth in terms of food is equal to the relative price of cloth, PC/PF. Answer: A Difficulty: Easy AACSB: Application of knowledge 12) In each sector, the ratio of labor to capital used in production depends on A) the cost of labor relative to the cost of capital. B) the cost of capital relative to the cost of labor. C) the opportunity cost of labor relative to the opportunity cost of capital. D) the opportunity cost of capital relative to the opportunity cost of labor. Answer: A Difficulty: Easy AACSB: Application of knowledge
3 Copyright © 2022 Pearson Education, Inc.
13) At any given wage-rental ratio, cloth production uses a higher labor-capital ratio than food production; then cloth production is ________ and food production is ________. A) labor-intensive; capital-intensive B) capital-intensive; labor-intensive C) labor-intensive; labor-intensive D) capital-intensive; capital-intensive Answer: A Difficulty: Easy AACSB: Application of knowledge 14) If a good is labor intensive it means that the good is produced A) using relatively more labor than goods that are not labor intensive. B) using labor as the only input. C) using more labor per unit of output than goods that are not labor intensive. D) using labor such that the total cost of labor is greater than the total cost of capital. E) using labor such that the cost of labor is more than 50% of total cost. Answer: A Difficulty: Easy AACSB: Application of knowledge 15) If a good is capital intensive it means that the good is produced A) using capital as the only input. B) using capital such that the cost of capital is more than 50% of total cost. C) using capital such that the total cost of capital is greater than the total cost of labor. D) using relatively more capital than goods that are not labor intensive. E) using more capital per unit of output than goods that are not capital intensive. Answer: D Difficulty: Easy AACSB: Application of knowledge 16) The upward sloping SS curve illustrates there is a one-to-one relationship between A) the factor price ratio and the relative price of the good. B) the cost of labor and the price of the labor-intensive good. C) the cost of land and the price of the land-intensive good. D) the relative price of the good and the relative income of workers. Answer: A Difficulty: Easy AACSB: Application of knowledge
4 Copyright © 2022 Pearson Education, Inc.
17) In the 2-factor, 2-good Heckscher-Ohlin model, the country with a relative abundance of ________ will have a production possibility frontier that is biased toward production of the ________ good. A) labor; labor intensive B) labor; capital intensive C) land; labor intensive D) land; capital intensive E) capital; land intensive Answer: A Difficulty: Easy AACSB: Application of knowledge 18) In the 2-factor, 2-good Heckscher-Ohlin model, the country with a relative abundance of ________ will have a production possibility frontier that is biased toward production of the ________ good. A) capital; capital intensive B) labor; capital intensive C) land; labor intensive D) land; capital intensive E) labor; land intensive Answer: A Difficulty: Easy AACSB: Application of knowledge 19) In the 2-factor, 2-good Heckscher-Ohlin model, an influx of workers from across the border would A) move the point of production along the production possibility curve. B) shift the production possibility curve outward, and increase the production of both goods. C) shift the production possibility curve outward and decrease the production of the laborintensive product. D) shift the production possibility curve outward and much larger in the direction of the laborintensive product than in the direction of the capital-intensive product. E) shift the possibility curve outward and displace preexisting labor. Answer: D Difficulty: Moderate AACSB: Application of knowledge 20) Biased expansion of production possibilities occurs A) when the production possibility frontier shifts outward much more in one direction than in the other. B) when the production possibility frontier shifts outward equally in the production of all goods. C) when the production possibility frontier shifts inward more in one direction than in the other. D) when the production possibility frontier shifts inward equally in the production of all goods. Answer: A Difficulty: Easy AACSB: Application of knowledge 5 Copyright © 2022 Pearson Education, Inc.
21) An increase in the supply of labor expands production possibilities disproportionately in the direction of ________ production, while an increase in the supply of capital expands them disproportionately in the direction of ________ production. A) cloth; food B) food; cloth C) domestic; foreign D) foreign; domestic Answer: A Difficulty: Easy AACSB: Application of knowledge 22) An economy with a high relative supply of labor to capital will be relatively better at A) producing cloth than an economy with a low relative supply of labor to capital. B) producing food than an economy with a low relative supply of labor to capital. C) producing both cloth and food than an economy with a low relative supply of labor to capital. D) producing food than an economy with a high relative supply of capital to labor. Answer: A Difficulty: Easy AACSB: Application of knowledge 23) Which of the following statements is TRUE? A) An economy will tend to be relatively effective at producing goods that are intensive in the factors with which the country is relatively well endowed. B) An economy will tend to be relatively ineffective at producing goods that are intensive in the factors with which the country is relatively well endowed. C) An economy will tend to be relatively effective at producing goods that are intensive in the factors with which the country is relatively not well endowed. D) An economy will tend to be relatively effective at producing goods that are intensive in the factors, no matter whether the country is relatively well endowed and not. Answer: A Difficulty: Easy AACSB: Application of knowledge 24) Why is the H.O. model called the factor-proportions theory? Answer: The H.O. model explores the nature and the limitations of assuming that the sole determinant of comparative advantage is inter-country differences in (relative) factor proportions. Difficulty: Easy AACSB: Application of knowledge
6 Copyright © 2022 Pearson Education, Inc.
25) Differentiate the shapes of production possibility frontier with and without factor substitution. Answer: If capital cannot be substituted for labor or vice versa, the production possibility frontier in the factor-proportions model would be defined by two resource constraints: The economy can't use more than the available supply of labor or capital. So the production possibility frontier is defined by the solid line with a kink (Figure 5.1). If capital can be substituted for labor or vice versa, the production possibility frontier no longer has a kink and has a bowed shape (Figure 5.2). But it remains true that the opportunity cost of cloth in terms of food rises as the economy's production mix shifts toward cloth and away from food. Difficulty: Moderate AACSB: Application of knowledge 26) "A good cannot be both land- and labor-intensive." Discuss. Answer: In a two-good, two-factor model, such as the original Heckscher-Ohlin framework, the factor intensities are relative intensities. Hence, the relevant statistic is either workers per acre (or acres per worker); or wage per rental unit (or rental per wage). In order to illustrate the logic of the statement above, let us assume that the production of a broom requires 4 workers and 1 acre. Also, let us assume that the production of one bushel of wheat requires 40 workers and 80 acres. In this case the acres per person required to produce a broom is one quarter, whereas to produce a bushel of wheat requires 2 acres per person. The wheat is therefore (relatively) land intensive, and the broom is (relatively) labor intensive. Difficulty: Moderate AACSB: Application of knowledge 5.2
Effects of International Trade Between Two-Factor Economies
1) One way in which the Heckscher-Ohlin model differs from the Ricardo model of comparative advantage is by assuming that ________ is (are) identical in all countries. A) factor endowments B) scale of production C) factor intensities D) technology E) opportunity costs Answer: D Difficulty: Easy AACSB: Application of knowledge 2) In the Heckscher-Ohlin model, when two countries begin to trade with each other A) the relative prices of traded goods in the two countries converge. B) relative factor prices in the two countries diverge. C) benefits from trade are evenly distributed between the two countries. D) all factors in both countries will gain from trade. E) all factors in one country will gain, but there may be no gains in the other country. Answer: A Difficulty: Easy AACSB: Application of knowledge 7 Copyright © 2022 Pearson Education, Inc.
3) In the Heckscher-Ohlin model, when there is international-trade equilibrium A) the capital-rich country will charge more for the capital-intensive good than the price paid by the capital-poor country for the capital-intensive good. B) workers in the capital-rich country will earn more than those in the poor country. C) the workers in the capital-rich country will earn less than those in the poor country. D) the relative price of the capital-intensive good in the capital-rich country will be the same as that in the capital-poor country. E) the capital-rich country will charge less for the capital-intensive good than the price paid by the capital-poor country for the capital-intensive good. Answer: D Difficulty: Moderate AACSB: Application of knowledge 4) According to the Heckscher-Ohlin model, A) the country that is abundant in a factor exports the good whose production is intensive in that factor. B) the country that is not abundant in a factor exports the good whose production is intensive in that factor. C) the country that is abundant in a factor imports the good whose production is intensive in that factor. D) the country that is not abundant in a factor imports the good whose production is not intensive in that factor. Answer: A Difficulty: Easy AACSB: Application of knowledge 5) Countries tend to A) export goods whose production is intensive in factors with which the countries are abundantly endowed. B) import goods whose production is intensive in factors with which the countries are abundantly endowed. C) export goods whose production is intensive in factors with which the countries are not abundantly endowed. D) import goods whose production is not intensive in factors with which the countries are not abundantly endowed. Answer: A Difficulty: Easy AACSB: Application of knowledge
8 Copyright © 2022 Pearson Education, Inc.
6) The resource of which a country has a relatively large supply is the ________ in that country, and the resource of which it has a relatively small supply is the ________. A) abundant factor; scarce factor B) scarce factor; abundant factor C) abundant factor; abundant factor D) scarce factor; scarce factor Answer: A Difficulty: Easy AACSB: Application of knowledge 7) In terms of income distribution effects of international trade in the long run, A) owners of a country's abundant factors gain from trade, but owners of a country's scarce factors lose. B) owners of a country's abundant factors lose from trade, but owners of a country's scarce factors gain. C) both owners of a country's abundant factors and owners of a country's scarce factors lose from trade. D) both owners of a country's abundant factors and owners of a country's scarce factors gain from trade. Answer: A Difficulty: Easy AACSB: Application of knowledge 8) In the 2-factor, 2-good Heckscher-Ohlin model, trade will ________ the owners of a country's ________ factor and will ________ the good that uses that factor intensively. A) benefit; abundant; export B) harm; abundant; import C) benefit; scarce; export D) benefit; scarce; import E) harm; scarce; export Answer: A Difficulty: Easy AACSB: Application of knowledge 9) In the 2-factor, 2-good Heckscher-Ohlin model, trade will ________ the owners of a country's ________ factor and will ________ the good that uses that factor intensively. A) harm; scarce; import B) harm; abundant; import C) benefit; scarce; export D) benefit; scarce; import E) harm; scarce; export Answer: A Difficulty: Easy AACSB: Application of knowledge
9 Copyright © 2022 Pearson Education, Inc.
10) Trade benefits a country by A) increasing available consumption choices. B) reducing the relative price of the exported good. C) increasing the wage rate. D) increasing the real income of all resource owners. E) reducing the need for specialization in production. Answer: A Difficulty: Easy AACSB: Application of knowledge 11) According to the Heckscher-Ohlin model A) the gainers from trade could compensate the losers and still retain gains. B) everyone gains from trade. C) the scarce factor gains from trade and the abundant factor loses. D) a country gains from trade if its exports have a high value added. E) only the country with the more advanced technology gains from trade. Answer: A Difficulty: Easy AACSB: Application of knowledge Assume that only two countries, A and B, exist.
12) Refer to the table above. If good S is capital intensive, then following the Heckscher-Ohlin Theory A) country B will export good S. B) country A will export good S. C) both countries will export good S. D) trade will not occur between these two countries. E) both countries will import good S. Answer: A Difficulty: Moderate AACSB: Application of knowledge
10 Copyright © 2022 Pearson Education, Inc.
13) Refer to the table above. If you are told that Country B is very much richer than Country A, then the correct answer is A) country B will export good S. B) country A will export good S. C) both countries will export good S. D) trade will not occur between these two countries. E) both countries will import good S. Answer: A Difficulty: Moderate AACSB: Application of knowledge 14) Refer to the table above. You are told that Country B is very much larger than country A. The correct answer is A) country B will export good S. B) country A will export good S. C) both countries will export good S. D) trade will not occur between these two countries. E) both countries will import good S. Answer: A Difficulty: Moderate AACSB: Application of knowledge 15) Refer to the table above. You are told that Country B has no minimum wage or child labor laws. Now the correct answer is A) country B will export good S. B) country A will export good S. C) both countries will export good S. D) trade will not occur between these two countries. E) both countries will import good S. Answer: A Difficulty: Moderate AACSB: Application of knowledge 16) The Heckscher-Ohlin model predicts all of the following EXCEPT A) that relative wages will tend to become equal in both trading countries. B) which factor of production within each country will gain from trade. C) which country will export which product. D) that trade increases a country's overall welfare. E) the volume of trade. Answer: E Difficulty: Moderate AACSB: Application of knowledge
11 Copyright © 2022 Pearson Education, Inc.
17) If Australia has relatively more land per worker, and Belgium has relatively more capital per worker, then if trade began between these two countries A) the relative price of the capital-intensive product would decrease in Belgium. B) relative product prices would diverge between Australia and Belgium. C) the relative price of the land-intensive product would increase in Australia. D) the relative price of the land-intensive product would increase in Belgium. E) the relative price of the capital-intensive product would increase in Australia. Answer: C Difficulty: Moderate AACSB: Application of knowledge 18) If Australia has more land per worker, and Belgium has more capital per worker,then if trade began between these two countries A) the real income of labor in Australia would decline. B) the real income of labor in Belgium would decline. C) the real income of landowners in Belgium would decline. D) the real income of capital owners in Australia would increase. E) the real income of labor in both countries would decline. Answer: C Difficulty: Moderate AACSB: Application of knowledge 19) If Japan is relatively capital rich and the United States is relatively land rich, and if food is relatively land intensive then trade between these two, formerly autarkic countries will result in A) a global increase in the relative price of food. B) an increase in the relative price of food in the U.S. C) a decrease in the relative price of food in both countries. D) an increase in the relative price of food in Japan. E) an increase in the relative price of food in both countries. Answer: B Difficulty: Moderate AACSB: Application of knowledge 20) If Gambinia has many workers but very little land and even less productive capital, then, following the Heckscher-Ohlin model, we predict that Gambinia will export A) capital-intensive goods. B) labor-intensive goods. C) both labor- and land-intensive goods. D) both capital- and land-intensive goods. E) land-intensive goods. Answer: B Difficulty: Moderate AACSB: Application of knowledge
12 Copyright © 2022 Pearson Education, Inc.
21) If Gambinia has many workers but very little land and even less productive capital, then, following the Heckscher-Ohlin model, in order to improve the country's economic welfare, the Gambinian government should A) discontinue all international trade. B) protect the labor-intensive product. C) engage in free trade. D) protect the capital-intensive product. E) protect the land-intensive product. Answer: C Difficulty: Easy AACSB: Application of knowledge 22) Starting from an autarky (no-trade) situation with Heckscher-Ohlin model, if Country H is relatively labor abundant, then once trade begins A) wages should fall and rents should rise in H. B) wages and rents should fall in H. C) wages should rise and rents should fall in H. D) rent will be unchanged but wages will rise in H. E) wages and rents should rise in H. Answer: C Difficulty: Moderate AACSB: Application of knowledge 23) Suppose that there are two factors, capital and land, and that the United States is relatively land endowed while the European Union is relatively capital-endowed. According to the Heckscher-Ohlin model A) European landowners should support U.S.-European free trade. B) the U.S. should compensate European countries once trade commences. C) European capitalists should support U.S.-European free trade. D) all landowners should support free trade. E) all capitalists in both countries should support free trade. Answer: C Difficulty: Easy AACSB: Application of knowledge 24) International trade has strong effects on income distributions. Therefore, international trade A) will be beneficial to all those engaged in international trade. B) will tend to hurt one trading country. C) will tend to hurt everyone in both countries. D) will tend to hurt some groups in each trading country. E) is beneficial to everyone in both trading countries. Answer: D Difficulty: Easy AACSB: Application of knowledge
13 Copyright © 2022 Pearson Education, Inc.
25) Factors tend to be specific to certain uses and products A) in capital-intensive industries. B) in labor-intensive industries. C) in the short run. D) in countries lacking fair labor laws. E) in countries lacking comparative advantage. Answer: C Difficulty: Moderate AACSB: Application of knowledge 26) If the price of the capital-intensive product rises more than does the price of the landintensive product, then A) demand will shift away from the capital-intensive product, and its production will decrease. B) the relative price of the capital-intensive product will fall to some point between the pretrade relative prices. C) the country that exports the capital-intensive good will lose its comparative advantage. D) demand will shift away from the capital-intensive product, and its production will decrease relative to that of the land-intensive product. E) the production of the capital-intensive product will decrease, but by less than production of the land-intensive product. Answer: B Difficulty: Easy AACSB: Application of knowledge 27) If trade opens up between the two formerly autarkic countries, Australia and Belgium, then A) the real income of both countries will increase. B) the real income of neither country will increase. C) the real income of Australia but not of Belgium will increase. D) the real income of Australia and of Belgium will increase. E) the real income of both countries may increase. Answer: E Difficulty: Moderate AACSB: Application of knowledge 28) Since the 1970s, the distribution of wages in the United States has become A) considerably more unequal. B) considerably more equal. C) unchanged. D) unpredictable. Answer: A Difficulty: Easy AACSB: Application of knowledge
14 Copyright © 2022 Pearson Education, Inc.
29) Until the 1970s, trade between advanced industrial nations and less-developed economies was often referred to as A) North-South trade. B) Center-Periphery trade. C) West-East trade. D) Autarky trade. Answer: A Difficulty: Easy AACSB: Application of knowledge 30) What is responsible for the growing gap between skilled and unskilled workers in the United States? A) Many economists believe it is the skill-biased technological change. B) Many economists believe it is the growing trade with low-wage countries. C) Many economists believe it is the growing exports of manufactured goods from newly industrializing economies (NIEs) such as Mexico and China. D) Many economists believe it is the steady rise in education premium. Answer: A Difficulty: Moderate AACSB: Application of knowledge 31) In the U.S. economy over the last half-century, A) skilled-based technological change has been associated with a higher skilled-unskilled wage ratio, leading to increasing skilled-unskilled employment ratios in all sectors. B) skilled-based technological change has been associated with a higher skilled-unskilled wage ratio, leading to decreasing skilled-unskilled employment ratios in all sectors. C) trade-based technological change has been associated with a higher relative price of high-tech goods, leading to increasing skilled-unskilled employment ratios in all sectors. D) trade-based technological change has been associated with a lower relative cost of skilled workers, leading to increasing skilled-unskilled employment ratios in all sectors. Answer: A Difficulty: Easy AACSB: Application of knowledge 32) The outsourcing process from the United States to Mexico can explain A) the increase in the wage premium between non-production and production. B) the decrease in the wage premium between non-production and production. C) the unchanged level in the wage premium between non-production and production. D) none of the change in the wage premium between non-production and production. Answer: A Difficulty: Easy AACSB: Application of knowledge
15 Copyright © 2022 Pearson Education, Inc.
33) Capital-skill complementarity refers to a model where A) capital is a much closer substitute for unskilled labor than for skilled labor. B) capital is a much closer substitute for skilled labor than for unskilled labor. C) capital is not a substitute factor for both unskilled and skilled labor. D) capital is not a compliment factor for both unskilled and skilled labor. Answer: A Difficulty: Easy AACSB: Analytical thinking 34) Equalization of factor prices model assumes that A) when the relative prices of goods converge, the relative prices of factors of production will be equalized across countries as a result of international trade. B) when the relative prices of goods converge, the relative prices of factors of production will not be equalized across countries as a result of international trade. C) when the relative prices of goods do not converge, the relative prices of factors of production will still be equalized across countries as a result of international trade. D) when the relative prices of goods do not converge, they may or may not have effect on the relative prices of factors of production as a result of international trade. Answer: A Difficulty: Easy AACSB: Application of knowledge 35) "No country is abundant in everything." Discuss. Answer: The concept of factor abundance is (like factor intensities) a relative concept. When we identify a country as being capital abundant, we mean that it has more capital per worker than does the other country. If one country has more capital worker than another, it is an arithmetic impossibility that it also has more workers per unit of capital. Difficulty: Moderate AACSB: Application of knowledge 36) International trade leads to complete equalization of factor prices. Discuss. Answer: This statement is typically "true . . . but." Under a strict and limited set of assumptions, such as the original Heckscher-Ohlin model which excludes country specific technologies; nonhomothetic tastes; factor intensity reversals; large country differences in (relative) factor abundances, more factors than goods, and an equilibrium solution within the "cone of specialization"; then it may be demonstrated that internal consistency demands that the above stated sentence is "true." However, the minute one relaxes any of the above listed assumptions one may easily identify solutions, which contradict the factor price equalization theorem. Difficulty: Moderate AACSB: Analytical thinking
16 Copyright © 2022 Pearson Education, Inc.
37) Refer to above figure. Two countries exist in this model, P and R. P is relatively labor (L) abundant, as is evident in the bottom right horizontal axis. If Country P were to be completely specialized in the labor-intensive product, C, it would be producing at point 4. In fact, it produces both C and P, at point 5. The (autarky) relative price of C (in terms of F) of Country P is at point 3; and of Country R at point 1. If trade were to open up between these two countries, which would export C and which would export F? Is this consistent with the Heckscher-Ohlin model? Explain. Answer: Country R would export F. This is consistent with the H-O model. The country which is relatively capital abundant exports the product which is relatively capital intensive. Difficulty: Difficult AACSB: Application of knowledge 38) Refer to above figure. If trade were to open up between P and R, where would the terms of trade locate in the figure above (somewhere on the PC/PF axis)? Would relative wages (w/r) in the two countries become equal? Is this consistent with the Heckscher-Ohlin model? Explain. Answer: The terms of trade would settle somewhere between the two autarky relative prices on the PC/PF axis. The relative wages (w/r) will be lower than the highest and higher than the lowest on the vertical axis above, but will not coincide. This last result is in contradiction to the factor price equalization expectation we have from the model. Difficulty: Difficult AACSB: Application of knowledge
17 Copyright © 2022 Pearson Education, Inc.
39) Refer to above figure. Would you expect to find that the real wages become equalized in both countries? Explain the reason for any differences you note. Answer: We would expect that one single relative wage will be established for both countries. This happens because the two countries do not differ in relative factor availability by much, and hence a zone of overlap exists which allows for this result. Difficulty: Difficult AACSB: Application of knowledge 40) Refer to above figure. In autarky, Country P was producing at point 5. With trade, would its production point be found above or below point 5? Explain why. What must happen in the K/L intensity ratio in the production of each of the products in this country when moving from autarky to free trade? Answer: The point of production with trade will be above point 5. The country will be shifting its production composition to be more heavily weighted in labor intensive good, C. Within each industry, the production technique will be more capital intensive, since with the rising relative wage, the optimal point of production will involve sliding around the isoquants in the direction of saving on the now relatively more expensive labor. Difficulty: Difficult AACSB: Application of knowledge 41) One of the commonly used assumptions in deriving the Heckscher-Ohlin model is that tastes are homothetic, or that if the per capita incomes were the same in two countries, the proportions of their expenditures allocated to each product would be the same as it is in the other country. Imagine that this assumption is false, and that in fact, the tastes in each country are strongly biased in favor of the product in which it has a comparative advantage. How would this affect the relationship between relative factor abundance between the two countries, and the nature (factorintensity) of the product each exports? What if the taste bias favored the imported good? Answer: If in fact national tastes were strongly biased in favor of the product in which the country enjoyed a comparative advantage, then we would expect a bias in favor of rejecting the Heckscher-Ohlin Theorem in actual trade data. The engine driving the H-O model is that a country should be expected to have a relatively low cost of producing the good in which it has a comparative advantage. However, the respective demand forces would tend to raise the price of this good, so that the expected pattern would not generally be observed. However, if the tastes were biased in favor of the imported good, then the predictions of the Heckscher-Ohlin Theorem would be expected to be generally observed. Difficulty: Difficult AACSB: Application of knowledge
18 Copyright © 2022 Pearson Education, Inc.
Suppose Australia, a land (K)-abundant country, and Sri Lanka, a labor(L)-abundant country, both produce labor and land intensive goods with the same technology.
42) Use the diagram above to identify the pre-trade situation for Australia and Sri Lanka. Where on the K/L axis will you find each of the two countries? Which of the two countries has a higher relative wage, w/r? Which product is the labor intensive, and which is the land intensive one? Show where the relative price of cloth to food will be found once trade opens between these two countries. Show where the relative wages of each will appear. Answer: You will find Sri Lanka to the left of Australia on the K/L axis. Australia has a higher relative wage. Food is the land intensive product. The relative price PC/PF is found between the two autarkic prices. The post-trade relative wage is between the two autarkic ones on the vertical axis. Difficulty: Difficult AACSB: Application of knowledge
19 Copyright © 2022 Pearson Education, Inc.
43) Using the figure above, demonstrate what happens to the composition of production (that is quantity of cloth per 1 unit of food) in Australia once trade is established between the two countries. Which country will export cloth? What happens to the relative income of workers in Australia as a result of trade? Does it increase or decrease? Would land owners in Australia lobby for or against free trade? Would land owners in Australia lobby for or against free admittance of immigrant workers? Answer: The proportion of food to cloth will increase in the production of Australia. Sri Lanka will export cloth. The relative (and real) incomes of workers will fall in Australia as a result of trade. Land owners in Australia should lobby in favor of trade. They would also lobby for free labor mobility (of workers into Australia), since the marginal product of labor is high, the owners of land have much (Ricardian) rents to gain from an inflow of workers. Difficulty: Difficult AACSB: Application of knowledge 44) Refer to above figure. Imagine that the relative capital abundance of Australia was so much greater than that of Sri Lanka, that we would have to locate Australia far to the right on the K/L axis. If this were so far to the right that there was no area of overlap on the w/r axis, then what product would Australia export? Which product will each of the trade partners export? Will the relative wages as calculated now be the same or different in both Australia and Sri Lanka? Answer: Australia would still export food. As a result of trade, wages will fall in Australia and will rise in Sri Lanka. However, in this case, the wages in Australia will remain higher than in Sri Lanka, creating an incentive for migration from the latter to the former country. Difficulty: Difficult AACSB: Application of knowledge 45) Why has wage inequality increased in the United States since the 1970s? Answer: While some economists believe that growing trade with low-wage countries has been the main cause of rising income inequality in the United States, many believe that it is the new production technologies that put a greater emphasis on worker skills (such as the widespread introduction of computers and other advanced technologies in the workplace), often referred to as a technology-skill complementarity or skill-biased technological change. Difficulty: Moderate AACSB: Application of knowledge 46) Why do most empirical researchers believe that growing trade with low-wage countries is not the main cause of rising income inequality in the United States? Answer: Their skepticism rests on three main observations. (1) There is no clear evidence of change in relative prices of goods (a rise in the prices of skill-intensive products compared to those of unskilled-labor-intensive goods) as the factor-proportions model predicts. (2) Except for some particular cases of Mexico and China, there is still a lack of evidence of relative factor price convergence as the model predicts that wages of skilled workers are rising and those of unskilled workers are falling in the skill-abundant country, and the reverse should be happening in the labor-abundant country. (3) Trade flows couldn't have had a very large impact on income distribution as trade between advanced countries and NIEs still constitutes only a small percentage of total spending in the advanced nations. Difficulty: Moderate AACSB: Application of knowledge 20 Copyright © 2022 Pearson Education, Inc.
5.3
Empirical Evidence on the Heckscher-Ohlin Model
1) If two countries are very different in relative factor abundance, then empirical support for which of the following would be less likely? A) the Factor Price Equalization Theorem B) the Heckscher-Ohlin Theorem C) the Law of One Price D) the Law of Demand E) the Gravity Theorem Answer: A Difficulty: Easy AACSB: Application of knowledge 2) Which of the following empirical studies cast the most doubt on the Heckscher-Ohlin model? A) the study by Wassily Leontief B) the study by Bowen, Leamer, and Sveikauskas C) the study by David Ricardo D) the study by Adam Smith E) the study by Davis and Weinstein Answer: A Difficulty: Easy AACSB: Application of knowledge 3) The Leontief Paradox A) proved that the U.S. economy is different from all others. B) failed to support the validity of the Ricardian theory. C) supported the validity of the Ricardian theory of comparative advantage. D) supported the validity of the Heckscher-Ohlin model. E) failed to support the validity of the Heckscher-Ohlin model. Answer: E Difficulty: Easy AACSB: Application of knowledge 4) The Leontief Paradox A) still accurately applies to today's pattern of U.S. international trade. B) refers to the fact that Leontief—an American economist—had a Russian name. C) refers to the finding that the U.S. produces outside its Edgeworth Box. D) refers to the finding that U.S. exports were more labor intensive than its imports. E) refers to the finding that U.S. Exports were more capital intensive than its exports. Answer: D Difficulty: Easy AACSB: Application of knowledge
21 Copyright © 2022 Pearson Education, Inc.
5) The 1987 study by Bowen, Leamer and Sveikauskas A) proved that the U.S.'s comparative advantage relied on skilled labor. B) supported the validity of the Leontief Paradox. C) supported the validity of the Heckscher-Ohlin model. D) demonstrated that in fact countries tend to use different technologies. E) used a two-country and two-product framework. Answer: B Difficulty: Easy AACSB: Application of knowledge 6) Empirical observations on actual North-South trade patterns tend to A) support the validity of the Rybczynski Theorem. B) support the validity of the wage equalization theorem. C) support the validity of the neo-imperialism exploitation theory. D) support the validity of the Leontief Paradox. E) support the validity of the Heckscher-Ohlin model. Answer: E Difficulty: Moderate AACSB: Application of knowledge 7) The Case of the Missing Trade refers to A) the fact that world exports does not equal world imports. B) the fact that factor trade is less than predicted by the Heckscher-Ohlin theory. C) the fact that the Heckscher Ohlin theory never applies to China-U.S. trade practices. D) the fact that the Heckscher Ohlin theory predicts much less volume of trade than actually exists. Answer: B Difficulty: Moderate AACSB: Application of knowledge 8) Empirical support for the Heckscher-Ohlin model was weakest when the study applied A) all of the assumptions of the model. B) all of the assumptions of the model except that regarding technology. C) all of the assumptions of the model except those regarding technology, goods and shipping costs. D) all of the assumptions of the model except those regarding technology, shipping costs and gravity. E) all of the assumptions of the model except those regarding shipping costs. Answer: C Difficulty: Easy AACSB: Application of knowledge
22 Copyright © 2022 Pearson Education, Inc.
9) Which of the following empirical studies provided the most support for the Heckscher-Ohlin model? A) the study by Wassily Leontief B) the study by Bowen, Leamer, and Sveikauskas C) the study by David Ricardo D) the study by Adam Smith E) the study by Davis and Weinstein Answer: E Difficulty: Easy AACSB: Application of knowledge 10) John Romalis showed how the Heckscher-Ohlin model's prediction for the pattern of exports A) can be extended to multiple countries producing multiple goods. B) can only be applied to a 2 goods, 2 countries, 2 factors scenario. C) cannot be extended to multiple countries producing multiple goods. D) cannot be proved in the real world. Answer: A Difficulty: Easy AACSB: Application of knowledge 11) John Romalis showed that exports of the ________ to the United States are overwhelmingly concentrated in sectors with the ________. A) developing countries; lowest skill-intensity B) developed countries; lowest skill-intensity C) developing countries; highest skill-intensity D) developed countries; labor-intensity Answer: A Difficulty: Easy AACSB: Application of knowledge 12) As predicted by the Chinese change in factor proportions, the concentration of exports A) in high-skill sectors steadily increases over time. B) in low-skill sectors steadily increases over time. C) in high-skill sectors steadily decreases over time. D) in low-skill sectors still dominate over time. Answer: A Difficulty: Easy AACSB: Application of knowledge
23 Copyright © 2022 Pearson Education, Inc.
13) Which of the following statements is NOT true? A) Factor-price equalization are observed across countries. B) Factor-price equalization are not observed across countries. C) Given all the assumptions behind factor-price equalization, a country's factor content of trade bears little resemblance to the theoretical predictions based on that country's factor abundance. D) A less restrictive version of the factor proportions model fits the predicted patterns for the factor content of trade. E) The pattern of goods trade between developed and developing countries fits the predictions of the model quite well. Answer: A Difficulty: Easy AACSB: Application of knowledge 14) Countries do not in fact export the goods the H.O. theory predicts. Discuss. Answer: This statement is not true. Although one may find many cases where it seems to be true (e.g., the Leontief Paradox), all one needs to do in order to render the above statement not (generally) true is to find one counter example. In fact, one can find large subsets of agricultural and commodity products in which the H.O predictions are generally fulfilled. Labor-intensive countries such as Bangladesh do in fact export relatively labor-intensive goods. Capital-intensive countries such as Germany do in fact export capital-intensive products (at least to South countries). Countries such as Costa Rica ("sunshine abundant") tend to export bananas (sunshine-intensive products). The U.S. (a wheat-land-abundant country) does indeed export wheat (a wheat-land intensive product). In fact, since the early 1980s, the Leontief Paradox was not found to describe the U.S. trade data (hence ratifying the H.O. theory). Difficulty: Moderate AACSB: Application of knowledge 15) Why do we observe the Leontief paradox? Answer: There are many possible answers. They may be classified into three groups. One would argue that the model, or theory is wrong. The second would argue that the theory is correct (internally consistent and descriptive of real world data), but the real world data is incorrectly perceived, defined or measured. The third would argue that the statement itself is wrong, and that in fact the Leontief paradox itself is not actually observed, but rather is due to faulty logical rendering of the original model. Empirical studies conducted since Leontief's work was published suggest that, by relaxing the model's restrictive assumptions regarding technology, goods, and trade costs, evidence in support of the HO model was strengthened and evidence of the Leontief effect was diminished. Difficulty: Moderate AACSB: Application of knowledge
24 Copyright © 2022 Pearson Education, Inc.
16) Why are prices of factors of production NOT equalized? Answer: Again this statement may or may not be argued to be true. On the one hand, the large volume and growth in world trade between the United States and other OECD countries during the 50 years since World War II has clearly been related to a near universal (average) convergence in real wage levels in these countries, whereas the most obvious cases in which such a convergence did not take place (North-South countries) also happened to be cases in which trade was relatively small and "missing." There are many theoretical reasons why factor price equalization may not occur. If the relative country relative abundances are very different, then the theory itself does not predict that the wage equalization will occur. The same is true if factor intensity reversals exist within relevant relative wage ranges. Dynamic migration models such as Harris-Todaro are another class of theory that may suggest that even if the static equilibrium solution does include the factor-price equalization, the dynamic path of the model may never reach this solution, so that when observed within any finite time frame, a lack of equalization would exist. Difficulty: Difficult AACSB: Application of knowledge 17) The Heckscher-Ohlin model is famous for being elegant and mathematically sophisticated, yet failing to describe reality. One manifestation of this fact is Trefler's Case of Missing Trade. Explain what exactly is missing. In what sense is it missing? How would you explain why it is missing? How can a relaxation of the identical production functions explain the case of the missing trade? How did the results obtained by Davis and Weinstein strengthen support for the validity of the HO model? Answer: Trefler demonstrated that the actual volume of world trade is significantly less than that which would be predicted by the Heckscher-Ohlin model. One explanation is that NorthSouth trade is especially less than would be predicted by a factor proportions model. If technologies differ in the poorer countries, then it is possible that the cost of producing a product, which uses relatively much of their abundant factor may still be higher than the cost of producing it in the other country. Davis and Weinstein showed that relaxing the restrictive assumptions regarding technology, goods, and trade costs, yielded empirical results that support the model. Difficulty: Difficult AACSB: Application of knowledge 18) Factor-intensity reversals describe a situation in which the production of a product may be land intensive in one country, and relatively labor intensive in another (at given relative wage levels). For example, cotton may be land intensive in the U.S., and labor intensive in Egypt where land is relatively scarce and expensive. Suppose factor-intensity reversals were common. How would that affect the conclusion that a country in which land is relatively scarce will not be the country with a comparative advantage in the land-intensive product? Answer: The answer here is straightforward (though it has various interesting implications). In this case we cannot define or identify a product in terms of its relative factor intensity (at all or any relative wage level). Therefore, the Heckscher-Ohlin Theorem is ipso-facto inapplicable. Difficulty: Difficult AACSB: Application of knowledge
25 Copyright © 2022 Pearson Education, Inc.
19) Why is it that North-South trade in manufactures seems to be consistent with the results or expectations generated by the factor-proportions theory of international trade, whereas NorthNorth trade is not? Answer: There is a clear difference in relative factor availabilities between North and South countries, no matter how we define and measure the factors of production. Hence, the factorproportions theory of trade may be sensibly expected to explain the pattern (though not the volume) of trade between these two groups of countries. However, the North-North trade partners do not vary significantly in their relative factor availabilities, so that other forces, such as scale economies play a relatively large role in determining trade patterns. Difficulty: Difficult AACSB: Application of knowledge 20) Why do you suppose that South-South trade does NOT conform in volume, but does conform in pattern with expectations generated by the Heckscher-Ohlin model? Answer: The pattern of trade is generally observed to conform to the Heckscher-Ohlin models expectations. That is, the developing countries tend to export labor-intensive goods, such as textiles, and import capital-intensive goods such as machinery. The volume however is quite lower than what would be expected from the Neoclassical model. There are many possible reasons, such as financial crises necessitating premia in the financing of this trade. Difficulty: Difficult AACSB: Application of knowledge
26 Copyright © 2022 Pearson Education, Inc.
5.4
Appendix to Chapter 5: Factor Prices, Goods Prices, and Production Decisions
1) Which of the following is an assertion of the Heckscher-Ohlin model? A) An increase in a country's labor supply will increase production of both the capital-intensive and the labor-intensive good. B) Factor endowments determine the technology that is available to a country, which determines the good in which the country will have a comparative advantage. C) In the long run, labor is mobile and capital is not. D) Factor price equalization will occur only if there is costless mobility of all factors across borders. E) The wage-rental ratio is determined by relative product prices. Answer: E Difficulty: Moderate AACSB: Application of knowledge 2) Which of the following is an assertion of the Heckscher-Ohlin model? A) Factor endowments determine the technology that is available to a country, which determines the good in which the country will have a comparative advantage. B) An increase in a country's labor supply will increase production of both the capital-intensive and the labor-intensive good. C) Factor price equalization will occur only if there is costless mobility of all factors across borders. D) The wage-rental ratio determines the capital-labor ratio in a country's industries. E) In the long run, labor is mobile and capital is not. Answer: D Difficulty: Moderate AACSB: Application of knowledge 3) Which of the following is an assertion of the Heckscher-Ohlin model? A) In the long run, labor is mobile and capital is not. B) Factor endowments determine the technology that is available to a country, which determines the good in which the country will have a comparative advantage. C) An increase in a country's labor supply will increase production of both the capital-intensive and the labor-intensive good. D) Factor price equalization will occur only if there is costless mobility of all factors across borders. E) An increase in a country's labor supply will increase production of the labor-intensive good and decrease production of the capital-intensive good. Answer: E Difficulty: Moderate AACSB: Application of knowledge
27 Copyright © 2022 Pearson Education, Inc.
4) To minimize costs, a producer must get to A) the lowest possible isocost line, which means the lowest possible cost given the technological trade-off outline by the unit isoquant curve. B) the highest possible isocost line, which means the highest possible cost given the technological trade-off outline by the unit isoquant curve. C) the lowest possible isocost line, which means the highest possible cost given the technological trade-off outline by the unit isoquant curve. D) the highest possible isocost line, which means the lowest possible cost given the technological trade-off outline by the unit isoquant curve. Answer: A Difficulty: Easy AACSB: Application of knowledge 5) Assume cloth production is labor intensive, which of the following statements is TRUE? A) A higher relative price of cloth implies a higher wage-rental ratio. B) A higher wage-rental ratio leads to the choice of a higher labor-capital ratio. C) A higher relative price of cloth implies a lower wage-rental ratio. D) A lower wage-rental ratio leads to the choice of a lower labor-capital ratio. Answer: A Difficulty: Easy AACSB: Analytical thinking 6) As the economy's labor supply increases, the economy concentrates A) more and more of both factors in the labor-intensive cloth sector. B) more and more of both factors in the capital-intensive food sector. C) less and less of both factors in the labor-intensive cloth sector. D) more labor in the labor-intensive cloth sector and more capital in the capital-intensive food sector. Answer: A Difficulty: Easy AACSB: Application of knowledge 7) List three assertions regarding factor prices, good prices, and production decisions in the Heckscher-Ohlin model. Answer: First, the ratio of labor to capital employed in each industry depends on the wagerental ratio w/r. Second, there is a one-to-one relationship between relative goods prices PC/PF and the wage-rental ratio. Third, an increase in a country's labor supply (at a given relative goods price PC/PF) will lead to movements of both labor and capital from the food sector to the cloth sector (the labor-intensive sector). Difficulty: Easy AACSB: Application of knowledge
28 Copyright © 2022 Pearson Education, Inc.
International Economics: Theory and Policy, 12e (Krugman) Chapter 6 The Standard Trade Model 6.1
A Standard Model of a Trading Economy
1) The standard trade model is NOT built on A) the relationship between geography and the volume of trade. B) the relationship between the production possibility frontier and the relative supply curve. C) the relationship between relative prices and relative demand. D) the determination of world equilibrium by world relative supply and world relative demand. E) the effect of the terms of trade on a nation's welfare. Answer: A Difficulty: Easy AACSB: Application of knowledge 2) The meaning of "terms of trade" is A) the amount of exports sold by a country. B) the tariffs in place between two trading countries. C) the quantities of imports received in free trade. D) the price of a country's exports divided by the price of its imports. E) the price conditions bargained for in international markets. Answer: D Difficulty: Easy AACSB: Application of knowledge 3) Terms of trade refers to A) what goods are exported. B) the volume of trade. C) the relative price at which trade occurs. D) the tariffs applied to trade. E) what goods are imported. Answer: C Difficulty: Easy AACSB: Application of knowledge 4) Isovalue lines are A) lines along which the value of output is constant. B) lines along which the production cost is constant. C) lines along which the utility is constant. D) lines along which the value of input is constant. Answer: A Difficulty: Easy AACSB: Application of knowledge
1 Copyright © 2022 Pearson Education, Inc.
5) The economy produces at a point A) where the isovalue line is tangent to the production possibility frontier. B) which is on the lowest possible the isovalue line. C) where the production possibility frontier intersects with one of the axes. D) which is on the highest isovalue line outside the production possibility frontier. Answer: A Difficulty: Easy AACSB: Application of knowledge 6) A rise in the relative price of cloth leads the economy to produce ________ cloth and ________ food, therefore ________ the relative supply of cloth. A) more; less; increase B) more; less; decrease C) less; more; increase D) less; more; decrease Answer: A Difficulty: Easy AACSB: Application of knowledge 7) Which of the following is NOT a property of the indifference curve? A) They are upward sloping. B) They are downward sloping. C) The farther up and to the right an indifference curve lies, the higher the level of welfare to which it corresponds. D) They are bowed-out to the origin, i.e. each indifference curve gets flatter as we move to the right. Answer: A Difficulty: Easy AACSB: Application of knowledge 8) If points A and B are two locations on a country's production possibility frontier, then A) at any point in time, the country could produce both. B) both bundles must have the same relative cost. C) consumers are indifferent between the two bundles. D) the country could produce either of the two bundles. E) producers are indifferent between the two bundles. Answer: D Difficulty: Easy AACSB: Application of knowledge
2 Copyright © 2022 Pearson Education, Inc.
9) If the economy is producing at point a on its production possibility frontier, then A) all of its capital is used, but not efficiently. B) all of the country's capital is used for one product. C) all of the country's workers are employed. D) all of the country's workers are specialized in one product. E) all of the country's exports are produced in equal amounts. Answer: C Difficulty: Easy AACSB: Application of knowledge
10) Refer to the figure above, which shows a country's possible production possibility frontiers and indifference curves. If the country is producing at ________, then moving to ________ will cause utility to ________. A) point c; point b; decrease B) point b; point c; remain unchanged C) point a; point b; increase D) point a; point c; remain unchanged E) point c; point b; increase Answer: B Difficulty: Easy AACSB: Application of knowledge 11) Refer to the figure above, which shows a country's possible production possibility frontiers and indifference curves. If the country is producing at ________, then moving to ________ will cause utility to ________. A) point c; point b; remain unchanged B) point a; point b; increase C) point c; point b; increase D) point c; point b; decrease E) point a; point c; remain unchanged Answer: A Difficulty: Easy AACSB: Application of knowledge 3 Copyright © 2022 Pearson Education, Inc.
12) Refer to the figure above, which shows a country's possible production possibility frontiers and indifference curves. If the country is producing at ________, then moving to ________ will cause utility to ________. A) point b; point a; increase B) point a; point b; increase C) point c; point b; increase D) point c; point b; decrease E) point a; point c; remain unchanged Answer: A Difficulty: Easy AACSB: Application of knowledge 13) A country cannot produce a mix of products with a higher value than where A) the isovalue line is above the production possibility frontier. B) the isovalue line is below the production possibility frontier. C) the isovalue line is tangent with the indifference curve. D) the isovalue line is tangent to the production possibility frontier. E) the isovalue line intersects the production possibility frontier. Answer: D Difficulty: Easy AACSB: Application of knowledge 14) Tastes of individuals are represented by A) isovalue lines. B) production possibility frontiers. C) production functions. D) indifference curves. E) the terms of trade. Answer: D Difficulty: Easy AACSB: Application of knowledge 15) If the ratio of price of cloth (PC) divided by the price of food (PF) increases in the international marketplace, then A) the terms of trade of food exporters will improve. B) the terms of trade of all countries will improve. C) the terms of trade of cloth exporters will worsen. D) all countries would be better off. E) the terms of trade of cloth exporters will improve. Answer: E Difficulty: Easy AACSB: Application of knowledge
4 Copyright © 2022 Pearson Education, Inc.
16) If the ratio of price of cloth (PC) divided by the price of food (PF) increases in the international marketplace, then A) the country would import more cloth. B) the food exporter will increase the quantity of food exported. C) the cloth exporter will increase the quantity of cloth produced. D) the cloth exporter will increase the quantity of cloth exported. E) the cloth exporter will decrease the quantity of cloth exported. Answer: C Difficulty: Easy AACSB: Application of knowledge 17) If the ratio of price of cloth (PC) divided by the price of food (PF) increases in the international marketplace, then A) world relative quantity of cloth demanded will decrease. B) world relative quantity of cloth supplied will increase. C) world relative quantity of cloth supplied and demanded will decrease. D) world relative quantity of cloth supplied and demanded will increase. E) world relative quantity of food will increase. Answer: B Difficulty: Easy AACSB: Application of knowledge 18) A country will be able to consume a combination of goods that is NOT attainable solely from domestic production if A) the country avoids international trade. B) the country specializes in one product. C) the world terms of trade equal the domestic relative costs. D) the world terms of trade differ from its domestic relative costs. E) the country's domestic production value equals world relative value. Answer: D Difficulty: Moderate AACSB: Application of knowledge 19) Increased trade with China between 2000 and 2007 induced large reductions in U.S. import prices, A) generating substantial increases in the purchasing power of U.S. households. B) generating substantial decreases in the purchasing power of U.S. households. C) creating a loss of $1,500 on average in purchasing power per U.S. household per year. D) creating neither gains nor losses for U.S. consumers. Answer: A Difficulty: Easy AACSB: Application of knowledge
5 Copyright © 2022 Pearson Education, Inc.
20) A ________ in the terms of trade ________ a country's welfare, while a ________ in the terms of trade ________ its welfare. A) rise; increases; decline; reduces B) rise; reduces; decline; increases C) decline; increases; rise; reduces D) rise; increases; decline; increases Answer: A Difficulty: Easy AACSB: Application of knowledge 21) The equilibrium relative price (PC/PF) is determined by A) the intersection of the world relative supply and demand curves. B) the intersection of the world supply and demand curves. C) the intersection of the Home supply and demand curves. D) the intersection of the Foreign supply and demand curves. Answer: A Difficulty: Easy AACSB: Application of knowledge 22) Which of the following statements is NOT true? A) economic growth in one country has no impact on other countries during open trade. B) economic growth in the rest of the world means larger markets for our exports and lower prices for our imports. C) economic growth in other countries may mean increased competition for our exporters and domestic producers. D) growth in an economy's production capacity should be more valuable when that country can sell some of its increased production to the world market. E) the benefits of economic growth may be passed on to foreigners in the form of lower prices for the country's exports rather than retained at home. Answer: A Difficulty: Easy AACSB: Application of knowledge 23) When the production possibility frontier shifts out relatively more in one direction, we have A) immiserizing growth. B) unbiased growth. C) imbalanced growth. D) biased growth. E) balanced growth. Answer: D Difficulty: Easy AACSB: Application of knowledge
6 Copyright © 2022 Pearson Education, Inc.
24) Suppose that a country experiences growth strongly biased toward its export, cloth, A) this will tend to leave the country's terms of trade unchanged. B) this will tend to improve the country's terms of trade. C) this will increase the price of cloth relative to the imported good. D) this will tend to worsen the terms of trade for the country's trading partner. E) this will tend to worsen the country's terms of trade. Answer: E Difficulty: Moderate AACSB: Application of knowledge 25) An increase in a country's net commodity terms of trade will A) not always guarantee positive changes in the country's economy. B) always increase the country's economic welfare. C) always increase the country's real income. D) never increase the country's quantity of exports. E) always increase the country's production of its import competing good. Answer: A Difficulty: Moderate AACSB: Application of knowledge 26) Growth that disproportionately expands a country's production possibilities in the direction of the good it exports is called A) export-biased growth. B) import-biased growth. C) foreign-biased growth. D) domestic-biased growth. Answer: A Difficulty: Easy AACSB: Application of knowledge 27) Growth that disproportionately expands a country's production possibilities in the direction of the good a country imports is called A) import-biased growth. B) export-biased growth. C) foreign-biased growth. D) domestic-biased growth. Answer: A Difficulty: Easy AACSB: Application of knowledge
7 Copyright © 2022 Pearson Education, Inc.
28) Which of the following statements is TRUE? A) Export-biased growth tends to worsen a growing country's terms of trade to the benefit of the rest of the world. B) Export-biased growth tends to improve a growing country's terms of trade at the rest of the world's expense. C) Import-biased growth expands a country's production possibilities in the direction of the good it exports. D) Import-biased growth tends to worsen a growing country's terms of trade to the benefit of the rest of the world. Answer: A Difficulty: Easy AACSB: Application of knowledge 29) Which of the following statements is TRUE? A) Import-biased growth tends to improve a growing country's terms of trade at the rest of the world's expense. B) Export-biased growth tends to improve a growing country's terms of trade at the rest of the world's expense. C) Import-biased growth tends to worsen a growing country's terms of trade to the benefit of the rest of the world. D) Export-biased growth expands a country's production possibilities in the direction of the good it imports. Answer: A Difficulty: Easy AACSB: Application of knowledge 30) Which of the following statements is NOT true? A) Export-biased growth in the U.S. improves the U.S.'s terms of trade. B) Import-biased growth in the U.S. improves the U.S.'s terms of trade. C) Export-biased growth in the rest of the world improves the U.S.'s terms of trade. D) Import-biased growth in the rest of the world worsens the U.S.'s terms of trade. E) Export-biased growth in the U.S. worsens the U.S.'s terms of trade. Answer: A Difficulty: Easy AACSB: Application of knowledge 31) The situation when export-biased growth by poor nations would worsen their terms of trade so much that they would be worse off than if they had not grown at all is called A) immiserizing growth. B) imbalanced growth. C) biased growth. D) balanced growth. E) unbiased growth. Answer: A Difficulty: Easy AACSB: Application of knowledge 8 Copyright © 2022 Pearson Education, Inc.
32) Has the United States suffered from a deterioration in its terms of trade as some of its main trading partners, especially China, experienced rapid growth? A) No, there is no evidence that the United States has suffered any kind of sustained loss from a long-term deterioration in its terms of trade. B) Yes, there is evidence that the United States has suffered some sustained loss from a longterm deterioration in its terms of trade. C) No, there is no evidence that China's terms of trade have steadily appreciated as China has become increasingly integrated into the world economy. D) Yes, there is evidence that China's terms of trade have deteriorated over the past decade. Answer: A Difficulty: Easy AACSB: Application of knowledge 33) While most developed countries tend to experience ________ swings in their terms of trade, some developing countries tend to experience ________ swings in their terms of trade due to the fact that ________. A) mild; large; their major exports concentrated in mineral and agricultural sectors. B) mild; large; their major exports concentrated in manufactured sectors. C) large; mild; their major exports concentrated in manufactured sectors. D) large; mild; their major exports concentrated in mineral and agricultural sectors. Answer: A Difficulty: Easy AACSB: Application of knowledge 34) In 38 low-income and emerging economies with GDP per capita below $25,000 tracked by World Bank, the induced terms of trade movements accounted for ________ fluctuations on average for all those countries. A) 40 percent of GDP B) 30 percent of GDP C) 20 percent of GDP D) 10 percent of GDP Answer: A Difficulty: Easy AACSB: Application of knowledge 35) Other things being equal, a rise in a country's terms of trade increases its welfare. What would happen if we relax the ceteris paribus assumption, and allow for the law of demand to operate internationally? Answer: Let us assume that the terms of trade (or technically the net commodity terms of trade) improve, thus the relative price of a country's exports increase. This would, logically, lead to a shift away by world consumers to substitute goods. If the demand for a country's exports is elastic, the quantity decrease would be proportionally larger than the per unit price increase. This term of trade effect would actually lower the country's real income and economic welfare. Difficulty: Difficult AACSB: Application of knowledge
9 Copyright © 2022 Pearson Education, Inc.
36) Refer to above figure. Albania refused to engage in international trade for ideological reasons. To maximize its economic welfare it would choose to produce at which point in the diagram above? Suppose the PA/PB at point a was equal to 1. Given this information, in which good (A or B) does Albania enjoy a comparative advantage? Now that the Cold War is over, Albania is interested in obtaining economic welfare gains from trade. The relevant international relative price is PA/PB = 2. Albania would therefore choose to produce at which point (a, b, or c)? Given this additional information, in which good does Albania enjoy a comparative advantage? Answer: Albania would choose to produce at point a. With no reference to world terms of trade, one cannot establish Albania's comparative advantage. Later, when Albania discovers that the relative price of A equals twice the price of B, it knows that it has a comparative advantage in A. Therefore Albania would produce at production point b. Difficulty: Moderate AACSB: Application of knowledge 37) Refer to above figure. Now, suppose that the relative price of A is actually not higher than Albania's autarkic level of 1, but quite the opposite (e.g., PA/PB = 0.5). Would Albania still be able to gain from trade? If so, where would be its production point? Given the information in this question, where is Albania's comparative advantage? Answer: Yes. As long as the world's terms of trade differed from those of Albania, that country stands to gain from international trade. In this particular case, its point of production with trade would be at point c. Difficulty: Moderate AACSB: Application of knowledge
10 Copyright © 2022 Pearson Education, Inc.
38) Suppose, as a result of various dynamic factors associated with exposure to international competition, Albania's economy grew, and is now represented by the rightmost production possibility frontier in the figure above. If its point of production with trade was point c, would you consider this growth to be export-biased or import biased? If Albania were a large country with respect to the world trade of A and B, how would this growth affect Albania's terms of trade? Its real income? Answer: If point c is the production point with trade, then Albania has a comparative advantage in good B. Therefore, from the shape of the new production possibility frontier (as compared to the original one), this is clearly an export-biased growth. This ceteris paribus would tend to worsen Albania's terms of trade. The terms of trade effect would, again ceteris paribus, worsen its real income. However, the growth itself acts in the opposite direction. Difficulty: Moderate AACSB: Application of knowledge 39) Suppose, as a result of various dynamic factors associated with exposure to international competition, Albania's economy grew, and is now represented by the rightmost production possibility frontier in the figure above. If its point of production with trade was point b, would you consider this growth to be export-biased or import biased? If Albania were a large country with respect to the world trade of A and B, how would this growth affect Albania's terms of trade? Its real income? What if Albania were a small country? Answer: If the production with trade point was point b, then the observed growth is a case of import-biased growth, and would improve Albania's terms of trade. If Albania were a small country, the world's terms of trade would not change at all. In such a case, economic growth (with no induced change in income distributions) would always increase its real income. Difficulty: Moderate AACSB: Application of knowledge
11 Copyright © 2022 Pearson Education, Inc.
40) Suppose Albania is exporting product B, and experienced economic growth biased in favor of product B as seen in the figure above. We are also told that Albania's new consumption point is at point d. Would you still consider the economic growth, which took place biased in favor of B? If Albania were a large country how would this growth affect its terms of trade? Answer: This is a relatively difficult case. On the one hand, the growth is still technically export biased. However, Albania's consumption clearly shifted in favor of its import product, A. In this case, the deterioration in the terms of trade would be much more pronounced than before, and may lead to a case of immiserizing growth. However, for this to occur, there must have been a major shift in the taste patterns (the old community indifference map is not longer applicable). Therefore, when we try to judge the direction and magnitude of the welfare change, we are comparing the old versus new taste preferences, which raises the classic index number problem. Difficulty: Difficult AACSB: Application of knowledge 41) Explain the main reasons why growth may be biased. Answer: (1) The Ricardian model showed that technological progress in one sector of the economy will expand the economy's production possibilities in the direction of that sector's output. (2) The Heckscher-Ohlin model showed that an increase in a country's supply of a factor of production will produce biased expansion of production possibilities. The bias will be in the direction of either the good to which the factor is specific or the good whose production is intensive in the factor whose supply has increased. Thus, the same considerations that give rise to international trade will also lead to biased growth in a trading economy. Difficulty: Easy AACSB: Application of knowledge 42) Discuss why some developing countries' experienced large swings in the terms of trade. Answer: While most developed countries tend to experience mild swings in their terms of trade of around 1 percent or less a year on average, some developing countries' exports are heavily concentrated in mineral and agricultural sectors. The prices of those goods on world markets are very volatile, leading to large swings in the terms of trade. These swings in turn translate into substantial changes in welfare because trade is concentrated in a small number of sectors and represents a substantial percentage of GDP. Difficulty: Easy AACSB: Application of knowledge 43) How has Covid-19 pandemic impacted developing countries' terms of trade? Answer: Beyond the direct public health impact of COVID-19, the worldwide pandemic has also induced large fluctuations in commodity prices that are hitting some developing countries particularly hard. These swings in turn translate into substantial changes in welfare because trade is concentrated in a small number of sectors and represents a substantial percentage of GDP in those countries. Difficulty: Easy AACSB: Application of knowledge
12 Copyright © 2022 Pearson Education, Inc.
6.2
Tariffs and Export Subsidies: Simultaneous Shifts in RS and RD
1) The distinctive feature of tariffs and export subsidies is that A) they create a difference between prices at which goods are traded on the world market and prices at which those goods can be purchased within a country. B) they equalize prices at which goods are traded on the world market and prices at which those goods can be purchased within a country. C) they make imported and exported goods cheaper. D) they give incentives for importers and exporters. Answer: A Difficulty: Easy AACSB: Application of knowledge 2) The direct effect of a tariff is to make A) imported goods more expensive inside a country than they are outside the country. B) imported goods cheaper inside a country than they are outside the country. C) imported goods more expensive outside a country than they are inside the country. D) imported goods the same price inside a country as they are outside the country. Answer: A Difficulty: Easy AACSB: Application of knowledge 3) The effect of an export subsidy is to A) give producers an incentive to export. B) raise the prices of exported goods outside a country. C) make it less profitable to sell abroad than at home. D) raise government's revenue. Answer: A Difficulty: Easy AACSB: Application of knowledge 4) The price changes caused by tariffs and subsidies A) change both world relative supply and world relative demand, resulting in a shift in the terms of trade of the country imposing the policy change and of the rest of the world. B) have no impact on world relative supply and world relative demand, as well as the terms of trade of the country imposing the policy change. C) change both world relative supply and world relative demand, but have no impact on the terms of trade of the country imposing the policy change. D) have no impact on world relative supply and world relative demand, but change the terms of trade of the country imposing the policy change and of the rest of the world. Answer: A Difficulty: Easy AACSB: Application of knowledge
13 Copyright © 2022 Pearson Education, Inc.
5) If the U.S. (a large country) imposes a tariff on its imported good, this will tend to A) cause a deterioration of U.S. terms of trade. B) have no effect on terms of trade. C) improve the terms of trade of all countries. D) raise the world price of the good imported by the United States. E) improve the terms of trade of the United States. Answer: E Difficulty: Easy AACSB: Application of knowledge 6) If Slovenia is a small country in world trade terms, then if it imposes a large series of tariffs on many of its imports, this would A) decrease its marginal propensity to consume. B) deteriorate its terms of trade. C) increase its exports. D) have no effect on its terms of trade. E) improve its terms of trade. Answer: D Difficulty: Easy AACSB: Application of knowledge 7) If Slovenia is a large country in world trade, then if it imposes a large set of tariffs on many of its imports, this would A) increase its exports. B) have no effect on its terms of trade. C) harm its terms of trade. D) improve its terms of trade. E) decrease its marginal propensity to consume. Answer: D Difficulty: Easy AACSB: Application of knowledge 8) If Slovenia were a large country in world trade, then if it imposes a large set of tariffs on its imports, this must A) decrease the internal price of imports below the world market rate. B) improve the real income of its trade partners. C) harm Slovenia's real income. D) improve Slovenia's real income. E) cause retaliation on the part of its trade partners. Answer: A Difficulty: Easy AACSB: Application of knowledge
14 Copyright © 2022 Pearson Education, Inc.
9) If Slovenia were a large country in world trade, then if it instituted a large set of subsidies for its exports, this must A) have no effect on its terms of trade. B) harm world terms of trade. C) harm its terms of trade. D) decrease its marginal propensity to consume. E) improve its terms of trade. Answer: C Difficulty: Easy AACSB: Application of knowledge 10) If Slovenia were a large country in world trade, then if it instituted a large set of subsidies for its exports, this must A) increase internal prices above the world market rate. B) harm Slovenia's real income. C) improve Slovenia's real income. D) improve the real income of its trade partners. E) cause retaliation on the part of its trade partners. Answer: D Difficulty: Easy AACSB: Application of knowledge 11) If a small country were to levy a tariff on its imports then this would A) make the terms of trade effect too small to measure. B) change the terms of trade. C) raise prices on its exports in other countries. D) increase the country's economic welfare. Answer: A Difficulty: Easy AACSB: Application of knowledge 12) An import tariff on food will cause the world relative demand for cloth to ________ and the world relative supply for cloth to ________. A) decrease; increase B) increase; increase C) decrease; decrease D) increase; decrease Answer: D Difficulty: Moderate AACSB: Application of knowledge
15 Copyright © 2022 Pearson Education, Inc.
13) An export subsidy on cloth will cause the world relative demand for cloth to ________ and the world relative supply for cloth to ________. A) decrease; decrease B) increase; decrease C) increase; increase D) decrease; increase Answer: D Difficulty: Moderate AACSB: Analytical thinking 14) If a country is large relative to the rest of the world, an import tariff will cause the terms of trade of the ________ country to ________ and will ________ the country. A) importing; improve; benefit B) importing; improve; harm C) exporting; improve; benefit D) exporting; improve; harm E) importing; suffer; harm Answer: A Difficulty: Moderate AACSB: Application of knowledge 15) An export subsidy will cause the terms of trade of the ________ country to ________ and will ________ the country. A) importing; suffer; benefit B) importing; improve; harm C) exporting; suffer; harm D) importing; suffer; harm E) exporting; improve; benefit Answer: C Difficulty: Moderate AACSB: Application of knowledge 16) An export subsidy has the opposite effect on terms of trade to the effect of an import tariff. Domestically a tariff will raise the price of the import good, deteriorating the domestic terms of trade. A production subsidy for the export product will lower the local price of the export good, lowering the domestic terms of trade for the country. Hence the export subsidy and the import tariff have the same effect. This analysis seems to contradict the first sentence in this paragraph. Discuss this paradox. Answer: While this (Lerner) equivalence may well occur domestically, internationally the tariff will improve a country's terms of trade. An export subsidy on the other hand will in fact lower the international price of the (now readily available) export good, hence hurting a country's terms of trade. Difficulty: Moderate AACSB: Application of knowledge
16 Copyright © 2022 Pearson Education, Inc.
17) Discuss the winners and losers when Chinese government subsidized exports of solar panel to the United States, and the consequence when United States responded by imposing a tariff on solar panel imports from China. Answer: The subsidy's biggest impact falls on the distribution of income within the United States. If China subsidizes exports of solar panels to the United States, most U.S. residents gain from cheaper solar power, and the world benefits from lower carbon emissions. However, workers and investors in the U.S. solar panel industry are hurt by the lower import prices. Yet those workers are outnumbered by a ratio of nearly 6 to 1 by workers employed in the sales, distribution, and installation of solar panels, who benefit from lower import prices. Another consequence of the U.S. tariffs on imports of solar panels from China was trade diversion: The higher price of solar panels from China fueled an investment boom in the production of solar panels in Malaysia. By 2016, Malaysia supplanted China as the leading import source of U.S. solar panels. In 2018, the United States imposed a 30 percent tariff on imports of solar panels from all countries. Difficulty: Easy AACSB: Application of knowledge 6.3
International Borrowing and Lending
1) International borrowing and lending may be interpreted as one form of A) unrequited international transfers. B) aid to offset trade advantages. C) intertemporal trade. D) intermediate trade. E) trade in services. Answer: C Difficulty: Easy AACSB: Application of knowledge 2) Which of the following statements is TRUE? A) A country can trade current consumption for future consumption in the same way that it can produce more of one good by producing less of another. B) A country can produce more of one good by producing less of another but cannot trade current consumption for future consumption. C) A country can increase both current consumption and future consumption given its limited resources and fixed technology. D) A country can increase the production of all goods given its limited resources and fixed technology. Answer: A Difficulty: Easy AACSB: Application of knowledge
17 Copyright © 2022 Pearson Education, Inc.
3) If one observes that Japan was traditionally a net foreign lender, one could conclude that relative to its international trade and financial partners A) Japan's intertemporal production possibilities are biased toward present consumption. B) Japan's intertemporal production possibilities are not biased. C) Japan preferred to consume beyond its production in the present. D) Japan's intertemporal production possibilities are larger than that of the other countries. E) Japan's intertemporal production possibilities are biased toward future consumption. Answer: A Difficulty: Easy AACSB: Application of knowledge 4) If borrowing and lending are allowed, the relative price of future consumption, and thus the world real interest rate, will be determined by A) the world relative supply and demand for future consumption. B) the world relative supply and demand for current consumption. C) the world supply and demand for future consumption. D) the world supply and demand for current consumption. Answer: A Difficulty: Easy AACSB: Application of knowledge 5) Given r as the real interest rate on borrowing, the relative price of future consumption is A) 1/(1 + r) B) 1/(1 - r) C) (1 + r) D) (1 - r) Answer: A Difficulty: Easy AACSB: Application of knowledge 6) The intertemporal tradeoff between present and future consumption is measured by the A) nominal interest rate. B) real interest rate. C) terms of trade. D) rate of economic growth. E) inflation rate. Answer: B Difficulty: Moderate AACSB: Application of knowledge
18 Copyright © 2022 Pearson Education, Inc.
7) A fall in the real interest rate, all other things held constant, will cause a country's ________ to ________. A) current consumption; decrease B) welfare level; improve C) current consumption; increase D) terms of trade; worsen E) terms of trade; improve Answer: C Difficulty: Easy AACSB: Application of knowledge 8) An increase in the real interest rate, all other things held constant, will cause a country's ________ to ________. A) current consumption; increase B) current consumption; decrease C) terms of trade; improve D) terms of trade; worsen E) welfare level; improve Answer: B Difficulty: Easy AACSB: Application of knowledge 9) Countries that ________ in the international market will be those where highly productive investment opportunities are available relative to current productive capacity, while countries that ________ will be those where such opportunities are not available domestically. A) borrow; lend B) lend; borrow C) borrow; borrow D) lend; lend Answer: A Difficulty: Easy AACSB: Application of knowledge 10) Rapidly growing developing countries tend to be borrowers on the international capital markets. From this information we may surmise that they have a comparative advantage in A) capital goods. B) future income. C) consumer goods. D) present income. E) disposable income. Answer: B Difficulty: Easy AACSB: Application of knowledge
19 Copyright © 2022 Pearson Education, Inc.
11) It may be argued that theoretically, international capital movements A) tend to hurt the recipient countries. B) tend to hurt labor in donor countries. C) increase future production in donor countries. D) tend to hurt the donor countries. E) tend to hurt labor in recipient countries. Answer: B Difficulty: Moderate AACSB: Application of knowledge 12) What is intertemporal comparative advantage? Answer: Intertemporal comparative advantage arises when a country can produce goods for future consumption at a relatively low cost in terms of current consumption. Such a country can import investments (loans) from other countries with intertemporal comparative disadvantages at terms of trade that benefit both countries. Difficulty: Moderate AACSB: Application of knowledge 6.4
Appendix to Chapter 6: More on Intertemporal Trade
1) Given r as the real interest rate, the price of ________ consumption in terms of ________ consumption is ________. A) future; current; r B) present; future; r C) future; current; 1 + r D) future; current; 1/(1 + r) E) present; future; 1/(1 + r) Answer: D Difficulty: Easy AACSB: Application of knowledge 2) A rise in the world real interest rate r, which steepens the isovalue lines, A) causes investment to fall. B) causes investment to increase. C) has no impact on investment. D) has no impact on current and future consumption. Answer: A Difficulty: Easy AACSB: Application of knowledge
20 Copyright © 2022 Pearson Education, Inc.
3) The intertemporal budget constraint is defined as A) V = DP + DF/(1 + r) B) DP + DF(1 + r) = QP + QF(1 + r) C) DP + DF/(1 + r) = QP + QF/(1 + r) D) DF + DP/(1 + r) = QF + QP/(1 + r) E) V = QP + QF/(1 + r) Answer: C Difficulty: Easy AACSB: Application of knowledge 4) The intertemporal budget constraint means that the value of Home's consumption over the two periods ________ the value of consumption goods produced in the two periods. A) equals B) is greater than C) is less than D) is different from Answer: A Difficulty: Easy AACSB: Application of knowledge 5) Describe the nature of trade between two countries based on intertemporal comparative advantage. Answer: Intertemporal comparative advantage arises when a country can produce goods for future consumption at a relatively low cost in terms of current consumption when compared with its trading partner. This implies that the first country offers a relatively high return on investment when compared to the second. As a result, the first country will import goods for current consumption (investments or loans) and will export goods for future consumption (return on investment or interest). The resulting pattern of trade is one which will tend to equalize returns on investment in the two countries. Difficulty: Moderate AACSB: Application of knowledge
21 Copyright © 2022 Pearson Education, Inc.
International Economics: Theory and Policy, 12e (Krugman) Chapter 7 External Economies of Scale and the International Location of Production 7.1
Economies of Scale and International Trade: An Overview
1) If a firm's output more than doubles when all inputs are doubled, production is said to occur under conditions of A) intra-industry equilibrium. B) decreasing returns to scale. C) imperfect competition. D) increasing returns to scale. E) constant returns to scale. Answer: D Difficulty: Easy AACSB: Application of knowledge 2) Doubling the input of labor of 15 to 30 hours, the firm's output increases from 10 to 25 widgets, production is said to occur under conditions of A) increasing returns to scale. B) decreasing returns to scale. C) constant returns to scale. D) dynamic returns to scale. Answer: A Difficulty: Easy AACSB: Application of knowledge 3) One advantage of the specialization that results from international trade is that countries can take advantage of A) scale economies. B) taste reversals. C) production diversification. D) smaller countries. E) lower transport costs. Answer: A Difficulty: Moderate AACSB: Application of knowledge 4) If a firm's output doubles when all inputs are doubled, production is said to occur under conditions of A) increasing returns to scale. B) imperfect competition. C) intra-industry equilibrium. D) constant returns to scale. E) decreasing returns to scale. Answer: D Difficulty: Easy AACSB: Application of knowledge 1 Copyright © 2022 Pearson Education, Inc.
5) If a firm's output less than doubles when all inputs are doubled, production is said to occur under conditions of A) increasing returns to scale. B) imperfect competition. C) intra-industry equilibrium. D) constant returns to scale. E) decreasing returns to scale. Answer: E Difficulty: Easy AACSB: Application of knowledge 6) Explain why economies of scale provide an incentive for international trade. Answer: Mutually beneficial trade can arise as a result of economies of scale. Economies scale means higher productivity or higher output from the same amount of input. When each country concentrates on producing only a limited number of goods, each good can be produced more efficiently at a larger scale than would be the case if each country tried to produce everything for itself. As a result, the world economy can produce more of each good. This also allows each country not to have to sacrifice the variety in consumption. When specialized economies trade, this leads to an increase in the variety of goods available to world consumers. Difficulty: Moderate AACSB: Application of knowledge 7) How is trade beneficial to consumers as a result of economies of scale? Answer: When each country specializes in producing a limited range of products, this enables it to produce these goods more efficiently than it tried to produce everything for itself. These specialized countries then trade with each other and as a result, consumers can enjoy a lower price, higher quality, and more variety of goods available. Difficulty: Moderate AACSB: Reflective thinking
2 Copyright © 2022 Pearson Education, Inc.
7.2
Economies of Scale and Market Structure
1) The existence of external economies of scale A) tends to result in large profits for each firm. B) may be associated with a perfectly competitive industry. C) cannot be associated with a perfectly competitive industry. D) tends to result in one huge monopoly. E) focuses more on individual firms than the industry as a whole. Answer: B Difficulty: Moderate AACSB: Application of knowledge 2) The existence of internal economies of scale A) may be associated with a perfectly competitive industry. B) is associated only with sophisticated products such as aircraft. C) cannot be associated with a perfectly competitive industry. D) cannot form the basis for international trade. E) focuses more on the industry than individual firms. Answer: C Difficulty: Moderate AACSB: Application of knowledge 3) When there are external economies of scale, an increase in the size of the market will A) not affect the number of firms, but will lower the cost per unit. B) decrease the number of firms and lower the cost per unit. C) decrease the number of firms and raise the cost per unit. D) increase the number of firms and raise the cost per unit. E) increase the number of firms and lower the cost per unit. Answer: E Difficulty: Easy AACSB: Application of knowledge 4) If some industries exhibit internal increasing returns to scale in each country, we should not expect to see A) intra-industry trade between countries. B) high levels of specialization in both countries. C) inter-industry trade between countries. D) perfect competition in these industries. E) increased productivity in both countries. Answer: D Difficulty: Moderate AACSB: Application of knowledge
3 Copyright © 2022 Pearson Education, Inc.
5) External economies of scale arise when the cost per unit A) rises as the industry and the average firm grows larger. B) remains constant over a broad range of output. C) falls as the industry and the average firm grows larger. D) falls as the industry grows larger and rises as the average firm grows larger. E) rises as the industry grows larger and falls as the average firm grows larger. Answer: D Difficulty: Easy AACSB: Application of knowledge 6) Internal economies of scale arise when the cost per unit A) falls as the industry grows larger. B) remains constant over a broad range of output. C) rises as the industry grows larger. D) falls as the size of an individual firm grows larger. E) rises as the size of an individual firm grows larger. Answer: D Difficulty: Easy AACSB: Application of knowledge 7) Internal economies of scale will ________ cost per unit when output is ________ by ________. A) reduce; increased; a firm B) reduce; increased; the industry C) increase; increased; a firm D) reduce; reduce; the industry E) increase; increased; the industry Answer: A Difficulty: Moderate AACSB: Application of knowledge 8) External economies of scale will ________ cost per unit when output is ________ by ________. A) reduce; increased; the industry B) reduce; increased; a firm C) increase; increased; a firm D) increase; increased; the industry E) reduce; reduce; the industry Answer: A Difficulty: Moderate AACSB: Application of knowledge
4 Copyright © 2022 Pearson Education, Inc.
9) Why is it that if an industry is operating under conditions of internal scale economies then the resultant equilibrium cannot be consistent with the pure competition model? Answer: Because once one firm becomes bigger than another, or if one firm began the industry, then no other firm will be able to match its per unit cost, so that they would be driven out of the industry. The firm would become a natural monopoly. Difficulty: Moderate AACSB: Application of knowledge 10) Distinguish external economies of scale and internal economies of scale in terms of efficiency and the size of firm(s). Answer: Regarding external economies of scale, the efficiency of firms is increased by having a larger industry, even though each firm is the same size as before. Regarding internal economies of scale, a firm is more efficient if its output is larger. Difficulty: Easy AACSB: Application of knowledge 11) Explain how external and internal economies of scale have different implications for the structure of industries. Answer: An industry where economies of scale are purely external (no advantage to large firms) will typically consist of many small firms and be perfectly competitive. Internal economies of scale, by contrast, give large firms a cost advantage over small firms and lead to an imperfectly market structure. Difficulty: Moderate AACSB: Application of knowledge 7.3
The Theory of External Economies
1) When economies of scale apply at the level of the industry rather than at the level of the individual firms, they are called A) external economies. B) internal economies. C) mixed economies. D) scope economies. E) scale economies. Answer: A Difficulty: Easy AACSB: Application of knowledge 2) A cluster of firms may be more efficient than an individual firm in isolation because it can take advantage of A) specialized suppliers, labor market pooling, and knowledge spillovers. B) specialized suppliers, government policies, geographical location. C) labor market pooling, tax policies, internal economies of scale. D) knowledge spillovers, network externalities, government support. Answer: A Difficulty: Easy AACSB: Application of knowledge 5 Copyright © 2022 Pearson Education, Inc.
3) External economies of scale often arise because similar firms A) have excellent internal logistics. B) locate in the same geographic region. C) collude to fix prices and increase profits. D) agree to cooperate to expand global trade. E) have economies of scale in production. Answer: B Difficulty: Easy AACSB: Application of knowledge 4) If output is increased in the long run, average production costs in the presence of internal economies of scale will ________, and in the presence of external economies of scale, will ________. A) increase; decrease B) increase; remain constant C) remain constant; increase D) decrease; decrease E) decrease; remain constant Answer: D Difficulty: Moderate AACSB: Application of knowledge 5) A localized industrial cluster improves efficiency because A) it can bring together many firms that collectively provide a large enough market to support a wide range of specialized suppliers. B) it limits the market size by only attracting local firms and local suppliers. C) it creates more hostility among firms in the same industry. D) it may lower the number of small start-ups to join the cluster. Answer: A Difficulty: Moderate AACSB: Application of knowledge 6) Which of the following statements is NOT TRUE? A) A localized industrial cluster improves efficiency because it limits the market size by only attracting local firms and local suppliers. B) A localized industrial cluster improves efficiency because it can bring together many firms that collectively provide a large enough market to support a wide range of specialized suppliers. C) A localized industrial cluster improves efficiency because it creates more competition, availability, and affordability of key inputs from a network of specialized suppliers. D) A localized industrial cluster improves efficiency because it frees individual producers from the expense of developing capital equipment internally and by spreading the costs of development by using specialized suppliers. Answer: A Difficulty: Moderate AACSB: Application of knowledge
6 Copyright © 2022 Pearson Education, Inc.
7) Labor market pooling from a cluster of firms can ________ labor shortages for producers and ________ unemployment risk for workers. A) decrease; decrease B) increase; increase C) increase; decrease D) decrease; increase Answer: A Difficulty: Easy AACSB: Application of knowledge 8) A geographically concentrated industry can facilitate ________ that allow informal exchange of information and ideas at a personal level. A) knowledge spillovers B) creative commons C) social gathering D) technical gossiping Answer: A Difficulty: Easy AACSB: Application of knowledge 9) When there are external economies of scale, the average cost of production ________ as the industry output ________. A) falls; rises B) rises; falls C) falls; falls D) rises; rises Answer: A Difficulty: Easy AACSB: Application of knowledge 10) In presence of external economies of scale, given competition among producers, the downward-sloping average cost curve can be interpreted as A) a forward-falling supply curve. B) an upward-sloping supply curve. C) a horizontal supply curve. D) a vertical supply curve. Answer: A Difficulty: Easy AACSB: Application of knowledge 11) What is meant by "a cluster of firms" and what are the three main sources of the economic advantages derived from locating in such a district? Answer: A cluster of firms is a geographical concentration of firms in the same industry. Silicon Valley and Bollywood are modern examples. The advantages are (1) specialized suppliers, (2) labor market pooling, and (3) knowledge spillovers. Difficulty: Easy AACSB: Application of knowledge 7 Copyright © 2022 Pearson Education, Inc.
12) Explain how a localized industrial cluster can solve the problem of specialized suppliers. Answer: A localized industrial cluster can bring together many firms that collectively provide a large enough market to support a wide range of specialized suppliers. Key inputs are cheaper and more easily available because there are many firms competing to provide them, and firms can concentrate on what they do best, contracting out other aspects of their business. A company that tried to enter the industry in another location–for example, in a country that did not have a comparable industrial cluster–would be at an immediate disadvantage because it would lack easy access to specialized suppliers and would either have to provide them for itself or be faced with the task of trying to deal with specialized suppliers at long distance. Difficulty: Easy AACSB: Application of knowledge 13) Justify how a localized industrial cluster can create a pooled market that is advantageous for employers and employees. Answer: If firms are in different locations, whenever one of the firms is doing well, it will be confronted with a labor shortage at some point. If the firms are near each other, in case one will be doing well when the other doing badly, both firms may be able to hire as many workers as they want. By locating near each other, the companies increase the likelihood that they will be able to take advantage of business opportunities. From the workers' point of view, if the industry is concentrated in a single city, low labor demand from one firm will at least sometimes be offset by high demand from the other. As a result, workers will have a lower risk of unemployment. Difficulty: Moderate AACSB: Application of knowledge 14) Describe how a localized industrial cluster can facilitate knowledge spillovers. Answer: An important source of technical know-how, is the informal exchange of information and ideas that takes place at a personal level. And this kind of informal diffusion of knowledge often seems to take place most effectively when an industry is concentrated in a fairly small area, so that employees of different companies mix socially and talk freely about technical issues. This kind of informal information flow means it is easier for companies in the cluster to stay near the technological frontier than it is for companies elsewhere. Difficulty: Easy AACSB: Application of knowledge
8 Copyright © 2022 Pearson Education, Inc.
7.4
External Economies and International Trade
1) When trade is opened, the scale of production possible in a country is constrained by A) the size of the domestic plus the foreign market. B) the size of the country. C) the size of the domestic market. D) the size of the trading partner's country. E) the size of the foreign market. Answer: A Difficulty: Easy AACSB: Application of knowledge 2) Because of forward-falling supply curve, China's ________ production output as a result of trade leads to button price that is ________ than the price before trade. A) higher; lower B) lower; higher C) higher; higher D) lower; lower Answer: A Difficulty: Easy AACSB: Application of knowledge 3) Apart from comparative advantage, ________ can play a key role in determines the pattern of specialization and trade in industries with external economies of scale. A) historical accident B) civil wars C) natural disasters D) decreasing returns to scale Answer: A Difficulty: Easy AACSB: Application of knowledge 4) Why does a pattern of specialization established by historical accident persist even when new producers could potentially have lower costs? A) New producers face with higher initial production cost than the established producers. B) New producers incur lower production cost than the established producers. C) New producers own better technology than the established producers. D) New producers are more competitive than the established producers. Answer: A Difficulty: Moderate AACSB: Application of knowledge
9 Copyright © 2022 Pearson Education, Inc.
5) Which of the following statements is TRUE? A) It is possible that trade based on external economies may make a country worse off than it would have been in the absence of trade. B) It is impossible that trade based on external economies will make a country worse off than it would have been in the absence of trade. C) It is impossible that trade based on external economies will make a country better off than it would have been in the absence of trade. D) It is guaranteed that trade based on external economies will make a country better off than it would have been in the absence of trade. Answer: A Difficulty: Moderate AACSB: Application of knowledge 6) The spillover of knowledge gives rise to a situation in which the production costs of individual firms ________ as the industry as a whole accumulates experience. A) fall B) rise C) stay the same D) are unpredictable Answer: A Difficulty: Easy AACSB: Application of knowledge 7) In presence of external economies of scale, a country's industry with ________ cumulative output to date will experience a ________ unit cost. A) higher; lower B) higher; higher C) lower; lower D) higher; constant Answer: A Difficulty: Easy AACSB: Application of knowledge 8) When costs fall with cumulative production over time rather than with the current rate of production, this is the case of A) dynamic increasing returns. B) dynamic decreasing returns. C) constant increasing returns. D) constant decreasing returns. Answer: A Difficulty: Easy AACSB: Application of knowledge
10 Copyright © 2022 Pearson Education, Inc.
9) In the presence of external economies of scale, trade A) will unambiguously improve welfare in both countries. B) will unambiguously worsen welfare in the exporting country and improve welfare in the importing country. C) may or may not improve welfare in both countries. D) will unambiguously improve welfare in the exporting country and worsen welfare in the importing country. E) will unambiguously worsen welfare in both countries. Answer: C Difficulty: Easy AACSB: Application of knowledge 10) A learning curve relates ________ to ________ and is a case of ________ returns. A) unit cost; cumulative production; dynamic increasing B) output per time period; long-run marginal cost; dynamic decreasing C) output per time period; long-run marginal cost; dynamic increasing D) unit cost; cumulative production; dynamic decreasing E) labor productivity; education; increasing marginal Answer: A Difficulty: Easy AACSB: Application of knowledge 11) The learning curve describes the ________ relationship between ________ and ________. A) inverse; education; annual income B) direct; education; annual income C) direct; education; labor productivity D) inverse; unit cost; cumulative output E) direct; unit cost; cumulative output Answer: D Difficulty: Moderate AACSB: Application of knowledge 12) The argument for temporary protection of industries from foreign competition to enable them to gain experience until the industry can stand on its own feet is known as A) infant industry argument. B) national security argument. C) anti-dumping argument. D) cultural preservation argument. Answer: A Difficulty: Easy AACSB: Application of knowledge
11 Copyright © 2022 Pearson Education, Inc.
13) If a scale economy is the dominant technological factor defining or establishing comparative advantage, then the underlying facts explaining why a particular country dominates world markets in some product may be pure chance, or historical accident. Explain, and compare this with the answer you would give for the Heckscher-Ohlin model of comparative advantage. Answer: This statement is true, since the reason the seller is a monopolist may be that it happened to have been the first to produce this product in this country. It may have no connection to any supply or demand related factors; nor to any natural or man-made availability. This is all exactly the opposite of the Heckscher-Ohlin Neo-Classical model's explanation of the determinants of comparative advantage. Difficulty: Moderate AACSB: Application of knowledge 14) Using external economies of scale, explain how international trade led to lower button prices than the prices both in China and the U.S. before trade. Answer: Before the opening of trade, China supplied only its own domestic button market. After trade, it supplies the world market, producing buttons for both China and U.S. consumers. Because of external economies of scale, international trade makes it possible to concentrate world production in a single location and therefore to reduce costs by reaping the benefits of even stronger external economies. China's supply curve is forward falling, increased production as a result of trade leads to a button price that is lower than the price before trade. Difficulty: Moderate AACSB: Analytical thinking 15) What is historical contingency? How does it play a role in determining the pattern of specialization and trade in industries with external economies of scale? Answer: Something gives a particular location an initial advantage in a particular industry, and this advantage gets "locked in" by external economies of scale even after the circumstances that created the initial advantage are no longer relevant. Examples include financial centers in London and New York, carpet manufacturing in Dalton, Georgia, semiconductor and high-tech center in the Silicon Valley, information service call centers in Bangalore. Difficulty: Easy AACSB: Analytical thinking 16) Explain how trade based on external economies may leave a country worse off than it would have been in the absence of trade. Answer: The importance of established advantage means that there is no guarantee that the right country will produce a good subject to external economies. A pattern of specialization established by historical accident may persist even when new producers could potentially have lower costs. For example, Thailand would be able to supply its domestic market with watches at a lower price if there were no trade and the country were forced to produce the good for itself. However, with international trade, Switzerland's watch manufacturers are able to supply the world market at a price low enough to block entry by Thai producers who do not have the established advantage with as large of a quantity of watches produced and demanded as their Swiss counterparts. Difficulty: Moderate AACSB: Analytical thinking 12 Copyright © 2022 Pearson Education, Inc.
7.5
Interregional Trade and Economic Geography
1) Trade that takes place between regions within countries is called A) interregional trade. B) intra-industry trade. C) domestic trade. D) intercontinental trade. Answer: A Difficulty: Easy AACSB: Application of knowledge 2) More than ________ percent of U.S. workers are employed by industries whose output is nontradable even within the United States. A) 60 B) 50 C) 40 D) 30 Answer: A Difficulty: Easy AACSB: Application of knowledge 3) Restaurant meals are an example of a ________ good and clothing is an example of a ________ good. The pattern of interregional trade is determined primarily by ________. A) durable; nondurable; natural resources B) consumer; style; population C) nontradable; tradable; external economies D) nondurable; durable; natural resource E) traded; nontraded; internal economies Answer: C Difficulty: Easy AACSB: Application of knowledge 4) The share of ________ goods in employment is ________ across the country. The share of ________ goods in employment is ________ across the country. A) nontradable; uniform; tradable; variable B) nondurable; uniform; durable; variable C) durable; uniform; nondurable; variable D) traded; uniform; nontraded; variable E) nontraded; variable; traded; uniform Answer: A Difficulty: Moderate AACSB: Application of knowledge
13 Copyright © 2022 Pearson Education, Inc.
5) Patterns of interregional trade are primarily determined by ________ rather than ________ because factors of production are generally ________. A) external economies; population; immobile B) external economies; natural resources; mobile C) internal economies; population; immobile D) population; external economies; immobile E) internal economies; external economies; mobile Answer: B Difficulty: Easy AACSB: Application of knowledge 6) The primary determinant of patterns of interregional trade is A) accidents of history. B) centralized optimization. C) resource allocations. D) weather. E) factor abundance. Answer: A Difficulty: Easy AACSB: Application of knowledge 7) The study of factors that influence economic interaction across space related to both interregional and international trade is referred to as A) accidents of history. B) economic geography. C) factor abundance theory. D) weather analysis. E) centralized optimization. Answer: B Difficulty: Easy AACSB: Application of knowledge 8) Explain how the City in London and the Wall Street in New York derive their advantages from external economies in the past. How have Brexit and Covid-19 impacted the special roles of the City and the Street recently and in the years ahead? Answer: Both established dominant roles in the distant past, when the British Empire ruled much of the world and New York had a unique advantage as the terminus of the Erie Canal. Both may benefit a bit from being located in English-speaking countries, since English remains the dominant language of international business. During the pandemic, it turns out that much of the business formerly conducted in urban highrises can be done equally well from home offices, the great financial centers, which are monuments to the power of external economies of scale, may emerge much diminished. Difficulty: Moderate AACSB: Reflective thinking
14 Copyright © 2022 Pearson Education, Inc.
International Economics: Theory and Policy, 12e (Krugman) Chapter 8 Firms in the Global Economy: Export and Foreign Sourcing Decisions and Multinational Enterprises 8.1
The Theory of Imperfect Competition
1) Internal economies of scale imply that A) a firm's average cost of production decreases the more output it produces. B) a firm's average cost of production increases the more output it produces. C) a firm's average cost of production stays the same as more output is produced. D) a firm's average cost of production may increase or decrease the more output it produces. Answer: A Difficulty: Easy AACSB: Application of knowledge 2) Internal economies of scale and product differentiation combine A) to generate some new sources of gains of trade via economic integration. B) to generate some new sources of losses of trade via economic integration. C) to generate some new trade barriers among countries. D) to generate some new obstacles for economic growth. Answer: A Difficulty: Easy AACSB: Application of knowledge 3) A perfectly competitive market is a market in which A) there are many buyers and sellers, none of whom represents a large part of the market, and firms are price takers. B) there are only a few large buyers and sellers, who represent a large part of the market, and firms are price takers. C) there are many buyers and sellers, none of whom represents a large part of the market, and firms are price setters. D) there are only a few large firms, who represent a large part of the market, and firms are price setters. Answer: A Difficulty: Easy AACSB: Application of knowledge 4) In imperfect competition, A) firms can influence the prices of their differentiated products and they are price setters. B) firms can influence the prices of their differentiated products and they are price takers. C) firms cannot influence the prices of their differentiated products and they are price takers. D) firms cannot influence the prices of their differentiated products and they are price setters. Answer: A Difficulty: Easy AACSB: Application of knowledge
1 Copyright © 2022 Pearson Education, Inc.
5) A market in which a firm faces no competition is called A) a pure monopoly. B) an oligopoly. C) a duopoly. D) a monopolistic competitor. Answer: A Difficulty: Easy AACSB: Application of knowledge 6) International trade based solely on internal scale economies in both countries is likely to be carried out by A) a relatively small number of imperfect competitors. B) a large number of oligopolists in each country. C) monopolists in each country. D) a relatively small number of price competing firms. E) a relatively large number of price competing firms. Answer: C Difficulty: Easy AACSB: Application of knowledge 7) Marginal revenue is A) the extra revenue the firm gains from selling an additional unit. B) the extra utility the firm gains from selling an additional unit. C) the extra cost the firm incurs from producing an additional unit. D) the extra profit the firm gains from selling an additional unit. Answer: A Difficulty: Easy AACSB: Application of knowledge 8) For a monopolist, the marginal revenue curve always lies below the demand curve or the marginal revenue is always less than the price because A) to sell an additional unit, the firm must lower the price of all units. B) to sell an additional unit, the firm must raise the price of all units. C) to sell an additional unit, the firm can keep the price of all units the same. D) to sell an additional unit, the firm can lower the price of just the marginal one. Answer: A Difficulty: Easy AACSB: Application of knowledge
2 Copyright © 2022 Pearson Education, Inc.
9) A monopolistic firm A) can sell as much as it wants for any price it determines in the market. B) cannot sell additional quantity unless it raises the price on each unit. C) chooses an output at which marginal revenue equals marginal cost. D) cannot determine the price, which is determined by consumer demand. E) will always earn a profit in the long run. Answer: C Difficulty: Moderate AACSB: Application of knowledge 10) A monopoly firm engaged in international trade will A) equate marginal costs with the highest price the market will bear. B) equate average to local costs. C) equate marginal costs with foreign marginal revenues. D) equate marginal costs with marginal revenues in both domestic and foreign markets. E) equate marginal costs with the relative world prices. Answer: D Difficulty: Easy AACSB: Application of knowledge 11) A monopoly firm will maximize profits by producing where A) total revenue from domestic and foreign sales is maximized. B) marginal revenue is the same in domestic and foreign markets. C) prices are the same in domestic and foreign markets. D) marginal revenue is higher in the domestic market. E) marginal revenue is higher in foreign markets. Answer: B Difficulty: Moderate AACSB: Application of knowledge 12) If a firm increases its output in the ________ and unit costs ________, then the firm is experiencing ________ of scale. A) long run; decrease; economies B) long run; increase; economies C) short run; decrease; diseconomies D) long run; decrease; diseconomies E) short run; decrease; economies Answer: A Difficulty: Easy AACSB: Application of knowledge
3 Copyright © 2022 Pearson Education, Inc.
13) If a firm increases its output in the ________ and unit costs ________, then the firm is experiencing ________ of scale. A) short run; decrease; economies B) long run; increase; diseconomies C) short run; decrease; diseconomies D) long run; increase; economies E) long run; decrease; diseconomies Answer: B Difficulty: Easy AACSB: Application of knowledge 14) Modeling trade in imperfectly competitive industries is problematic because A) collusion among imperfectly competitive firms makes usable data rare. B) there is no single generally accepted model of behavior by imperfectly competitive firms. C) it is difficult to find an imperfectly competitive firm in the real world. D) there is only a single model of imperfect competition (monopoly) but imperfect competition can take many forms in the real world. E) there are no models of imperfectly competitive behavior. Answer: B Difficulty: Easy AACSB: Application of knowledge 15) Firms that produce ________ products must be ________ competitive. A) differentiated; imperfectly B) differentiated; perfectly C) exported; imperfectly D) standardized; imperfectly E) standardized; perfectly Answer: A Difficulty: Moderate AACSB: Application of knowledge 16) Imperfectly competitive firms have a demand curve that ________ and a marginal revenue curve that ________ and is ________ the demand curve. A) is horizontal; is horizontal; the same as B) is horizontal; slopes downward; below C) slopes downward; slopes downward; below D) slopes downward; is horizontal; above E) slopes downward; slopes downward; the same as Answer: C Difficulty: Easy AACSB: Application of knowledge
4 Copyright © 2022 Pearson Education, Inc.
17) Monopolistic competition is associated with A) product differentiation. B) price-taking behavior. C) increasing returns to scale. D) high profit margins in the long run. E) explicit consideration at the firm level of the strategic impact of other firms' pricing decisions. Answer: A Difficulty: Easy AACSB: Application of knowledge 18) If there are a large number of firms in a monopolistically competitive industry A) the country in which the firms are located can be expected to export the goods they produce. B) long-run profit will be equal to zero. C) there will be a small number of firms that are very large and the rest will be very small. D) the firms will converge production on a standardized product. E) there will be barriers to entry that prevent additional firms from entering the industry. Answer: B Difficulty: Moderate AACSB: Application of knowledge 19) A firm in long-run equilibrium under monopolistic competition will earn A) positive oligopoly profits because each firm sells a differentiated product. B) zero economic profits because of free entry. C) negative economic profits because it has economies of scale. D) positive economic profit if it engages in international trade. E) positive monopoly profits because each sells a differentiated product. Answer: B Difficulty: Easy AACSB: Application of knowledge 20) A product is produced in a monopolistically competitive industry with scale economies. If this industry exists in two countries, and these two countries engage in trade with each other, then we would expect A) the country with a relative abundance of the factor of production in which production of the product is intensive will export this product. B) the countries will trade only with other nations they are not in competition with. C) the country in which the price of the product is lower will export the product. D) neither country will export this product since there is no comparative advantage. E) each country will export different varieties of the product to the other. Answer: E Difficulty: Easy AACSB: Application of knowledge
5 Copyright © 2022 Pearson Education, Inc.
21) If the market for products produced by firms in a monopolistically competitive industry becomes ________, then there will be ________ firms and each firm will produce ________ output and charge a ________ price. A) smaller; fewer; less; higher B) smaller; fewer; less; lower C) smaller; more; less; lower D) smaller; more; less; higher E) smaller; fewer; more; higher Answer: A Difficulty: Easy AACSB: Application of knowledge 22) Under oligopoly, firms' pricing policies are ________ and, under monopolistic competition, they are ________. A) interdependent; independent B) independent; interdependent C) cooperative; uncooperative D) uncooperative; cooperative E) profit maximizing; revenue maximizing Answer: A Difficulty: Moderate AACSB: Application of knowledge 23) If a firm that uses a production process that yields economies of scale charges a price less than ________, then profit will be ________. A) marginal cost; positive B) average cost; negative C) marginal revenue; positive D) marginal cost; maximized E) marginal revenue; maximized Answer: B Difficulty: Moderate AACSB: Application of knowledge 24) Under the model of monopolistic competition, a(an) ________ in the number of firms in the industry will cause ________ to ________. A) increase; average cost; increase B) increase; average price; increase C) increase; average cost; decrease D) decrease; markup; decrease E) increase; marginal cost; decrease Answer: A Difficulty: Easy AACSB: Application of knowledge
6 Copyright © 2022 Pearson Education, Inc.
25) Under the model of monopolistic competition, a(an) ________ in the number of firms in the industry will cause ________ charged by each firm to ________. A) increase; price; decrease B) increase; price; increase C) increase; average cost; decrease D) decrease; markup; decrease E) increase; marginal cost; decrease Answer: A Difficulty: Moderate AACSB: Application of knowledge 26) Under the model of monopolistic competition, a(an) ________ in the number of firms in the industry will cause ________ to ________. A) increase; markup; decrease B) increase; average price; increase C) increase; average cost; decrease D) decrease; markup; decrease E) increase; marginal cost; decrease Answer: A Difficulty: Moderate AACSB: Application of knowledge 27) When a country both exports and imports a type of commodity, the country is engaged in A) inter-industry trade. B) an attempt to monopolize the relevant industry. C) increasing returns to scale. D) intra-industry trade. E) imperfect competition. Answer: D Difficulty: Easy AACSB: Application of knowledge 28) It is possible that trade based on external scale economies may leave a country worse off than it would have been without trade. Explain how this could happen. Answer: One answer is that the terms of trade effects may dominate any other factors. Difficulty: Difficult AACSB: Application of knowledge 29) An imperfectly competitive firm has the following demand curve: Q = 100 - 2P. What is marginal revenue equal to when P = 30? Answer: Q = 40, so MR = 30 - (40/2) = 10. Difficulty: Moderate AACSB: Application of knowledge
7 Copyright © 2022 Pearson Education, Inc.
30) An imperfectly competitive firm has the following demand curve: Q = 100 - 2P. What is marginal revenue equal to when P = 40? Answer: Q = 20, so MR = 40 - (20/2) = 30. Difficulty: Moderate AACSB: Application of knowledge 31) An imperfectly competitive firm has the following total cost curve: C = 100 + 4Q. What is marginal cost equal to when Q = 10? Answer: MC = 4 for any Q Difficulty: Moderate AACSB: Application of knowledge 32) An imperfectly competitive firm has the following total cost curve: C = 100 + 4Q. What is total cost equal to when Q = 10? Answer: C = 100 + (4)(10) = 140 Difficulty: Moderate AACSB: Application of knowledge 33) An imperfectly competitive firm has the following total cost curve: C = 100 + 4Q. What is average total cost equal to when Q = 10? Answer: C/Q = [100 + (4)(10)]/10 = 14 Difficulty: Moderate AACSB: Application of knowledge 34) An imperfectly competitive firm has the following total cost curve: C = 100 + 4Q. What is average fixed cost equal to when Q = 10? Answer: F/Q = 100/10 = 10 Difficulty: Moderate AACSB: Application of knowledge
8 Copyright © 2022 Pearson Education, Inc.
8.2
Monopolistic Competition and Trade
1) The monopolistic competition model can be used to show the following EXCEPT A) how trade limits the size of the national market and the variety of goods available to domestic consumers. B) how trade improves the trade-off between scale and variety that individual nation face. C) how trade creates a world market larger than any of the national markets that comprise it. D) how a larger market leads to both a lower average price and the availability of a greater variety of goods. Answer: A Difficulty: Easy AACSB: Application of knowledge 2) In larger markets there will be both more firms and more sales per firm, A) consumers in a large market will be offered both lower prices and a greater variety of products than consumers in small markets. B) consumers in a small market will be offered both lower prices and a greater variety of products than consumers in large markets. C) consumers in a large market will be offered both higher prices and a very limited variety of products than consumers in small markets. D) consumers in a small market will be offered the same prices and same variety of products as consumers in small markets. Answer: A Difficulty: Easy AACSB: Application of knowledge 3) If the market for products produced by firms in a monopolistically competitive industry becomes ________, then there will be ________ firms and each firm will produce ________ output and charge a ________ price. A) larger; fewer; more; higher B) larger; more; less; higher C) larger; fewer; more; lower D) larger; more; more; lower E) larger; more; more; higher Answer: D Difficulty: Easy AACSB: Application of knowledge
9 Copyright © 2022 Pearson Education, Inc.
4) International trade based on external scale economies in both countries is likely to be carried out by A) a large number of oligopolists in each country. B) a relatively large number of price competing firms. C) a relatively small number of price competing firms. D) a relatively small number of imperfect competitors. E) monopolists in each country. Answer: B Difficulty: Easy AACSB: Application of knowledge 5) Product differentiation and internal economies of scale lead to A) trade between similar countries with no comparative advantage differences between them. B) no trade between similar countries with no comparative advantage differences between them. C) a smaller variety of goods available to consumers. D) higher price that consumers have to pay. Answer: A Difficulty: Easy AACSB: Application of knowledge 6) The integrated market supports ________ firms, each producing at a ________ scale and selling at a ________ price than either national market does on its own. A) more; larger; lower B) more; smaller; higher C) fewer; smaller; higher D) fewer; larger; higher Answer: A Difficulty: Easy AACSB: Application of knowledge 7) An industry is characterized by scale economies, and exists in two countries. Should these two countries engage in trade such that the combined market is supplied by one country's industry, then A) consumers in both countries would have more varieties and lower prices. B) consumers in both countries would have higher prices and fewer varieties. C) consumers in the exporting country only would have higher prices and fewer varieties. D) consumers in both countries would have fewer varieties at lower prices. E) consumers in the importing country only would have higher prices and fewer varieties. Answer: A Difficulty: Easy AACSB: Application of knowledge
10 Copyright © 2022 Pearson Education, Inc.
8) In the integrated market after trade, both Home and Foreign consumers A) benefit from a greater variety of goods at a lower price. B) suffer from fewer variety of goods at a higher price. C) suffer from fewer quality of goods at a higher price. D) benefit from higher quality of goods at a higher price. Answer: A Difficulty: Easy AACSB: Application of knowledge 9) An industry is characterized by scale economies and exists in two countries. In order for consumers of its products to enjoy both lower prices and more variety of choice A) the two countries must engage in international trade with each other. B) each country's marginal cost must equal that of the other country. C) the monopoly must lower prices in order to sell more. D) they must combine to become a multinational corporation. E) the marginal cost of this industry must equal marginal revenue in the other. Answer: A Difficulty: Moderate AACSB: Application of knowledge 10) In an industry where firms experience internal scale economies, the long-run cost of production will depend on A) individual firms' fixed costs. B) the size of the labor force. C) whether the country engages in intra-industry trade. D) the size of the market. E) whether the country engages in inter-industry trade. Answer: D Difficulty: Easy AACSB: Application of knowledge 11) The simultaneous export and import of widgets by the United States is an example of A) inter-industry trade. B) imperfect competition. C) the effect of a monopoly on international trade. D) intra-industry trade. E) increasing returns to scale. Answer: D Difficulty: Easy AACSB: Application of knowledge
11 Copyright © 2022 Pearson Education, Inc.
12) Intra-industry trade is a very important component of trade for the United States in many different industries EXCEPT A) footwear and apparel. B) chemicals. C) pharmaceuticals. D) specialized machinery. Answer: A Difficulty: Easy AACSB: Application of knowledge 13) Intra-industry trade plays an even more prominent role in the trade of A) manufactured goods among advanced industrial nations. B) labor-intensive goods among less advanced industrial nations. C) intermediate goods among advanced industrial nations. D) agricultural goods among less advanced industrial nations. Answer: A Difficulty: Easy AACSB: Application of knowledge 14) The gains from integration generated by economies of scale were most pronounced for A) smaller economies. B) raw material producers. C) the developing countries of Asia and Africa. D) labor-intensive products. E) China and not the rest of the world. Answer: A Difficulty: Easy AACSB: Application of knowledge 15) Intra-industry trade is most common in the trade patterns of A) raw material producers. B) the developing countries of Asia and Africa. C) labor-intensive products. D) the industrial countries of Western Europe. E) China with the rest of the world. Answer: D Difficulty: Easy AACSB: Application of knowledge
12 Copyright © 2022 Pearson Education, Inc.
16) Before 1965, tariff protection by Canada and the United States produced a Canadian auto industry A) that was largely self-sufficient, neither importing nor exporting much. B) where imports exceeded exports. C) where exports exceeded imports. D) that was completely dependent on the U.S. auto industry. Answer: A Difficulty: Easy AACSB: Application of knowledge 17) After the free trade agreement in 1964, A) Canada's auto imports and exports both increased sharply. B) Canada's auto imports increased and exports decreased. C) Canada's auto exports increased and imports decreased. D) Canada's auto imports and exports both decreased sharply. Answer: A Difficulty: Easy AACSB: Application of knowledge 18) With the implementation of NAFTA, the transformation of the auto industry with substantial productivity gain expanded to A) Mexico. B) Canada. C) South America. D) Japan. Answer: A Difficulty: Easy AACSB: Application of knowledge 19) The dismantling of NAFTA A) hit Mexico and Canada's automotive production the hardest. B) led to higher car production in the United States. C) decrease in car production outside North America. D) decreased in price of cars produced outside North America. Answer: A Difficulty: Easy AACSB: Application of knowledge 20) Imagine scale economies were not only external to firms, but were also external to individual countries. That is, the larger the worldwide industry (regardless of where firms or plants are located), the cheaper would be the per-unit cost of production. Describe what world trade would look like in this case. Answer: Presumably each country would specialize in some component of the final product. This would result in in a high volume of intra-industry trade. Difficulty: Difficult AACSB: Application of knowledge 13 Copyright © 2022 Pearson Education, Inc.
Dollars
Quantity 21) Refer to above figure. The monopolist can export as much as it likes of its steel at the world price of $5/ton. How much steel will the monopolist sell, and at what price? Answer: It would sell 10 million tons at $5/ton. Difficulty: Moderate AACSB: Application of knowledge 22) Refer to above figure. Given the opportunity to sell at world prices, the marginal (opportunity) cost of selling a ton domestically is what? Answer: $5/ton. Difficulty: Easy AACSB: Application of knowledge 23) Refer to above figure. While selling exports it would also maximize its domestic sales by equating its marginal (opportunity) cost to its marginal revenue of $5. How much steel would the firm sell domestically, and at what price? Answer: 4 million tons at $10/ton. Difficulty: Easy AACSB: Application of knowledge
14 Copyright © 2022 Pearson Education, Inc.
24) Everyone is better off as a result of integration. Explain why and how to realize these gains. Answer: The integrated market supports more firms, each producing at a larger scale and selling at a lower price than either national market does on its own. Clearly everyone is better off because of integration. In the larger market, consumers have a wider range of choices, yet each firm produces more and is therefore able to offer its product at a lower price. To realize these gains from integration, the countries must engage in international trade. To achieve economies of scale, each firm must concentrate its production in one country–either Home or Foreign. Yet it must sell its output to customers in both markets. As firms are able to consolidate their production destined for both locations and take advantage of economies of scale, each product will be produced in only one country and exported to the other. Difficulty: Easy AACSB: Application of knowledge 8.3
Firm Responses to Trade: Winners, Losers, and Industry Performance
1) Trade (or economic integration) A) led to an increase in competition between firms. B) led to a decrease in competition between firms. C) did not affect competition between firms. D) may or may not affect competition between firms. Answer: A Difficulty: Easy AACSB: Application of knowledge 2) Increased competition from trade (or economic integration) tends to A) force the worst-performing firms to exit the market. B) force the best performing firms to exit the market. C) provide fewer sales opportunities in new markets for surviving firms. D) provide more incentives for new firms to join the market. Answer: A Difficulty: Easy AACSB: Application of knowledge 3) Which of the following statements is NOT true? Compared to a firm with a higher marginal cost, a firm with a lower marginal cost will A) produce less output. B) set a lower price but at a higher markup over marginal cost. C) produce more output. D) earn higher profit. Answer: A Difficulty: Easy AACSB: Application of knowledge
15 Copyright © 2022 Pearson Education, Inc.
4) In the model of monopolistic competition, if firms have ________ average cost curves, then opening trade will ________ the total number of firms and ________ the average price. A) downward sloping; decrease; decrease B) downward sloping; decrease; increase C) upward sloping; decrease; increase D) downward sloping; increase; decrease E) upward sloping; increase; decrease Answer: A Difficulty: Moderate AACSB: Application of knowledge 5) In the model of monopolistic competition, if firms have ________ average cost curves, then opening trade will cause ________ firms to ________ the industry. A) different; more efficient; enter B) symmetric; less efficient; enter C) different; less efficient; exit D) symmetric; less efficient; exit E) symmetric; more efficient; enter Answer: C Difficulty: Easy AACSB: Application of knowledge 6) In the model of monopolistic competition, compared to a firm with a higher marginal cost, a firm with a lower marginal cost will set a ________ price, produce ________ output, and earn ________ profits. A) lower; less; less B) higher; less; more C) lower; more; more D) higher; more; more E) higher; less; less Answer: C Difficulty: Easy AACSB: Application of knowledge 7) In the model of monopolistic competition, compared to a firm with a lower marginal cost, a firm with a higher marginal cost will set a ________ price, produce ________ output, and earn ________ profits. A) higher; less; more B) higher; less; less C) lower; less; less D) lower; more; more E) higher; more; more Answer: B Difficulty: Easy AACSB: Application of knowledge
16 Copyright © 2022 Pearson Education, Inc.
8) In the model of monopolistic competition, an increase in industry output will cause individual firms' demand curves to become ________, which will ________ demand for higher-priced goods and ________ demand for lower-priced goods. A) steeper; reduce; increase B) steeper; increase; reduce C) horizontal; reduce; reduce D) flatter; increase; reduce E) flatter; reduce; increase Answer: E Difficulty: Easy AACSB: Application of knowledge 9) In the model of monopolistic competition, an increase in industry output will ________ producers of higher-priced goods and ________ producers of lower-priced goods. A) harm; benefit B) benefit; benefit C) benefit; harm D) benefit; have no effect on E) harm; harm Answer: A Difficulty: Easy AACSB: Application of knowledge 10) In the model of monopolistic competition, an increase in industry output will ________ market shares and ________ profits of producers of higher-priced goods and will ________ market shares and ________ profits of producers of lower-priced goods. A) increase; reduce; increase; reduce B) increase; increase; reduce; reduce C) reduce; increase; reduce; increase D) reduce; reduce; increase; increase E) reduce; increase; increase; reduce Answer: D Difficulty: Easy AACSB: Application of knowledge 11) How can trade and economic integration have a direct impact on industry performance? Answer: Increased competition from trade (or economic integration) tends to force the worstperforming firms to exit the market, provide sales opportunities in new markets for the surviving or best performing firms. When the better-performing firms expand and the worse-performing ones contract or exit, overall industry performance improves. These composition changes generate substantial improvements in industry productivity. Difficulty: Easy AACSB: Application of knowledge
17 Copyright © 2022 Pearson Education, Inc.
12) Who are the winners and losers generated by increased market size among firms in an industry? Answer: The low-cost firms thrive and increase their profits and market shares, while the highcost firms contract and the highest-cost firms exit. These composition changes imply that overall productivity in the industry is increasing as production is concentrated among the more productive (low-cost) firms. Difficulty: Easy AACSB: Application of knowledge 8.4
Trade Costs and Export Decisions
1) In the model of monopolistic competition, trade costs A) explain why some firms choose not to export. B) raise firms' profitability. C) explain why some firms choose to import. D) associated with national borders increase trade among countries. Answer: A Difficulty: Easy AACSB: Application of knowledge 2) In the model of monopolistic competition, trade costs between countries will cause domestic and foreign markets to have ________ prices, ________ quantities sold, and ________ profit levels. A) identical; identical; different B) different; different; identical C) identical; different; identical D) different; different; different E) identical; different; different Answer: D Difficulty: Easy AACSB: Application of knowledge 3) In the model of monopolistic competition, trade costs between countries cause A) marginal costs of exported goods to exceed the marginal costs of goods sold domestically. B) all firms that can earn a profit on domestic sales to export their goods at higher prices. C) countries to negotiate the elimination of trade costs by mutual subsidization of trade. D) marginal costs of goods sold domestically to exceed the marginal costs of exported goods. E) all firms that can earn a profit on domestic sales to export their goods at lower prices. Answer: A Difficulty: Easy AACSB: Application of knowledge
18 Copyright © 2022 Pearson Education, Inc.
4) In the model of monopolistic competition, trade costs between countries cause A) marginal costs of goods sold domestically to exceed the marginal costs of exported goods. B) countries to negotiate the elimination of trade costs by mutual subsidization of trade. C) prices of goods sold domestically to exceed the prices of exported goods. D) some firms that can earn a profit on domestic sales to refrain from exporting their goods. E) all firms that can earn a profit on domestic sales to export their goods at higher prices. Answer: D Difficulty: Easy AACSB: Application of knowledge 5) Why would some firms choose not to export? Answer: The trade costs reduce the profitability of exporting for all firms. For some, that reduction in profitability makes exporting unprofitable. Difficulty: Easy AACSB: Application of knowledge 6) What are the important predictions of the modeling of trade costs to monopolistic competition and trade? Answer: Trade costs explain why only a subset of firms export, and they also explain why this subset of firms will consist of relatively larger and more productive firms (those firms with lower marginal cost). Empirical analyses of firms' export decisions from numerous countries have provided overwhelming support for this prediction that exporting firms are bigger and more productive than firms in the same industry that do not export. Difficulty: Easy AACSB: Application of knowledge
19 Copyright © 2022 Pearson Education, Inc.
8.5
Dumping
1) Trade costs affect the following EXCEPT A) how a firms will let go of profit for the benefit of domestic and overseas customers. B) how a firm responds to competition in a market. C) whether a firm will export their products. D) how much a firm will lower its markup for the export market. Answer: A Difficulty: Easy AACSB: Application of knowledge 2) In response to trade costs, A) a firm with a higher marginal cost will choose to set a lower markup over marginal cost for the export market. B) a firm with a lower marginal cost will choose to set a lower markup over marginal cost for the export market. C) a firm with a higher marginal cost will choose to set a higher markup over marginal cost for the export market. D) a firm with a lower marginal cost will not export to the foreign market. Answer: A Difficulty: Easy AACSB: Application of knowledge 3) The trade practice when a firm sets an export price lower than its domestic price is called A) preferential trade arrangements. B) non-tariff barriers. C) product boycotts. D) voluntary export restraints. E) dumping. Answer: E Difficulty: Easy AACSB: Application of knowledge 4) Any firm from Foreign can appeal to its local authorities and seek punitive damages against the unfair trade practice of dumping in the form of A) antidumping duty. B) antidumping quota. C) antidumping punishment. D) antidumping demonstration. Answer: A Difficulty: Easy AACSB: Application of knowledge
20 Copyright © 2022 Pearson Education, Inc.
5) Economists believe that A) there is no good economic justification for dumping to be considered particularly harmful. B) enforcement of dumping claims is well-guided. C) dumping is not a controversial issue in trade policy. D) trade costs have no impact on firms' pricing decisions in the export market. Answer: A Difficulty: Easy AACSB: Application of knowledge 6) Complaints are often made to the International Trade Commission concerning foreign "dumping" practices. These complaints typically claim that A) foreign companies are charging prices that are lower than prices they charge countries other than the U.S. B) foreign companies are charging exorbitant prices that are higher than the true value of the products. C) U.S. consumers cannot differentiate between the foreign and domestic goods. D) U.S. firms are harmed by the unfair pricing of foreign exporters. E) U.S. consumers are harmed by the lack of quality control or health concerns in foreign countries. Answer: D Difficulty: Easy AACSB: Application of knowledge 7) Most antidumping complaints were directed at ________ in the early 1990s, at ________ since 1995. A) developed countries; developing countries B) developing countries; developed countries C) developed countries; developed countries D) developing countries; developing countries Answer: A Difficulty: Easy AACSB: Application of knowledge 8) Antidumping laws A) allow domestic firms to be protected from foreign competition by raising their competitors' costs. B) allow foreign firms to easily enter the domestic market. C) allows domestic firms to be protected from foreign competition by lowering their competitors' costs. D) allows foreign firms to be more competitive in the domestic market. Answer: A Difficulty: Easy AACSB: Application of knowledge
21 Copyright © 2022 Pearson Education, Inc.
9) When U.S. tire manufacturers successfully lobbied for antidumping duties on Chinese tires in 2009, A) production shifted to South Korea, Thailand, Indonesia, and exports from those countries doubled. B) tire imports from China continued to increase. C) tire exports from China and all other countries to the U.S. decreased. D) tire exports from U.S. tire manufacturers increased. Answer: A Difficulty: Easy AACSB: Application of knowledge 10) In 2012, after Whirlpool won its antidumping case against the imports of washing machines produced by LG and Samsung in South Korea, LG and Samsung A) shifted their washing machine production for the U.S. market to China. B) shifted their washing machine production for the U.S. market to Vietnam and Thailand. C) shifted their washing machine production for the U.S. market to the U.S. D) shifted their washing machine production for the U.S. market to Canada and Mexico. Answer: A Difficulty: Easy AACSB: Application of knowledge Dollars
Quantity 11) The figure above represents the demand and cost functions facing a Brazilian steel producing monopolist. If it were unable to export, and was constrained by its domestic market, what quantity would it sell at what price? Answer: It would sell 5 (million tons) at a price of $8/ton. Difficulty: Moderate AACSB: Application of knowledge 22 Copyright © 2022 Pearson Education, Inc.
12) The figure above represents the demand and cost functions facing a Brazilian steel producing monopolist. The Brazilian firm is charging its foreign (U.S.) customers one half the price it is charging its domestic customers. Is this good or bad for the real income or economic welfare of the United States? Is the Brazilian firm engaged in dumping? Is this predatory behavior on the part of the Brazilian steel company? Answer: It is good for U.S. customers.Yes, this is dumping if you define dumping as selling abroad at a price lower than domestically. No, it is not dumping if by dumping you mean selling below marginal cost. No, this is not predatory, since it is not being done in order to capture market share, but rather is "mere" static profit maximization behavior, as is expected of any selfrespecting monopolist. Difficulty: Moderate AACSB: Application of knowledge 13) Describe the Whirlpool's antidumping case and discuss the winners and losers. Answer: In In 2012, after Whirlpool won its antidumping case against the imports of washing machines produced by LG and Samsung in South Korea, LG and Samsung shifted their washing machine production for the U.S. market to China. In 2016, after Whirlpool won a new antidumping case against those Chinese-produced washing machines, this then prompted LG and Samsung to invest in production facilities in Vietnam and Thailand. In 2018 when the Trump administration enacted a global "safeguard" tariff against the import of washing machines from all destinations, LG and Samsung started producing washing machines for the U.S. market in the United States. The higher production costs in the United States, along with higher markups induced by lower competition in the protected U.S. market, led to significantly higher prices for washing machines: A recent study estimates that the tariffs were responsible for an $86 increase in the median price of a washing machine (the price of drying machines also increased due to the tariff). The relocation to LG's and Samsung's production did result in 1,800 additional U.S. jobs. But the annual cost to U.S. consumers from higher prices amounts to $815,000 per job created. Difficulty: Easy AACSB: Application of knowledge
23 Copyright © 2022 Pearson Education, Inc.
8.6
Multinationals and Foreign Direct Investment
1) A corporation is considered a multinational ________ if ________. A) parent; more than 10% of its stock is held by a foreign company B) child; more than 50% of its stock is held by a foreign company C) monopolist; it owns more than 50% of a foreign firm D) parent; it owns more than 10% of a foreign firm E) child; more than 10% of its stock is held by a foreign company Answer: D Difficulty: Easy AACSB: Application of knowledge 2) A corporation is considered a multinational ________ if ________. A) child; more than 50% of its stock is held by a foreign company B) child; more than 10% of its stock is held by a foreign company C) parent; more than 10% of its stock is held by a foreign company D) affiliate; more than 10% of its stock is held by a foreign company E) monopolist; it owns more than 50% of a foreign firm Answer: D Difficulty: Easy AACSB: Application of knowledge 3) Consider the following two cases. In the first, a U.S. firm purchases 18% of a foreign firm. In the second, a U.S. firm builds a new production facility in a foreign country. Both are ________, with the first referred to as ________ and the second as ________. A) foreign direct investment (FDI); inflows; outflows B) foreign direct investment (FDI) inflows; brownfield; greenfield C) foreign direct investment (FDI) outflows; brownfield; greenfield D) foreign direct investment (FDI) outflows; greenfield; brownfield E) foreign direct investment (FDI) inflows; greenfield; brownfield Answer: D Difficulty: Moderate AACSB: Application of knowledge 4) When a multinational affiliate replicates production in a foreign country it is called ________ foreign direct investment. A) bisectional B) direct C) horizontal D) transitional E) vertical Answer: C Difficulty: Easy AACSB: Application of knowledge
24 Copyright © 2022 Pearson Education, Inc.
5) When a multinational affiliate replicates elements of a production process in a foreign country it is called ________ foreign direct investment. A) horizontal B) vertical C) transitional D) bisectional E) direct Answer: B Difficulty: Easy AACSB: Application of knowledge 6) Vertical FDI is mainly driven by A) production cost differences between countries. B) geographical location. C) absolute advantage. D) political power. Answer: A Difficulty: Easy AACSB: Application of knowledge 7) In vertical FDI, when a multinational ships intermediate goods across its network of domestic and foreign affiliates, the resulting trade is classified as A) intra-firm trade. B) intra-industry trade. C) intra-continent trade. D) intra-nation trade. Answer: A Difficulty: Easy AACSB: Application of knowledge 8) Horizontal FDI is mainly driven by A) trade and transport costs. B) production cost differences between countries. C) political wills. D) comparative advantage. Answer: A Difficulty: Easy AACSB: Application of knowledge
25 Copyright © 2022 Pearson Education, Inc.
9) Toyota's replicating the production for its most popular car model, the Corolla, in assembly plants in many countries all over the world is an example of A) horizontal FDI. B) vertical FDI. C) circular FDI. D) triangular FDI. Answer: A Difficulty: Easy AACSB: Application of knowledge 10) A firm's foreign direct investment decisions are, in the case of horizontal FDI, strongly influenced by ________ and, in the case of vertical FDI, strongly influenced by ________. A) materials costs; labor costs B) trade costs; production costs C) labor costs; trade costs D) production costs; trade costs E) production costs; materials costs Answer: B Difficulty: Easy AACSB: Application of knowledge 11) Fluctuations in FDI flows are strongly influenced by the following factors EXCEPT A) the ups and downs of the domestic business cycles. B) changes in corporate tax policy. C) the gyrations of stock markets worldwide. D) the financial crisis. E) the waves of cross-border mergers and acquisitions. Answer: A Difficulty: Easy AACSB: Application of knowledge 12) Historically, ________ have been the biggest recipients of inward FDI. A) the richest OECD B) the low-income countries C) the middle-income countries D) the poorest outside OECD Answer: A Difficulty: Easy AACSB: Application of knowledge
26 Copyright © 2022 Pearson Education, Inc.
13) From 2017 to 2019, FDI outflows are dominated by A) the developed economies. B) Brazil and India. C) developing countries. D) Cayman Islands and British Virgin Islands. Answer: A Difficulty: Easy AACSB: Application of knowledge 14) What is the nature of the proximity-concentration tradeoff that firms have to deal with then making decisions regarding foreign direct investment? Answer: If the firm has numerous production facilities close to their various international markets, trade costs will be relatively low. However, when there are numerous production facilities, each will be relatively small, and opportunities for economies of scale will be foregone. Difficulty: Moderate AACSB: Application of knowledge 15) Explain the difference between horizontal and vertical FDI. Answer: Horizontal FDI occurs when the multinational undertakes the production process in its domestic facilities and replicates the same process elsewhere in the world. Vertical FDI takes place when the multinational breaks up the product chain internationally, locating each stage of production in the country where it can be done at the least cost. Horizontal FDI is mainly driven by trade and transport costs and is dominated by flows between developed countries. Vertical FDI is mainly driven by production cost differences between countries and is behind the large increase in FDI inflows to developing countries. Difficulty: Easy AACSB: Application of knowledge
27 Copyright © 2022 Pearson Education, Inc.
16) Describe the inflows of FDI from 1970 to 2019 from Figure 8.9 below.
Answer: Worldwide flows of FDI have significantly increased since the mid-1990s, though the rates of increase have been very uneven. Historically, most of the inflows of FDI have gone to the developed countries in the OECD. However, the proportion of FDI inflows going to developing and transition economies has steadily increased over time and accounted for roughly half of worldwide FDI flows since 2009. Difficulty: Easy AACSB: Application of knowledge 17) Describe the outward FDI for top 25 countries from 2017 to 2019 from Figure 8.10 below.
Answer: Developed countries dominate the list of the top countries whose firms engage in outward FDI. More recently, firms from some big developing countries such as China and India have performed significantly more FDI. Difficulty: Easy AACSB: Application of knowledge 28 Copyright © 2022 Pearson Education, Inc.
18) Considering the patterns of FDI flows around the world from mid 1990s to 2017, list some of the factors that influence the fluctuations of FDI. Answer: There was a massive increase in multinational activity in the mid- to late-1990s, when worldwide FDI flows more than quadrupled relative to world GDP. The growth rate of FDI is very uneven, with huge peaks and troughs correlate with the gyrations of stock markets worldwide (strongly dominated by fluctuations in the U.S. stock market) and the related waves of cross-border mergers and acquisitions (greenfield FDI is much more stable over time). The financial collapse in 2000 (the bursting of the dot-com bubble) and the most recent financial crisis in 2007—2009 also induced huge crashes in worldwide FDI flows. Fluctuations in FDI flows are also strongly influenced by changes in corporate tax policy. The U.S. "Tax Cuts and Jobs Act" enacted in 2017 strongly cut the corporate tax rate for U.S. multinationals. In response, those multinationals repatriated a substantial proportion of their foreign earnings, which led to a sharp reduction in the value of their foreign investments and a drop in the value of world FDI. Difficulty: Easy AACSB: Application of knowledge 8.7
Foreign Direct Investment and Foreign Sourcing Decisions
1) A firm is more likely to engage in horizontal foreign direct investment if A) trade costs are low and firms experience constant returns to scale in production. B) trade costs are low and there are increasing returns to scale in production. C) trade costs are high and there are increasing returns to scale in production. D) trade costs are high and there are decreasing returns to scale in production. E) trade costs are low and there are decreasing returns to scale in production. Answer: C Difficulty: Easy AACSB: Application of knowledge 2) The proximity-concentration trade-off for FDI happens when A) firms have an incentive to locate a production facility near their foreign customers to avoid the high trade costs associated with exports to that foreign destination. B) trade costs are low and exporting is less expensive. C) it is cost effective to replicate the production process many times. D) firms operate facilities that produce too little output to take advantage of increasing returns to scale in production. Answer: A Difficulty: Easy AACSB: Application of knowledge
29 Copyright © 2022 Pearson Education, Inc.
3) If the total trade-related cost to export exceeds the alternative of the fixed cost of setting up an additional production facility in the foreign market, then A) exporting is more expensive and FDI is the profit-maximizing choice. B) FDI alternative is more expensive, and exporting is the profit-maximizing choice. C) there should be no trade between the two countries. D) neither exporting nor FDI is the profit-maximizing choice. Answer: A Difficulty: Easy AACSB: Application of knowledge 4) Higher trade costs on one hand and lower fixed production costs on the other hand A) lower the FDI cutoff. B) raise the FDI cutoff. C) keeps the FDI cutoff the same. D) have no impact on the FDI cutoff. Answer: A Difficulty: Easy AACSB: Application of knowledge 5) A firm's decision to break up its production chain and move parts of that chain to a foreign supplier is called A) foreign sourcing. B) foreign transferring. C) foreign sharing. D) foreign assembling. Answer: A Difficulty: Easy AACSB: Application of knowledge 6) Foreign outsourcing is A) the transfer of operations to foreign contractors. B) an example of foreign direct investment. C) an example of internalization. D) currently illegal in the U.S. E) the substitution of immigration for foreign direct investment. Answer: A Difficulty: Moderate AACSB: Application of knowledge
30 Copyright © 2022 Pearson Education, Inc.
7) During the past decade, U.S. imports of business services have ________, U.S. exports of business services have ________, and U.S. net exports of business services have ________. A) increased; decreased; decreased B) decreased; increased; increased C) decreased; decreased; increased D) increased; increased; increased E) increased; increased; not changed Answer: D Difficulty: Moderate AACSB: Application of knowledge 8) A scale cutoff for foreign sourcing depends on A) the production cost differentials and the fixed cost of coordinating with a foreign supplier. B) transportation and trade costs. C) the corporate tax rates and government subsidies. D) the number of jobs to be created in the foreign market. Answer: A Difficulty: Easy AACSB: Application of knowledge 9) When the scale effect is strong, A) the decisions to use some foreign suppliers make the firm more likely to use others. B) the decisions to use some foreign suppliers will have no impact on whether the firm will use others. C) the decisions to use some no foreign suppliers will never change. D) the decisions to use some foreign suppliers will make the firm less likely to use others. Answer: A Difficulty: Easy AACSB: Application of knowledge 10) U.S. firms that source from a wider range of countries A) enjoy greater cost advantages and have higher scales of production. B) enjoy smaller cost advantages and have lower scales of production. C) incur higher fixed costs and bear higher risks. D) incur higher tax rates and bear higher risks. Answer: A Difficulty: Easy AACSB: Application of knowledge 11) Sourcing choices are based on different trade-offs between A) cost savings and the fixed costs of coordinating with suppliers in foreign destinations. B) producers' surplus and consumers' surplus. C) deadweight loss and social welfare. D) gains and loss from trade. Answer: A Difficulty: Easy AACSB: Application of knowledge 31 Copyright © 2022 Pearson Education, Inc.
12) In terms of foreign sourcing, China and Mexico offer ________ cost savings with relatively ________ fixed costs while Germany and Canada offer substantially ________ fixed costs. A) highest; high; lower B) lowest; low; higher C) highest; low; higher D) lowest; high; lower Answer: A Difficulty: Easy AACSB: Application of knowledge 13) As a substitute for horizontal FDI, A) a parent could license an independent firm to produce and sell its products in a foreign location. B) a parent could contract an independent firm to perform a specific part of the production process in a foreign location. C) a parent could build more manufacturing facilities in their home country. D) a parent could expand the current manufacturing facilities in their home country. Answer: A Difficulty: Easy AACSB: Application of knowledge 14) As a substitute for vertical FDI, A) a parent could contract with an independent firm to perform specific parts of the production process in a foreign location. B) a parent could license an independent firm to produce and sell its products in a foreign location. C) a parent could contract with more independent suppliers in their home country. D) a parent could expand the current manufacturing facilities in their home country. Answer: A Difficulty: Easy AACSB: Application of knowledge 15) A firm selects a make-or buy internationalization choice by considering the following EXCEPT A) the difference between the unemployment rates at home and abroad. B) the different trade-offs between production cost savings and the fixed cost of moving parts of the production process abroad. C) the risks of losing some proprietary technology during licensing. D) the potential arising conflict regarding specific attributes of the inputs that cannot be specified in a legal contract between the buying and supplying firms. Answer: A Difficulty: Easy AACSB: Application of knowledge
32 Copyright © 2022 Pearson Education, Inc.
16) Which of the following statements is NOT true? In a world where production chains increasingly stretch around the world, the rise of intermediate good trade A) makes measures of bilateral trade deficits based on standard trade flows (at gross value) still irrelevant. B) makes aggregate trade statistics very misleading. C) makes measures of bilateral trade deficits based on standard trade flows (at gross value) irrelevant. D) affects the measures of the bilateral trade deficit across various U.S. trading partners. Answer: A Difficulty: Easy AACSB: Application of knowledge 17) The reported U.S. iPhone imports from China A) represent imports from many other countries–including the United States–that export iPhone components to China. B) represent manufacturing costs with components produced solely in China. C) represent value of iPhone components exported to China from the U.S. only. D) represent only the value added of assembly, testing cost, and battery produced in China. Answer: A Difficulty: Easy AACSB: Application of knowledge 18) When a firm switches from domestic sourcing to foreign sourcing, it is commonly referred to as A) offshoring. B) domestic sourcing. C) forwarding. D) transferring. Answer: A Difficulty: Easy AACSB: Application of knowledge 19) In many cases, the overall employment effect for the offshoring firm is positive because A) firms expand their overseas employment as well as U.S. employment thanks to higher productivity. B) jobs used to be performed in the U.S. are now moved overseas. C) call center or manufacturing workers displaced by offshoring are not the ones being hired by the expanding firms. D) U.S. service offshoring has grown faster than inshoring (exports of business services). Answer: A Difficulty: Easy AACSB: Application of knowledge
33 Copyright © 2022 Pearson Education, Inc.
20) Manufacturing employment has been steadily decreasing over the past 30 years in the US, A) but offshoring played a very minor role in this trend. B) and offshoring played a very significant role in this trend. C) but offshoring has nothing to do with this trend. D) and offshoring is the main reason for this trend. Answer: A Difficulty: Easy AACSB: Application of knowledge 21) As with other forms of trade, trade in intermediates has substantial consequences for A) the distribution of income. B) climate change. C) culture. D) political stability. Answer: A Difficulty: Easy AACSB: Application of knowledge 22) During the past decade, U.S. imports of business services have ________, U.S. exports of business services have ________, and U.S. net exports of business services have ________. A) increased; decreased; decreased B) decreased; increased; increased C) decreased; decreased; increased D) increased; increased; increased E) increased; increased; not changed Answer: D Difficulty: Moderate AACSB: Application of knowledge 23) Product differentiation and internal economies of scale yield gains from trade in the form of A) higher profits and lower trade costs. B) lower production costs and a greater variety of goods. C) the proximity-concentration effect. D) the substitution of immigration for foreign direct investment. E) a proliferation of competitive firms. Answer: B Difficulty: Moderate AACSB: Application of knowledge
34 Copyright © 2022 Pearson Education, Inc.
24) Discuss the role of multinationals in terms of foreign sourcing and exports. Answer: With the case of horizontal FDI, foreign sourcing requires a substantial fixed cost investment and there is a scale cutoff that depends on the production cost differentials and the fixed cost of coordinating with a foreign supplier. Only those firms operating at a scale above that cutoff will choose to source from a foreign supplier. Similarly, manufacturing firms that import intermediate goods are substantially bigger and more productive than firms in the same industries that do not import. In the United States in a typical manufacturing industry, an importing firm is two and a half times larger than a firm does not import. There is a huge overlap between those sets of firms that foreign source and export–because both of those activities involve a trade-off in favor of a larger scale. Amongst this selected subset of firms, the largest and most productive are overwhelmingly multinationals. For the world, multinationals are responsible for 80 percent of all trade flows. Difficulty: Moderate AACSB: Application of knowledge 25) What are the key elements that determine a firm's make-or-buy internationalization choice? Answer: Students can talk about various factors that boil down to different trade-offs between production cost savings and the fixed cost of moving parts of the production process abroad. Difficulty: Moderate AACSB: Application of knowledge 26) Using the example of iPhones imported from China, explain why trade statistics can be misleading. Answer: The rise of intermediate good trade makes the aggregate statistics very misleading. When an iPhone 11 ProMax was imported from China, its total manufacturing cost was recorded as $490. However, less than $10 of the $490 total cost represents assembly and testing costs (performed in China). The battery for the iPhone produced in China adds another $10 of Chinese valued added. The remaining cost represents the iPhone's component costs, which are overwhelmingly produced throughout Asia (Korea, Japan, and Taiwan are the largest suppliers), Europe, and the Americas. Thus, the reported U.S. iPhone imports from China actually represent imports from many other countries–including the United States–that export iPhone components to China. Difficulty: Moderate AACSB: Application of knowledge 27) Do you agree or disagree with the statement that offshoring simply amounts to "shipping jobs overseas"? Justify your answer. Answer: Students may agree or disagree. If they disagree, they may elaborate on why the view that offshoring simply amounts to "shipping jobs overseas" is misleading. True, when a firm based in the United States moves a call center to India, or moves the assembly of its product to China, then some specific jobs that used to be performed in the United States are now performed other countries like India or China. However, the evidence shows that in terms of overall employment, those jobs are replaced by other ones in the United States: some related to the expansion effect at the offshoring firms and others by firms providing intermediate goods and services to firms located abroad (inshoring). Difficulty: Moderate AACSB: Application of knowledge 35 Copyright © 2022 Pearson Education, Inc.
28) What are the consequences of outsourcing production on the welfare of countries? Answer: By taking advantage of cost differentials between countries, both countries can enjoy gains from trade. However, income distribution effects will result in winners and losers. Gains from trade are therefore thought of in terms of net gains in which the winners could compensate the losers and still be better off. Difficulty: Moderate AACSB: Application of knowledge
36 Copyright © 2022 Pearson Education, Inc.
International Economics: Theory and Policy, 12e (Krugman) Chapter 9 The Instruments of Trade Policy 9.1
Basic Tariff Analysis
1) A tariff is a tax levied when A) a good is imported. B) a good is exported. C) a good is produced. D) a good is exchanged in the domestic market. Answer: A Difficulty: Easy AACSB: Application of knowledge 2) Specific tariffs are A) import taxes stated in specific legal statutes. B) import taxes calculated as a fixed charge for each unit of imported goods. C) import taxes calculated as a fraction of the value of the imported goods. D) the same as import quotas. E) import taxes calculated based solely on the origin country. Answer: B Difficulty: Easy AACSB: Application of knowledge 3) Ad valorem tariffs are A) import taxes stated in ads in industry publications. B) import taxes calculated as a fixed charge for each unit of imported goods. C) import taxes calculated as a fraction of the value of the imported goods. D) the same as import quotas. E) import taxes calculated solely on the origin country. Answer: C Difficulty: Easy AACSB: Application of knowledge 4) Which of the following is a fixed percentage of the value of an imported product? A) specific tariff B) ad valorem tariff C) nominal tariff D) effective protection tariff E) infant industry tariff Answer: B Difficulty: Easy AACSB: Application of knowledge
1 Copyright © 2022 Pearson Education, Inc.
5) A tax of 20 cents per unit of imported garlic is an example of a(n) A) specific tariff. B) ad valorem tariff. C) nominal tariff. D) effective protection tariff. E) a disadvantageous tariff. Answer: A Difficulty: Easy AACSB: Application of knowledge 6) A tax of 20 percent per unit of imported garlic is an example of a(n) A) specific tariff. B) ad valorem tariff. C) nominal tariff. D) effective protection tariff. E) a disadvantageous tariff Answer: B Difficulty: Easy AACSB: Application of knowledge 7) Throughout the post-World War II era, the importance of tariffs as a trade barrier has A) increased. B) decreased. C) remained the same. D) fluctuated wildly. E) demonstrated a classic random walk with a mean-reversion tendency. Answer: B Difficulty: Easy AACSB: Application of knowledge 8) In an inflationary environment, then over time A) a specific tariff will tend to raise more revenue than an ad valorem tariff. B) an ad valorem tariff will tend to raise more revenue than a specific tariff. C) an optimum tariff will tend to raise more revenue than an escalating tariff. D) a tariff quota will tend to raise more revenue than a specific tariff. E) an import quota would raise more revenue than a specific tariff. Answer: B Difficulty: Easy AACSB: Application of knowledge
2 Copyright © 2022 Pearson Education, Inc.
9) The purpose of tariffs is A) to provide revenue and to protect particular domestic sectors. B) to limit the quantity of exports. C) to provide subsidies to domestic producers. D) to diversity domestic consumers' choices of goods and services. E) to lower the cost of shipping goods to a country. Answer: A Difficulty: Easy AACSB: Application of knowledge 10) A specific tariff provides home producers more protection when A) the home market buys cheaper products rather than expensive products. B) it is applied to a commodity with many grade variations. C) the home demand for a good is elastic with respect to price changes. D) it is levied on manufactured goods rather than primary products. E) the home supply outnumbers the foreign imports. Answer: A Difficulty: Easy AACSB: Application of knowledge 11) Examples of nontariff barriers include the following EXCEPT A) tariffs for the long haul. B) import quotas. C) export restraints. D) export subsidies. Answer: A Difficulty: Easy AACSB: Application of knowledge 12) What is a TRUE statement concerning the imposition in the U.S. of a tariff on cheese? A) It lowers the price of cheese domestically. B) It raises the price of cheese internationally. C) It raises revenue for the government. D) It will always result in retaliation from abroad. E) It leads to higher domestic demand for cheese. Answer: C Difficulty: Easy AACSB: Application of knowledge
3 Copyright © 2022 Pearson Education, Inc.
13) Which of the following statements is TRUE? A) As the price of the good increases, Home consumers demand less, while Home producers supply more, so that the demand for imports declines. B) As the price of the good increases, Home consumers demand more, while Home producers supply less, so that the demand for imports increases. C) As the price of the good decreases, Home consumers demand less, while Home producers supply more, so that the demand for imports declines. D) As the price of the good decreases, Home consumers demand more, while Home producers supply less, so that the demand for imports declines. Answer: A Difficulty: Easy AACSB: Application of knowledge 14) Which of the following statements is TRUE? A) As the price of the good rises, Foreign producers supply more while Foreign consumers demand less, so that the supply available for export rises. B) As the price of the good rises, Foreign producers supply less while Foreign consumers demand more, so that the supply available for export decreases. C) As the price of the good decreases, Foreign producers supply more while Foreign consumers demand less, so that the supply available for export rises. D) As the price of the good decreases, Foreign producers supply less while Foreign consumers demand more, so that the supply available for export rises. Answer: A Difficulty: Easy AACSB: Application of knowledge 15) The equilibrium world price is NOT where A) Home demand + Home supply = Foreign supply + Foreign demand. B) Home import demand equals Foreign export supply. C) World supply equals world demand. D) Home demand + Foreign demand = Home supply + Foreign supply E) Home demand - Home supply = Foreign supply - Foreign demand. Answer: A Difficulty: Easy AACSB: Application of knowledge 16) A tariff ________ the price in Home while ________ the price in Foreign, therefore the volume traded ________. A) raises; lowering; declines B) lowers; increasing; increases C) raises; increasing; stays the same D) lowers; lowers; stays the same Answer: A Difficulty: Easy AACSB: Application of knowledge
4 Copyright © 2022 Pearson Education, Inc.
17) Suppose the United States eliminates its tariff on ball bearings used in producing exports. Ball bearing prices in the United States would be expected to A) increase, and the foreign demand for U.S. exports would increase. B) increase, and the foreign demand for U.S. exports would decrease. C) decrease, and the foreign demand for U.S. exports would increase. D) decrease, and the foreign demand for U.S. exports would decrease. E) decrease, and the foreign demand would be unchanged. Answer: C Difficulty: Easy AACSB: Application of knowledge 18) A lower tariff on imported steel would most likely benefit A) foreign producers at the expense of domestic consumers. B) domestic manufacturers of steel. C) domestic consumers of steel. D) workers in the steel industry. E) foreign consumers of steel. Answer: C Difficulty: Easy AACSB: Application of knowledge 19) The tariff levied in a "large country" (Home), lowers the world price of the imported good. This causes A) foreign consumers to demand less of the good on which was levied a tariff. B) domestic demand for imports to decrease. C) domestic demand for imports to increase. D) foreign suppliers to produce less of the good on which was levied a tariff. E) no change in the foreign price of the good it imports. Answer: D Difficulty: Easy AACSB: Application of knowledge 20) When a country is small, a tariff it imposes A) cannot lower the foreign price of the good it imports. B) can lower the foreign price of the good it imports. C) can raise the foreign price of the good it imports. D) cannot raise the price of the goods it imports. Answer: A Difficulty: Easy AACSB: Application of knowledge
5 Copyright © 2022 Pearson Education, Inc.
21) Tariffs are NOT defended on the grounds that they A) improve the terms of trade of foreign nations. B) protect jobs and reduce unemployment. C) promote growth and development of young industries. D) prevent over-dependence of a country on only a few industries. E) protect domestic producers from foreign low prices. Answer: A Difficulty: Easy AACSB: Application of knowledge 22) The effective rate of protection measures A) the "true" ad valorem value of a tariff. B) the quota equivalent value of a tariff. C) the efficiency with which the tariff is collected at the customhouse. D) the protection given by the tariff to domestic value added. E) the difference between domestic and foreign prices of the import. Answer: D Difficulty: Easy AACSB: Application of knowledge 23) If the tariff on computers is not changed, but domestic computer producers shift from domestically produced semiconductors to imported components, then the effective rate of protection in the computer industry will A) increase. B) decrease. C) remain the same. D) depend on whether computers are PCs or "Supercomputers." E) no longer apply. Answer: A Difficulty: Easy AACSB: Application of knowledge 24) If the tariff on computers is not changed, but the government then adds hitherto nonexistent tariffs on imported semiconductor components, then the effective rate of protection in the computer industry will A) increase. B) decrease. C) remain the same. D) depend on whether computers are PCs or "Supercomputers." E) no longer apply. Answer: B Difficulty: Easy AACSB: Application of knowledge
6 Copyright © 2022 Pearson Education, Inc.
25) When a government allows raw materials and other intermediate products to enter a country duty free, this generally results in a(an) A) effective tariff rate less than the nominal tariff rate. B) nominal tariff rate less than the effective tariff rate. C) rise in both nominal and effective tariff rates. D) fall in both nominal and effective tariff rates. E) rise in only the effective tariff rate. Answer: B Difficulty: Easy AACSB: Application of knowledge 26) As globalization tends to increase the proportion of imported inputs relative to domestically supplied components A) the nominal tariff automatically increases. B) the rate of (effective) protection automatically decreases. C) the nominal tariff automatically decreases. D) the rate of (effective) protection automatically increases. E) the amount of tariffs levied increases. Answer: D Difficulty: Easy AACSB: Application of knowledge 27) Trade policies aimed at promoting economic development often lead to rates of effective protection A) much higher than the tariff rates themselves. B) much lower than the tariff rates themselves. C) the same as the tariff rates themselves. D) unpredictable with respect to the tariff rates themselves. Answer: A Difficulty: Easy AACSB: Application of knowledge 28) The most vocal political pressure for tariffs is generally made by A) consumers lobbying for export tariffs. B) consumers lobbying for import tariffs. C) consumers lobbying for lower import tariffs. D) producers lobbying for export tariffs. E) producers lobbying for import tariffs. Answer: E Difficulty: Easy AACSB: Application of knowledge
7 Copyright © 2022 Pearson Education, Inc.
29) Tariff rates on products imported into the U.S. A) have dropped substantially over the past 50 years. B) were prohibited by the Constitution. C) reached an all time high in 2002. D) have risen steadily since 1920. E) were the government's main source of income in 2006. Answer: A Difficulty: Easy AACSB: Application of knowledge
30) Refer to above figure. In the absence of trade, how many Widgets does this country produce? Answer: 60 Difficulty: Easy AACSB: Application of knowledge 31) Refer to above figure. In the absence of trade, how many Widgets does this country consume? Answer: 60 Difficulty: Easy AACSB: Application of knowledge 32) Refer to above figure. With free trade and no tariffs, what is the quantity of Widgets imported? Answer: 90 Difficulty: Easy AACSB: Application of knowledge 33) Refer to above figure. With a specific tariff of $3 per unit, what is the quantity of Widgets imported? Answer: 40 Difficulty: Easy AACSB: Application of knowledge 8 Copyright © 2022 Pearson Education, Inc.
34) Refer to above figure. With free trade and no tariffs, what is the quantity of Widgets produced domestically? Answer: 10 Difficulty: Easy AACSB: Application of knowledge 35) Refer to above figure. The lowest specific tariff which would be considered prohibitive is ________. Answer: $5 Difficulty: Easy AACSB: Application of knowledge 36) Refer to above figure. With a specific tariff of $3 per unit, what is the quantity of Widgets produced domestically? Answer: 40 Difficulty: Easy AACSB: Application of knowledge 37) Refer to above figure. With free trade and no tariffs, what is the quantity of Widgets consumed domestically? Answer: 100 Difficulty: Easy AACSB: Application of knowledge 38) Refer to above figure. With a specific tariff of $3 per unit, what is the quantity of Widgets consumed domestically? Answer: 80 Difficulty: Easy AACSB: Application of knowledge 39) Some argue that tariffs always hurt the imposing country's economic welfare, and are typically designed to shift resources from one sector to another, protected or preferred one, within an economy. Find and discuss a counter example to this argument. Answer: The optimum tariff is theoretically a first-best trade policy. Difficulty: Moderate AACSB: Application of knowledge
9 Copyright © 2022 Pearson Education, Inc.
40) The effective rate of protection is a weighted average of nominal tariffs and tariffs on imported inputs. It has been noted that in most industrialized countries, the nominal tariffs on raw materials or intermediate components or products are lower than on final-stage products meant for final markets. Why would countries design their tariff structures in this manner? Who tends to be helped, and who is harmed by this cascading tariff structure? Answer: The cascading tariff structure is probably the result of systematic lobbying on the part of manufacturing interests and lobbies to lower costs of production (in terms of imported inputs). The end result is in fact to create effective rates of protection for downstream, or final manufacturing processes that are often much higher than nominal tariffs on these products. An important group, which is hurt by this are exporters of raw materials and components in developing countries. Difficulty: Moderate AACSB: Application of knowledge 41) What is the purpose of tariffs? Answer: The true purpose of tariffs has usually been twofold: to provide revenue and to protect particular domestic sectors. Difficulty: Easy AACSB: Application of knowledge 42) What are the potential problems with trying to calculate the rate of protection? Answer: First, if the small-country assumption is not a good approximation, part of the effect of a tariff will be to lower foreign export prices rather than to raise domestic prices. This effect of trade policies on foreign export prices is sometimes significant. The second problem is that tariffs may have very different effects on different stages of production of a good. Difficulty: Easy AACSB: Application of knowledge
10 Copyright © 2022 Pearson Education, Inc.
9.2
Costs and Benefits of a Tariff
1) Which of the following statements is TRUE? A) A tariff raises the price of a good in the importing country and lowers it in the exporting country. B) A tariff creates gain for consumers in the importing country and loss for consumers in the exporting country. C) A tariff creates loss for producers in the importing country and gain for producers in the exporting country. D) The government imposing the tariff loses revenue. Answer: A Difficulty: Easy AACSB: Application of knowledge 2) If a good is imported into (large) country H from country F, then the imposition of a tariff in country H A) raises the price of the good in both countries (the "Law of One Price"). B) raises the price in country H and cannot affect its price in country F. C) lowers the price of the good in both countries. D) lowers the price of the good in H and could raise it in F. E) raises the price of the good in H and lowers it in F. Answer: E Difficulty: Easy AACSB: Application of knowledge 3) If a good is imported into (small) country H from country F, then the imposition of a tariff In country H A) raises the price of the good in both countries (the "Law of One Price"). B) raises the price in country H and does not affect its price in country F. C) lowers the price of the good in both countries. D) lowers the price of the good in H and could raise it in F. E) raises the price of the good in H and lowers it in F. Answer: B Difficulty: Easy AACSB: Application of knowledge 4) If a small country imposes a tariff, then A) the producers must suffer a loss. B) the consumers must suffer a loss. C) the government revenue must suffer a loss. D) the demand curve must shift to the left. E) the world price on that item will shift. Answer: B Difficulty: Easy AACSB: Application of knowledge
11 Copyright © 2022 Pearson Education, Inc.
5) Consumer surplus measures A) the amount a consumer gains or the difference between the actual price and what consumers would have been willing to pay. B) the amount a consumer loses or the difference between the actual price and what consumers would have been willing to pay. C) the amount a consumer gains or the difference between the world price and the domestic price. D) the amount a consumer loses or the difference between the price without tariffs and the price with tariffs. Answer: A Difficulty: Easy AACSB: Application of knowledge 6) Consumer surplus is equal to the area A) under the demand curve and above the price. B) above the supply curve and under the price. C) above the demand curve and below the price. D) under the supply curve and above the price. Answer: A Difficulty: Easy AACSB: Application of knowledge 7) Producer surplus measures A) the amount a producer gains or the difference between the actual price and what producers would have been willing to sell. B) the amount a producer loses or the difference between the actual price and what producers would have been willing to sell. C) the amount a producer gains or the difference between the world price and the domestic price. D) the amount a producer loses or the difference between the price without tariffs and the price with tariffs. Answer: A Difficulty: Easy AACSB: Application of knowledge 8) Producer surplus is equal to the area A) above the supply curve and below the price. B) under the demand curve and above the price. C) above the demand curve and below the price. D) under the supply curve and above the price. Answer: A Difficulty: Easy AACSB: Application of knowledge
12 Copyright © 2022 Pearson Education, Inc.
9) The imposition of tariffs on imports results in deadweight (triangle) losses. These are A) production and consumption distortion effects. B) redistribution effects. C) revenue effects D) efficiency effects. E) distortion of incentives. Answer: E Difficulty: Easy AACSB: Application of knowledge 10) The deadweight loss of a tariff A) is a social loss because it promotes inefficient use of national resources. B) is a social loss because it reduces the revenue of the government. C) is not a social loss because it merely redistributes revenue from one sector to another. D) is not a social loss because it is paid for by rich corporations. E) is not a social loss because it aids domestic consumers. Answer: A Difficulty: Easy AACSB: Application of knowledge 11) The main redistribution effect of a tariff is the transfer of income from A) domestic producers to domestic buyers. B) domestic buyers to domestic producers. C) domestic producers to domestic government. D) domestic government to domestic consumers. E) foreign producers to domestic consumers. Answer: B Difficulty: Moderate AACSB: Application of knowledge 12) The principle benefit of tariff protection goes to A) domestic consumers of the good produced. B) foreign consumers of the good produced. C) domestic producers of the good produced. D) foreign producers of the good produced. E) the domestic government. Answer: C Difficulty: Moderate AACSB: Application of knowledge
13 Copyright © 2022 Pearson Education, Inc.
13) In the country levying the tariff, the tariff will A) increase both consumer and producer surplus. B) decrease both the consumer and producer surplus. C) decrease consumer surplus and increase producer surplus. D) increase consumer surplus and decrease producer surplus. E) decrease consumer surplus but leave producers surplus unchanged. Answer: C Difficulty: Easy AACSB: Application of knowledge 14) The imposition of tariffs will help a nation attain which of the following goals? A) decreased domestic consumer prices B) increased domestic employment C) increased amount and variety of goods available for consumers D) increased competition between domestic and foreign producers E) gains for domestic producers Answer: E Difficulty: Easy AACSB: Application of knowledge 15) Should the home country be "large" relative to its trade partners, its imposition of a tariff on imports would lead to an increase in domestic welfare if the terms of the trade rectangle exceed the sum of the A) revenue effect plus redistribution effect. B) protective effect plus revenue effect. C) consumption effect plus redistribution effect. D) production distortion effect plus consumption distortion effect. E) terms of trade gain. Answer: D Difficulty: Easy AACSB: Application of knowledge 16) A policy of tariff reduction in the computer industry is A) in the interest of the United States as a whole and in the interest of computer producing regions of the country. B) in the interest of the United States as a whole but not in the interest of computer producing regions of the country. C) not in the interest of the United States as a whole but in the interests of computer producing regions of the country. D) not in the interest of the United States as a whole and not in the interests of computer consumers. E) not in the interest of the United States as a whole but in the interests of foreign computer producers. Answer: B Difficulty: Moderate AACSB: Application of knowledge 14 Copyright © 2022 Pearson Education, Inc.
17) The fact that industrialized countries levy very low or no tariff on raw materials and semi processed goods A) helps developing countries export manufactured products. B) has no effect on developing country exports. C) hurts developing country efforts to export manufactured goods. D) hurts developing country efforts to export raw materials. E) does not affect industrialized countries' exports. Answer: C Difficulty: Moderate AACSB: Application of knowledge 18) The net cost of a tariff is equal to A) consumer loss - producer gain - government revenue. B) consumer loss + producer gain + government revenue. C) consumer loss + producer gain - government revenue. D) consumer loss - producer gain + government revenue. Answer: A Difficulty: Easy AACSB: Application of knowledge 19) The change in the economic welfare of a country associated with an increase in a tariff equals A) efficiency loss - terms of trade gain. B) efficiency gain - terms of trade loss. C) efficiency loss + tax revenue gain. D) efficiency loss + tax revenue gain + terms of trade gain. E) efficiency loss - tax revenue gain. Answer: A Difficulty: Easy AACSB: Application of knowledge 20) Over the 2018-2019 period, the Trump administration A) increased tariffs targeted both at specific goods as well as specific trading partners. B) lowered tariffs targeted both at specific goods as well as specific trading partners. C) kept tariffs the same without any increase or decrease for U.S. major trading partners. D) removed tariffs on all imports to the U.S. Answer: A Difficulty: Easy AACSB: Application of knowledge
15 Copyright © 2022 Pearson Education, Inc.
21) Over the 2018-2019 period, the Trump administration's new tariffs on some specific goods led to A) higher prices for imported goods and a substantial annual cost increase for U.S. households. B) lower prices for imported goods and a substantial annual cost decrease for U.S. households. C) higher prices for imported goods and a substantial annual cost decrease for U.S. households. D) lower prices for imported goods and a substantial annual cost increase for U.S. households. Answer: A Difficulty: Easy AACSB: Application of knowledge 22) In response to the Trump trade war, all of the U.S. major trading partners responded to the tariffs on their exports with A) tariffs of their own targeting U.S. exports. B) import quota on U.S. goods. C) trade embargo on U.S. goods. D) export subsidies on their exports to the U.S. Answer: A Difficulty: Easy AACSB: Application of knowledge 23) Comparing consumer loss and producer gain, the Trump trade war A) generated an overall efficiency loss. B) generated an overall efficiency gain. C) did not affect U.S. social welfare. D) made U.S. farmers better off and decreased the number of farm bankruptcies in 2019. Answer: A Difficulty: Easy AACSB: Application of knowledge 24) Under "chicken tax," a retaliation by U.S. President Johnson's administration against a tariff on U.S. chicken exports imposed by Western Europe in the early 1960s, the U.S. imposed A) a 25 percent tariff on imports of light commercial truck vehicles. B) a 25 percent tariff on imports of chicken from Europe. C) a 25 percent tariff on imports of agricultural products from Europe. D) a 25 percent tariff on imports of heavy commercial truck vehicles. Answer: A Difficulty: Easy AACSB: Application of knowledge
16 Copyright © 2022 Pearson Education, Inc.
25) The tariff known as "chicken tax" ended up hurting the US producers including A) Ford. B) Volkswagen. C) Isuzu. D) Mazda. E) Mercedes-Benz. Answer: A Difficulty: Easy AACSB: Application of knowledge 26) Which of the following statements is NOT true? A) The Japanese truck producers responded to the chicken tax by stopping exports to the United States. B) The Japanese truck producers responded to the chicken tax by opening production facilities in the United States. C) Subaru built the light commercial trucks in the United States. D) Subaru bolted two plastic seats to the open bed of its pickup truck exported to the U.S. to be classified as a passenger vehicle to avoid the tariff. Answer: A Difficulty: Easy AACSB: Application of knowledge 27) To get around the 25 percent tariff on light commercial truck vehicles, Ford A) converted commercial vans produced in Europe into passenger vehicles prior to shipping to the U.S. B) lobbied to lower the tariff to 2.5 percent. C) removed the rear seats, rear windows, seat belts prior to shipping to the U.S. D) stopped producing light commercial trucks outside the U.S. border. Answer: A Difficulty: Easy AACSB: Application of knowledge 28) The two deadweight triangles are the Consumption distortion and Production distortion losses. It is easy to understand why the Consumption distortion constitutes a loss for society. After all, it raises the prices of goods to consumers, and even causes some consumers to drop out of the market altogether. It seems paradoxical that the Production distortion is considered an equivalent burden on society. After all, in this case, profits increase, and additional production (with its associated employment) comes on line. This would seem to be an offset rather than an addition to the burden or loss borne by society. Explain why the Production distortion is indeed a loss to society, and what is wrong with the logic that leads to the apparent paradox. Answer: The Production Distortion represents an inefficient shift of society's resources to produce a good, which it could not sell profitably at world prices. Since (with full employment assumed) these resources were formerly used to produce export goods, which could compete profitably, the net result is a loss in real income to the country. Difficulty: Moderate AACSB: Application of knowledge 17 Copyright © 2022 Pearson Education, Inc.
29) Refer to above figure. In the absence of trade, what is the country's consumer surplus? Answer: $180 Difficulty: Easy AACSB: Application of knowledge 30) Refer to above figure. In the absence of trade, what is the country's producer surplus? Answer: $180 Difficulty: Easy AACSB: Application of knowledge 31) Refer to above figure. The loss of Consumer Surplus due to the tariff equals ________. Answer: $230 Difficulty: Easy AACSB: Application of knowledge 32) Refer to above figure. In the absence of a tariff and in the presence of trade, what is the country's consumer surplus? Answer: $550 Difficulty: Easy AACSB: Application of knowledge 33) Refer to above figure. Given a tariff of $3 per unit, what is the country's consumer surplus? Answer: $320 Difficulty: Easy AACSB: Application of knowledge 34) Refer to above figure. What is the amount of efficiency loss resulting from imposition of the tariff? Answer: $75 Difficulty: Easy AACSB: Application of knowledge
18 Copyright © 2022 Pearson Education, Inc.
35) Refer to above figure. What is the amount of government revenue resulting from imposition of the tariff? Answer: $120 Difficulty: Easy AACSB: Application of knowledge 36) Discuss the winners and losers of the Trump trade war. Answer: Over the 2018-19 period, the Trump administration drastically increased tariffs targeted both at specific goods as well as specific trading partners. As a result, the higher prices for imported goods due to the new tariffs represented a substantial annual cost increase for U.S. households, for government services that imported those goods, for US firms that imported goods as capital investment. The tariffs also led to higher prices for U.S. produced goods that compete with the imported goods hit by the tariff. Although firms producing steel, aluminum, solar panels, and washing machines — as well as others producing products that completed directly with Chinese imports were winners from the trade war, but not enough to offset the loss to consumers and the U.S. economy. Furthermore, all of the United States' major trading partners responded to the tariffs on their exports with tariffs of their own targeting U.S. exports. In this case, U.S. farmers and ranchers were especially hard hit. Difficulty: Easy AACSB: Application of knowledge 37) What was the "chicken tax"? How did it end up hurting one of the "big three" U.S. producers who had intensively lobbied to maintain the tariff in the first place? Answer: The tariff known as chicken tax was a retaliation by U.S. President Lyndon Johnson's administration against a tariff on U.S. chicken exports imposed by Western Europe in the early 1960s. The U.S. then imposed a 25 percent tariff on imports of light commercial truck vehicles that affected German producers like Volkswagen and Japanese truck producers over the years. However, the latest company to be hit by the consequences of the tariff is Ford, one of those "big three" U.S. producers! Ford produces a small commercial van in Europe, and started selling these vehicles in the U.S. in 2009. To get around the 25 percent tariff, Ford installs rear windows, rear seats, and seat belts prior to shipping the vehicles to the U.S. to convert them into passenger vehicles, which are subject to the much lower 2.5 percent tariff. Upon arrival in Baltimore, Maryland, the rear seats are promptly removed and the rear windows replaced with metal panels–before delivery to the Ford dealers. The U.S. Supreme Court denied a petition to hear the case of Ford in 2020, and Ford paid a $196 million penalty to U.S. Customs. Difficulty: Easy AACSB: Application of knowledge
19 Copyright © 2022 Pearson Education, Inc.
9.3
Other Instruments of Trade Policy
1) Beside tariffs, other instruments of trade policy include the following EXCEPT A) embargoes sanctions. B) export subsidies. C) import quotas. D) voluntary export restraints. E) local content requirements. Answer: A Difficulty: Easy AACSB: Application of knowledge 2) Payment to a firm or individual that ships a good abroad is called A) an export subsidy. B) an import quota. C) a tariff. D) a voluntary export restraint. Answer: A Difficulty: Easy AACSB: Application of knowledge 3) An export subsidy is A) a payment to a firm or individual that ships a good abroad. B) a fee that is charged to a country that ships goods to the U.S. C) a payment made to a foreign government in return for preferential trade treatment. D) illegal in the U.S. but is fairly common in the rest of the world. E) a limit on the quantity of a good or service that can be sold abroad. Answer: A Difficulty: Easy AACSB: Application of knowledge 4) An export subsidy ________ prices in the exporting country while ________ them in the importing country. A) raises; lowering B) lowers; raising C) raises; raising D) lowers; lowering Answer: A Difficulty: Easy AACSB: Application of knowledge
20 Copyright © 2022 Pearson Education, Inc.
5) In the exporting country, an export subsidy will A) help consumers and raise the overall economic welfare of the exporting country. B) hurt consumers but raise the overall economic welfare of the exporting country. C) hurt consumers and lower the overall economic welfare of the exporting country. D) help consumers but lower economic welfare of the exporting country. E) help consumers and have no effect on the economic welfare of the exporting country. Answer: C Difficulty: Easy AACSB: Application of knowledge 6) An export subsidy differs from a tariff in each of the following ways EXCEPT A) a tariff generates revenue. B) a tariff is applied to imports. C) a tariff results in an efficiency loss. D) a tariff is a tax. E) a tariff discourages imports. Answer: C Difficulty: Easy AACSB: Application of knowledge 7) The European Union's Common Agricultural Policy (CAP) is, in effect A) a tariff imposed on agricultural exports. B) a tariff imposed on agricultural imports. C) a subsidy that reduces the cost of agricultural exports. D) a subsidy that increases the cost of agricultural exports. E) a quota that limits production of agricultural goods by EU nations. Answer: C Difficulty: Easy AACSB: Application of knowledge 8) Which of the following statements is TRUE? A) When a government's export subsidies harm producers in other countries, WTO allows the impacted countries to impose countervailing duties against the subsidizing countries. B) When a government's export subsidies harm producers in other countries, WTO do not interfere and let the countries find solutions together. C) When a government's export subsidies harm producers in other countries, governments of the impacted countries can together impose trade sanctions against the subsidizing country. D) When a government's export subsidies harm producers in other countries, governments of the impacted countries can stop trading with the subsidizing country. Answer: A Difficulty: Easy AACSB: Application of knowledge
21 Copyright © 2022 Pearson Education, Inc.
9) Which of the following statements is NOT true? A) In 2004, WTO ruled in favor of both United States and the European Union, allowing them to impose countervailing duties due to actionable export subsidies. B) In 2004, the United States filed a case with the WTO complaining that European-owned Airbus had received favorable loan agreements for the development of their new A380 and A350 jumbo jets. C) In 2004, The European Union filed a claim that Boeing received tax rebates and favorable contracting terms from the U.S. government. D) In 2004, the United States and the European Union both claimed to WTO that the assistance received by the airplane manufacturers Airbus and Boeing amounted to actionable export subsidies. Answer: A Difficulty: Easy AACSB: Application of knowledge 10) In 2019, the United States imposed tariffs on A) European wines and specialty food items. B) European whiskey. C) European nuts. D) European tobacco. Answer: A Difficulty: Easy AACSB: Application of knowledge 11) In 2019, the European Union retaliated with tariffs on A) U.S. whiskey, nuts, and tobacco. B) U.S. wines. C) U.S. specialty food items. D) U.S. chicken. Answer: A Difficulty: Easy AACSB: Application of knowledge 12) In 2021, the United States and the European Union A) finally agreed to end their 17-year trade dispute regarding the production subsidies received by Boeing and Airbus. B) still continued their 17-year trade dispute regarding the production subsidies received by Boeing and Airbus. C) filed new cases with WTO regarding the production subsidies received by Boeing and Airbus. D) disagreed to remove tariffs and address the favorable financial treatments received by Boeing and Airbus. Answer: A Difficulty: Easy AACSB: Application of knowledge
22 Copyright © 2022 Pearson Education, Inc.
13) An import quota is A) a direct restriction on the quantity of some good that may be imported. B) a payment to a firm or individual that ships a good abroad. C) a fee that is charged to a country that imports goods from the U.S. D) a limit on the quantity of a good or service that can be sold abroad. Answer: A Difficulty: Easy AACSB: Application of knowledge 14) An import quota A) always raises the domestic price of the imported goods. B) always lowers the domestic price of the imported goods. C) has no impact on the domestic price of the imported goods. D) limits import without raising the domestic price of the imported goods. Answer: A Difficulty: Easy AACSB: Application of knowledge 15) An important difference between tariffs and quotas is that tariffs A) raise the price of the good. B) generate tax revenue for the government. C) stimulate international trade. D) help domestic producers. E) are paid by foreign producers. Answer: B Difficulty: Easy AACSB: Application of knowledge 16) An import quota is similar to a ________ in its effect on imports, EXCEPT that an import quota ________. A) tariff; does not generate revenue B) tariff; generates revenue C) subsidy; does not generate revenue D) subsidy; generates revenue E) tariff; does not result in an efficiency loss. Answer: A Difficulty: Easy AACSB: Application of knowledge
23 Copyright © 2022 Pearson Education, Inc.
17) Import license holders are able to buy imports and resell them at a higher price in the domestic market. The profits received by the holders of import licenses are known as A) quota rents. B) import quota. C) license margin. D) opportunity cost. Answer: A Difficulty: Easy AACSB: Application of knowledge 18) The U.S. sugar quota A) restricts the imports of both raw sugar as well as refined sugar to the U.S. market. B) promotes imports of both raw sugar as well as refined sugar to the U.S. market. C) imposes fees on the imports of both raw sugar as well as refined sugar to the U.S. market. D) removes limits of the imports of both raw sugar as well as refined sugar to the U.S. market. Answer: A Difficulty: Easy AACSB: Application of knowledge 19) The U.S. sugar quota import restrictions A) raised the U.S. domestic price above the world price. B) lowered the U.S. domestic price above the world price. C) lowered U.S. sugar production and increased U.S. sugar consumption. D) benefited U.S. consumers. Answer: A Difficulty: Easy AACSB: Application of knowledge 20) The U.S. sugar quota A) generates government revenue. B) results in net welfare benefits to the U.S. economy. C) results in benefits to sugar producers that exceed the cost to consumers. D) results in costs to consumers that exceed the benefits to sugar producers. E) does not result in an efficiency loss. Answer: D Difficulty: Easy AACSB: Application of knowledge 21) A voluntary export restraint is A) a quota on trade imposed from the exporting country's side instead of the importer's. B) a quota on trade imposed from the importing country's side instead of the exporter's. C) generally imposed at the request of the exporter and agreed to by the importer to forestall other trade restrictions. D) an agreement to voluntarily remove all quotas imposed on trade between two countries. Answer: A Difficulty: Easy AACSB: Application of knowledge 24 Copyright © 2022 Pearson Education, Inc.
22) Which of the following are examples of goods that have been subject to voluntary export restraints? A) Japanese cars and Chinese solar panels B) Belgian chocolates and French wines C) French wines and cheeses D) Japanese sushi and German cars E) Taiwanese electronics and Canadian barley Answer: A Difficulty: Easy AACSB: Application of knowledge 23) A local content requirement (also called rules of origin) A) is a regulation that requires some specified fraction of a final good to be produced domestically. B) provides no protection for domestic producers. C) produces government revenue and quota rents. D) places a strict limit on imports. Answer: A Difficulty: Easy AACSB: Application of knowledge 24) Local content requirements are A) common in free trade areas such as NAFTA-USMCA to address differences in each country's tariffs with trading partners outside the region. B) not common in the automotive sector. C) not common in free trade areas such as NAFTA-USMCA to address differences in each country's tariffs with trading partners outside the region. D) common to impose additional restrictions on vehicles assembled in Mexico intended for sale in the U.S. and vehicles assembled and sold in the U.S. Answer: A Difficulty: Easy AACSB: Application of knowledge 25) Which of the following statements is TRUE? A) The U.S. Buy American Act triggered similar protectionist clauses from other foreign governments. B) The U.S. Buy American Act saved costs for many U.S. government agencies' infrastructure projects. C) The U.S. Buy American Act lowered the cost to U.S. taxpayers. D) The U.S. Buy American Act allowed access to cheaper imported immediate goods. Answer: A Difficulty: Easy AACSB: Application of knowledge
25 Copyright © 2022 Pearson Education, Inc.
26) Which of the followings is NOT true? A) Export credit subsidies is a government's policy tool to discourage exports. B) Export credit subsidies act like an export subsidy in the form of a subsidized loan to the foreign buyer. C) Export credit subsidies offered by U.S. Export-Import Bank provide slightly subsidized loans to aid exports. D) Export credit subsidies facilitate the sale of a good from the exporting country to the overseas country. Answer: A Difficulty: Easy AACSB: Application of knowledge 27) The process where the purchases by the government or strongly regulated firms are directed toward domestically produced goods even when these goods are more expensive than imports is called A) national procurement. B) nationalism. C) national protectionism. D) national sovereignty. Answer: A Difficulty: Easy AACSB: Application of knowledge 28) Red-tape barriers happen when a government wants to restrict imports by A) twisting normal health, safety, and customs procedures to place substantial obstacles in the way of trade. B) giving credit to foreign buyers to raise the volume of exports. C) imposing more administrative paperwork and costs on domestic producers. D) giving credit to domestic buyers who buy domestically produced goods. Answer: A Difficulty: Easy AACSB: Application of knowledge 29) Economic theory in general, and trade theory in particular are replete with equivalencies. For example, it is argued that for any specific tariff one can find an equivalent ad valorem tariff; and that for any quota one can calculate a tariff equivalent. Discuss conditions or situations under which a specific and an ad valorem tariff are not equivalent. Discuss conditions or situations when a tariff and a quota are not equivalent. Answer: E.g., during a period of price inflation, an ad valorem tariff would become increasingly more effective. The government does not receive any of the quota revenues, unless the import licenses are sold or auctioned. Difficulty: Moderate AACSB: Application of knowledge
26 Copyright © 2022 Pearson Education, Inc.
30) What is the difference between a tariff and a quota? Answer: The difference between a quota and a tariff is that with a quota, the government receives no revenue. When a quota instead of a tariff is used to restrict imports, the sum of money that would have appeared with a tariff as government revenue is collected by whoever receives the import licenses. Difficulty: Easy AACSB: Application of knowledge 31) Explain the misconception that import quotas somehow limit imports without raising domestic prices. Answer: An import quota always raises the domestic price of the imported good. When imports are limited, the immediate result is that at the initial price, the demand for the good exceeds domestic supply plus imports. This causes the price to be bid up until the market clears. In the end, an import quota will raise domestic prices by the same amount as a tariff that limits imports to the same level. Difficulty: Easy AACSB: Application of knowledge 32) Explain the winners and losers from the U.S. sugar import quota and why there is little effective opposition from the losers. Answer: U.S. sugar producers gain from the higher sugar prices. U.S. consumers and sugarusing food producers lose from paying higher prices for sugar and products using sugar as ingredient. The cost to consumers well exceeds the benefits to a small group of sugar producers. However, the yearly consumer loss amounts to "only" $11 per capita, or a little under $30 for a typical household. Therefore, the average American voter is unaware that the sugar quota exists, and so there is little effective opposition. On the other hand, the sugar producers are organized, and they donated more than $4.5 million in the 2012 congressional races. The American Sugar Alliance spent an additional $3 million on lobbying expenses in the 12-month period leading up to the 2013 congressional vote on the U.S. farm bill (which reauthorizes the restrictions on U.S. imports of sugar). The sugar restrictions were reauthorized in 2013 and then again in the 2018 farm bill. Difficulty: Easy AACSB: Application of knowledge 33) What is the difference between voluntary export restraint (VER) and import quota? Answer: From an economic point of view, a voluntary export restraint is exactly like an import quota where the licenses are assigned to foreign governments and is therefore very costly to the importing country. A VER is always more costly to the importing country than a tariff that limits imports by the same amount. The difference is that what would have been revenue under a tariff becomes rents earned by foreigners under the VER, so that the VER clearly produces a loss for the importing country. Difficulty: Easy AACSB: Application of knowledge
27 Copyright © 2022 Pearson Education, Inc.
34) Discuss some of the impacts of the U.S. Buy American Act. Answer: The U.S. Buy American Act, originally passed in 1933, requires those government agencies to purchase many specific inputs from U.S. firms - unless the foreign bid for that input is more than 25 percent below the lowest bid from a U.S. firm. This results in a cost increase well below the 25 percent maximum. For the construction of the new Bay Bridge linking San Francisco and Oakland, the 23 percent difference between the Chinese bid and the lone U.S. bid for some key steel components amounted to a $400 million cost difference. The Buy American provisions therefore not only raise the costs for U.S. government agencies' projects, but also cost to U.S. taxpayers, delays in some essential projects as managers navigate the administrative paperwork to show that some key components are not available in the U.S, trigger similar protection clauses from other foreign governments, shutting out U.S. firms from those business opportunities. Difficulty: Easy AACSB: Application of knowledge 9.4
The Effects of Trade Policy: A Summary
1) An export tariff will ________ producer surplus, ________ consumer surplus, ________ government revenue, and ________ overall domestic national welfare. A) increase; decrease; increase; have an ambiguous effect on B) increase; decrease; decrease; decrease C) increase; decrease; have no effect on; have an ambiguous effect on D) increase; decrease; have no effect on; decrease E) increase; increase; decrease; have an ambiguous effect on Answer: A Difficulty: Easy AACSB: Application of knowledge 2) An export subsidy will ________ producer surplus, ________ consumer surplus, ________ government revenue, and ________ overall domestic national welfare. A) increase; decrease; increase; have an ambiguous effect on B) increase; decrease; decrease; decrease C) increase; decrease; have no effect on; have an ambiguous effect on D) increase; decrease; have no effect on; decrease E) increase; increase; decrease; have an ambiguous effect on Answer: B Difficulty: Easy AACSB: Application of knowledge
28 Copyright © 2022 Pearson Education, Inc.
3) An import quota will ________ producer surplus, ________ consumer surplus, ________ government revenue, and ________ overall domestic national welfare. A) increase; decrease; increase; have an ambiguous effect on B) increase; decrease; decrease; decrease C) increase; decrease; have no effect on; have an ambiguous effect on D) increase; decrease; have no effect on; decrease E) increase; increase; decrease; have an ambiguous effect on Answer: C Difficulty: Easy AACSB: Application of knowledge 4) A voluntary export restraint will ________ producer surplus, ________ consumer surplus, ________ government revenue, and ________ overall domestic national welfare. A) increase; decrease; increase; have an ambiguous effect on B) increase; decrease; decrease; decrease C) increase; decrease; have no effect on; have an ambiguous effect on D) increase; decrease; have no effect on; decrease E) increase; increase; decrease; have an ambiguous effect on Answer: D Difficulty: Easy AACSB: Application of knowledge 5) What do export tariff, export subsidy, import quota and voluntary export restraint polices have in common? A) They benefit producers and hurt consumers. B) They benefit both producers and consumers. C) They hurt both producers and consumers. D) They benefit consumers and hurt producers. Answer: A Difficulty: Easy AACSB: Application of knowledge
29 Copyright © 2022 Pearson Education, Inc.
9.5
Appendix to Chapter 9: Tariffs and Import Quotas in the Presence of Monopoly
1) International trade ________ monopoly power, and policies that ________ trade may therefore ________ monopoly power. A) limits; limit; increase B) increases; limit; decrease C) increases; limit; increase D) limits; limit; decrease Answer: A Difficulty: Easy AACSB: Application of knowledge 2) If a firm is the only producer of a good in a country, A) it will have little ability to raise prices if there are many foreign suppliers and free trade. B) it will be free to raise prices without fear of competition even with free trade. C) it will be unaffected and fully protected by trade policies during free trade. D) it will retain its full monopoly power even with free trade. Answer: A Difficulty: Easy AACSB: Application of knowledge 3) Under free trade, the threat of import competition A) forces the monopolist to behave like a perfectly competitive industry. B) has no impact on the monopolist's market power and behavior. C) keeps the monopolist to behave like an ordinary profit-maximizing monopolist. D) forces the monopolist to behave like in an imperfectly competitive industry. Answer: A Difficulty: Easy AACSB: Application of knowledge 4) If an import-competing firm is the only domestic producer of a good, then a transition from autarky to free trade will ________ domestic price, ________ producer surplus, ________ consumer surplus, and ________ overall domestic national welfare. A) increase; increase; increase; increase B) decrease; decrease; decrease; decrease C) decrease; decrease; increase; increase D) increase; increase; decrease; decrease E) increase; increase; decrease; increase Answer: C Difficulty: Easy AACSB: Application of knowledge
30 Copyright © 2022 Pearson Education, Inc.
5) The tariff allows the monopolist A) to raise its price, but the price is still limited by the threat of imports. B) to still charge its ordinary profit-maximizing monopoly price. C) in free trade to keep its price the same as before free trade. D) to decrease its price even lower than the world price. Answer: A Difficulty: Easy AACSB: Application of knowledge 6) If an import-competing firm is imperfectly competitive, then under free trade an export tariff will ________ domestic market price, ________ producer surplus, ________ consumer surplus, ________ government revenue, and ________ overall domestic national welfare. A) have no effect on; have no effect on; decrease; increase; decrease B) increase; have no effect on; increase; decrease; increase C) increase; have no effect on; decrease; increase; decrease D) decrease; increase; decrease; increase; decrease E) decrease; decrease; increase; decrease; have no effect on Answer: C Difficulty: Easy AACSB: Application of knowledge 7) If imports are limited by a quota, the monopolist A) will be free to raise prices without fear of competition, knowing that the domestic price of imports will rise too. B) knows that when it charges a price above world price PW, it will lose all its sales. C) will not be free to raise prices without fear of competition. D) will not be able to maximize profit by setting marginal cost equal to this new marginal revenue. Answer: A Difficulty: Easy AACSB: Application of knowledge 8) If an import-competing firm is imperfectly competitive, than under free trade an import quota will ________ domestic market price, ________ producer surplus, ________ consumer surplus, ________ government revenue, and ________ overall domestic national welfare. A) decrease; decrease; increase; decrease; have no effect on B) increase; have no effect on; decrease; increase; decrease C) have no effect on; have no effect on; decrease; increase; decrease D) increase; increase; decrease; have no effect on; decrease E) decrease; increase; decrease; increase; decrease Answer: D Difficulty: Easy AACSB: Application of knowledge
31 Copyright © 2022 Pearson Education, Inc.
9) The difference between a tariff and a quota is that A) an import quota creates more monopoly power than a tariff. B) a tariff creates more monopoly power than an import quota. C) a quota leads to higher domestic output and a lower price than a tariff that yields the same level of imports. D) an import quota creates the same monopoly power as a tariff. Answer: A Difficulty: Easy AACSB: Application of knowledge 10) Suppose an import-competing firm is imperfectly competitive. Replacement of an export tariff with an import quota that yields the same level of imports will ________ market price, ________ producer surplus, ________ consumer surplus, ________ government revenue, and ________ overall domestic national welfare. A) have no effect on; have no effect on; have no effect on; decrease; decrease B) increase; increase; increase; decrease; have an ambiguous effect on C) decrease; decrease; increase; decrease; increase D) increase; increase; decrease; decrease; decrease E) increase; have no effect on; decrease; decrease; increase Answer: D Difficulty: Easy AACSB: Application of knowledge 11) Will free trade limit or increase monopoly power? Explain. Answer: If there were no trade in the market, the domestic monopolistic firm would behave as an ordinary profit-maximizing monopolist. With free trade, this monopoly behavior is not possible. If the firm tried to charge the monopoly price PM, or indeed any price above world price PW, nobody would buy its product because cheaper imports would be available. Thus international trade puts a lid on the monopolist's price at PW, limits the monopoly power and gives it little ability to raise prices if there are many foreign suppliers and free trade. Difficulty: Easy AACSB: Application of knowledge 12) How do tariffs impact an import-competing firm who is the only domestic producer of a good? Answer: The effect of a tariff is to raise the maximum price the domestic industry can charge. However, the price is still limited by the threat of imports, as the industry still is not free to raise its price all the way to the monopoly price because consumers will still turn to imports if the price rises above the world price plus the tariff. Difficulty: Easy AACSB: Application of knowledge
32 Copyright © 2022 Pearson Education, Inc.
13) How do import quotas impact an import-competing firm who is the only domestic producer of a good? Answer: Suppose the government imposes a limit on imports, restricting their quantity to a fixed level. Then the monopolist knows that when it charges a price above world price PW, it will not lose all its sales. Instead, it will sell whatever domestic demand is at that price, minus the allowed imports. The firm protected by an import quota maximizes profit by setting marginal cost equal to this new marginal revenue. The import quotas therefore create more monopoly power for the firm. Difficulty: Easy AACSB: Application of knowledge 14) How do tariff and import quota policies impact a monopolistic domestic firm differently? Answer: When protected by a tariff, the monopolistic domestic industry behaves as if it were perfectly competitive; when protected by a quota, it clearly does not. The reason for this difference is that an import quota creates more monopoly power than a tariff. When monopolistic industries are protected by tariffs, domestic firms know that if they raise their prices too high, they will still be undercut by imports. An import quota, on the other hand, provides absolute protection: No matter how high the domestic price, imports cannot exceed the quota level. Protected by import quotas, the monopolist is now free to raise prices, knowing that the domestic price of imports will rise as well. A quota therefore leads to lower domestic output and a higher price than a tariff that yields the same level of imports. Difficulty: Moderate AACSB: Application of knowledge
33 Copyright © 2022 Pearson Education, Inc.
International Economics: Theory and Policy, 12e (Krugman) Chapter 10 The Political Economy of Trade Policy 10.1
The Case for Free Trade
1) Over the course of 2018 and 2019, the United States and China engaged in A) a trade war. B) a trade embargo. C) a trade sanction. D) no trade at all. Answer: A Difficulty: Easy AACSB: Application of knowledge 2) A series of tariff increases on both sides in which each nation asserted that it was responding to the other nation's previous actions is called A) a trade war. B) a trade embargo. C) a trade agreement. D) a trade sanction. Answer: A Difficulty: Easy AACSB: Application of knowledge 3) By the end of 2019, the average U.S. tariff on imports from China had risen from 3% to ________, while the average Chinese tariff on U.S. exports had risen from 8% to ________. A) 21%; 21% B) 11%; 11% C) 5%; 10% D) 13%; 18% Answer: A Difficulty: Easy AACSB: Application of knowledge 4) ________ countries have approached completely free trade. A) Few B) Many C) Some D) No Answer: A Difficulty: Easy AACSB: Application of knowledge
1 Copyright © 2022 Pearson Education, Inc.
5) The only modern economy with no tariffs or imports quotas is A) the city of Hong Kong. B) Taiwan. C) Singapore. D) South Korea. Answer: A Difficulty: Easy AACSB: Application of knowledge 6) Since the time of Adam Smith, economists have advocated A) free trade as an ideal toward which trade policy should strive. B) protectionism as an ideal toward which trade policy should strive. C) self-sufficient economies without trade are the best models for growth. D) central planning will help trade and economies thrive. Answer: A Difficulty: Easy AACSB: Application of knowledge 7) Which of the following statements is TRUE? A) A trade restriction, such as a tariff, leads to production and consumption distortions. B) A trade restriction, such as a tariff, leads to economic growth and prosperity. C) A trade restriction, such as a tariff, leads strong domestic industries. D) A trade restriction, such as a tariff, leads to higher productivity. Answer: A Difficulty: Easy AACSB: Application of knowledge 8) The efficiency case made for free trade is that as trade distortions such as tariffs are dismantled and removed A) government tariff revenue will decrease, and therefore national economic welfare will decrease. B) government tariff revenue will decrease, and therefore national economic welfare will increase. C) deadweight losses for producers and consumers will decrease, hence increasing national economic welfare. D) deadweight losses for producers and consumers will decrease, hence decreasing national economic welfare. E) government tariff revenue will increase, hence increasing national economic welfare. Answer: C Difficulty: Easy AACSB: Application of knowledge
2 Copyright © 2022 Pearson Education, Inc.
9) In the case of small countries in general and developing countries in particular, A) there are important gains from free trade not accounted for in conventional cost-benefit analysis. B) important gains from free trade are all accounted for in conventional cost-benefit analysis. C) there are no gains from free trade. D) the costs are higher than gains from free trade. Answer: A Difficulty: Easy AACSB: Application of knowledge 10) The opportunity to exploit economies of scale is one of the gains to be derived by removing tariffs and other trade distortions. These gains will be the result of a decrease in A) world prices of imports. B) the consumption distortion loss triangle. C) the production distortion loss triangle. D) international labor mobility. E) excessive entry and inefficient business practices. Answer: E Difficulty: Easy AACSB: Application of knowledge 11) Which of the following statements is NOT true? A) Free trade leads to inefficient scale production. B) Free trade provides entrepreneurs with an incentive to seek new ways to export or compete with imports. C) Free trade offers more opportunities for learning and innovation than are provided by a system of "managed" trade. D) Free trade eliminates production and consumption distortions. Answer: A Difficulty: Easy AACSB: Application of knowledge 12) When imports are restricted with a quota rather than a tariff, rent-seeking occurs because A) individuals and companies may waste some of the economy's productive resources to try to get import licenses. B) it is good for the economy. C) companies do not want to take advantage of the import restricting policy. D) individuals do not seek to increase their wealth without contributing to the society's wealth. Answer: A Difficulty: Easy AACSB: Application of knowledge
3 Copyright © 2022 Pearson Education, Inc.
13) Trade policies in practice are dominated by A) special-interest politics rather than by consideration of national costs and benefits. B) public interest. C) tragedy of the commons. D) trade wars' retaliation. Answer: A Difficulty: Easy AACSB: Application of knowledge 14) Any government agency attempting to pursue a sophisticated program of intervention in trade A) would probably be captured by interest groups and converted into a device for redistributing income to politically influential sectors. B) would find it easy to achieve equality in distribution of income. C) would always be willing to sacrifice small groups' special interests for the majority public interest. D) would prioritize consumers' benefits over corporates' interests. Answer: A Difficulty: Easy AACSB: Application of knowledge 15) Judging by the ongoing changes in tariff rates in major trading countries, the world has been experiencing a great A) trade liberalization. B) surge of protectionism. C) lack of progress in the trade-policy area. D) move towards regional integration. E) shift from export subsidies to specific tariffs. Answer: A Difficulty: Easy AACSB: Application of knowledge 16) Trade theory suggests that Japan would gain from a subsidy the United States provides its grain farmers if the gains to Japanese consumers of wheat products more than offsets the losses to Japanese wheat farmers. This would occur as long as Japan A) is a net importer in bilateral trade flows with the United States. B) is a net importer of wheat. C) has a comparative advantage in wheat. D) has an absolute advantage in producing wheat. E) is involved in intra-industry trade with the United States. Answer: B Difficulty: Easy AACSB: Application of knowledge
4 Copyright © 2022 Pearson Education, Inc.
17) Why have most economists since the time of Adam Smit advocated free trade? Answer: The reasons for this advocacy are not quite as simple as the idea itself. At one level, theoretical models suggest that free trade will avoid the efficiency losses associated with protection. Many economists believe free trade produces additional gains beyond the elimination of production and consumption distortions. Finally, even among economists who believe free trade is a less-than-perfect policy, many believe free trade is usually better than any other policy a government is likely to follow. Difficulty: Easy AACSB: Application of knowledge 18) Explain how protected markets lead to inefficient scale production? Answer: Protected markets limit gains from external economies of scale by inhibiting the concentration of industries; when the economies of scale are internal, they not only fragment production internationally, but by reducing competition and raising profits, they also lead too many firms to enter the protected industry. With a proliferation of firms in narrow domestic markets, the scale of production of each firm becomes inefficient. Difficulty: Easy AACSB: Application of knowledge 19) List three arguments for free trade by most international economists. Answer: (1) The conventionally measured costs of deviating from free trade are large. (2) There are other benefits from free trade that add to the costs of protectionist policies. (3) Any attempt to pursue sophisticated deviations from free trade will be subverted by the political process. Difficulty: Easy AACSB: Application of knowledge 20) It is argued that the United States would be foolish to maintain a free-trade stance in a world in which all other countries exploit child or prisoner labor, or are protectionist. On the other hand, Ricardo's classic demonstration of the sources and effects of comparative advantage cogently demonstrates that regardless of other country policy, free trade remains the first best policy for a country to follow, since it will maximize its consumption possibilities (conditional upon other country policies). Explain. Discuss the contradiction with the argument in the preceding paragraph. Answer: In the context of the Ricardian model, it is true that gains from trade are strictly a result of world terms of trade, which differ from domestic marginal rates of substitution. In such a world, the reason why foreign goods are cheap is of no concern to domestic consumers. However, in a world which allows for large-scale labor migration, ignoring labor conditions abroad may ultimately result in living standards for domestic workers to be dragged down. Difficulty: Moderate AACSB: Application of knowledge
5 Copyright © 2022 Pearson Education, Inc.
10.2
National Welfare Arguments Against Free Trade
1) Although politicians often claim that the policies are being undertaken in the interest of the nation as a whole, A) most tariffs, import quotas, and other trade policy measures are undertaken primarily to protect the income of particular interest groups. B) most tariffs, import quotas, and other trade policy measures are undertaken primarily to avoid trade deficits. C) most tariffs, import quotas, and other trade policy measures are undertaken primarily to protect the consumers and disadvantaged groups in the society. D) most tariffs, import quotas, and other trade policy measures are undertaken primarily to protect the country's sovereignty. Answer: A Difficulty: Easy AACSB: Application of knowledge 2) In some cases when the terms of trade benefits of a tariff outweigh its costs, there is a A) terms of trade argument for a tariff. B) cost-benefit argument for a tariff. C) nationalism argument for a tariff. D) infant-industry argument for a tariff. Answer: A Difficulty: Easy AACSB: Application of knowledge 3) The tariff rate that maximizes national welfare is the called A) optimum tariff. B) prohibitive tariff. C) most favored tariff. D) welfare tariff. Answer: A Difficulty: Easy AACSB: Application of knowledge 4) The prohibitive tariff is a tariff that A) is so high that it eliminates imports. B) is so high that it causes undue harm to trade-partner economies. C) is so high that it causes undue harm to import competing sectors. D) is so low that the government prohibits its use since it would lose an important revenue source. E) is so low that it causes domestic producers to leave the industry. Answer: A Difficulty: Easy AACSB: Application of knowledge
6 Copyright © 2022 Pearson Education, Inc.
5) The optimum tariff is A) the best tariff a country can obtain via a WTO negotiated round of compromises. B) the tariff, which maximizes the terms of trade gains. C) the tariff, which maximizes the difference between terms of trade gains and terms of trade loses. D) not practical for a small country due to the likelihood of retaliation. E) not practical for a large country due to the likelihood of retaliation. Answer: E Difficulty: Easy AACSB: Application of knowledge 6) The optimum tariff is most likely to apply to A) a small tariff imposed by a small country. B) a small tariff imposed by a large country. C) a large tariff imposed by a small country. D) a large tariff imposed by a large country. E) an ad valorem tariff on a small country. Answer: B Difficulty: Easy AACSB: Application of knowledge 7) Which of the following statements is TRUE? A) Most small countries have very little ability to affect the world prices of either their imports or their exports. B) Most small countries find the terms of trade of very high importance to them. C) Most small countries have the ability to affect the world prices of either their imports or their exports. D) Most small countries can avoid the impacts of the world prices on either their imports or their exports when trading. Answer: A Difficulty: Easy AACSB: Application of knowledge 8) The existence of marginal social benefits which are NOT marginal benefits for the industry producing the import substitutes A) is an argument supporting free trade and non-governmental involvement. B) is an argument supporting the use of an optimum tariff. C) is an argument supporting the use of market failures as a trade-policy strategy. D) is an argument rejecting free trade and supporting governmental involvement. E) is an argument rejecting the domestic market failure concept. Answer: D Difficulty: Easy AACSB: Application of knowledge
7 Copyright © 2022 Pearson Education, Inc.
9) The domestic market failure argument is a particular case of the theory of A) the optimum, or first-best. B) the second best. C) the third best. D) the sufficing principle. E) the efficiency case for free trade. Answer: B Difficulty: Easy AACSB: Application of knowledge 10) The authors of the text believe that A) second-best policy is worse than optimal policy. B) special interest groups generally enhance national welfare. C) national welfare is likely to be enhanced by the imposition of an optimal tariff. D) market failure arguments tend to support free-trade policy. E) there is no such thing as national welfare. Answer: D Difficulty: Easy AACSB: Application of knowledge 11) The difficulty of ascertaining the right second-best trade policy to follow A) reinforces support for the third-best policy approach. B) reinforces support for increasing research capabilities of government agencies. C) reinforces support for abandoning trade policy as an option. D) reinforces support for free-trade options. E) reinforces support for the domestic market failure argument. Answer: D Difficulty: Easy AACSB: Application of knowledge 12) What is the difference between optimum tariff and prohibitive tariff? Answer: The tariff rate that maximizes national welfare is the optimum tariff. A prohibitive tariff is one so high that no one imports any of that item.The optimum tariff rate is always positive but less than the prohibitive rate that would eliminate all imports. The optimum tariff creates revenues for the government, but the prohibitive tariff creates no tariff revenue, which implies that terms of trade gain is also zero. Difficulty: Easy AACSB: Application of knowledge
8 Copyright © 2022 Pearson Education, Inc.
13) The United States appears at times to have a totally schizophrenic attitude toward protectionism. The United States was the country that proposed the establishment of the World Trade Organization as early as the late 1940s, and was also the only industrialized country that refused to ratify this at that time. The United States has consistently argued on the side of multinational free trade in GATT Rounds, and yet maintains many protectionist laws such as those which reserve oil shipments from Alaska to U.S. flag carriers. How can you explain this apparent lack of national consistency on this issue? Answer: This reflects the fact that international trade typically has many winners and relatively fewer, but politically powerful losers. Short of guaranteed (constitutional?) non-conditional compensatory mechanisms, the basic conflict between these two groups will always be there. Difficulty: Moderate AACSB: Application of knowledge
Assume that Boeing (U.S.) and Airbus (European Union) both wish to enter the Hungarian market with the next new generation airliner. They both have identical cost and demand conditions (as indicated in the graph above). 14) Refer to above figure. Assume that Boeing is the first to enter the Hungarian market. Without a government subsidy what price would they demand, and what would be their total profits? Answer: $12 Million, $16. Difficulty: Moderate AACSB: Application of knowledge
9 Copyright © 2022 Pearson Education, Inc.
15) Refer to above figure. Assume that Boeing is the first to enter the Hungarian market. Without a government subsidy what price would they demand, and what would be their total profits? Answer: $12 Million, $16. Difficulty: Moderate AACSB: Application of knowledge 16) Refer to above figure. What is the consumer surplus enjoyed by Hungarian consumers of Boeing aircraft in the situation? Answer: $8 Million. Difficulty: Moderate AACSB: Application of knowledge 17) Refer to above figure. Suppose the European government provides Airbus with a subsidy of $4 for each airplane sold, and that the subsidy convinces Boeing to exit the Hungarian market. Now Airbus would be the monopolist in this market. What price would they charge, and what would be their total profits? Answer: $10 Million, and $36 Million. Difficulty: Moderate AACSB: Application of knowledge 18) Refer to above figure. What would be the cost of the subsidy to European taxpayers? Answer: $24 Million. Difficulty: Moderate AACSB: Application of knowledge 19) Refer to above figure. What happens to the Consumer Surplus of Hungarian customers as a result of this subsidy? Answer: An increase of $10 Million. Difficulty: Moderate AACSB: Application of knowledge 20) Refer to above figure. What is the revenue gain or loss for Europe as a whole (including taxpayers)? Answer: A gain of $12 Million. Difficulty: Moderate AACSB: Application of knowledge
10 Copyright © 2022 Pearson Education, Inc.
10.3
Income Distribution and Trade Policy
1) The simple model of competition among political parties long used by political scientists tends to lead to the practical solution of selecting the A) optimal tariff. B) prohibitive tariff. C) zero (free-trade) tariff. D) the tariff rate favored by the median voter. E) the tariff rate supported by exporters. Answer: D Difficulty: Easy AACSB: Application of knowledge 2) The political wisdom of choosing a tariff acceptable to the median U.S. voter is A) a good example of the principle of the second best. B) a good example of the way in which actual tariff policies are determined. C) a good example of the principle of political negotiation. D) not evident in actual tariff determination. E) usually evident in actual tariff determination. Answer: D Difficulty: Easy AACSB: Application of knowledge 3) The median voter model A) works well in the area of trade policy. B) is not intuitively reasonable. C) tends to result in biased tariff rates. D) does not seem to work well in the area of trade policy. E) is not widely practiced in the United States. Answer: D Difficulty: Easy AACSB: Application of knowledge 4) The fact that trade policy often imposes harm on large numbers of people, and benefits only a few may be explained by A) the lack of political involvement of the public. B) the power of advertisement. C) the problem of collective action. D) the basic impossibility of the democratic system to reach a fair solution. E) a cycle of political corruption. Answer: C Difficulty: Easy AACSB: Application of knowledge
11 Copyright © 2022 Pearson Education, Inc.
5) The reason protectionism remains strong in the United States is that A) economists can produce any result they are hired to produce. B) economists cannot persuade the general public that free trade is beneficial. C) economists do not really understand how the real world works. D) the losses associated with protectionism are diffuse, making lobbying by the public impractical. E) economists cannot agree on trade policy recommendations. Answer: D Difficulty: Easy AACSB: Application of knowledge 6) A 1998 study by Robert Baldwin and Christopher Magee found A) a strong impact of money on the voting pattern of NAFTA and GATT. B) contribution money did not impact the voting pattern of NAFTA and GATT. C) NAFTA and GATT would not have passed in the absence of contributions from either labor or business. D) contributions had very minor impacts on the vote totals. Answer: A Difficulty: Easy AACSB: Application of knowledge 7) It is argued that special interest groups are likely to take over and promote protectionist policies, which may lead to a decrease in national economic welfare. This argument leads to A) a presumption that in practice a free trade policy is likely to be better than alternatives. B) a presumption that trade policy should be shifted to Non-Governmental Organizations, so as to limit taxpayer burden. C) a presumption that free trade is generally a second-best policy, to be avoided if feasible alternatives are available. D) a presumption that free trade is the likely equilibrium solution if the government allows special interest groups to dictate its trade policy. E) a presumption that protectionist policies will better serve a country as a whole than free trade policies. Answer: A Difficulty: Easy AACSB: Application of knowledge 8) Well-organized groups – that is, groups that are able to overcome the problem of collective action – A) will be able to get policies that favor their interests at the expense of the public as a whole. B) will not be able to get policies that favor their interests at the expense of the public as a whole. C) will give up their interests for the public interest. D) will no longer take actions to favor their interests at the expense of the public as a whole. Answer: A Difficulty: Easy AACSB: Application of knowledge 12 Copyright © 2022 Pearson Education, Inc.
9) Protectionism tends to be concentrated in two sectors A) agriculture and clothing. B) high-tech and national security sensitive industries. C) capital and skill intensive industries. D) industries concentrated in the South and in the Midwest of the country. E) financial services and manufacturing based in the Midwest. Answer: A Difficulty: Easy AACSB: Application of knowledge 10) The quantitative importance of U.S. protection of the agriculture industry is best explained by the fact that A) this industry is a well-organized and politically powerful group in the U.S. B) this industry is an important employer of highly skilled labor. C) this industry is an important employer of low skilled labor. D) most of the exporters of agriculture into the U.S. are poor countries. E) the technology involved is very advanced. Answer: A Difficulty: Easy AACSB: Application of knowledge 11) In Japan before mid-1990s, the government had traditionally banned imports of rice, thus driving up internal prices of the country's staple food to more than ________ as high as the world price. A) five times B) four times C) three times D) twice Answer: A Difficulty: Easy AACSB: Application of knowledge 12) Until 2005, the clothing industry were subject to the ________, which set both export and import quotas for a large number of countries. A) Multi-Fiber Arrangement B) Multi-Fashion Arrangement C) Textile and Clothing Agreement D) Global Apparel Agreement Answer: A Difficulty: Easy AACSB: Application of knowledge
13 Copyright © 2022 Pearson Education, Inc.
13) The strongest political pressure for a trade policy that results in higher protectionism comes from A) domestic workers lobbying for import restrictions. B) domestic workers lobbying for export restrictions. C) domestic workers lobbying for free trade. D) domestic consumers lobbying for export restrictions. E) domestic consumers lobbying for import restrictions. Answer: A Difficulty: Easy AACSB: Application of knowledge 14) Today U.S. protectionism is concentrated in A) high-tech industries. B) labor-intensive industries. C) industries in which Japan has a comparative advantage. D) computer intensive industries. E) capital-intensive industries. Answer: B Difficulty: Easy AACSB: Application of knowledge 15) The quantitative importance of U.S. protection of the domestic clothing industry is best explained by the fact that A) this industry is an important employer of highly skilled labor. B) this industry is an important employer of low skilled labor. C) most of the exporters of clothing into the U.S. are poor countries. D) this industry is a politically well-organized sector in the U.S. E) the technology involved is very advanced. Answer: D Difficulty: Easy AACSB: Application of knowledge 16) What does the median voter model of electoral competition predict? Why is it helpful? Answer: Self- interested politicians will always want to find a middle ground and converge on the tariff rate preferred by the median voter who is exactly halfway up the lineup in order to win the next election. The median voter model of electoral competition has been very helpful as a way of thinking about how political decisions get made in the real world, where the effects of policy on income distribution may be more important than their effects on efficiency. Difficulty: Easy AACSB: Application of knowledge
14 Copyright © 2022 Pearson Education, Inc.
17) Give an example and explain why the median voter model does not seem to work well in trade policy. Answer: The U.S. dairy industry is protected from foreign competition by an elaborate system of tariffs and quotas. These restrictions impose losses on just about every family in America while providing much smaller benefits to a dairy industry that employs only about 0.1 percent of the nation's workforce. This is not what the median voter model predicts that a policy winner should inflict large losses on a few people and benefit a large number of people. The reason is because of collective action: While it is in the interests of the group as a whole to press for favorable policies, it is not in any individual's interest to do so. As a result, well-organized groups–that is, groups that are able to overcome the problem of collective action–will be able to get policies that favor their interests at the expense of the public as a whole. Difficulty: Easy AACSB: Application of knowledge 18) It may be demonstrated that any protectionist policy, which effectively shifts real resources to import competing industries or sectors will harm export industries or sectors. This may, for example, happen by the strengthening U.S. dollar in the foreign exchange market. Would you propose therefore that export industries lobby against protectionism in International Trade Commission proceedings? What of consumer advocates? Discuss the pros and the problems of such a suggestion. Answer: Actually this is an interesting idea. It is well known that the public interest is put on hold as the ITC considers only the squeaky wheels of those allegedly hurt by trade. While "consumers" may be too amorphous a group to successfully organize and pursue a political agenda, the exporters and consumer advocates may be able to form a counter weight to the import competing industries. Difficulty: Moderate AACSB: Application of knowledge
15 Copyright © 2022 Pearson Education, Inc.
10.4
International Negotiations and Trade Policy
1) The average tariff rate dutiable imports in the United States is approximately A) less than 10 % of the value of imports. B) 15% of the value of imports. C) 20 % of the value of imports. D) 25% of the value of imports. E) more than 30% of the value of imports. Answer: A Difficulty: Easy AACSB: Application of knowledge 2) There are at least two reasons why it is easier to lower tariffs as part of a mutual agreement than to do so as a unilateral policy. A) It helps countries mobilize support for freer trade and helps governments avoid getting caught in destructive trade wars. B) It helps countries improve their welfare policies and increase governments' revenues. C) It helps countries avoid trade deficits and improve trade balances. D) It helps countries gain popularity in the world arena and avoid global competition. Answer: A Difficulty: Easy AACSB: Application of knowledge 3) When the U.S. placed tariffs on French wine, France placed high tariffs on U.S. chickens. This is an example of A) deadweight losses. B) multilateral negotiations. C) bilateral trade negotiations. D) international market failures. E) a trade war. Answer: E Difficulty: Easy AACSB: Application of knowledge 4) A game-theory explanation of the paradox that even though all countries would benefit if each chose free trade, in fact each tends to follow protectionist policies is A) trade war. B) collective action. C) prisoner's dilemma. D) benefit-cost analysis. E) rent seeking. Answer: C Difficulty: Easy AACSB: Application of knowledge
16 Copyright © 2022 Pearson Education, Inc.
5) The Smoot-Hawley Tariff Act of 1930 has generally been associated with A) falling tariffs. B) free trade. C) intensifying the worldwide depression. D) recovery from the worldwide depression. E) non-tariff barriers. Answer: C Difficulty: Easy AACSB: Application of knowledge 6) The General Agreement on Tariffs and Trade and the World Trade Organization have resulted in A) termination of export subsidies applied to manufactured goods. B) termination of import tariffs applied to manufactures. C) termination of import tariffs applied to agricultural commodities. D) termination of international theft of copyrights. E) a number of rounds of multilateral trade agreements. Answer: E Difficulty: Easy AACSB: Application of knowledge 7) The General Agreement on Tariffs and Trade and the World Trade Organization have resulted in A) the establishment of universal trade adjustment assistance policies. B) the establishment of the European Union. C) the reciprocal trade clause. D) reductions in trade barriers via multilateral negotiations. E) the total protection of all intellectual property rights. Answer: D Difficulty: Easy AACSB: Application of knowledge 8) An issue never confronted effectively by GATT, but considered an important issue for WTO is that of A) the promotion of freer World trade. B) the promotion of freer World commodity trade. C) the promotion of freer World services trade. D) the lowering of tariff rates. E) the liberalization of trade. Answer: C Difficulty: Easy AACSB: Application of knowledge
17 Copyright © 2022 Pearson Education, Inc.
9) The most two important results of the Uruguay round of global trade negotiations carried out under the GATT are A) trade liberalization and administrative reforms. B) threats and counter threats. C) offers and counteroffers. D) political consensus and trade barriers. Answer: A Difficulty: Easy AACSB: Application of knowledge 10) As a result of the Uruguay Round, average tariff rates imposed by advanced countries fell almost ________. A) 40 percent B) 30 percent C) 20 percent D) 10 percent Answer: A Difficulty: Easy AACSB: Application of knowledge 11) The Uruguay Round liberalized trade in two important sectors: A) agriculture and clothing. B) auto industry and finance. C) steel and sugar. D) sugar and dairy products. Answer: A Difficulty: Easy AACSB: Application of knowledge 12) The Uruguay Round set new rules that should open up a wide range of A) government contracts for imported products. B) government contracts for exported products. C) private firms' contracts for imported products. D) private firms' contracts for exported products. Answer: A Difficulty: Easy AACSB: Application of knowledge 13) The World Trade Organization (WTO) was organized as a successor to the A) IMF. B) UN. C) UNCTAD. D) GATT. E) the World Bank. Answer: D Difficulty: Easy AACSB: Application of knowledge 18 Copyright © 2022 Pearson Education, Inc.
14) The WTO was established by the ________ of multilateral trade negotiations. A) Kennedy Round B) Tokyo Round C) Uruguay Round D) Dillon Round E) NAFTA Round Answer: C Difficulty: Easy AACSB: Application of knowledge 15) Rules on trade in services A) were part of WTO's agreements. B) were included in GATT. C) were not part of GATT and WTO agreements. D) were added to GATT later on. Answer: A Difficulty: Easy AACSB: Application of knowledge 16) The World Trade Organization provides for all of the following EXCEPT A) the usage of the most favored nation clause. B) assistance in the settlement of trade disagreements. C) bilateral tariff reductions. D) multilateral tariff reductions. E) the prevention of nontariff interventions in trade. Answer: C Difficulty: Easy AACSB: Application of knowledge 17) Which organization determines procedures for the settlement of international trade disputes? A) World Bank B) World Trade Organization C) International Monetary Organization D) International Bank for Reconstruction and Development E) The League of Nations Answer: B Difficulty: Easy AACSB: Application of knowledge
19 Copyright © 2022 Pearson Education, Inc.
18) The economic impact of the Uruguay Round is difficult to estimate, but it is clear that the costs were felt by ________, while the benefit accrued to ________. A) concentrated and well-organized groups; broad and diffuse populations B) broad and diffuse populations; concentrated and well-organized groups C) broad and organized populations; scattered and unorganized groups D) consumers and taxpayers; small influential groups of farmers Answer: A Difficulty: Easy AACSB: Application of knowledge 19) The WTO's intervention against clean air standards A) has earned it universal approval. B) was done in order to limit national sovereignty. C) has resulted in much criticism. D) has resulted in much criticism among professional economists. E) was championed in developing countries. Answer: C Difficulty: Easy AACSB: Application of knowledge 20) The U.S.—Venezuela dispute over imported gasoline has become a prime example for those A) who accuse the WTO of undermining national sovereignty. B) who support WTO in its role of environmental protection. C) who support WTO in blocking important domestic social goals. D) who accuse WTO of widening the gap between the haves and have nots. Answer: A Difficulty: Easy AACSB: Application of knowledge 21) What was the real reason behind U.S. government's imposing 30 percent tariffs on a range of imported steel products in March 2002? A) It was politics in crucial "swing states" where the steel industry is concentrated in the 2004 election. B) It was a solution for the U.S. steel industry that faced a surge in imports and needed time to restructure. C) It was to retaliate other countries' tariff on U.S. steel exports. D) It was to increase the U.S. government's revenue and solve the budget deficit problem. Answer: A Difficulty: Easy AACSB: Application of knowledge
20 Copyright © 2022 Pearson Education, Inc.
22) What was the WTO's ruling on the U.S. tariffs on imported steel in 2003? A) U.S. action was unjustified. B) U.S. action was justified. C) U.S. action was legal. D) U.S. action was in compliance with WTO's principles. Answer: A Difficulty: Easy AACSB: Application of knowledge 23) Why did Brazil withdraw the complaint and did not retaliate with sanctions on U.S. exports after WTO's ruling of illegal U.S. subsidies to cotton producers in March 2005? A) Because the U.S. made a side deal to pay Brazil hundreds of millions of dollars in compensation. B) Because the United States complied with WTO's ruling and eliminated the subsidies. C) Because Brazil gained from the U.S. subsidies. D) Because Brazil cotton producers stopped lobbying. Answer: A Difficulty: Easy AACSB: Application of knowledge 24) What is the difference between GATT and WTO? Answer: From a legal point of view, the GATT was a provisional agreement, whereas the WTO is a full-fledged international organization. An updated version of the original GATT text has been incorporated into the WTO rules. The GATT applied only to trade in goods and not trade in services, while the WTO agreement includes rules on trade in services (the General Agreement on Trade in Services, or GATS). Difficulty: Easy AACSB: Application of knowledge 25) Presumably, since the United States is a large country in many of its international markets, a positive optimum tariff exists for this country. It follows therefore that when any legislator or government official who promotes zero-tariff free trade policies, is by definition not acting in the public's best interest. Discuss. Answer: Technically this is true. However, this is true only within the context of a generally myopic view of international relations. If the tariff imposing country is large enough to make a substantial difference in its welfare by seeking an optimum tariff, then it cannot hope to remain invisible, as its policies are substantially harming its trade partners. Foreign repercussions are almost a certainty. In such a "game" it is not at all certain that seeking the optimum tariff dominates alternative strategies. Difficulty: Moderate AACSB: Application of knowledge
21 Copyright © 2022 Pearson Education, Inc.
26) It has been claimed that foreign governments have attempted to influence votes in the U.S. that would promote policies of protectionism within the U.S. On the surface this appears to be totally illogical and counter intuitive, as this would presumably lessen the possibilities of foreigners' exports to the U.S. Answer: This would make sense only if the form of protectionism is a tariff. However, if it is a quota, then the scarcity rents may be captured by established foreign producers. Hence, the reaction of the Japanese to automobile quotas was to dramatically increase the high-end, highly profitable automobiles. This would be even more self-evident if the protectionism took the form of a Voluntary Export Restraint (VER), or a detailed formalized bilateral cartel, such as the old Multi-Fibre Agreement. Difficulty: Moderate AACSB: Application of knowledge 27) The U.S. producer Boeing, and the European Airbus are contemplating the next generation mid-sized fuel-efficient generation of air carrier. If both produce their respective models, then each would lose $50 million (because the world market is just not large enough to enable either to capture potential scale economies if they had to share the world market). If neither produce, then each one's net gain would of course be zero. If either one produces while the other does not, then the producer will gain $500 million. (a) What is the correct strategy for either company? (b) What is the correct strategy for a government seeking to maximize national economic welfare? (c) If a national government decides to subsidize its aircraft producer, how high should be the subsidy? Answer: (a) enter the market first. Then the other company knows that if it also enters, it will not be able to cover costs. (b) Subsidize its producer. If this allows it to enter first, then we get the same solution as answer (a) above. (c) Any figure above $50 million (e.g., $55 million). This would promise positive profits regardless of the decision of the competitor. The "winner" then may turn out to be that country whose voters are least sensitive to on-the-books, transparent subsidies given to rich corporations (these subsidies will have to continue year after year until the other country stops its subsidies). Difficulty: Moderate AACSB: Application of knowledge 28) In recent cases, the U.S. placed quotas or protectionist tariffs on imported steel and imported microchips. In both cases the damage to "downstream" industries was obvious to all and relatively easy to quantify and demonstrate. Assuming that the U.S. lawmakers are not plain dumb, why did they enact these protectionist policies? Answer: The system by which these protectionist policies are set into law is biased in favor of the producers of import competitive goods. Other sectors of the economy that may be affected are not parties in the petitions made to the ITC seeking redress. Difficulty: Moderate AACSB: Application of knowledge
22 Copyright © 2022 Pearson Education, Inc.
29) Describe the costs and benefits of the Uruguay Round? Answer: The costs of the Uruguay Round were felt by concentrated, often well-organized groups, while the benefit accrued to broad, diffuse populations. The progress on agriculture hurt the small but influential populations of farmers in Europe, Japan, and other countries where agricultural prices are far above world levels. These losses were much more than offset by gains to consumers and taxpayers in those countries, but because these benefits were very widely spread, they were little noticed. Similarly, the liberalization of trade in textiles and clothing produced some concentrated pain for workers and companies in those industries, offset by considerably larger but far less visible consumer gains. Difficulty: Easy AACSB: Application of knowledge 10.5
The End of Trade Agreements?
1) After the failure of the Doha round of world trade negotiations, A) the reductions in tariff rates that took place in the previous eight rounds remain in effect. B) the reductions in tariff rates that took place in the previous eight rounds stopped. C) more reductions in tariff rates take place all over the world. D) more trade barriers took effect than ever before. Answer: A Difficulty: Easy AACSB: Application of knowledge 2) The world trading system combines negotiated agreements that promote trade liberalization called ________ with binding agreements called ________ that block tariff increases. A) fiscal policies; monetary policies B) truces; aggressions C) free trade; enforcement contracts D) levers; ratchets E) wheels; walls Answer: D Difficulty: Moderate AACSB: Application of knowledge 3) According to the World Bank's estimate, liberalizing agricultural trade would produce ________ of the total world gains from free trade for the world as a whole; and these gains are very hard to get at. A) 63 percent B) 10 percent C) 17 percent D) 46 percent Answer: A Difficulty: Easy AACSB: Application of knowledge
23 Copyright © 2022 Pearson Education, Inc.
4) For most developing countries A) productivity is high among domestic workers. B) population growth and illiteracy rates are low. C) saving and investment levels are high. D) agricultural goods and raw materials constitute a high proportion of domestic output. E) pollution emissions are relatively low. Answer: D Difficulty: Moderate AACSB: Application of knowledge 5) One of the major issues that arose during the Doha round of negotiations involved complaints by ________ about ________. A) developing countries; agricultural subsidies. B) manufacturers; intellectual property C) industrialized countries; enforcement of contracts D) Eastern European countries; European Union tariffs E) South and Central American countries; domestic content requirements Answer: A Difficulty: Moderate AACSB: Application of knowledge 6) A successful Doha Round would actually have hurt China A) because it would have ended up paying higher prices for imported agricultural goods without rich country's export subsidies. B) because it would have ended up paying higher prices for imported agricultural goods with rich country's export subsidies. C) because it would have ended up paying lower prices for imported agricultural goods without rich country's export subsidies. D) because it would have ended up paying the same prices for imported agricultural goods with rich country's export subsidies. Answer: A Difficulty: Easy AACSB: Application of knowledge 7) Under preferential trading agreements, A) the tariffs that countries apply to each other's products are lower than the rates on the same goods coming from other countries. B) the tariffs that countries apply to each other's products are higher than the rates on the same goods coming from other countries. C) the tariffs that countries apply to each other's products are the same as the rates on the same goods coming from other countries. D) there are no tariffs that countries apply to each other's products. Answer: A Difficulty: Easy AACSB: Application of knowledge
24 Copyright © 2022 Pearson Education, Inc.
8) Countries agree to establish free trade by A) establishing a free trade area or a customs union. B) using a common currency. C) eliminating borders and border control. D) allowing unlimited labor migration without restrictions. Answer: A Difficulty: Easy AACSB: Application of knowledge 9) The North American Free Trade Agreement established free trade among Canada, the United States, and Mexico. It was later renegotiated and renamed the U.S-Mexico-Canada Agreement. This is an example of A) a free trade area. B) a customs union. C) a currency union. D) a trade union. Answer: A Difficulty: Easy AACSB: Application of knowledge 10) The European Union, in which all countries must agree to charge the same tariff rate on each imported good, is an example of A) a customs union. B) a currency union. C) a free trade area. D) a trade union. Answer: A Difficulty: Easy AACSB: Application of knowledge 11) A customs union is desirable or undesirable depends on A) whether it may lead to trade creation or trade diversion. B) member countries' governments' choices and decisions. C) the speed of globalization. D) the outcome of trade negotiations. Answer: A Difficulty: Easy AACSB: Application of knowledge
25 Copyright © 2022 Pearson Education, Inc.
12) The difference between a free trade area and a customs union is A) that the first is politically straightforward but an administrative headache, while the second is just the opposite. B) that in a customs union, each country can charge different tariffs. C) that in a free trade area, countries adopt a common external tariff on goods they import from other countries. D) that free trade agreements do not impose a large burden of paperwork. Answer: A Difficulty: Moderate AACSB: Application of knowledge 13) A customs union requires that Germany, France, the Netherlands, and all the other countries in the European Union A) agree to charge the same tariff on goods they import from other countries. B) be able to charge different tariffs on goods they import from other countries. C) to remove all tariffs on goods they import from other countries. D) be able to fluctuate their own tariff on goods they import from other countries. Answer: A Difficulty: Easy AACSB: Application of knowledge 14) Which of the following statement is NOT true? A) The USMCA is a customs union. B) The USMCA is a free trade area. C) The USMCA permits Mexican goods to enter the United States without tariffs and vice versa. D) The USMCA does not require that Mexico and the United States adopt a common external tariff on goods they import from other countries. Answer: A Difficulty: Easy AACSB: Application of knowledge 15) For over half a century since the European began in 1957, the economic integration of Europe was A) widened and deepened. B) narrowed and shallowed. C) an unsuccessful story. D) impossible to achieve. Answer: A Difficulty: Easy AACSB: Application of knowledge
26 Copyright © 2022 Pearson Education, Inc.
16) Brexit happened because of A) the populist backlash against the ways Europe tried to become more than a customs union. B) the trade in goods and services. C) the currency union. D) the peace and solidarity among European nations. Answer: A Difficulty: Easy AACSB: Application of knowledge 17) In 1991, four South American nations, Argentina, Brazil, Paraguay, and Uruguay, formed a free trade area known as A) MERCOSUR B) NAFTA C) USMCA D) SAFTA Answer: A Difficulty: Easy AACSB: Application of knowledge 18) Within four years after Mescosur was formed in 1991, A) the value of trade among the nations tripled. B) the pact had no immediate or dramatic effect on interregional trade. C) the value of trade among the nations decreased. D) the value of trade among the nations remained the same. Answer: A Difficulty: Easy AACSB: Application of knowledge 19) In 1996, a World Bank's study concluded that despite Mercosur's success in increasing regional trade, A) it produced a considerable amount of trade diversion. B) the net effects on the economies involved were positive. C) there was no impact on the economies' welfare. D) consumers benefited from cheaper but heavily tariffed goods from other countries outside the free trade area. Answer: A Difficulty: Easy AACSB: Application of knowledge 20) TPP stands for A) Trans-Pacific Partnership. B) Transportation Policy Plan. C) Technology and Policy Program. D) Transition Partnership Program. Answer: A Difficulty: Easy AACSB: Application of knowledge 27 Copyright © 2022 Pearson Education, Inc.
21) Which of the following statements is NOT true about TPP? A) TPP may weaken "intellectual property rights." B) TPP may help resolve disputes between private businesses and national governments. C) TPP would provide businesses with an assurance of fair treatment and foster greater trade and investment. D) TPP might reinforce corporate interests at the expense of workers. Answer: A Difficulty: Easy AACSB: Application of knowledge 22) Explain the impact of rich country export subsidies on China and other third world countries? Answer: China may actually benefit from rich-country export subsidies. Some third-world farmers are hurt by low prices of subsidized food exports from Europe and the United States–but urban residents in the third world benefit, and so do those farmers producing goods, such as coffee, that don't compete with the subsidized products. Similarly, African countries would actually be made worse off without the subsidized food exports from rich countries because the negative effects of higher food prices would more than offset the gains from higher prices for crops such as cotton. Difficulty: Easy AACSB: Application of knowledge 23) Why the Doha round of world trade negotiations failed? Answer: Doha's failure owes a lot to the success of previous trade negotiations. Because previous negotiations had been so successful at reducing trade barriers, the remaining barriers to trade are fairly low, so that the potential gains from further trade liberalization are modest. Indeed, barriers to trade in most manufactured goods other than apparel and textiles are now more or less trivial. Most of the potential gains from a move to freer trade would come from reducing tariffs and export subsidies in agriculture–which has been the last sector to be liberalized because it's the most sensitive sector politically. Poor countries saw little in the proposals for them; they pressed for much bigger concessions from rich countries. The governments of rich countries, in turn, refused to take the political risk of crossing powerful interest groups, especially farmers, without something in return–and poor countries were unwilling to offer the deep cuts in their remaining tariffs that might have been sufficient. Difficulty: Moderate AACSB: Application of knowledge 24) What is the difference between a free trade area and a customs union? Answer: In a free trade area, each country's goods can be shipped to the other without tariffs, and countries set tariffs against the outside world independently. In a customs union, all countries must agree and charge the same tariff rates on each imported good. Therefore, in brief, that the first is politically straightforward but an administrative headache, while the second is just the opposite. Difficulty: Easy AACSB: Application of knowledge 28 Copyright © 2022 Pearson Education, Inc.
25) What are the advantages and disadvantages of a free trade area and a customs union? Answer: In a free trade area, each country's goods can be shipped to the other without tariffs, and countries set tariffs against the outside world independently. In a customs union, all countries must agree and charge the same tariff rates on each imported good. The first is politically straightforward but an administrative headache, while the second is just the opposite. Free trade agreements impose a large burden of paperwork, which may be a significant obstacle to trade even when such trade is, in principle, free. Difficulty: Easy AACSB: Analytical thinking 26) How did Brexit happen? Answer: In 2016 the European Union experienced a shocking reversal: Britain held a referendum on whether to leave the union–a proposition that came to be known as "Brexit" (for British exit)–and a narrow majority of the public voted to leave. What happened? It probably wasn't about trade in goods and services; that is, it wasn't about the customs union. Instead, there was a backlash against the ways Europe tried to become more than a customs union, an effort symbolized by the change in name from Common Market to European Union. More specifically, in 1992 the group known by then as the European Economic Community established new rules that harmonized regulations and, perhaps more important, guaranteed free movement of people among member countries. At first, this seemed to cause few problems. But after 2004 the Union was also expanded substantially, adding a number of former Communist countries in Eastern Europe. These countries are relatively poor– for example, per capita income in both Romania and Bulgaria is less than half its level in Britain. As a result, significant numbers of workers began migrating to richer European nations. There is a widespread perception in the countries experiencing inward migration that the migrants are having an adverse effect on native-born citizens: taking jobs, putting strain on public services, and so on. Most economic analyses suggest that this perception greatly overstates the reality and fails to take account of the benefits of additional workers, but it's not hard to see why such claims get popular traction, especially given declining wages for many blue-collar workers. Add in fears that national identity is being undermined, and the conditions were there for a populist backlash. Difficulty: Moderate AACSB: Application of knowledge 27) What is trade diversion? Why was Mescosur concluded to produce trade diversion in South America? Answer: Trade diversion occurs when tariff agreements cause imports to shift from low-cost countries to higher-cost countries. As a result of Mercosur, consumers in the member countries were being induced to buy expensively produced manufactured goods from their neighbors rather than cheaper but heavily tariffed goods from other countries. In particular, Brazil's highly protected and somewhat inefficient auto industry had in effect acquired a captive market in Argentina, thus displacing imports from elsewhere. Difficulty: Easy AACSB: Application of knowledge
29 Copyright © 2022 Pearson Education, Inc.
28) What were the initial purposes of TPP? Why did TPP agreement fail to pass? Answer: One important aspect of TPP was a strengthening of "intellectual property rights"–the ability to enforce patents and copyrights across borders. Another aspect was "investor-state dispute settlement"–dealing with arguments between private businesses and national governments. TPP would have set up special panels, with representatives from both sectors, to resolve such disputes. There was a case to be made for both aspects, which would arguably provide businesses with an assurance of fair treatment and foster greater trade and investment. However, there was also a reasonable case against the agreement, which was that it might reinforce corporate interests at the expense of workers–for example, making it easier for drug companies to charge high prices. The point is not that one side was right and the other wrong, but rather that the simple logic of free trade offered little guidance to the desirability of TPP. This murkiness, combined with growing skepticism about trade in general, made TPP a more or less impossible sell. Difficulty: Moderate AACSB: Application of knowledge 29) The U.S. is probably the most open international market among the industrialized countries. What then does the U.S. have to gain by joining the WTO? Answer: There are two answers. First, the U.S. exporters stand to gain profitable markets if foreign protectionism in areas of U.S. comparative advantage (e.g., soy) is removed due to WTO efforts. The second is that the WTO offers the U.S. government administration a counterweight to regional and sectoral interests demanding protection. It is always politically easier to bring about more efficient resource allocations if the complaints of the losers may be deflected by the presence of a binding treaty with an international organization ("our hands are tied"). Difficulty: Moderate AACSB: Application of knowledge
30 Copyright © 2022 Pearson Education, Inc.
10.6
Appendix to Chapter 10: Proving That the Optimum Tariff Is Positive
1) The effect of an export tariff on a large country is to ________ the terms of trade. A) always improve B) sometimes improve C) leave unchanged D) sometimes worsen E) always worsen Answer: A Difficulty: Easy AACSB: Application of knowledge 2) Which of the following statements is TRUE? A) A tariff always improves the terms of trade of a large country but at the same time distorts production and consumption. B) A tariff never distorts production and consumption. C) A tariff may or may not improve the terms of trade of a large country. D) A tariff always harms the terms of trade of a large country. Answer: A Difficulty: Easy AACSB: Application of knowledge 3) Assume that a country has a domestic demand curve defined as Qd = 100 - 2P and a domestic supply curve defined as Qs = -20 + 3P. What is the autarchy equilibrium price and quantity? Answer: 100 - 2P = -20 + 3P => P = 120/5 = 24 and Q = 52 Difficulty: Moderate AACSB: Application of knowledge 4) Assume that a country has a domestic demand curve defined as Qd = 100 - 2P and a domestic supply curve defined as Qs = -20 + 3P. What is the country's import demand curve (Qm)? Answer: Qm = Qd - Qs = (100 - 2P) - (-20 + 3P) = 120 - 5P Difficulty: Moderate AACSB: Application of knowledge
31 Copyright © 2022 Pearson Education, Inc.
5) Refer to the figure above, which shows domestic supply and demand. If P1 is equal to P2 (the world price) plus a tariff, then the social loss from the tariff is equal to A) a + c. B) b. C) P1 ( Q3 - Q2). D) P2 [(Q2 - Q1) + (Q4 - Q3)]. E) a + b + c. Answer: A Difficulty: Easy AACSB: Application of knowledge 6) Refer to the figure above, which shows domestic supply and demand. If P1 is equal to P2 (the world price) plus a tariff, then government revenue from the tariff is equal to A) a + c. B) b. C) P1 ( Q3 - Q2). D) P2 [(Q2 - Q1) + (Q4 - Q3)]. E) a + b + c. Answer: B Difficulty: Easy AACSB: Application of knowledge
32 Copyright © 2022 Pearson Education, Inc.
International Economics: Theory and Policy, 12e (Krugman) Chapter 11 Trade Policy in Developing Countries 11.1
Import-Substituting Industrialization
1) Nations differ greatly in their per-capita incomes. Among developing countries alone, the income range is very A) wide. B) narrow. C) hard to measure. D) insignificant. Answer: A Difficulty: Easy AACSB: Application of knowledge 2) For all developing countries, ________ has been a central concern of economic policy. A) the attempt to close the income gap with more advanced nations B) the attempt to be economically independent from more advanced nations C) the attempt to be politically independent from more advanced nations D) the attempt to be self-sufficient without relying on more advanced nations Answer: A Difficulty: Easy AACSB: Application of knowledge 3) From World War II until the 1970s, many developing countries attempted to accelerate their development by A) limiting imports of manufactured goods and fostering a manufacturing sector serving the domestic market. B) promoting exports of manufactured goods. C) lowering trade barriers to integrate in the global market. D) increasing imports of manufactured goods and ignoring the manufacturing sector serving the domestic market. Answer: A Difficulty: Easy AACSB: Application of knowledge 4) The infant industry argument is that A) comparative advantage is irrelevant to economic growth. B) developing countries have a comparative advantage in agricultural goods. C) developing countries have a comparative advantage in manufacturing. D) developing countries have a potential comparative advantage in manufacturing. E) developing countries have no chance to compete with industrialized countries. Answer: D Difficulty: Easy AACSB: Application of knowledge
1 Copyright © 2022 Pearson Education, Inc.
5) The infant industry argument was an important theoretical basis for A) neo-colonialist theory of international exploitation. B) import-substituting industrialization. C) historiography of the industrial revolution in Western Europe. D) the East-Asian miracle. E) the reduction of tariffs on Western Europe. Answer: B Difficulty: Easy AACSB: Application of knowledge 6) The infant industry argument calls for active government involvement A) only if the government forecasts are accurate. B) only if some market failure can be identified. C) only if the industry is not one already dominated by industrial countries. D) only if the industry has a high value added. E) only if the industry is independently able to earn high returns. Answer: B Difficulty: Easy AACSB: Application of knowledge 7) The imperfect capital market justification for infant industry promotion A) assumes that new industries will tend to have low profits. B) assumes that infant industries will soon mature. C) assumes that infant industries will be in products of comparative advantage. D) assumes that banks can allocate resources efficiently. E) assumes that developing country will reward the donor country. Answer: A Difficulty: Easy AACSB: Application of knowledge 8) Sophisticated theoretical arguments supporting import-substitution policies include A) terms of trade effects. B) learning curve considerations. C) the problem of appropriability. D) domestic market failure arguments. Answer: C Difficulty: Easy AACSB: Application of knowledge
2 Copyright © 2022 Pearson Education, Inc.
9) The appropriability argument for infant industry promotion A) assumes that firms in a new industry generate social benefits for which they are not compensated. B) assumes that pioneering firms' start-up costs are compensated. C) assumes that firms in a new industry create knowledge and new markets in which they can establish property rights. D) assumes that social benefits from creation of a new industry will always exceed its costs. Answer: A Difficulty: Easy AACSB: Application of knowledge 10) Which trade strategy have developing countries used to restrict imports of manufactured goods so that the domestic market is preserved for home producers? A) international commodity agreement B) export promotion C) multilateral contract D) import substitution E) export subsidies Answer: D Difficulty: Easy AACSB: Application of knowledge 11) Import-substituting industrialization A) is the strategy of encouraging domestic industry by limiting imports of manufactured goods. B) is the strategy of promoting exports of manufactured goods and substituting imports at the same time. C) is the strategy of importing manufactured goods as much as possible to meet the need of industrialization. D) is the strategy of lifting all import restrictions to promote free trade. Answer: A Difficulty: Easy AACSB: Application of knowledge 12) Import substitution policies make use of A) tariffs that discourage goods from entering a country. B) quotas applied to goods that are shipped abroad. C) production subsidies granted to industries with comparative advantage. D) tax breaks granted to industries with comparative advantage. E) production facilities provided by industrialized countries. Answer: A Difficulty: Easy AACSB: Application of knowledge
3 Copyright © 2022 Pearson Education, Inc.
13) General equilibrium considerations lead to the realization that import-substituting policies have the effect of A) discouraging exports. B) encouraging exports. C) encouraging an efficient use of a country's resources. D) generating large tariff revenues for the government. E) creating competitive manufacturing sectors. Answer: A Difficulty: Easy AACSB: Application of knowledge 14) Ten years before joining NAFTA in 1994, A) Mexico turned from import-substituting industrialization to relatively free trade. B) Mexico turned from export-promoting industrialization to relatively close to trade. C) Mexico was still an import-substituting industrialization and close to global trade. D) Mexico turned from export-promoting industrialization to import-substituting industrialization. Answer: A Difficulty: Easy AACSB: Application of knowledge 15) Half a century after the Great Depression of the 1930s, Mexico followed A) import-substituting industrialization strategy. B) export-subsidizing industrialization strategy. C) free trade strategy. D) no trade strategy. Answer: A Difficulty: Easy AACSB: Application of knowledge 16) Throughout the 1950s and 1960s, trade barriers were ________, as Mexican industry became increasingly self-sufficient. A) raised higher B) lowered C) non-existent D) completely removed Answer: A Difficulty: Easy AACSB: Application of knowledge
4 Copyright © 2022 Pearson Education, Inc.
17) How did Mexico's policy change from import-substitution to export-promotion industrialization work? A) Mexican exports increased rapidly, primarily manufactures. B) Mexican rate of growth has been higher than that achieved when Mexico was pursuing a policy of import-substituting industrialization. C) Mexican per-capita income has decreased over the past 30 years. D) Mexican manufacturing is still devoted to serving only the small domestic market, not yet part of an integrated North American manufacturing system. Answer: A Difficulty: Easy AACSB: Application of knowledge 18) The United States, as it began its long and successful growth in the early 19th century, consciously promoted domestic production through such activities as tariffs, Clay's American System, and many direct subsidies to railroads, canal companies, farmers (free land) etc. Today we view this blatant example of large scale and extensive import-substitution industrialization as having been very successful. Comment on this. Answer: This is an interesting point and emphasizes that economic models tend to be ahistorical. That is, they lack the historic perspective; and thus may be misleading as guides to long-run issues, such as economic growth. This also suggests that trade policy per se is almost certainly not sufficient to explain why some countries grow and others do not. Difficulty: Moderate AACSB: Application of knowledge 19) Describe one of the arguments to support import-substitution as a popular development strategy for many developing countries from World War II until the 1970s? Answer: Probably the most important of these arguments was the infant industry argument. Developing countries have a potential comparative advantage in manufacturing, but new manufacturing industries in developing countries cannot initially compete with well-established manufacturing in developed countries. To allow manufacturing to get a toehold, then, governments should temporarily support new industries until they have grown strong enough to meet international competition. Thus, it makes sense, according to this argument, to use tariffs or import quotas as temporary measures to get industrialization started. Difficulty: Easy AACSB: Application of knowledge 20) What are major problems with the infant industry argument? Answer: Economists have pointed out many pitfalls in the argument. First, it is not always a good idea to try to move today into the industries that will have a comparative advantage in the future. Second, protecting manufacturing does no good unless the protection itself helps make industry competitive. There are many stories of infant industries that have never grown up and remain dependent on protection. Difficulty: Easy AACSB: Application of knowledge
5 Copyright © 2022 Pearson Education, Inc.
21) Discuss the reasons why import substitution rather than export growth had usually been chosen as an industrialization strategy of many developing countries until the 1970s? Answer: The reasons were a mixture of economics and politics. First, many developing countries were skeptical about the possibility of exporting manufactured goods. Second, in many cases, import-substituting industrialization policies dovetailed naturally with existing political biases. Some believed that the advantages of established industrial nations were simply too great to be overcome by newly industrializing economies. Difficulty: Easy AACSB: Application of knowledge 22) How did Mexico's policy change from import-substitution to export-promotion industrialization work? Answer: Mexican exports increased rapidly from 10.7 percent of GDP in 1980 to 34 percent of GDP, primarily manufactures in 2012. Today, Mexican manufacturing, rather than being devoted to serving the small domestic market, is very much part of an integrated North American manufacturing system. The results for the overall Mexican economy have, however, been somewhat disappointing. Per-capita income has risen over the past 30 years, but the rate of growth has actually been lower than that achieved when Mexico was pursuing a policy of import-substituting industrialization. Difficulty: Easy AACSB: Application of knowledge 11.2
Results of Favoring Manufacturing: Problems of Import-Substituting Industrialization
1) Which of the following statements is TRUE? A) Countries pursuing import substitution were not catching up with advanced countries, and some lagged even further behind advanced countries as they developed a domestic manufacturing base. B) Countries pursuing import substitution all found that the industrialization strategy worked well for them. C) Countries pursuing import substitution could persuade other advanced countries in the world to adopt the same strategy. D) Countries pursuing import substitution could catch up with advanced countries, and some even grew faster than advanced countries as they developed a domestic manufacturing base. Answer: A Difficulty: Easy AACSB: Application of knowledge 2) The disappointment with import-substitution policies is in part because A) of the rapid and continuous growth record of South American countries. B) many countries pursuing this strategy experienced stagnation in their growth. C) this policy is inconsistent with sophisticated economic growth models. D) this policy tended to create world-class industrial competitors. E) of the financial investment lost by the U.S. Answer: B Difficulty: Easy AACSB: Application of knowledge 6 Copyright © 2022 Pearson Education, Inc.
3) The protectionist policies of many less-developed countries A) badly distorted incentives. B) strengthened domestic manufacturing sectors. C) promoted exports. D) created world class competitors. Answer: A Difficulty: Easy AACSB: Application of knowledge 4) The high rates of effective protection in some industries in Latin America and South Asia before 1970s allowed these industries A) to exist even when their cost of production was three or four times the price of the imports they replaced. B) to be more globally competitive. C) to have highly efficient scale production. D) to thrive and promote further exports. Answer: A Difficulty: Easy AACSB: Application of knowledge 5) Import-substituting industrialization has aggravated other problems, such as A) income inequality and unemployment. B) violence and crime. C) gambling and overspending. D) corruption and brain drain. Answer: A Difficulty: Easy AACSB: Application of knowledge 6) It is argued that import substitution is a misguided trade policy if the intent is to promote longterm economic growth. Explain the reasons underlying this argument. Answer: Import substitution promotes that economic activity in which the country is relatively inefficient. This lowers the real income at any given time and decreases the resources which can be used for investment purposes, hence lower growth rates. An additional answer is that import substitution by creating a protected domestic market fails to provide incentives to produce high, or world-class quality-which means this country cannot market in foreign countries. With such a (perceived) limited market, endogenous economic growth will not be forthcoming. Finally, it may be that exposure to world competition has its own dynamic effect promoting economic growth. Difficulty: Moderate AACSB: Application of knowledge
7 Copyright © 2022 Pearson Education, Inc.
11.3
Trade Liberalization Since 1985
1) Since the beginning of the mid-1980s, many developing countries have A) shifted toward freer trade with lower tariff and fewer restrictions. B) remained consistent with import-substituting policy. C) raised higher tariffs, imposed more import quotas and trade restrictions. D) closed their economies to import competition. Answer: A Difficulty: Easy AACSB: Application of knowledge 2) Trade liberalization in developing countries has led to A) a large rise in exports as a percentage of GDP. B) a large decline in exports as a percentage of GDP. C) a large rise in imports as a percentage of GDP. D) a large decrease in imports as a percentage of GDP. Answer: A Difficulty: Easy AACSB: Application of knowledge 3) Trade liberalization in developing countries has led to A) a larger share of manufactured goods in developing countries' exports. B) a smaller share of manufactured goods in developing countries' exports. C) a larger share of agricultural and mining products in developing countries' exports. D) no change in the nature of trade. Answer: A Difficulty: Easy AACSB: Application of knowledge 4) Growth rates in Brazil and other Latin American countries have actually been slower since the trade liberalization of the late 1980s than they were during import-substituting industrialization A) proves that export promoting trade policy leads to relatively rapid economic growth. B) proves that a free-trade orientation of trade policy results in rapid economic growth. C) proves that exports help growth, whereas imports impede growth. D) proves that trade policy is the most important policy area for promotion of economic development. E) proves that trade liberalization does not always lead to rapid economic growth. Answer: E Difficulty: Easy AACSB: Application of knowledge
8 Copyright © 2022 Pearson Education, Inc.
5) ________ experienced an impressive acceleration of growth after shifting away from importsubstitution. A) India B) Brazil C) Latin American countries D) Brazil and other Latin American countries Answer: A Difficulty: Easy AACSB: Application of knowledge 6) Describe the effects of trade liberalization. Answer: Trade liberalization in developing countries had two clear effects. One was a dramatic increase in the volume of trade. The share of trade in GDP of developing countries has tripled over that period, with most of the growth happening after 1985. The other effect was a change in the nature of trade. Before the change in trade policy, developing countries mainly exported agricultural and mining products. After 1980, the share of manufactured goods in developingcountry exports surged, coming to dominate the exports of the biggest developing economies. Difficulty: Easy AACSB: Application of knowledge 7) Has the switch from import-substitution to more open trade delivered better results for developing countries? Explain. Answer: The answer is that the picture is mixed. Growth rates in Brazil and other Latin American countries have actually been slower since the trade liberalization of the late 1980s than they were during import-substituting industrialization. India, on the other hand, has experienced an impressive acceleration of growth–although there is an intense dispute about how much of that acceleration can be attributed to trade liberalization. In addition, there is growing concern about rising inequality in developing countries. In Latin America at least, the switch away from import-substituting industrialization seems to have been associated with declining real wages for blue-collar workers, even as earnings of highly skilled workers have risen. Difficulty: Moderate AACSB: Application of knowledge
9 Copyright © 2022 Pearson Education, Inc.
11.4
Trade and Growth: Takeoff in Asia
1) Surprising success stories in the developing world in the 1970s involved a group of relatively small East Asian economies converging on advanced-country levels of income that included A) South Korea, Taiwan, Hong Kong, and Singapore. B) Vietnam, Laos, Cambodia, and Thailand. C) Brunei, Burma (Myanmar), Timor-Leste, and Indonesia. D) Malaysia, the Philippines, Indonesia, and Macau. Answer: A Difficulty: Easy AACSB: Application of knowledge 2) The "East Asian Miracle" of the original group of Asian tigers in the 1960s was spread and replicated by A) other Asian countries including China and India. B) developing countries around the world. C) Sub Sahara African countries. D) industrialized countries. E) Eastern European countries. Answer: A Difficulty: Easy AACSB: Application of knowledge 3) In China, Deng Xiaoping, who had taken power in 1978, converted A) a centrally planned economy into a market economy. B) a market economy to a centrally planned economy. C) a capitalist economy into a socialist economy. D) a market economy into a mixed economy. Answer: A Difficulty: Easy AACSB: Application of knowledge 4) China's recent experience supports the proposition that A) "economic miracles" are solely to be expected in small countries. B) central planning and socialism can promote sustained economic growth. C) a lessening of income disparities is a prerequisite for economic growth. D) growth in a large country cannot be affected by its foreign sector. E) policy changes can dramatically prompt export oriented growth. Answer: E Difficulty: Easy AACSB: Application of knowledge
10 Copyright © 2022 Pearson Education, Inc.
5) Latin American nations like Mexico and Brazil, which sharply liberalized trade and shifted toward exports, A) did not see comparable economic takeoffs as in the Asian miracle. B) experienced comparable economic takeoffs as in the Asian miracle. C) saw dramatic economic decline after the implementation. D) experienced higher economic takeoffs than in the Asian miracle. Answer: A Difficulty: Easy AACSB: Application of knowledge 6) Historically those few developing countries which have succeeded in significantly raising their per-capita income levels A) did not accomplish this with import-substituting industrialization. B) did accomplish this with import-substituting industrialization. C) tended to provide heavy protection to domestic industrial sectors. D) favored industrial to agricultural or service sectors. E) did so to the detriment of their nearest neighbors. Answer: A Difficulty: Easy AACSB: Application of knowledge 7) Asian success stories demonstrated that A) it is possible to achieve development through export-oriented growth. B) it is impossible to achieve development through export-oriented growth. C) it is possible to achieve development through import-substitution growth. D) it is impossible to achieve development through trade liberalization. Answer: A Difficulty: Easy AACSB: Application of knowledge 8) The growth successes of the high performance Asian economies A) supports the belief that economic development requires import substitution policies. B) rejects the belief that export-oriented industrialization is likely to promote economic development. C) rejects the belief that economic development requires import substitution policies. D) suggests that free trade policies are required for successful economic development. E) enforces United States' hesitation to trade with developing countries. Answer: C Difficulty: Easy AACSB: Application of knowledge
11 Copyright © 2022 Pearson Education, Inc.
9) Which industrialization policy used by developing countries places emphasis on the comparative advantage principle as a guide to resource allocation? A) export promotion B) import substitution C) international commodity agreements D) infant industry promotion E) intra-industry trade practice Answer: A Difficulty: Easy AACSB: Application of knowledge 10) Taiwan and South Korea are examples of developing nations that have recently pursued these industrialization policies: A) import substitution. B) export promotion. C) commercial dumping. D) multilateral contract. E) trade embargoes. Answer: B Difficulty: Easy AACSB: Application of knowledge 11) All of the following nations EXCEPT ________ have recently utilized export-led growth policies. A) Hong Kong B) South Korea C) Argentina D) Singapore E) Taiwan Answer: C Difficulty: Easy AACSB: Application of knowledge 12) Asia's economic takeoff proves that A) trade policy is the key to successful economic growth. B) trade policy is irrelevant to successful economic growth. C) high educational standards is the key to successful economic growth. D) dual economies must suffer economic stagnation. E) trade policy can lead to a rapid rise in the standard of living of many people in developing countries. Answer: E Difficulty: Easy AACSB: Application of knowledge
12 Copyright © 2022 Pearson Education, Inc.
13) India, the world's second most populous country, since 1980 has become A) a growing force in world trade, especially in new forms of trade that involve information rather than physical goods. B) an insignificant part in world trade, mainly trading physical goods. C) a closed economy to world trade, especially with higher trade barriers and stricter government regulations. D) a minor player in world trade, mainly trading agricultural goods and raw materials. Answer: A Difficulty: Easy AACSB: Application of knowledge 14) The Indian city of Bangalore has become famous for its growing role in the A) global information technology industry. B) global food and agriculture industry. C) global steel industry. D) global movie industry. Answer: A Difficulty: Easy AACSB: Application of knowledge 15) Discuss reasons for India's slow growth before 1980s and the major policy changes since then. Answer: India's economy performed very poorly before 1980s with an economic growth rate of only about 1 percent point higher than population growth. This slow growth was widely attributed to the stifling effect of bureaucratic restrictions and little participation in world trade. After the country achieved independence in 1948, its leaders adopted a particularly extreme form of import-substituting industrialization as the country's development strategy: India imported almost nothing that it could produce domestically, even if the domestic product was far more expensive and of lower quality than what could be bought abroad. High costs, in turn, crimped exports. So India was a very "closed" economy. In the 1970s, imports and exports averaged only about 5 percent of GDP, close to the lowest levels of any major nation. Then everything changed when India's growth accelerated dramatically: GDP per capita, which had risen at an annual rate of only 1.3 percent from 1960 to 1980, has grown at close to 4 percent annually since 1980. And India's participation in world trade surged as tariffs were brought down and import quotas were removed. In short, India has become a high-performance economy. Why India's growth rate has increased so dramatically is still debatable among economists. Some have argued that trade liberalization, which allowed India to participate in the global economy, was crucial. Others point out that India's growth began accelerating around 1980, whereas the big changes in trade policy didn't occur until the beginning of the 1990s. Difficulty: Easy AACSB: Application of knowledge
13 Copyright © 2022 Pearson Education, Inc.
16) Classical and Neoclassical trade theory makes the case that free trade can bring a country to an optimum and economically efficient use of its resources; and hence is an optimal trade-policy, if the objective is maximizing long term economic growth. There are those who argue that the experience of the Asian Miracle countries, such as Taiwan, South Korea and Singapore verify this argument in the real world. Explain. There are others who argue that the experience of these countries cannot be used to verify or support the argument above. Explain. Answer: Both arguments may indeed be made. These countries did in fact tend to promote exports in a consistent set of policies. However, they tended to do this via conscious National industrial-policy (e.g., South Korea using "the Japanese system"), and hence (with the possible exception of Hong Kong) do not provide a good test-tube for the long run effects of a free-trade stance. Difficulty: Moderate AACSB: Application of knowledge 17) The consensus today is that import-substitution protectionist industrial policy has not served the developing countries' growth ambitions well. This fact proves that policies relying on exportdriven growth are the "winning ticket" for these countries. Answer: Although there are many who draw precisely this lesson from the " East Asian Miracle" of the past half-century, such a conclusion does not necessarily follow logically. Although the four HPAEs succeeded in their economic as well as in their export sector growth, they differed among themselves considerably in the degree and manner with which they abjured protectionist policies. In any case, export-promotion policies may distort relative prices to the same extent as import protectionist policies, and hence may lead to the same waste and misallocation of national resources. Difficulty: Moderate AACSB: Application of knowledge
14 Copyright © 2022 Pearson Education, Inc.
International Economics: Theory and Policy, 12e (Krugman) Chapter 12 Controversies in Trade Policy 12.1
Sophisticated Arguments for Activist Trade Policy
1) Benefits that accrue to parties other than the firms that produce them are called A) positive externalities. B) free rider. C) public goods. D) marginal utility. Answer: A Difficulty: Easy AACSB: Application of knowledge 2) There is a good case for subsidizing the industry A) if firms in an industry generate knowledge that other firms can use as externalities. B) if firms in an industry generate knowledge that other firms cannot use. C) if firms in an industry do not generate any new knowledge to be shared. D) if firms in an industry generate knowledge at a higher cost than benefits. Answer: A Difficulty: Easy AACSB: Application of knowledge 3) If firms in an industry are generating knowledge that other firms can use without paying for it, this industry is characterized by A) social costs that exceed private costs. B) social benefits that exceed private benefits. C) social costs that exceed social benefits. D) private benefits that exceed social benefits. E) social benefits that undermine private benefits. Answer: B Difficulty: Easy AACSB: Application of knowledge 4) There is a potential market failure arising from difficulties of appropriating knowledge A) supports the conclusions of the Heckscher-Ohlin model. B) rejects the usefulness of government protectionism. C) supports the concept that the government should support only high-tech industries. D) provides support for government protectionism. E) supports arguments for free trade. Answer: D Difficulty: Easy AACSB: Application of knowledge
1 Copyright © 2022 Pearson Education, Inc.
5) Which of the following statements is NOT true? A) It is always easy to identify the knowledge generation and let the government subsidize. B) Trade and industrial policy should be targeted specifically on the activity in which the market failure occurs. C) Policy should seek to subsidize the generation of knowledge that firms cannot appropriate. D) Innovation and technological spillovers can happen even in non-high-tech industries. Answer: A Difficulty: Easy AACSB: Application of knowledge 6) It is argued that high-tech industries typically generate new technologies but cannot fully appropriate the commercial benefits associated with their inventions or discoveries. If this is TRUE, then in order to maximize a country's real income, the government should A) tax the high-tech firms. B) subsidize the high-tech firms. C) protect the high-tech firms. D) outsource high-tech production. E) discourage high-tech investments. Answer: B Difficulty: Easy AACSB: Application of knowledge 7) Since 2015, the United States has developed A) a large trade deficit in R&D-intensive products. B) a large trade surplus in R&D-intensive products. C) a small trade deficit in R&D-intensive products. D) a small trade surplus in R&D-intensive products. Answer: A Difficulty: Easy AACSB: Application of knowledge 8) Since 2000, the number of workers producing computers and related goods in the United States A) has fallen sharply, outpacing the general decline in manufacturing employment. B) has increased sharply, outpacing the general increase in manufacturing employment. C) has fallen sharply despite the general increase in manufacturing employment. D) has increased sharply despite the general decline in manufacturing employment. Answer: A Difficulty: Easy AACSB: Application of knowledge
2 Copyright © 2022 Pearson Education, Inc.
9) The Brander-Spencer model identified market failure in certain industries due to A) unfair competition. B) wildcat destructive competition. C) environmental negative externalities associated with pollution. D) limited competition. E) lack of excess returns. Answer: D Difficulty: Easy AACSB: Application of knowledge 10) Profits firms will make above what equally risky investments elsewhere in the economy can earn are called A) excess returns. B) normal returns. C) economic returns. D) opportunity cost. Answer: A Difficulty: Easy AACSB: Application of knowledge 11) Spencer and Brander's model highlights the existence of A) aircraft industries. B) excess returns present in highly competitive markets. C) excess returns, or rents, available in non-competitive markets. D) the futility of government bureaucrats' attempts to build an airplane. E) natural advantages in foreign technology firms. Answer: C Difficulty: Easy AACSB: Application of knowledge 12) The best economic case one can make for an active industrial policy involves A) the national security argument. B) the technological spillover argument. C) the environment preservation argument. D) the high value added argument. E) raising the national income. Answer: E Difficulty: Easy AACSB: Application of knowledge
3 Copyright © 2022 Pearson Education, Inc.
13) The reason Airbus succeeded in the Brander Spencer example is that A) Boeing made the first move in this strategic game. B) Europeans tend to be better strategists than corn-fed Americans. C) the Airbus actually was a better plane than the Boeing 747. D) U.S. laws actually prohibit U.S. exporters from bribing foreign officials. E) the subsidy removed the advantage that Boeing gained with their head start in production. Answer: E Difficulty: Easy AACSB: Application of knowledge 14) The reason Airbus succeeded in the Brander Spencer example is that A) the European government made an explicit subsidy offer, but the U.S. government did not. B) Airbus' prices were better when adjusted for quality and warranty services. C) Boeing traditionally refused to undertake any exchange rate risk in its transactions. D) the U.S. acted in accordance with its ideological reliance on market solutions, whereas the Europeans ignored market and technological factors. E) the Airbus plane benefited from more advanced technology. Answer: A Difficulty: Easy AACSB: Application of knowledge 15) In the Brander-Spencer model the subsidy raises profits by more than the subsidy because of A) the "multiplier" effect of government expenditures. B) the military-industrial complex. C) the forward and backward linkage effects of certain industries. D) the deterrent effect of the subsidy on foreign competition. E) the economies of scale once the company enters the market. Answer: D Difficulty: Easy AACSB: Application of knowledge 16) Spencer and Brander's model highlights the conventional assumption that A) government involvement in business or in the economy tends to fail. B) government subsidies tend to waste taxpayer's money. C) government subsidies cannot create a successfully competing export. D) government tends to distort when it displaces Adam Smith's Invisible Hand. E) government subsidies can produce profits that exceed the subsidy's value. Answer: E Difficulty: Easy AACSB: Application of knowledge
4 Copyright © 2022 Pearson Education, Inc.
17) Criticisms of the Brander-Spencer model include all EXCEPT which of the following? A) the problem of insufficient information B) the problem of likely foreign retaliation C) the problem of harm to interests of consumers D) the problem of adverse effects of trade policy politics E) the problem of simultaneously causing harm to other industries Answer: C Difficulty: Easy AACSB: Application of knowledge 18) The argument that strategic planning is NOT likely to be practical due to insufficient information means that A) because of trade secrets, the government does not know true cost relationships in any given industry. B) if the government had all the relevant information in a given industry then it could decide whether a subsidy would enhance the public's welfare. C) even if the government had all the relevant information in a given industry, it still could not decide whether a subsidy would enhance the public's welfare. D) due to recent cuts in the Department of the Census' sampling budgets, industry surveys are no longer reliable, so that there is no way to determine if a subsidy is in the public's interest. E) the government would need to employ its intelligence agencies in order to gain a complete understanding of the market. Answer: C Difficulty: Easy AACSB: Application of knowledge 19) The invocation of beggar-thy-neighbor arguments with respect to industrial policies A) strengthens the argument for subsidies. B) makes sense if the international Keynesian multipliers exceed unity. C) applies only to rich countries most of whose trade partners are very poor countries. D) weakens the argument for subsidies. E) does not apply to rich countries who can influence relative world prices. Answer: D Difficulty: Easy AACSB: Application of knowledge 20) Andy Grove, co-founder of Intel, argued that A) technological spillovers require more than researchers; they require the presence of large numbers of workers putting new ideas to work. B) startups are a wonderful thing, and they can themselves increase tech employment. C) scaling is easy work, especially for young startups. D) as technology goes from prototype to mass production, they do not require the presence of large numbers of workers putting new ideas to work. Answer: A Difficulty: Easy AACSB: Application of knowledge 5 Copyright © 2022 Pearson Education, Inc.
21) When Japan's MITI (Ministry of International Trade and Industry) focused resources on the semiconductor industry, and in particular on Random Access Memory (RAM), it was viewed as a typically successful Japanese foray into a new dynamic strategic sector. The results, as viewed by the late 1990s A) justified this view. B) led to similar structuring of industrial policy in the U.S. C) lent support to the Brander-Spencer model. D) helped shift the focus of economists away from Japanese-style industrial policy. E) propelled Japan into the leading country in high-tech manufacturing. Answer: D Difficulty: Easy AACSB: Application of knowledge 22) Japan's protection of its semiconductor (RAM) producers is today seen as an object lesson in A) how strategic planning may backfire and cause a large waste of resources. B) how externalities may be successfully exploited by protectionist policies. C) how excess returns may be successfully exploited by protectionist policies. D) how government intervention may create a meaningful comparative advantage. E) how monopolies can outlast government intervention. Answer: A Difficulty: Easy AACSB: Application of knowledge 23) It may be argued that Japan's explicit promotion of its microchip industry was an excellent example of successful industrial policy. What criteria would you apply to determine whether such a policy is or is not successful? Judging from your own stated criteria, was Japan's exercise successful? Why or why not? What information would a government require in order to increase the probability that its industrial policy would promote long-term self-generated economic growth? Answer: It is argued that Japan's subsidies to its nascent microchip industry was an important factor in putting Japan on the world map in this area. However, a minimal criteria for a successful industrial policy would be that the infant industry mature, and that it prove to be a profitable area of the country's comparative advantage. In this case, one might argue that the latter part of the above statement was not fulfilled, since the microchip industry was adopted by so many countries, that it became a "commodity." That is, it became a product with a very low profit margin, which was not really a good use of Japan's resources, given their alternative uses. Difficulty: Moderate AACSB: Application of knowledge
6 Copyright © 2022 Pearson Education, Inc.
Suppose there is a new product, a superjumbo aircraft, that both firms Boeing and Airbus are capable of making. Assume each firm can make only a yes/no decision: to either produce superjumbo aircraft or not. Either firm alone could earn profits making superjumbo aircraft, but if both firms try to produce them, both will incur losses. Table 12-1 below illustrates how the profits earned by the two firms might depend on their decisions.
24) Refer to the above table. Suppose Airbus is able to get a small head start and commits itself to produce superjumbo aircraft before Boeing can get going. Will Boeing enter the market? Explain why. Answer: Only Airbus will produce, and Boeing will not. Boeing will find that it has no incentive to enter the market. The outcome will be in the lower left corner of the table, with Airbus earning profits. Difficulty: Moderate AACSB: Application of knowledge 25) Refer to the above table. Suppose Airbus is able to get a small head start and commits itself to produce superjumbo aircraft before Boeing can get going. Assume both European and US. governments offer their respective company a subsidy of $4 (million). Will Boeing enter the market? Explain why. Answer: Only Airbus will produce since it knows that the subsidy would not be sufficiently large to entice Boeing to also enter the market. To avoid a potential loss of $1 million to produce the superjumbo aircraft even after the government's subsidy, Boeing will find that it has no incentive to enter. Difficulty: Moderate AACSB: Application of knowledge 26) Refer to the above table. Suppose both governments offer their respective company a $10 million subsidy. Which company will enter the market? Answer: Both companies would enter the market, since each knows that regardless of the other's decision, it will make some profit here. Difficulty: Moderate AACSB: Application of knowledge
7 Copyright © 2022 Pearson Education, Inc.
27) Refer to the above table. Suppose the U.S. government (but not Europe) offers a $10 million subsidy. Which company will enter the market? Answer: With the U.S. government's $10 million subsidy, it will be profitable for Boeing to produce the superjumbo aircraft whatever Airbus does. In this case Airbus would decide not to enter the market since it knows Boeing will, and that therefore its own production will entail a loss of $5 million. Difficulty: Moderate AACSB: Application of knowledge 28) Refer to the above table. How could the U.S. government justify its decision to offer a subsidy to a profitable and successful business? Answer: It could point out that this $10 million pump-priming expenditure results in a profit of $110 million. If Boeing paid a marginal income tax of 20%, this would net the government $55 million, which is more than 5 times the original subsidy, so that the decision may be justified not only in terms of benefit/cost considerations, but even in terms of pure budgetary terms. Difficulty: Moderate AACSB: Application of knowledge 12.2
Globalization and Low-Wage Labor
1) In today's world markets, poor developing countries have seen the rise in exports of A) agricultural products. B) primary products. C) mineral products. D) manufactured products. E) high-tech products. Answer: D Difficulty: Easy AACSB: Application of knowledge 2) Before 1995, most complaints about international trade targeted its effects A) on wages of less-skilled workers of advanced countries. B) on wages of less-skilled workers of developing countries. C) on wages of high-skilled workers of advanced countries. D) on wages of high-skilled workers of developing countries. Answer: A Difficulty: Easy AACSB: Application of knowledge
8 Copyright © 2022 Pearson Education, Inc.
3) In the second half of the 1990s a rapidly growing movement focused on the harm caused by international trade to A) land owners in poor countries. B) capital owners in rich industrialized countries. C) land owners in rich industrialized countries. D) production workers in the developing countries. E) terms of trade in developing countries. Answer: D Difficulty: Easy AACSB: Application of knowledge 4) The Ricardian model of comparative advantage lends support to the argument that A) trade tends to worsen the conditions of unskilled labor in rich countries. B) trade tends to worsen the conditions of owners of capital in rich countries. C) trade tends to worsen the conditions of workers in poor countries. D) trade tends to worsen the conditions of workers in rich countries. E) trade is mutually beneficial to the countries that engage in it. Answer: E Difficulty: Easy AACSB: Application of knowledge 5) Critics argued that A) globalization was not helping workers in developing countries who suffered from low wages and associated poor working conditions. B) globalization was helping workers in developing countries to earn higher wages under better working conditions. C) globalization lowered further wages and worsened working conditions of workers in developing countries. D) globalization did not increase employment in developing countries. Answer: A Difficulty: Easy AACSB: Application of knowledge 6) Most developing countries oppose including labor standards in trade agreements because A) they believe this would involve a loss of their national sovereignty. B) they believe this would be used as a protectionist tool by advanced countries. C) they believe this would create an uneven playing field. D) multinational corporations control them. E) they do not want to improve wages for their workers. Answer: B Difficulty: Easy AACSB: Application of knowledge
9 Copyright © 2022 Pearson Education, Inc.
7) In the five years following the signing of the North American Free Trade Agreement, Mexico's maquiladoras, factories near the U.S. border A) had expanded rapidly, roughly doubling in employment. B) had shrunk rapidly, roughly cutting employment in half. C) had offered workers same wage levels as in the U.S. D) had recruited many American workers from across the border. Answer: A Difficulty: Easy AACSB: Application of knowledge 8) Opponents of the free trade agreement argued that by making it easier for employers to replace high-wage workers in the United States with lower-paid workers in Mexico, the NAFTA agreement A) had hurt labor on both sides of the border. B) had made only American workers winners. C) had made only Mexican workers winners. D) had made both America and Mexico's trade balance worse off. Answer: A Difficulty: Easy AACSB: Application of knowledge 9) Which of the following statements is a misconception? A) Trade must involve the exploitation of workers if they earn much lower wages than their counterparts in a richer country. B) Trade can be mutually beneficial to the countries that engage in it. C) Workers in a labor-abundant low-wage country should benefit from a shift in the distribution of income in their favor by trade. D) Workers in a labor-scarce high wage country might be hurt by trade with a labor-abundant country like Mexico. Answer: A Difficulty: Easy AACSB: Application of knowledge 10) In the specific case of the maquiladoras, some economists A) shrug their shoulders and ignore the issue. B) agree that trade theory is thus proven hollow and internally inconsistent. C) argue that U.S. consumers should not consume lettuce. D) argue that while wages and working conditions may appear terrible, they represent an improvement over the alternatives available in Mexico. E) argue that Mexico's generally high overall productivity offsets these conditions. Answer: D Difficulty: Easy AACSB: Application of knowledge
10 Copyright © 2022 Pearson Education, Inc.
11) The evidence usually cited to prove that globalization hurts workers in developing countries A) is inconclusive due to poor statistical design of the underlying samples. B) is inconclusive due to the poorly funded Central Statistical Office of Mexico. C) is inconclusive due to the ambiguous theoretical implications of the findings. D) is conclusive. E) does not take into account the Heckscher-Ohlin model. Answer: C Difficulty: Easy AACSB: Application of knowledge 12) When one applies the Heckscher-Ohlin model of trade to the issue of trade-related income redistributions, one must conclude that North-South trade, such as U.S.-Mexico trade, A) must help low-skilled workers on both sides of the border. B) is likely to hurt high-skilled workers in the U.S. C) is likely to involve higher overall national economic gains that will be greater than any harm done to low-skilled workers in the U.S. D) is likely to hurt low-skilled workers in Mexico. E) gives no advantage to the workers in either country. Answer: C Difficulty: Easy AACSB: Application of knowledge 13) When the WTO met in Seattle to initiate a further move towards free international trade, thousands of activists met A) in order to promote the WTO's goals of "Trade-not Aid." B) in order to laud the WTO policy orientation which would bust local monopolies and therefore help ordinary relatively poor consumers everywhere. C) in order to laud the WTO policy of disallowing government sweetheart deals, which typically meant that corrupt governments subsidized their in-laws' conglomerates on the backs of poor taxpayers. D) in order to support the WTO efforts of bringing about a universal shift of resources in poor countries to higher efficiency and productivity uses, which would raise the real incomes of everyone. E) in order to protest WTO free trade policies that they believed hurt workers. Answer: E Difficulty: Easy AACSB: Application of knowledge
11 Copyright © 2022 Pearson Education, Inc.
14) Labor standards in trade are A) strongly opposed by most developing countries, which believe that the standards would inevitably be used as a protectionist tool. B) strongly supported by most developing countries, which believe that the benefits outweigh the costs. C) strongly opposed by most developed countries, which believe that costs outweigh the benefits. D) of no concern and interest to both developing and developed nations. Answer: A Difficulty: Easy AACSB: Application of knowledge 15) The proposal that trade agreements should include a system which monitors worker conditions and make the results available to consumers in the rich importing country A) is consistent with the Invisible Hand paradigm. B) is consistent with the market failure approach. C) is consistent with the Ricardian theory of comparative advantage. D) is consistent with the scale economies approach to trade theory. E) is consistent with the principles laid out by the WTO. Answer: B Difficulty: Easy AACSB: Application of knowledge 16) Labor standards in trade are typically opposed by most developing countries who believe that they will be used A) to do harm to national history and civil society. B) as a protectionist tool by import-competing producers in industrial countries. C) as a means of spreading U.S. corporate values and destroying local cultures. D) to hinder investment in foreign-based multinational corporations. Answer: B Difficulty: Easy AACSB: Application of knowledge 17) Which of the following statements is NOT true? A) Globalization is always good for the environment. B) Environmental standards in developing-country export industries are much lower than in advanced country industries. C) Substantial environmental damage has been done to provide goods to advanced-country markets. D) Globalization has led to cultural homogenization. Answer: A Difficulty: Easy AACSB: Application of knowledge
12 Copyright © 2022 Pearson Education, Inc.
18) The WTO seems at times to be interfering in domestic policy since A) the line between domestic policies and de factor protectionism is often fuzzy. B) it is a supra-national organization with the power to overturn governments. C) it determines which nations may trade what with whom. D) it punishes naughty nations. E) it exempts the U.S. and other powerful member nations from many of its edicts. Answer: A Difficulty: Easy AACSB: Application of knowledge 19) One complaint from the anti-globalization movement is that A) the drive for free trade and free flow of capital has undermined national sovereignty. B) WTO does not look like a world government. C) the drive for free trade and free flow of capital has reinforced national sovereignty. D) WTO does not seem to interfere in domestic policy. Answer: A Difficulty: Easy AACSB: Application of knowledge 20) Bangladeshi impressive economic growth from 1992 to 2010 A) relied on rising exports, specifically exports of apparel. B) relied on rising imports, specifically imports of apparel. C) relied on its own resources without trade. D) relied on its high wages and excellent working conditions. Answer: A Difficulty: Easy AACSB: Application of knowledge 21) Working conditions for clothing workers in Bangladesh are very poor. If countries refuse to buy clothing from Bangladesh in order to encourage change, the effect is likely to be that A) firms will be forced to comply and workers will be better off. B) firms will refuse to comply, but workers will be better off. C) firms will try to comply and workers will be worse off. D) firms will try to comply and workers will be better off. E) regardless of how firms respond, workers will be better off. Answer: C Difficulty: Easy AACSB: Application of knowledge
13 Copyright © 2022 Pearson Education, Inc.
22) How did manufactured exports shift in the world economy over the last generation? Is it good or bad for developing countries? Explain. Answer: Developing countries has seen a major shifts to more exports of manufactured goods and less exports of traditional agricultural or mineral products. This seems to be a good trend for developing countries since prices of agricultural and mineral products are usually volatile. However, the concerns remain regarding the low wages and poor working conditions for workers in third-world factories. Difficulty: Easy AACSB: Application of knowledge 23) What was the difference in anti-globalization movement before and after 1995? Answer: Before 1995, most complaints about international trade made by citizens of advanced countries targeted its effects on people who were also citizens of advanced countries. Most critics of free trade in the 1980s focused on the alleged threat of competition from abroad and concern over the effects of imports from low-wage countries on the wages of less-skilled workers at home. After 1995, activists pointed to the low wages and poor working conditions in third-world factories that produced goods for Western markets, stressing the alleged harm that world trade was doing to workers in the developing countries. Difficulty: Easy AACSB: Application of knowledge 24) Describe the tragedy that happened on April 24, 2013 in Bangladesh. What can be done to avoid future tragedies like that? Answer: On April 24, 2013, an eight-story building in Bangladesh, containing a number of garment factories, had collapsed, killing more than 1,200 people. It showed that those people were under unsafe and poor working conditions with low wages. Clearly, Bangladesh needs to take steps to protect its workers, starting by enforcing its own building and worker-safety laws. Some of the suggestions on how this can be done include approaches either by law or simply through pressure from consumers, trade partners, and other countries in the global community to require basic standards for working conditions that apply not only to Bangladesh but also to its competitors. Provided that they're not too ambitious, such standards could make life better for Bangladeshi workers without destroying the exports the country relies on. Difficulty: Easy AACSB: Application of knowledge
14 Copyright © 2022 Pearson Education, Inc.
12.3
Globalization and the Environment
1) Free trade and globalization is generally believed A) to cause a degradation in the world's environment. B) to improve the environment by correcting for distortions caused by import competing policies. C) to help spread the best of each country's culture, so as to uplift global cultural standards. D) to help each country safeguard the best of its own culture. E) to make no difference in the economic welfare of the world. Answer: A Difficulty: Easy AACSB: Application of knowledge 2) Other things equal, economic growth, which increases both production and consumption, A) leads to greater environmental damage. B) leads to less environmental damage. C) has no impact on the environment. D) benefits the environment in the short run and long run. Answer: A Difficulty: Easy AACSB: Application of knowledge 3) Which of the following statements is NOT true? A) When countries become richer and strive for sustainable development, the negative environmental impact tend to increase. B) When countries become more devoted to the production of services rather than goods as they grow richer, they tend to use less energy and raw materials per dollar of GDP. C) Growing wealth tends to lead to growing political demands for environmental quality. D) Rich countries generally impose stricter regulations to ensure clean air and water than poorer countries. Answer: A Difficulty: Easy AACSB: Application of knowledge 4) A distinctive "inverted U" relationship between per-capita income and environmental damage is known as the A) environmental Kuznets curve. B) Laffer curve. C) Average Product of Labor curve. D) Lorenz curve. Answer: A Difficulty: Easy AACSB: Application of knowledge
15 Copyright © 2022 Pearson Education, Inc.
5) The Kuznets curve's inverted U shape illustrates that as economies grow, A) they initially do increasing environmental damage, but they become more environmentally friendly once they become sufficiently rich. B) the environmental damage keeps increasing forever. C) the environmental damage disappeared completely. D) they initially become more environmentally friendly, but they do increasing environmental damage once they become sufficiently rich. Answer: A Difficulty: Easy AACSB: Analytical thinking 6) At a world level, A) globalization had indeed harmed the environment. B) globalization has done no harm to the environment. C) globalization always protects the environment. D) globalization has no environmental impact. Answer: A Difficulty: Easy AACSB: Application of knowledge 7) The rapid economic growth of China A) has turned it from a minor factor in climate change to the world's largest emitter of carbon dioxide. B) has turned it from the world's largest emitter of carbon dioxide to a minor factor in climate change. C) has been associated with a huge decrease in its emission of carbon dioxide. D) has no impact on the world's emission of carbon dioxide. Answer: A Difficulty: Easy AACSB: Application of knowledge 8) A cost that individuals impose on others but don't pay for is an example of a A) negative externality. B) positive externality. C) neutral externality. D) environmental externality. Answer: A Difficulty: Easy AACSB: Application of knowledge
16 Copyright © 2022 Pearson Education, Inc.
9) Whether nations have a legitimate interest in each other's environmental policies depend on A) the nature of the environmental problem because different forms of pollution have very different geographical reach. B) the good wills of nations. C) the living standard in each nation because different living standards correspond to different levels of pollution tolerance. D) the priorities in each nation's policy agenda. Answer: A Difficulty: Easy AACSB: Application of knowledge 10) The Indian shipbreakers of Alang are A) a metaphysical representation of the WTO, deriving from Edgar Rice Burroughs' Princess of Mars. B) an early version of the Russian Ice-breaker of the Dnieper-Alang class. C) a capital-intensive industry. D) examples of what is going on in a pollution haven. E) doing environmentally conscious work. Answer: D Difficulty: Easy AACSB: Application of knowledge 11) The Shipbreakers of Alang utilize much labor and little capital, thereby supporting the applicability of the A) factor proportions explanation of the sources of comparative advantage. B) specific factor theory of comparative advantage. C) monopolistic competition theory of comparative advantage. D) scale economies theory of comparative advantage. E) basis of the non-dumping legislation. Answer: A Difficulty: Easy AACSB: Application of knowledge 12) The Shipbreakers of Alang arouse the ire of Greenpeace because of A) India's non-repentant nuclear stance. B) India's import-competing industrialization policies. C) the difficulty of avoiding ship accidents between Greenpeace's sailboat and the reconstructed container ships of Alang. D) the large amount of pollution associated with the operations at Alang. E) their competition with capital-intensive industries. Answer: D Difficulty: Easy AACSB: Application of knowledge
17 Copyright © 2022 Pearson Education, Inc.
13) The Shipbreakers of Alang represent a perfect example of how a developing country can apply the principles of the Heckscher-Ohlin model, since A) shipbreaking is generally considered to be a capital-intensive operation and India, being a large country has much capital. B) shipbreaking is a labor-intensive operation in India, and India has many workers since it is such a large country. C) shipbreaking is a labor-intensive operation in India, and India's availability of capital per worker is less than that of its trade partners. D) shipbreaking is a capital-intensive operation elsewhere in the world, and therefore represents a case of a factor intensity reversal. E) India's climate lends itself to the work involved in shipbreaking. Answer: C Difficulty: Moderate AACSB: Application of knowledge 14) The climate-change legislation can be effective only A) if all major economies adopt similar policies. B) if few countries take action. C) if no country reduces greenhouse gas emission. D) if there is a unilateral reduction in emission by the United States. Answer: A Difficulty: Easy AACSB: Application of knowledge 15) Carbon tariffs are supposed A) to charge importers of goods from countries without climate-change policies an amount proportional to the carbon dioxide emitted in the production of those goods. B) to increase greenhouse gas emission. C) to shift polluting production to countries with lax regulation. D) to require both producers and consumers pay the highest price for greenhouse emission. Answer: A Difficulty: Easy AACSB: Application of knowledge 16) It is still the conventional wisdom in the U.S. that compliance with NAFTA requirements is having a deleterious effect on U.S. highway safety standards, on U.S. pollution and other environmental standards, and on U.S. jobs. What facts would proponents of an expansion of NAFTA (e.g., to include all of Central and South American countries) need to marshall in order to convince you? Answer: The answer is subjective. Presumably the answer should include reasonable and objective counter-factual scenarios (what would be the job or pollution situation with and without NAFTA). Difficulty: Moderate AACSB: Application of knowledge
18 Copyright © 2022 Pearson Education, Inc.
17) Describe the environmental Kuznets curve. Answer: This curve shows the effect of increased per capita income on environmental damage. It has an inverted U shape that indicates that increased per capita income first increases and then decreases environmental damage. Difficulty: Moderate AACSB: Application of knowledge 18) What is a pollution haven? Answer: A pollution haven is a country with relatively lax environmental standards or enforcement. In effect, countries with strict standards export activities that pose high environmental risks to countries that are willing to accept the risks. Difficulty: Moderate AACSB: Application of knowledge 19) Why may U.S. carbon tariffs fail to create a cap-and-trade system for lowering global greenhouse gases? Answer: A unilateral reduction in emissions by the United States would have only a limited effect on global emissions, and hence on future climate change. Policies that put a high price on carbon might make the pollution haven effect much larger than it has been so far, leading to "carbon leakage" as emissions-intensive industries relocate to countries without strong climatechange policies. To make the initiative global, it is necessary to have all major economies adopt similar policies. Difficulty: Easy AACSB: Application of knowledge 12.4
Trade Shocks and Their Impacts on Communities
1) Which of the following statements is TRUE? A) Increased trade can shift the distribution of income within countries and create losers as well as winners. B) Increased trade has no effect on the distribution of income within countries. C) Increased trade can shift the distribution of income within countries and create only winners. D) Increased trade can shift the distribution of income within countries and create only losers. Answer: A Difficulty: Easy AACSB: Application of knowledge 2) Rapid growth of Chinese exports after 1990, and especially after 2001, when China joined the WTO, A) created much more hardship in the United States than most economists had realized. B) created only losers in the United States as most economists had expected. C) created only winners in the United States as most economists had expected. D) created much more trade favor in the United States than most economists had realized. Answer: A Difficulty: Easy AACSB: Application of knowledge 19 Copyright © 2022 Pearson Education, Inc.
3) Explain why David Autor, David Dorn, and Gordon Hanson argued that surging Chinese exports had a bigger impact on American workers than looking at overall numbers might have suggested? Answer: Chinese export growth was very uneven across industries. Many U.S. manufacturing industries are or were very highly concentrated geographically, so the outsized China impact on certain industries fell heavily on some communities while largely passing others by. Finally, U.S. workers and families are much less willing or able to move away from depressed regions than one might have expected. Therefore, rapid changes in international trade and the impacts from surging Chinese exports are more painful than economists had realized. Difficulty: Easy AACSB: Application of knowledge
20 Copyright © 2022 Pearson Education, Inc.
International Economics: Theory and Policy, 12e (Krugman) Chapter 13 National Income Accounting and the Balance of Payments 13.1
The National Income Accounts
1) Between 2014 and 2019, the world's economy total real product grew at an annual average rate of nearly ________ percent per year. A) 3.5 B) 3.9 C) 2.9 D) 2.5 Answer: A Difficulty: Easy AACSB: Application of knowledge 2) Macroeconomic analysis emphasizes four aspects of economic life which are A) unemployment, saving, trade imbalances, money and the price level. B) market structures, social inequality, inflation, balance of payments. C) firms' competition, growth, trade imbalances, inflation. D) happiness and utilities, trade imbalances, equality, GDP growth. Answer: A Difficulty: Easy AACSB: Application of knowledge 3) Two related and essential tools to get a complete picture of the macroeconomic linkages among economies that engage in international trade include A) national income accounting and balance of payment accounting. B) public accounting and government accounting. C) foreign currency accounting and balance of payment accounting. D) corporate accounting and foreign exchange accounting. Answer: A Difficulty: Easy AACSB: Application of knowledge
1 Copyright © 2022 Pearson Education, Inc.
4) A country's gross national product (GNP) is A) the value of all final goods and services produced by its factors of production and sold on the market in a given time period. B) the value of all intermediate goods and services produced by its factors of production and sold on the market in a given time period. C) the value of all final goods produced by its factors of production and sold on the market in a given time period. D) the value of all final goods and services produced by its factors of production and sold on the market. E) the value of all final goods and services produced by its factors of production, excluding land, and sold on the market in a given time period. Answer: A Difficulty: Easy AACSB: Application of knowledge 5) For most macroeconomists A) national income accounts is much more important than national output accounts. B) it is impossible to tell whether national income accounts equal to national output accounts. C) national output accounts exceed national income accounts. D) national income accounts exceed national output accounts. E) national income accounts, rather than national output accounts are used to describe types of expenditure that make up a country's GNP. Answer: E Difficulty: Easy AACSB: Application of knowledge 6) GNP is divided into four possible uses for which a country's final output is purchased: A) consumption, investment, government purchases, and the current account balance. B) salaries and wages, rent, interest, and profits. C) total national income, sales taxes, depreciation, and net foreign factor income. D) government purchases, corporate investment, exports, imports. Answer: A Difficulty: Easy AACSB: Application of knowledge 7) For most macroeconomists A) gross national income exceeds gross national product. B) it is hard to tell whether gross national income equals gross national product. C) gross national product exceeds gross national income. D) gross national product is much more important than gross national income. E) gross national income and gross national product are used interchangeably. Answer: E Difficulty: Easy AACSB: Application of knowledge
2 Copyright © 2022 Pearson Education, Inc.
8) The sale of A) a used textbook does enter GNP. B) a used textbook does not enter GNP, but the sale of a used house does. C) both a used textbook and a used house do not enter GNP. D) a used house does not enter GNP, but the sale of a used book does. E) the GNP does not include sale of used items priced below $1000. Answer: C Difficulty: Easy AACSB: Application of knowledge 9) Which one of the following statements is the MOST accurate? A) The sale of a used textbook does generate income for factors of production. B) The sale of a used textbook does not generate income for any factor of production. C) The sale of a used textbook sometimes does and sometimes does not generate income for factors of production. D) It is hard to tell whether a sale of a used textbook does or does not generate income for factors of production. E) The sale of a used textbook is a part of the GNP. Answer: B Difficulty: Easy AACSB: Application of knowledge 10) Which one of the following statements is the MOST accurate? A) GNP plus depreciation is called net national product (NNP). B) GNP less depreciation is called net national product (NNP). C) GNP less depreciation is called net factor product (NFP). D) GDP plus depreciation is called net national product (NNP). E) GDP less depreciation is called net national product (NNP). Answer: B Difficulty: Easy AACSB: Application of knowledge 11) Net unilateral transfers A) are part of a national income. B) are part of a country's product. C) must be added to NNP in calculations of national income. D) are part of a country's GNP. E) Only A and C. Answer: E Difficulty: Easy AACSB: Application of knowledge
3 Copyright © 2022 Pearson Education, Inc.
12) National income equals GNP A) less depreciation, less net unilateral transfers. B) less depreciation or net unilateral transfers. C) plus depreciation, less net unilateral transfers. D) plus depreciation, plus net unilateral transfers. E) less depreciation, plus net unilateral transfers. Answer: E Difficulty: Easy AACSB: Application of knowledge 13) The United States began to report its gross domestic product (GDP) only since A) 1900. B) 1921. C) 1931. D) 1941. E) 1991. Answer: E Difficulty: Easy AACSB: Application of knowledge 14) GDP is supposed to measure A) the volume of production within a country's borders. B) the volume of services generated within a country's borders. C) the volume of production of a country's output. D) GNP plus depreciation. E) net unilateral transfers from foreigners. Answer: A Difficulty: Easy AACSB: Application of knowledge 15) GNP equals GDP A) minus net receipts of factor income from the rest of the world. B) plus receipts of factor income from the rest of the world. C) minus receipts of factor income from the rest of the world. D) plus net receipts of factor income from the rest of the world. E) minus depreciation. Answer: D Difficulty: Easy AACSB: Application of knowledge
4 Copyright © 2022 Pearson Education, Inc.
16) GDP is different than GNP in that A) it accounts for net unilateral transfers. B) it does not account for indirect business taxes. C) it does not account for a country's production using services with foreign-owned capital. D) it accounts for depreciation. E) it is unhelpful when tracking national income. Answer: C Difficulty: Easy AACSB: Application of knowledge 17) An example of how GNP accounts for services provided by foreign-owned capital (and GDP does not) is A) earnings of a Spanish factory with British owners counts only in Spain's GDP. B) earnings of a Spanish factory with British owners counts only in Britain's GNP. C) earnings of a Spanish factory with British owners counts in Spain's GNP but are part of Britain's GDP. D) earnings of a Spanish factory with British owners counts in Spain's GDP but are part of Britain's GNP. E) earnings of a Spanish factory with British owners counts in Spain's GNP but not in Britain's GDP or GNP. Answer: D Difficulty: Moderate AACSB: Application of knowledge 18) The profits of a Spanish factory with British owners are A) counted in Spain's GNP. B) not part of Britain's GDP. C) counted in Britain's GDP. D) part of Spain's GNP. E) counted in Spain's GDP and are a part of Britain's GNP. Answer: E Difficulty: Moderate AACSB: Application of knowledge 19) The services British capital provides in Spain are a service export from Britain A) therefore they are subtracted from British GDP in calculating British GNP. B) therefore they are added to Spanish GDP in calculating Spanish GDP. C) therefore they are added to British GDP in calculating British GNP. D) therefore they are added to Spanish GNP in calculating Spanish GDP. E) therefore they are subtracted from Spanish GNP. Answer: C Difficulty: Difficult AACSB: Application of knowledge
5 Copyright © 2022 Pearson Education, Inc.
20) Movements in GDP A) differ greatly from movements in GNP. B) do not differ greatly from movements in GNP. C) are not allowed to differ at all from movements in GNP by definition. D) need to be inflation adjusted in order to match movements in GNP. E) are not relevant to an examination of national income. Answer: B Difficulty: Easy AACSB: Application of knowledge 21) What are the main aspects of economic life that macroeconomics analysis is most concerned with? Answer: There are four main aspects: unemployment, saving, trade imbalances, and money and the price level. Difficulty: Easy AACSB: Application of knowledge 22) What can you learn from the figure below (Figure 13.1 from the text) which depicts the U.S. GNP and its components for the 1st quarter of 2020?
Answer: The largest component of GNP is consumption and the U.S. has a negative current account balance. Difficulty: Easy AACSB: Application of knowledge 6 Copyright © 2022 Pearson Education, Inc.
13.2
National Income Accounting for an Open Economy
1) The largest component of GNP is A) the current account. B) investment. C) government purchases. D) consumption. E) trade. Answer: D Difficulty: Easy AACSB: Application of knowledge 2) In the United States over the past seventy years, the fraction of GNP devoted to consumption has fluctuated in a range of about A) 42 to 49 percent. B) 32 to 39 percent. C) 22 to 29 percent. D) 82 to 89 percent. E) 62 to 70 percent. Answer: E Difficulty: Easy AACSB: Application of knowledge 3) Purchases of inventories by A) firms are not counted in investment spending. B) firms are also counted in investment spending. C) households are also counted in investment spending. D) households and Firms are also counted in investment spending. E) foreign consumers are counter in investment spending. Answer: B Difficulty: Easy AACSB: Application of knowledge 4) Investment is usually A) more variable than consumption. B) less variable than consumption. C) as variable as consumption. D) a larger component of the GNP than consumption. E) It is hard to tell from the data whether investment is more or less variable than consumption. Answer: A Difficulty: Easy AACSB: Application of knowledge
7 Copyright © 2022 Pearson Education, Inc.
5) In the United States, (gross) investment has fluctuated between ________ of GNP in recent years. A) 2 and 12 percent B) 11 and 22 percent C) 22 and 32 percent D) 32 and 42 percent E) 42 and 52 percent Answer: B Difficulty: Easy AACSB: Application of knowledge 6) Government purchases are defined as A) only goods purchased by federal, state, or local governments. B) all goods and services purchased by the federal government. C) all goods and services purchased by the federal or state government. D) all goods and services purchased by the federal, state, or local government. E) goods and services purchased from the government. Answer: D Difficulty: Easy AACSB: Application of knowledge 7) Government transfer payments like social security and unemployment benefits are A) included in government purchases. B) not included in government purchases. C) not included in government purchases, but they are included in the consumption component of GNP. D) not included in government purchases, but they are part of the investment component of GNP. E) included in government purchases but not in the GNP. Answer: B Difficulty: Moderate AACSB: Application of knowledge 8) Government purchases currently take up about A) 18 percent of U.S. GNP, and this share has not changed much since the late 1950s. B) 38 percent of U.S. GNP, and this share has not changed much since the late 1950s. C) 18 percent of U.S. GNP, and this share has been increasing since the late 1950s. D) 18 percent of U.S. GNP, and this share has been decreasing since the late 1950s. E) 25 percent of U.S. GNP, and this share has been decreasing since the late 1950s. Answer: A Difficulty: Easy AACSB: Application of knowledge
8 Copyright © 2022 Pearson Education, Inc.
9) In 1929, government purchases accounted for A) only 18.5 percent of U.S. GNP. B) only 8.5 percent of U.S. GNP. C) 28.5 percent of U.S. GNP. D) 38.5 percent of U.S. GNP. E) 48.5 percent of U.S. GNP. Answer: B Difficulty: Easy AACSB: Application of knowledge 10) In 1959, government purchases accounted for A) 22 percent of U.S. GNP. B) 8.5 percent of U.S. GNP. C) 18 percent of U.S. GNP. D) 38.5 percent of U.S. GNP. E) 48.5 percent of U.S. GNP. Answer: A Difficulty: Easy AACSB: Application of knowledge 11) Which one of the following expressions is the MOST accurate? A) CA = EX - IM B) CA = IM - EX C) CA = EX = IM D) CA = EX + IM E) CA - IM = EX Answer: A Difficulty: Easy AACSB: Application of knowledge 12) A country's current account A) balance equals the change in its net foreign wealth. B) balance equals the change in its foreign wealth. C) surplus equals the change in its foreign wealth. D) deficit equals the change in its foreign wealth. E) balance equals its GNP. Answer: A Difficulty: Easy AACSB: Application of knowledge
9 Copyright © 2022 Pearson Education, Inc.
13) In an open economy, the CA is equal to A) Y - (C + I + G). B) Y + (C + I + G). C) Y - (C - I + G). D) Y - (C + I - G). E) Y + (C - I - G). Answer: A Difficulty: Easy AACSB: Application of knowledge 14) In 2019, the United States had A) a surplus in the current account. B) a balance in the current account. C) a deficit in the current account. D) a positive balance of net financial flows. Answer: C Difficulty: Easy AACSB: Application of knowledge 15) The position of the United States current account balance in 2019 was A) lent roughly 6 percent of its GNP, resulting in a large current account surplus. B) borrowed roughly 2.3 percent of its GNP, leading to a large current account deficit. C) achieved a current account balance of zero. D) borrowed roughly 10 percent of its GNP, leading to a large current account deficit. E) borrowed less then 5 percent of its GNP, leading to a large current account surplus. Answer: B Difficulty: Easy AACSB: Application of knowledge 16) Which of the following is TRUE? A) A country with a current account surplus is earning more from its exports than it spends on imports. B) A country could finance a current account deficit by using previously accumulated foreign wealth to pay for its imports. C) A country with a current account deficit must be increasing its net foreign debts by the amount of the deficit. D) We can describe the current account surplus as the difference between income and absorption. E) All of the above are true of current account balances. Answer: E Difficulty: Moderate AACSB: Application of knowledge
10 Copyright © 2022 Pearson Education, Inc.
17) Over the 1980s A) there is no question that a large increase in U.S. foreign assets did occur. B) there is a question whether a large decrease in U.S. foreign assets did occur. C) there is no question that a large decrease in U.S. foreign assets did occur. D) there is no question that there was almost no change in U.S. foreign assets. E) there is no question that rising exports exceeded U.S. foreign debt. Answer: C Difficulty: Easy AACSB: Application of knowledge 18) Which one of the following statements is FALSE? A) The United States had accumulated substantial foreign wealth by the early 1980s. B) The 1980s witnessed a sustained current account deficit of proportions unprecedented in the twentieth century. C) In 1989, the country became a net debtor to foreigners for the first time since World War I. D) U.S. foreign debt has continued to grow and now stands at 50 percent of GNP at the start of 2020. E) The U.S. foreign debt was paid off in the 1990s, allowing the U.S. to attain a current account surplus. However, the deficit has returned in recent years. Answer: E Difficulty: Moderate AACSB: Application of knowledge 19) In a closed economy, national saving A) sometimes equals investment. B) always equals investment. C) is always less than investment. D) is always more than investment. E) is never equal to investment. Answer: B Difficulty: Easy AACSB: Application of knowledge 20) In open economies A) saving and investment are necessarily equal. B) as in a closed economy, saving and investment are not necessarily equal. C) saving and investment are not necessarily equal as they are in a closed economy. D) saving and investment are necessarily equal contrary to the case of a closed economy. E) investment always refers to the domestic stock market. Answer: C Difficulty: Moderate AACSB: Application of knowledge
11 Copyright © 2022 Pearson Education, Inc.
21) For open economies, A) S = I. B) S = I + CA. C) S = I - CA. D) S > I + CA. E) S < I + CA. Answer: B Difficulty: Easy AACSB: Application of knowledge 22) An open economy A) can save only by building up its capital stock. B) can save only by acquiring foreign wealth. C) cannot save either by building up its capital stock or by acquiring foreign wealth. D) can save either by building up its capital stock or by acquiring foreign wealth. E) can save by avoiding excessive imports. Answer: D Difficulty: Easy AACSB: Application of knowledge 23) A closed economy A) can save either by building up its capital stock or by acquiring foreign wealth. B) can save only by building up its capital stock. C) can save only by acquiring foreign wealth. D) cannot save either by building up its capital stock or by acquiring foreign wealth. E) can save by avoiding excessive imports. Answer: B Difficulty: Easy AACSB: Application of knowledge 24) Disposable income is national income A) less taxes collected from households and firms by the government. B) plus net taxes collected from households and firms by the government. C) less net taxes collected from firms by the government. D) less net taxes collected from households by the government. E) less net taxes collected from households and firms by the government. Answer: E Difficulty: Easy AACSB: Application of knowledge
12 Copyright © 2022 Pearson Education, Inc.
25) Government savings, , is equal to A) T - G. B) T + G. C) T = G. D) T + G - I. E) T - G = I. Answer: A Difficulty: Easy AACSB: Application of knowledge 26) Which of the following is FALSE about private savings and government savings? A) SP = Y - T - C B) Unlike private saving decisions, government saving decisions are often made with an eye toward their effect on output and employment. C) Total savings (S) = SP + D) The national income identity can help us to analyze the channels through which government saving decisions influence macroeconomic conditions. E) None of the above; all statements are true. Answer: E Difficulty: Moderate AACSB: Application of knowledge 27) In a closed economy, private saving, A) I - (G - T). B) I + (G - T). C) I + (G + T). D) I - (G + T). E) I + (G - T) + C. Answer: B Difficulty: Moderate AACSB: Application of knowledge
, is equal to
28) In an open economy, private saving, A) I - CA + (G - T). B) I + CA - (G - T). C) I + CA + (G - T). D) I - CA - (G - T). E) I + CA + (G + T). Answer: C Difficulty: Moderate AACSB: Application of knowledge
, is equal to
29) What is the national income identity for a closed economy? Answer: Y = C + I + G Difficulty: Easy AACSB: Application of knowledge 13 Copyright © 2022 Pearson Education, Inc.
30) What is the national income identity for an open economy? Answer: Y = C + I + G + EX - IM Difficulty: Easy AACSB: Application of knowledge 31) Discuss the values of private saving in closed and open economies. Answer: In a closed economy, private saving, , is equals to, I + (G - T). In a open economy, private saving, , is equals to I + CA + (G - T). Open economy helps in extending the opportunities for private saving or dis-saving, or borrowing. Difficulty: Moderate AACSB: Application of knowledge 32) Assume in a closed economy: C = 40 + 0.8(Y - T) G = 10 I = 20 T = 0, where T are taxes. (a) Calculate Y at equilibrium. (b) Calculate C, I, and G at equilibrium. (c) Now assume the economy is open with exports and imports activities, EX = 5 + 4EP /P IM = 10 + 0.1 (Y - T) - 3EP /P E=3 P = 1.5 P=2 Find equilibrium Y. Answer: (a) Y = C + I + G Y = 350 (b) C = 40 + 0.8 Y = 320 I = 20 G = 10 (c) Y = 269.1667 Difficulty: Moderate AACSB: Analytical thinking
14 Copyright © 2022 Pearson Education, Inc.
33) Fill in the following table.
Answer:
Difficulty: Moderate AACSB: Analytical thinking
15 Copyright © 2022 Pearson Education, Inc.
34) Assume in a closed economy: C = 50 + 0.6 (Y - T) G = 15 I = 15 T=2 (a) Calculate Y at equilibrium (b) Calculate C (c) Assume the economy is open EX = 4 + 3EP /P IM = 8 + 0.1 (Y - T) - 2EP /P E=3 P =1 P = 1.5 Find equilibrium Y. Answer: (a) Y = C + I + G Y = 50 + 0.6(Y - 2 + 15 + 15) Y = 197 (b) C = 50 + 0.6 (197 - 2) = 50 + 117 = 167 (c) Y = 170 Difficulty: Moderate AACSB: Analytical thinking
16 Copyright © 2022 Pearson Education, Inc.
35) Fill in the following table.
Answer:
Difficulty: Moderate AACSB: Analytical thinking
17 Copyright © 2022 Pearson Education, Inc.
36) What can one learn from the following figure 13.2?
Answer: The figure shows the U.S. current account and net foreign wealth from 1976 until 2019. It shows that a string of current account deficits in the 1980s reduced America's net foreign wealth until, by the early 21st century, the country had accumulated a substantial net foreign debt. In 1989 the country became a net debtor to foreigners for the first time since World War I. Difficulty: Moderate AACSB: Analytical thinking
18 Copyright © 2022 Pearson Education, Inc.
37) Fill in the following table.
Answer:
Difficulty: Moderate AACSB: Analytical thinking 13.3
The Balance of Payments Accounts
1) Every international transaction automatically enters the balance of payments A) once either as a credit or as a debit. B) twice, once as a credit and once as a debit. C) once as a credit. D) twice, both times as debit. E) three times, once as a credit, once as a debit, and once as an exchange. Answer: B Difficulty: Easy AACSB: Application of knowledge 2) You travel to Paris and pay for a $100 dinner with your credit card. How is this accounted for in the balance of payments? A) current account, French service import B) current account, U.S. good export C) financial account, U.S. asset export D) financial account, U.S. asset import E) financial account, French asset export Answer: C Difficulty: Moderate AACSB: Application of knowledge 19 Copyright © 2022 Pearson Education, Inc.
3) The German government carries out an official foreign exchange intervention in which it uses dollars held in an American bank to buy French currency from its citizens. How is this accounted for in the balance of payments? A) current account, French good export B) current account, German good import C) financial account, French asset export D) financial account, German asset export E) financial account, German asset import Answer: C Difficulty: Difficult AACSB: Application of knowledge 4) The United States issues a $10,000 debt forgiveness to Argentina. How is this accounted for in the balance of payments? A) financial account, U.S. asset import B) current account, Argentina transfer payment C) current account, U.S. service export D) capital account, U.S. transfer payment E) current account, Argentina good import Answer: D Difficulty: Moderate AACSB: Application of knowledge 5) A U.S. citizen buys a newly issued share of stock in England, paying for his order with a check, which the British company deposits in its own U.S. bank account in New York. How is this transaction accounted for in the balance of payments? A) financial account, U.S. asset import B) current account, U.S. service import C) current account, British good export D) financial account, British asset import E) financial account, U.S. asset export Answer: A Difficulty: Difficult AACSB: Application of knowledge 6) Unilateral transfers between countries are A) long-term loans. B) part of national income, but not part of the current account C) part of the current account, but not a part of national income. D) known for reducing the income of capital owners. E) international gifts or payments that do not correspond to the purchase of any good, service, or asset. Answer: E Difficulty: Moderate AACSB: Application of knowledge
20 Copyright © 2022 Pearson Education, Inc.
7) How do we allocate statistical discrepancy among the current, capital, and financial accounts? A) We have no way of knowing exactly how to allocate this discrepancy. B) Depend on the degree of certainty which we attribute to these accounts. C) Divide it evenly amongst the three accounts. D) Depend on the convention adopted by the specific financial institution. E) Statistical discrepancy signals human errors made when dealing with financial accounts. Answer: A Difficulty: Easy AACSB: Application of knowledge 8) Which of the following statements was announced by Ireland's Central Statistics Office on July 12, 2016? A) Ireland's real GDP had risen by 26.3 percent between 2014 and 2015. B) Ireland's real GDP had risen by 7.8 percent between 2014 and 2015. C) Ireland's real GDP had risen by 5.8 percent between 2014 and 2015. D) Ireland's real GDP had risen by 2.3 percent between 2014 and 2015. Answer: A Difficulty: Easy AACSB: Application of knowledge 9) Ireland's remarkable spurt of GDP growth between 2014 and 2015 was A) in large part an accounting phenomenon reflecting tax avoidance by large multinationals from other countries. B) because of a sudden upsurge in Ireland's available supplies of factories, land, and labor. C) due to massive increase in factors of production. D) thanks to innovation and job creation. Answer: A Difficulty: Easy AACSB: Application of knowledge 10) Which of the following statements is TRUE? A) Ireland's low tax on corporate profits set up an incentive for large multinationals to relocate their intellectual property assets to subsidiaries in Ireland. B) Ireland's high tax on corporate profits set up an incentive for large multinationals to relocate their intellectual property assets to the United States of America. C) Ireland's high tax on corporate profits set up an incentive for Irish firms to relocate their intellectual property assets to other countries. D) Ireland's high tax on corporate profits make the Irish businesses very unhappy. Answer: A Difficulty: Easy AACSB: Application of knowledge
21 Copyright © 2022 Pearson Education, Inc.
11) The practice used by multinational corporations to relocate their intellectual property assets to subsidiaries in low-tax havens so that global profits are booked and taxed there is called A) profit shifting. B) offshore taxing. C) profit sharing. D) offshore relocating. Answer: A Difficulty: Easy AACSB: Application of knowledge 12) U.S. multinationals are estimated to shift roughly ________ percent of their total annual profit to low-tax jurisdictions, whereas the global average profit share that multinationals shift from their home countries is about ________ percent. A) 60; 40 B) 40; 60 C) 80; 20 D) 20; 80 Answer: A Difficulty: Easy AACSB: Application of knowledge 13) Facing the problem of multinational profit sharing, the United States and other large economies in 2021 agreed to establish a common minimum corporate tax rate of ________ percent. A) 15 B) 20 C) 25 D) 30 Answer: A Difficulty: Easy AACSB: Application of knowledge 14) Which of the following statements about the central bank is TRUE? A) Only the central bank may hold foreign reserves and intervene officially in exchange markets. B) Central banks have little power to affect macroeconomic conditions. C) Today, central banks' reserves consist largely of gold. D) The Federal Reserve holds only a small level of official reserve assets other than gold. E) Central banks never inject money into the economy. Answer: D Difficulty: Moderate AACSB: Application of knowledge
22 Copyright © 2022 Pearson Education, Inc.
15) An American buys a Japanese car, paying by writing a check on an account with a bank in New York. How would this be accounted for in the balance of payments? A) current account, a Japanese good import B) current account, a U.S. good import C) financial account, a U.S. asset import D) financial account, a U.S. asset export E) a current account as a U.S. good import and a financial account, a U.S. asset export Answer: E Difficulty: Moderate AACSB: Application of knowledge 16) The official settlements balance or balance of payments is the sum of A) the current account balance, and the capital account balance, less the non-reserve portion of the financial account balance. B) the current account balance and the capital account balance. C) the current account balance, the capital account balance, the non-reserve portion of the financial account balance, the statistical discrepancy. D) the current account balance and the non-reserve portion of the financial account balance. E) the current account balance and the interest in all investments. Answer: A Difficulty: Easy AACSB: Application of knowledge 17) Which of the following is TRUE about current cost method and market value method? A) They are used by the BEA to place current values on foreign indirect investments. B) These methods lead to the same valuations. C) Based on the current cost method, the BEA's 2019 estimate of U.S. net foreign wealth was $11,050.5 billion. D) The market value cost method is preferred by the BEA. E) Foreign direct investments of the U.S. are valued at their original purchase price. Answer: C Difficulty: Moderate AACSB: Application of knowledge 18) What types of international transactions are recorded in the balance of payment accounts? Answer: Three types: (1) transactions that involve exports and imports of goods and services; (2) transactions that involve the purchase or sale of financial assets; (3) other activities resulting in transfer of wealth between countries which are recorded in the capital account. Difficulty: Moderate AACSB: Application of knowledge
23 Copyright © 2022 Pearson Education, Inc.
19) "If the simple rule of double-entry bookkeeping holds true, the Balance of payments is always in balance." Discuss. Answer: True. Every international transaction automatically enters the balance of payments twice, once as a credit and once as a debit. Current account + financial account + capital account = 0 Difficulty: Moderate AACSB: Application of knowledge 20) Consider how the United States balance of payments accounts are affected when U.S. banks forgive two billion in debt owed to them by the government of Argentina. Answer: In this case, the United States makes a two billion dollar capital transfer to Argentina, which should appear as a negative two billion entry in the capital account. The associated credit is in the financial account, in the form of a two billion dollars reduction in U.S. assets held abroad, i.e., a net asset "export," and therefore a positive balance of payments entry. Difficulty: Moderate AACSB: Application of knowledge 21) "The balance of payments is seldom in balance in practice." Discuss. Answer: True. The main reasons are due to the fact that data collected or received from different sources may differ in coverage, accuracy, and timing. In addition, data on services are not reliable as well as data from the financial account. Moreover, accurate measurements of international interest and dividend receipts are particularly difficult. Difficulty: Moderate AACSB: Application of knowledge 22) What is profit shifting? How can governments including the U.S. solve this problem? Answer: Profit shifting is the practice used by multinational corporations to pay less profit tax by relocating operations from higher-tax jurisdictions to lower-tax jurisdictions. Governments can solve this problem only through substantial coordination of tax policies as well as detailed information sharing. Efforts to date had only limited success until 2021, when the United States (under President Joseph R. Biden, Jr.) and other large economies agreed to establish a common minimum corporate tax rate of 15 percent. Although that agreement was a major breakthrough, implementation faces political as well as technical obstacles. Difficulty: Easy AACSB: Application of knowledge 23) Explain the huge jump in Ireland's 2015 real GDP. Answer: The huge jump in Ireland's 2015 real GDP was mostly an artifact of creative accounting. The increase in GDP was in large part an accounting phenomenon reflecting tax avoidance by large multinationals form other countries. Ireland has a statutory tax on corporate profits of 12.5 percent, which is lower than other countries including the United States. This discrepancy sets up an incentive for large multinationals, notably in fields such as high tech and pharma, to relocate their intellectual property assets to subsidiaries in Ireland so that global profits are booked and taxed there and not at home. Difficulty: Easy AACSB: Application of knowledge 24 Copyright © 2022 Pearson Education, Inc.
24) How does an economy's central bank manage the supply of money through official reserve transactions? Answer: Official foreign exchange interventions are a way for the central bank to inject money into the economy or withdraw it from circulation. They can buy or sell international reserves in private asset markets in order to affect macroeconomic conditions without noticeably impacting the money supply. When a central bank purchases or sells a foreign asset, the transaction appears in its country's financial account as if a private citizen had carried out the same transaction. Difficulty: Moderate AACSB: Application of knowledge 25) How can changes in the market price of wealth previously acquired alter a country's net foreign wealth? Answer: When Japan's stock market lost value in the 1990s, American and European owners of Japanese assets lost value in their claims, while Japan's net foreign wealth increased. Similarly, exchange rate changes can have this effect. When the dollar depreciates against foreign currency, foreigners who hold dollar assets will see their wealth fall when measured in their home currencies. Difficulty: Moderate AACSB: Application of knowledge 26) Discuss the two different methods the Bureau of Economic Analysis (BEA) uses to place current values on foreign direct investments. Answer: The current cost method values direct investments at the cost of buying them today. The market value method is meant to measure the price at which the investments could be sold. These two methods can lead to different valuations because the cost of replacing a particular direct investment and the price it would command if sold on the market may be hard to measure. Difficulty: Moderate AACSB: Application of knowledge
25 Copyright © 2022 Pearson Education, Inc.
International Economics: Theory and Policy, 12e (Krugman) Chapter 14 Exchange Rates and the Foreign Exchange Market: An Asset Approach 14.1
Exchange Rates and International Transactions
1) The price of one currency in terms of another is called a(n) ________. A) exchange rate B) forward rate C) effective rate D) interest rate E) trading rate Answer: A Difficulty: Easy AACSB: Application of knowledge 2) How many dollars would it cost to buy an Edinburgh Woolen Mill sweater costing 50 British pounds if the exchange rate is 1.25 dollars per one British pound? A) 50 dollars B) 60 dollars C) 70 dollars D) 62.5 dollars E) 40 British pounds Answer: D Difficulty: Easy AACSB: Application of knowledge 3) How many dollars would it cost to buy an Edinburgh Woolen Mill sweater costing 50 British pounds if the exchange rate is 1.50 dollars per one British pound? A) 50 dollars B) 60 dollars C) 70 dollars D) 80 dollars E) 75 dollars Answer: E Difficulty: Easy AACSB: Application of knowledge 4) How many dollars would it cost to buy an Edinburgh Woolen Mill sweater costing 50 British pounds if the exchange rate is 1.80 dollars per one British pound? A) 40 dollars B) 90 dollars C) 50 dollars D) 100 dollars E) 95 dollars Answer: B Difficulty: Easy AACSB: Application of knowledge 1 Copyright © 2022 Pearson Education, Inc.
5) The Japanese currency is called the A) DM. B) Yen. C) Euro. D) Dollar. E) Pound. Answer: B Difficulty: Easy AACSB: Application of knowledge 6) How many British pounds would it cost to buy a pair of American designer jeans costing $45 if the exchange rate is 1.50 dollars per British pound? A) 440 British pounds B) 20 British pounds C) 30 British pounds D) 35 British pounds E) 25 British pounds Answer: C Difficulty: Easy AACSB: Application of knowledge 7) How many British pounds would it cost to buy a pair of American designer jeans costing $45 if the exchange rate is 1.80 dollars per British pound? A) 10 British pounds B) 25 British pounds C) 20 British pounds D) 30 British pounds E) 40 British pounds Answer: B Difficulty: Easy AACSB: Application of knowledge 8) How many British pounds would it cost to buy a pair of American designer jeans costing $45 if the exchange rate is 2.00 dollars per British pound? A) 22.5 British pounds B) 32.5 British pounds C) 12.5 British pounds D) 40 British pounds E) 30 British pounds Answer: A Difficulty: Easy AACSB: Application of knowledge
2 Copyright © 2022 Pearson Education, Inc.
9) How many British pounds would it cost to buy a pair of American designer jeans costing $45 if the exchange rate is 1.60 dollars per British pound? A) 38.125 British pounds B) 28.125 British pounds C) 48.125 British pounds D) 58.125 British pounds E) 18.125 British pounds Answer: B Difficulty: Easy AACSB: Application of knowledge 10) Exxon Mobil wants to pay €160,000 to a German supplier. They get an exchange rate quotation from its own commercial bank and instruct it to debit their dollar account and pay €160,000 to the supplier's German account. If the exchange rate quoted is $1.2 per euro, how much is debited to Exxon Mobil's account? A) $192,000 B) $160,000 C) $172,000 D) $180,000 E) $150,000 Answer: A Difficulty: Easy AACSB: Application of knowledge 11) Total Wine & More wants to pay ¥500,000 to a Japanese supplier of sake and plum wine. They get an exchange rate quotation from its own commercial bank and instruct it to debit their dollar account and pay ¥500,000 to the supplier's Japanese account. If the exchange rate quoted is $0.0092 per Japanese yen, how much is debited to Total Wine & More's account? A) $4,600 B) $1,150 C) $2,300 D) $9,200 E) $50,000 Answer: A Difficulty: Easy AACSB: Application of knowledge 12) What is the exchange rate between the dollar and the British pound if a pair of American jeans costs 50 dollars in New York and 100 Pounds in London? A) 1.5 dollars per British pound B) 0.5 dollars per British pound C) 2.5 dollars per British pound D) 3.5 dollars per British pound E) 2 dollars per British pound Answer: B Difficulty: Easy AACSB: Application of knowledge 3 Copyright © 2022 Pearson Education, Inc.
13) What is the exchange rate between the dollar and the British pound if a pair of American jeans costs 60 dollars in New York and 30 Pounds in London? A) 1.5 dollars per British pound B) 0.5 dollars per British pound C) 2.5 dollars per British pound D) 3.5 dollars per British pound E) 2 dollars per British pound Answer: E Difficulty: Easy AACSB: Application of knowledge 14) What is the exchange rate between the British pound and the dollar if a pair of American jeans costs 40 dollars in New York and 20 Pounds in London? A) 0.5 British pounds per U.S. dollar B) 1.5 British pounds per U.S. dollar C) 2.5 British pounds per U.S. dollar D) 3.5 British pounds per U.S. dollar E) 2 British pounds per U.S. dollar Answer: A Difficulty: Easy AACSB: Application of knowledge 15) When a country's currency depreciates A) foreigners find that its exports are more expensive, and domestic residents find that imports from abroad are more expensive. B) foreigners find that its exports are more expensive, and domestic residents find that imports from abroad are cheaper. C) foreigners find that its exports are cheaper; however, domestic residents are not affected. D) foreigners are not affected, but domestic residents find that imports from abroad are more expensive. E) foreigners find that its exports are cheaper and domestic residents find that imports from abroad are more expensive. Answer: E Difficulty: Easy AACSB: Application of knowledge
4 Copyright © 2022 Pearson Education, Inc.
16) When a country's currency appreciates A) foreigners find that its exports are more expensive, and domestic residents find that imports from abroad are cheaper. B) foreigners find that its exports are more expensive, and domestic residents find that imports from abroad are more expensive. C) foreigners find that its exports are cheaper; however, domestic residents are not affected. D) foreigners are not affected, but domestic residents find that imports from abroad are more expensive. E) foreigners find that its exports are cheaper and domestic residents find that imports from abroad are more expensive. Answer: A Difficulty: Easy AACSB: Application of knowledge 17) A depreciation of a country's currency A) lowers the relative price of its exports and raises the relative price of its imports. B) decreases the relative price of its exports and lowers the relative price of its imports. C) raises the relative price of its exports and raises the relative price of its imports. D) raises the relative price of its exports and lowers the relative price of its imports. E) raises the relative price of its exports and does not affect the relative price of its imports. Answer: A Difficulty: Easy AACSB: Application of knowledge 18) An appreciation of a country's currency A) decreases the relative price of its exports and lowers the relative price of its imports. B) raises the relative price of its exports and raises the relative price of its imports. C) lowers the relative price of its exports and raises the relative price of its imports. D) raises the relative price of its exports and lowers the relative price of its imports. E) raises the relative price of its exports and does not affect the relative price of its imports. Answer: D Difficulty: Easy AACSB: Application of knowledge 19) Which one of the following statements is the MOST accurate? A) A depreciation of a country's currency makes its goods cheaper for foreigners. B) A depreciation of a country's currency makes its goods more expensive for foreigners. C) A depreciation of a country's currency makes its goods cheaper for its own residents. D) A depreciation of a country's currency makes its goods cheaper. E) An appreciation of a country's currency makes its goods more expensive. Answer: A Difficulty: Easy AACSB: Application of knowledge
5 Copyright © 2022 Pearson Education, Inc.
20) A(n) ________ of a nation's currency will cause imports to ________ and exports to ________, all other things held constant. A) depreciation; increase; decrease B) appreciation; decrease; increase C) depreciation; decrease; increase D) appreciation; increase; increase E) depreciation; decrease; decrease Answer: C Difficulty: Moderate AACSB: Application of knowledge 21) A(n) ________ of a nation's currency will cause imports to ________ and exports to ________, all other things held constant. A) appreciation; increase; decrease B) depreciation; decrease; decrease C) depreciation; increase; decrease D) appreciation; decrease; increase E) depreciation; increase; increase Answer: A Difficulty: Moderate AACSB: Application of knowledge 22) If the goods' money prices do NOT change, an appreciation of the dollar against the pound A) makes British sweaters cheaper in terms of American jeans. B) makes British sweaters more expensive in terms of American jeans. C) doesn't change the relative price of sweaters and jeans. D) makes American jeans cheaper in terms of British sweaters. E) makes British jeans more expensive in Britain. Answer: A Difficulty: Moderate AACSB: Application of knowledge 23) If the goods' money prices do NOT change, a depreciation of the dollar against the pound A) makes British sweaters cheaper in terms of American jeans. B) makes British sweaters more expensive in terms of American jeans. C) makes American jeans more expensive in terms of British sweaters. D) doesn't change the relative price of sweaters and jeans. E) makes British jeans more expensive in Britain. Answer: B Difficulty: Moderate AACSB: Application of knowledge
6 Copyright © 2022 Pearson Education, Inc.
24) Compute how many dollars it would cost to buy an Edinburgh Woolen Mill sweater costing 50 British pounds for the following exchange rates.
Answer:
Difficulty: Easy AACSB: Application of knowledge
7 Copyright © 2022 Pearson Education, Inc.
25) Compute how many British pounds it would cost to buy a pair of American designer jeans costing $45.
Answer:
Difficulty: Easy AACSB: Application of knowledge
8 Copyright © 2022 Pearson Education, Inc.
26) Find the exchange rate between the dollar and the British pounds for the following cases.
Answer:
Difficulty: Easy AACSB: Application of knowledge
9 Copyright © 2022 Pearson Education, Inc.
14.2
The Foreign Exchange Market
1) The market in which households, firms, and financial institutions buy and sell foreign currencies to make international payments is called the A) foreign exchange market. B) currency market. C) money market. D) international market . E) trading market. Answer: A Difficulty: Easy AACSB: Application of knowledge 2) Which major actor is at the center of the foreign exchange market? A) corporations B) central banks C) commercial banks D) non-bank financial institutions E) individual firms Answer: C Difficulty: Easy AACSB: Application of knowledge 3) Which of the following is NOT a major actor in the foreign exchange market? A) corporations B) central banks C) commercial banks D) non-bank financial institutions E) tourists Answer: E Difficulty: Easy AACSB: Application of knowledge 4) Which of the following type of funds cater to wealthy individuals, are NOT bound by government regulations, and are actively traded in foreign exchange markets? A) pension funds B) mutual funds C) hedge funds D) exchange funds Answer: C Difficulty: Easy AACSB: Application of knowledge
10 Copyright © 2022 Pearson Education, Inc.
5) The largest trading of foreign exchange occurs in A) New York. B) London. C) Tokyo. D) Frankfurt. E) Singapore. Answer: B Difficulty: Easy AACSB: Application of knowledge 6) The action of arbitrage is A) the process of buying a currency cheap and selling it dear. B) the process of buying a currency dear and selling it cheap. C) the process of buying and selling currency at the same price. D) the process of selling currency at different prices in different markets. E) the process of buying a currency and holding onto it to take it off the market. Answer: A Difficulty: Easy AACSB: Application of knowledge 7) In 2019, about A) 4.3 percent of foreign exchange transactions involved exchanges of foreign currencies for U.S. dollars. B) 16.8 percent of foreign exchange transactions involved exchanges of foreign currencies for U.S. dollars. C) 32.3 percent of foreign exchange transactions involved exchanges of foreign currencies for U.S. dollars. D) 39 percent of foreign exchange transactions involved exchanges of foreign currencies for U.S. dollars. E) 88.3 percent of foreign exchange transactions involved exchanges of foreign currencies for U.S. dollars. Answer: E Difficulty: Easy AACSB: Application of knowledge 8) By April 2019 A) only about 0.9 percent of foreign exchange trades were against euros. B) only about 4.3 percent of foreign exchange trades were against euros. C) only about 32.3 percent of foreign exchange trades were against euros. D) only about 39 percent of foreign exchange trades were against euros. E) only about 88.3 percent of foreign exchange trades were against euros. Answer: C Difficulty: Easy AACSB: Application of knowledge
11 Copyright © 2022 Pearson Education, Inc.
9) Which of the following statements is TRUE about a vehicle currency? A) It is widely used to denominate contracts made by parties who reside in the country that issues the vehicle currency. B) The dollar is sometimes called a vehicle currency because of its pivotal role in many foreign exchange deals. C) There is much skepticism that the euro will ever evolve into a vehicle currency on par with the dollar. D) The pound sterling, once second only to the dollar as a key international currency, is beginning to rise in importance. E) Vehicle currencies include nondeliverable currencies like the renminbi. Answer: B Difficulty: Easy AACSB: Application of knowledge 10) The future date on which the currencies are actually exchanged is called what? A) the value date B) the spot exchange date C) the two-day window D) the commitment date E) the forward exchange rate Answer: A Difficulty: Easy AACSB: Application of knowledge 11) Which one of the following statements is the MOST accurate? A) Spot exchange rates are always higher than forward exchange rates. B) Spot exchange rates are always lower than forward exchange rates. C) Spot exchange rates and forward exchange rates are always equal. D) Spot exchange rates and forward exchange rates are equal when the value date and the date of the spot transaction are the same. E) Spot exchange rates and forward exchange rates never move closely together. Answer: D Difficulty: Moderate AACSB: Application of knowledge 12) Forward and spot exchange rates A) are necessarily equal. B) do not move closely together. C) are always such that the forward exchange rate is higher. D) move closely together and are equal on the value date when the value date and the date of the spot transaction are the same. E) are unrelated to the value date. Answer: D Difficulty: Moderate AACSB: Application of knowledge
12 Copyright © 2022 Pearson Education, Inc.
13) The following is an example of Radio Shack hedging its foreign currency risk: A) needing to pay 9,000 yen per radio to its suppliers in a month, Radio Shack makes a forwardexchange deal to buy yen. B) needing to pay 9,000 yen per radio to its suppliers in a month, Radio Shack makes a forwardexchange deal to sell yen. C) needing to pay 9,000 yen per radio to its suppliers in a month, Radio Shack buys yen at a spot-exchange 1 month from now. D) needing to pay 9,000 yen per radio to its suppliers in a month, Radio Shack sells yen at a spot-exchange 1 month from now. E) needing to pay 9,000 yen per radio to its suppliers in a month, Radio Shack sells yen in a forward-exchange deal. Answer: A Difficulty: Easy AACSB: Application of knowledge 14) A foreign exchange swap A) is a spot sale of a currency. B) is a forward repurchase of the currency. C) is a spot sale of a currency combined with a forward repurchase of the currency. D) is a spot sale of a currency combined with a forward sale of the currency. E) make up a negligible proportion of all foreign exchange trading. Answer: C Difficulty: Easy AACSB: Application of knowledge 15) Which of the following is NOT an example of a financial derivative? A) forwards B) bonds C) swaps D) futures E) options Answer: B Difficulty: Easy AACSB: Application of knowledge 16) Futures contracts differ from forward contracts in that A) future contracts ensures you will receive a certain amount of foreign currency at a specified future date. B) future contracts bind you into your end of the deal. C) future contracts allow you to sell your contract on an organized futures exchange. D) future contracts are a disadvantage if your views about the future spot exchange rate are to change. E) futures contracts don't allow you to realize a profit or a loss right away. Answer: C Difficulty: Moderate AACSB: Application of knowledge 13 Copyright © 2022 Pearson Education, Inc.
17) Who are the major participants in the foreign exchange market? Answer: (1) Commercial banks (2) Corporations (3) Nonbank financial institutions (4) Central banks Difficulty: Easy AACSB: Application of knowledge 18) Explain what is a "vehicle currency." Why is the U.S. dollar considered a vehicle currency? Answer: A vehicle currency is one that is widely used to denominate international contracts made by parties who do not reside in the country that issues the vehicle currency. Since in 2004, nearly 90 percent of foreign exchange transactions involve exchanges of foreign currencies for U.S. dollars; therefore, it is considered a vehicle currency. Difficulty: Moderate AACSB: Application of knowledge 19) Explain the purpose of the following figure.
Answer: To show that spot and forward exchange rates move closely together. Difficulty: Moderate AACSB: Application of knowledge
14 Copyright © 2022 Pearson Education, Inc.
14.3
The Demand for Foreign Currency Assets
1) What is the expected dollar rate of return on euro deposits if today's exchange rate is $1.10 per euro, next year's expected exchange rate is $1.165 per euro, the dollar interest rate is 10%, and the euro interest rate is 5%? A) 10% B) 11% C) -1% D) 0% E) 15% Answer: B Difficulty: Moderate AACSB: Application of knowledge 2) What is the expected dollar rate of return on dollar deposits if today's exchange rate is $1.10 per euro, next year's expected exchange rate is $1.165 per euro, the dollar interest rate is 10%, and the euro interest rate is 5%? A) 10% B) 11% C) -1% D) 0% E) 15% Answer: A Difficulty: Easy AACSB: Application of knowledge 3) What is the expected dollar rate of return on euro deposits if today's exchange rate is $1.167 per euro, next year's expected exchange rate is $1.10 per euro, the dollar interest rate is 10%, and the euro interest rate is 5%? A) 10% B) 11% C) -1% D) 0% Answer: C Difficulty: Moderate AACSB: Application of knowledge
15 Copyright © 2022 Pearson Education, Inc.
4) Which one of the following statements is the MOST accurate? A) Since dollar and yen interest rates are measured in comparable terms, they can move quite differently over time. B) Since dollar and yen interest rates are not measured in comparable terms, they can move quite differently over time. C) Since dollar and yen interest rates are measured in comparable terms, they move quite the same over time. D) Since dollar and yen interest rates are measured in comparable terms, they still move quite differently over time. E) Since dollar and yen interest rates are so similar, they move quite the same way over time. Answer: B Difficulty: Easy AACSB: Application of knowledge 5) The dollar rate of return on euro deposits is A) approximately the euro interest rate plus the rate of depreciation of the dollar against the euro. B) approximately the euro interest rate minus the rate of depreciation of the dollar against the euro. C) the euro interest rate minus the rate of inflation against the euro. D) the rate of appreciation of the dollar against the euro. E) the euro interest rate plus the rate of inflation against the euro. Answer: A Difficulty: Easy AACSB: Application of knowledge 6) If the dollar interest rate is 10 percent and the euro interest rate is 6 percent, then an investor should A) invest only in dollars. B) invest only in euros. C) be indifferent between dollars and euros. D) invest only in dollars if the exchange rate is expected to remain constant. E) invest only in euros if the exchange rate is expected to remain constant. Answer: D Difficulty: Easy AACSB: Application of knowledge 7) If the dollar interest rate is 5 percent and the euro interest rate is 3 percent, then an investor should A) invest only in dollars if the exchange rate is expected to remain constant. B) invest only in dollars. C) invest only in euros. D) be indifferent between dollars and euros. E) invest only in euros if the exchange rate is expected to remain constant. Answer: A Difficulty: Easy AACSB: Application of knowledge 16 Copyright © 2022 Pearson Education, Inc.
8) If the dollar interest rate is 4 percent, the euro interest rate is 6 percent, then A) an investor should invest only in dollars. B) an investor should invest only in euros. C) an investor should be indifferent between dollars and euros. D) invest only in dollars if the exchange rate is expected to remain constant. E) invest only in euros if the exchange rate is expected to remain constant. Answer: E Difficulty: Easy AACSB: Application of knowledge 9) If the dollar interest rate is 5 percent, the euro interest rate is 10 percent, then A) invest only in euros if the exchange rate is expected to remain constant. B) an investor should invest only in dollars. C) an investor should invest only in euros. D) an investor should be indifferent between dollars and euros. E) invest only in dollars if the exchange rate is expected to remain constant. Answer: A Difficulty: Easy AACSB: Application of knowledge 10) If the dollar interest rate is 10 percent, the euro interest rate is 6 percent, then A) an investor should invest only in dollars if the expected dollar depreciation against the euro is 4 percent. B) an investor should invest only in euros if the expected dollar depreciation against the euro is 4 percent. C) an investor should be indifferent between dollars and euros if the expected dollar depreciation against the euro is 4 percent. D) an investor should invest only in dollars. E) an investor should invest only in euros. Answer: C Difficulty: Moderate AACSB: Application of knowledge 11) If the dollar interest rate is 10 percent and the euro interest rate is 6 percent, then A) an investor should invest only in dollars if the expected dollar depreciation against the euro is 8 percent. B) an investor should invest only in euros if the expected dollar depreciation against the euro is 8 percent. C) an investor should be indifferent between dollars and euros if the expected dollar depreciation against the euro is 8 percent. D) an investor should invest only in dollars. E) an investor should invest only in euros. Answer: B Difficulty: Moderate AACSB: Application of knowledge
17 Copyright © 2022 Pearson Education, Inc.
12) If the dollar interest rate is 10 percent, the euro interest rate is 12 percent, then A) an investor should invest only in dollars if the expected dollar appreciation against the euro is 4 percent. B) an investor should invest only in euros if the expected dollar appreciation against the euro is 4 percent. C) an investor should be indifferent between dollars and euros if the expected dollar appreciation against the euro is 4 percent. D) an investor should invest only in dollars. E) an investor should invest only in euros. Answer: A Difficulty: Moderate AACSB: Application of knowledge 13) What are the three factors that affect the demand for foreign currency? Answer: The three factors that affect the demand for foreign currency are expected return, risk and liquidity. Difficulty: Moderate AACSB: Application of knowledge 14) Explain risk and liquidity of assets. Answer: Risk is the variability an asset contributes to a savers' wealth. An asset's real return can be unpredictable and savers dislike this uncertainty if the return fluctuates widely. Liquidity refers to the ease with which an asset can be sold or exchanged for goods. Cash is the most liquid of assets because it is always acceptable at face value as payment for goods or other assets. Thus, savers consider an asset's liquidity and its expected return and risk in deciding how much of it to hold. Difficulty: Moderate AACSB: Application of knowledge
18 Copyright © 2022 Pearson Education, Inc.
15) Explain the purpose of the following figure 14-2 from the text in the context of the interest rates on the dollar and the Japanese Yen between 1978 and 2020.
Answer: Since the dollar and the Yen interest rates are not measured in comparable terms, they can move quite differently over time. Except for a period from 1990 to 1993 when the Yen interest rate was higher than the dollar and the period between 2009 and 2011 when the interest rates were pretty close, dollar interest rates have been higher than the Yen, indicating depreciation of the dollar against the Yen. Difficulty: Moderate AACSB: Application of knowledge
19 Copyright © 2022 Pearson Education, Inc.
16) What is the expected dollar rate of return on euro deposits if today's exchange rate is $1.10 per euro, next year's expected exchange rate is $1.20 per euro, the dollar interest rate is 10%, and the euro interest rate is 5%? A) Calculate the expected dollar rate of return on euro deposits. B) Given the result above, if you were an investor, would you hold wealth in the form of euro or dollar deposits? Explain why. Answer: A) If today's exchange rate is $1.10 per euro, the dollar price of a euro is $1.10 If you purchase 1 euro deposit today, with interest rate of 5% a year, your deposit will be worth 1.05 euro at the end of the year. Since the dollar is expected to depreciate against the euro over the coming year, the exchange rate 12 months from today is $1.20 per euro, you expect the dollar value of your euro deposit after a year to be $1.05 Euro x $1.20 per euro = $1.26 After you know the dollar price of 1 euro deport today ($1.10) and can forecast its value in a year ($1.26), you can calculate the expected dollar rate of return on a euro deposit as ($1.26$1.10)/$1.10 = 0.15 or 15% a year. B) Since the dollar interest rate is only 10% per year and the expected rate of return on a euro deposits is 15% per year, you expect to do better by holding your wealth in the form of euro deposits. Despite the fact that dollar interest rate exceeds the euro interest rate by 5% a year, the euro's expected appreciation against the dollar gives euro holders a prospective capital gain that is large enough to make the euro deposits the higher-yield asset. Difficulty: Difficult AACSB: Application of knowledge
20 Copyright © 2022 Pearson Education, Inc.
17) For the following 15 cases, compare the dollar rates of return on dollar and euro deposits.
Answer:
Difficulty: Moderate AACSB: Application of knowledge 21 Copyright © 2022 Pearson Education, Inc.
18) For the table below calculate the EXACT relationship (i.e., EXACT calculated result of the rate of return difference between dollar and euro deposits, up to 4 digits after the decimal point).
22 Copyright © 2022 Pearson Education, Inc.
Answer: Using Figure (14-1) and Table 14-3 from the textbook as example, students can calculate the rate of return difference between dollar and euro deposits with up to 4 digits after the decimal point.
Difficulty: Moderate AACSB: Application of knowledge
23 Copyright © 2022 Pearson Education, Inc.
14.4
Equilibrium in the Foreign Exchange Market
1) Which one of the following statements is the MOST accurate? A) A rise in the interest rate offered by dollar deposits causes the dollar to appreciate. B) A rise in the interest rate offered by dollar deposits causes the dollar to depreciate. C) A rise in the interest rate offered by dollar deposits does not affect the U.S. dollar. D) For a given euro interest rate and constant expected exchange rate, a rise in the interest rate offered by dollar deposits causes the dollar to appreciate. E) A rise in the interest rate offered by the dollar causes the euro to appreciate. Answer: D Difficulty: Easy AACSB: Application of knowledge 2) Which one of the following statements is the MOST accurate? A) For a fixed interest rate, a rise in the expected future exchange rate causes a rise in the current exchange rate. B) For a fixed interest rate, a rise in the expected future exchange rate causes a fall in the current exchange rate. C) For a fixed interest rate, a rise in the expected future exchange rate does not cause a change in the current exchange rate. D) For a given dollar interest rate and a constant expected exchange rate, a rise in the interest rate of the euro causes the dollar to depreciate. E) For a fixed interest rate, a fall in the expected future exchange rate causes a rise in the current exchange rate. Answer: A Difficulty: Easy AACSB: Application of knowledge 3) A carry trade can be defined as A) a trading strategy that involves borrowing low-interest currencies and buying high-interest currencies. B) a trading strategy that involves borrowing high-interest currencies and buying low-interest currencies. C) a trading strategy that involves borrowing and investing in only low-interest currencies. D) a trading strategy that involves borrowing and investing in only high-interest currencies. Answer: A Difficulty: Easy AACSB: Application of knowledge
24 Copyright © 2022 Pearson Education, Inc.
4) A trade activity where international investors borrow low interest currencies and invest in high-interest currencies to seek profits is called A) carry trade. B) surplus trade. C) deficit trade. D) terms of trade. Answer: A Difficulty: Easy AACSB: Application of knowledge 5) In a carry trade, the low interest currencies are called ________ currencies, and the highinterest currencies are called ________ currencies. A) funding; investment B) borrowing; lending C) lending; borrowing D) investment; funding Answer: A Difficulty: Easy AACSB: Application of knowledge 6) What is the interest parity condition? Answer: The condition that the expected returns on deposits of any two currencies are equal when measured in the same currency is called the interest parity condition. It implies that potential holders of foreign currency deposits view them as equally desirable assets, i.e. risk is assumed away. In notational forms: R$ = RE + (
- E$/E)/E$/E
Difficulty: Easy AACSB: Application of knowledge 7) Explain why the interest parity condition must hold if the foreign exchange market is in equilibrium. Answer: The foreign exchange market is in equilibrium when deposits of all currencies offer the same expected rate of return. Potential holders of foreign currency deposits view them all as equally desirable assets. If expected rate of return on any currency deposit is higher or lower than the other, there will exist an excess supply or demand for that currency because one will yield a higher return than the other. Difficulty: Moderate AACSB: Application of knowledge
25 Copyright © 2022 Pearson Education, Inc.
8) Calculate the interest rate in the United States, if interest parity condition holds, for the following 15 cases.
Answer:
Difficulty: Moderate AACSB: Application of knowledge
26 Copyright © 2022 Pearson Education, Inc.
9) Calculate the interest rate in the euro zone if interest parity condition holds, for the following 15 cases.
Answer:
Difficulty: Moderate AACSB: Application of knowledge
27 Copyright © 2022 Pearson Education, Inc.
10) Determine for each, whether the interest parity condition holds or not, if
Answer:
Difficulty: Moderate AACSB: Application of knowledge
28 Copyright © 2022 Pearson Education, Inc.
= 1.10
11) Assume that the euro interest rate is constant at 5 percent, and that the expected exchange rate is 1.05 dollars per one euro. Find the expected dollar return on euro deposits for the following cases.
Answer:
Difficulty: Moderate AACSB: Application of knowledge
29 Copyright © 2022 Pearson Education, Inc.
12) Using the data in the table above, plot today's dollar/euro exchange rate against the expected dollar return on euro deposits. Answer:
Difficulty: Moderate AACSB: Application of knowledge 13) Calculate the Expected Dollar Depreciation Rate against the euro and the expected dollar return on euro deposits if the expected exchange rate is $1.10 per euro.
Answer:
Difficulty: Moderate AACSB: Application of knowledge 30 Copyright © 2022 Pearson Education, Inc.
14) Discuss the effects of a rise in the dollar interest rate on the exchange rate. Answer: There are two effects to consider. A rise in the interest rate offered by dollar deposits combined with a constant expected exchange rate will cause the dollar to appreciate (see Figure 14-5 from the text). However, the expected exchange rate will likely change. As Figure 14-6 from the text shows, if the expected exchange rate increases, the dollar will depreciate.
Figure 14-5
31 Copyright © 2022 Pearson Education, Inc.
Figure 14-6 Difficulty: Moderate AACSB: Application of knowledge
32 Copyright © 2022 Pearson Education, Inc.
15) Discuss the effects of a rise in the interest rate paid by euro deposits on the exchange rate. Answer: There are two effects to consider. If we make the unrealistic assumption that the expected exchange rate will not change, then a rise in the interest rate paid by euro deposits causes the dollar to depreciate. However, if the expected exchange rate were to rise, then the current exchange rate would also rise. (See Figure 14-6 from the text.)
Difficulty: Moderate AACSB: Application of knowledge 16) Explain why (holding interest rates constant), a rise in the expected depreciation in a country's currency leads to depreciation of that currency today. Answer: A rise in the expected depreciation rate of the dollar raises the expected dollar return on euro deposits. Now, there are excess supply of dollar deposits (euro deposits offer higher expected rate of return than do dollar deposits). The dollar must depreciate to remove this excess supply. Difficulty: Moderate AACSB: Application of knowledge
33 Copyright © 2022 Pearson Education, Inc.
17) Show graphically a drop in the interest rate paid by euro deposits. What is the effect on the dollar? Answer: A drop in the interest rate from R1$ to R2$ causes the dollar to depreciate from (point 2) to
(point 1). (See Figure 14-5 from the text.)
Difficulty: Difficult AACSB: Application of knowledge
34 Copyright © 2022 Pearson Education, Inc.
18) Show graphically a drop in the interest rate offered by dollar deposits, R$, and the effect on the exchange rate, . Answer: A drop in the interest rate paid by euro deposits causes the dollar to appreciate from (point 2) to
(point 1). The expected future exchange rate also drops. (See Figure 14-
6 from the text.)
Difficulty: Difficult AACSB: Application of knowledge 19) What is a carry trade? Answer: A carry trade is a trading strategy that involves borrowing a currency that has a low interest rate, then use that money to buy another currency that pays a higher interest rate. lowinterest currencies and re-investing in high-interest currencies. A carry trader may gain profits on the difference between the interest rates. Difficulty: Easy AACSB: Application of knowledge
35 Copyright © 2022 Pearson Education, Inc.
20) Explain why carry trade should not be systematically profitable and is a highly risky business. Answer: Carry trade involves high risks and liquidity factors. Investment currencies (the highinterest currencies that carry traders target) may experience abrupt crashes and funding currencies (the low interest currencies that carry traders borrow) may be subject to abrupt appreciation. Big carry trade positions emergence can result in great exchange volatility as well as the possibility of big trader losses with negative repercussions in stock markets, bond markets, and markets for interbank loans. Difficulty: Moderate AACSB: Application of knowledge 14.5
Forward Exchange Rates and Covered Interest Parity
1) The covered interest rate parity condition can be stated as follows: The interest rate on dollar deposits equals the interest rate on euro deposits ________ the forward ________ on euros against dollars. A) plus; premium B) minus; premium C) plus; discount D) minus; discount E) times; premium Answer: A Difficulty: Easy AACSB: Application of knowledge 2) The covered interest rate parity condition can be stated as follows: The interest rate on dollar deposits equals the interest rate on euro deposits ________ the forward ________ on dollars against euros. A) plus; premium B) plus; discount C) times; premium D) minus; premium E) minus; discount Answer: B Difficulty: Easy AACSB: Application of knowledge 3) Suppose that the one-year forward price of euros in terms of dollars is equal to $1.113 per euro. Further, assume that the spot exchange rate is $1.05 per euro, and the interest rate on dollar deposits is 10 percent and on euro it is 4 percent. Under these assumptions A) covered interest parity does hold. B) covered interest parity does not hold. C) it is hard to tell whether covered interest parity does or does not hold. D) covered interest parity fluctuates. E) Not enough information is given to answer the question. Answer: B Difficulty: Moderate AACSB: Application of knowledge 36 Copyright © 2022 Pearson Education, Inc.
4) Assume the U.S. interest rate is 10 percent, and the interest rate on euro deposits is 5 percent. For the following exchange rates, find the forward exchange rates.
Answer: Using the covered interest rate parity will yield the second column in the table: F$/E = (R$ - RE) E$/E + E$/E
Difficulty: Moderate AACSB: Application of knowledge
37 Copyright © 2022 Pearson Education, Inc.
International Economics: Theory and Policy, 12e (Krugman) Chapter 15 Money, Interest Rates, and Exchange Rates 15.1
Money Defined: A Brief Review
1) The exchange rate between currencies depends on A) the interest rate that can be earned on deposits of those currencies. B) the interest rate that can be earned on deposits of those currencies and the expected future exchange rate. C) the expected future exchange rate. D) national output. E) the interest rate that can be earned on deposits of those countries and the national output. Answer: B Difficulty: Easy AACSB: Application of knowledge 2) Which of the following statements is TRUE? A) Because exchange rates are the relative prices of national monies, factors that affect a country's money supply or demand are among the most powerful determinants of its currency's exchange rate against foreign currencies. B) Because exchange rates are the relative prices of national monies, factors that affect a country's imports and exports are among the most powerful determinants of its currency's exchange rate against foreign currencies. C) Because exchange rates are the relative prices of imports and exports, factors that affect a country's trade barriers are among the most powerful determinants of its currency's exchange rate against foreign currencies. D) Because exchange rates are the relative prices of imports and exports, factors that affect a country's trade openness are among the most powerful determinants of its currency's exchange rate against foreign currencies. Answer: A Difficulty: Easy AACSB: Application of knowledge 3) Expectations about future exchange rates are closely connected with A) expectations about the future money prices of countries' products; which, in turn, depend on changes in money supply and demand. B) expectations about the future unemployment rate; which, in turn, depends on changes in labor supply and demand. C) expectations about the future inflation rate; which, in turn, depends on changes in money supply. D) expectations about the future discount rate; which, in turn, depends on central bank's decisions. Answer: A Difficulty: Easy AACSB: Application of knowledge
1 Copyright © 2022 Pearson Education, Inc.
4) Money serves as all of the following EXCEPT A) a medium of exchange. B) a unit of account. C) a store of value. D) a symbol that is made of or can be redeemed for a fixed amount of precious metal. E) a highly liquid asset. Answer: D Difficulty: Easy AACSB: Application of knowledge 5) Money includes A) currency. B) checking deposits held by households and firms. C) deposits in the foreign exchange markets. D) currency and checking deposits held by households and firms. E) futures and deposits in the foreign exchange market. Answer: D Difficulty: Easy AACSB: Application of knowledge 6) In the United States in the last quarter of 2019, the total money supply, M1, amounted to approximately A) 19 percent of that year's GNP. B) 20 percent of that year's GNP. C) 30 percent of that year's GNP. D) 40 percent of that year's GNP. E) 50 percent of that year's GNP. Answer: A Difficulty: Easy AACSB: Application of knowledge 7) M1 is a measure of money supply that includes A) the total amount of currency and checking deposits held by households and firms. B) the large deposits traded by participants in the foreign exchange market. C) the total amount of savings deposits and certificates of deposit. D) the large time deposits, institutional money market funds, short-term repurchase agreements, and larger liquid funds. Answer: A Difficulty: Easy AACSB: Application of knowledge
2 Copyright © 2022 Pearson Education, Inc.
8) The money supply is A) controlled by central bank. B) controlled by the money market. C) regulated by demand and supply in the goods and service market. D) regulated by the U.S. Treasury. Answer: A Difficulty: Easy AACSB: Application of knowledge 9) What are the main functions of money? Answer: Money serves, in general, three important functions: a medium of exchange; a unit of account; and a store of value. As a medium of exchange, money avoids going back to a barter economy, with the enormous search costs connected with it. As a unit of account, the use of money economizes on the number of prices an individual faces. Consider an economy with N goods, then one needs only (N - 1) prices. As a store of value, the use of money in general ensures that you can transfer wealth between periods. Difficulty: Moderate AACSB: Application of knowledge 10) What are the factors that determine the amount of money an individual desires to hold? Answer: Three main factors: first, the expected return the asset offers compared with the returns offered by other assets; second, the riskiness of the asset's expected return; and third, the asset's liquidity. Difficulty: Moderate AACSB: Application of knowledge 15.2
The Demand for Money by Individuals
1) Individuals base their demand for an asset on A) the expected return the asset offers compared with the returns offered by other assets. B) the riskiness of the asset's expected return. C) the asset's liquidity. D) the expected return, the riskiness of the asset's expected return, and the asset's liquidity. E) the aesthetic qualities of the asset. Answer: D Difficulty: Easy AACSB: Application of knowledge 2) A family's summer house on Cape Cod pays a return in the form of A) interest rate. B) capital gains. C) the pleasure of vacations at the beach. D) stock options. E) capital gains and pleasure. Answer: E Difficulty: Easy AACSB: Application of knowledge 3 Copyright © 2022 Pearson Education, Inc.
3) Which of the following statements is TRUE? A) All else equal, a rise in interest rate causes the demand for money to fall. B) All else equal, a rise in interest rate causes the demand for money to rise. C) All else equal, a rise in interest rate has no effect on the demand for money. D) All else equal, a rise in interest rate causes people to hold more wealth in the form of money. Answer: A Difficulty: Easy AACSB: Application of knowledge 4) In a world with money and bonds, A) it is not risky to hold money. B) any change in the riskiness of money causes an equal change in the riskiness of bonds. C) risk is an important factor in the demand for money. D) there is no relationship between risk and holding money. E) it is not risky to hold bonds. Answer: B Difficulty: Easy AACSB: Application of knowledge 5) Opportunity cost is NOT A) the value of benefit you receive when you choose to take an action. B) the amount you sacrifice by taking one course of action rather than another. C) the value of the next best alternative forgone when a decision is made. D) the forgone benefit that would have been derived by an option not chosen. Answer: A Difficulty: Easy AACSB: Application of knowledge 6) It is risky to hold money because of the following reasons EXCEPT A) the government can confiscate individuals' money whenever it wants to. B) an unexpected increase in the prices of goods and services could reduce the value of the money in terms of the commodities to be consumed. C) money can be stolen, burnt, or lost. D) unexpected inflation can reduce the value of money any time. Answer: A Difficulty: Easy AACSB: Application of knowledge 7) Households and firms hold money because A) it is the easiest way of financing their everyday purchases. B) it is essential to hold money in a bartering economy. C) it is the safest way to save for the future generations. D) it is now the era of bitcoin and digital currency's popularity. Answer: A Difficulty: Easy AACSB: Application of knowledge 4 Copyright © 2022 Pearson Education, Inc.
8) Which one of the following statements is the MOST accurate? A) A rise in the average value of transactions carried out by a household or a firm causes its demand for money to fall. B) A reduction in the average value of transactions carried out by a household or a firm causes its demand for money to rise. C) A rise in the average value of transactions carried out by a household or a firm causes its demand for money to rise. D) A rise in the average value of transactions carried out by a household or a firm causes its demand for real money to rise. E) A decrease in the average value of transactions carried out by a household or a firm causes its demand for real money to rise. Answer: D Difficulty: Easy AACSB: Application of knowledge 9) An individual's need for liquidity would increase if A) the average value of transactions carried out by the individual fell. B) the average value of transactions carried out by the individual rose. C) the individual got a raise. D) the individual received a new ATM card. E) the individual wanted to avoid risks. Answer: B Difficulty: Easy AACSB: Application of knowledge 10) Explain how changes in the rate of interest affect the demand for money. Answer: Because an increase in the interest rate is a rise in the rate of return on less liquid assets relative to the rate of return on money, individuals will want to hold more of their wealth in nonmoney assets that pay the market interest rate and less of their wealth in the form of money if the interest rate rises. Difficulty: Easy AACSB: Application of knowledge 11) Why is risk not an important factor in money demand? Answer: People face the same risk whether holding money or bonds, whose real value may change. Because any change in the riskiness of money causes an equal change in the riskiness of bonds, changes in the risk of holding money need not cause individuals to reduce their demand for money and increase their demand for interest-paying assets. Difficulty: Easy AACSB: Application of knowledge 12) What is the main benefit of holding money? Answer: The main benefit of holding money comes from its liquidity. Households and firms hold money because it is the easiest way of financing their everyday purchases. Difficulty: Easy AACSB: Application of knowledge 5 Copyright © 2022 Pearson Education, Inc.
15.3
Aggregate Money Demand
1) The aggregate money demand depends on A) the interest rate. B) the price level. C) real national income. D) the interest rate, price level, and real national income. E) the price level and the liquidity of the asset. Answer: D Difficulty: Easy AACSB: Application of knowledge 2) All else equal, aggregate money demand ________ when interest rate ________. A) falls; rises B) rises; rises C) stays the same; rises D) falls; falls E) stays the same; falls Answer: A Difficulty: Easy AACSB: Application of knowledge 3) All else equal, if the price level ________, aggregate money demand will ________ because individual households and firms must spend more money than before to purchase their usual weekly baskets of goods and services. A) rises; rise B) rises; fall C) falls; rise D) rises; stay the same E) falls; stay the same Answer: A Difficulty: Easy AACSB: Application of knowledge 4) All else equal, when real national income (GNP) ________, more goods and services are sold in the economy and therefore aggregate money demand will ________. A) rises; rise B) rises; fall C) falls; rise D) rises; stay the same E) falls; stay the same Answer: A Difficulty: Easy AACSB: Application of knowledge
6 Copyright © 2022 Pearson Education, Inc.
5) The aggregate demand for money can be expressed by A) Md = P × L(R,Y). B) Md = L × P(R,Y). C) Md = P × Y(R, L). D) Md = R × L(P,Y). E) Md = R × L(R, P). Answer: A Difficulty: Easy AACSB: Application of knowledge 6) The aggregate real money demand schedule L(R,Y) A) slopes upward because a fall in the interest rate raises the desired real money holdings of each household and firm in the economy. B) slopes downward because a fall in the interest rate reduces the desired real money holdings of each household and firm in the economy. C) has a zero slope because a fall in the interest rate keeps constant the desired real money holdings of each household and firm in the economy. D) slopes downward because a fall in the interest rate raises the desired real money holdings of each household and firm in the economy. E) slopes downward because a rise in the interest rate makes consumers less focused on the liquidity of their assets. Answer: D Difficulty: Easy AACSB: Application of knowledge 7) For a given level of A) nominal GNP, changes in interest rates cause movements along the L(R,Y) schedule. B) real GNP, changes in interest rates cause a decrease of the L(R,Y) schedule. C) real GNP, changes in interest rates cause an increase of the L(R,Y) schedule. D) nominal GNP, changes in interest rates cause an increase in the L(R,Y) schedule. E) real GNP, changes in interest rates cause movements along the L(R,Y) schedule. Answer: E Difficulty: Easy AACSB: Application of knowledge 8) All else equal, A) changes in real GNP cause the L(R,Y) schedule to shift. B) changes in real GNP cause movements along the L(R,Y) schedule. C) changes in interest rates cause an increase of the L(R,Y) schedule. D) changes in interest rates cause a decrease in the L(R,Y) schedule. Answer: A Difficulty: Easy AACSB: Application of knowledge
7 Copyright © 2022 Pearson Education, Inc.
9) What are the main factors that determine aggregate money demand? Answer: The three main factors are interest rate, the price level and real national income. A rise in the interest rate causes individuals in the economy to reduce their demand for money. If the price level rises, individual households and firms will spend more money than before. When real national income (GNP) rises the demand for money will also rise. Difficulty: Moderate AACSB: Application of knowledge 10) Explain why one can write the demand for money as the price level times a function of the interest rate and real income as follows: = PxL (R, Y) Answer: The aggregate money demand is proportional to the price level. Imagine that all prices in an economy doubled, but the interest rate and everyone's real incomes remained unchanged. Then, the money value of each individual's average daily transactions would simply double, as would the amount of money each wishes to hold. Difficulty: Moderate AACSB: Application of knowledge 15.4
The Equilibrium Interest Rate: The Interaction of Money Supply and Demand
1) Which of the following statements is the MOST accurate? A) The money market is in equilibrium when the money supply set by the central bank equals aggregate money demand. B) The money market is in equilibrium when the aggregate money demand is sloping upward. C) The money market is in equilibrium when the money supply is set by the central bank. D) The money market is in equilibrium when the aggregate money demand is the sum of all the economy's individual money demands. Answer: A Difficulty: Easy AACSB: Application of knowledge 2) The money supply schedule is A) horizontal because is set by the central bank while P is taken as given. B) horizontal because is set by the central bank. C) vertical because is set by the households and firms while P is taken as given. D) vertical because and P are set by the central bank. E) vertical because is set by the central bank while P is taken as given. Answer: E Difficulty: Easy AACSB: Application of knowledge
8 Copyright © 2022 Pearson Education, Inc.
3) If individuals are holding more money than they desire A) they will attempt to reduce their liquidity by using money to purchase goods. B) they will attempt to reduce their liquidity by using money to purchase interest-bearing assets. C) they will attempt to reduce their liquidity by converting real money holdings into nominal money holdings. D) they will keep their holdings constant. Answer: B Difficulty: Easy AACSB: Application of knowledge 4) If there is an excess supply of money A) the interest rate falls. B) the interest rate rises. C) the real money supply shifts left to make an equilibrium. D) the real money supply shifts right to make an equilibrium. E) the interest rate stays constant, but consumer confidence falters. Answer: A Difficulty: Easy AACSB: Application of knowledge 5) Which of the following statements is the most ACCURATE? A) The market always moves toward an interest rate at which the real money supply equals aggregate real money demand. B) The market never moves toward an interest rate at which the real money supply equals aggregate real money demand. C) The market sometimes moves toward an interest rate at which the real money supply equals aggregate real money demand. D) The market rarely moves toward an interest rate at which the real money supply equals aggregate real money demand. Answer: A Difficulty: Easy AACSB: Application of knowledge 6) If there is initially an A) excess demand for money, the interest rate will fall, and the supply of money will rise. B) excess supply of money, the interest rate will fall, and if there is initially an excess demand, it will rise. C) excess supply of money, the interest rate will rise, and if there is initially an excess demand, it will fall. D) excess supply of money, the interest rate will fall, and if there is also an excess demand, it will fall rapidly. E) excess supply of money, the interest rate will rise, and if there is also an excess demand, it will rise rapidly. Answer: B Difficulty: Easy AACSB: Application of knowledge 9 Copyright © 2022 Pearson Education, Inc.
7) Which one of the following statements is the MOST accurate? A) A decrease in the money supply lowers the interest rate while an increase in the money supply raises the interest rate, given the price level and output. B) An increase in the money supply lowers the interest rate while a fall in the money supply raises the interest rate, given the price level. C) An increase in the money supply lowers the interest rate while a fall in the money supply raises the interest rate, given the output level. D) An increase in the money supply lowers the interest rate while a fall in the money supply raises the interest rate, given the price level and output. E) An increase in the money supply does not usually affect the interest rate. Answer: D Difficulty: Easy AACSB: Application of knowledge 8) An increase in A) nominal output raises the interest rate while a fall in real output lowers the interest rate, given the price level and the money supply. B) real output decreases the interest rate while a fall in real output increases the interest rate, given the price level. C) real output raises the interest rate while a fall in real output lowers the interest rate, given the money supply. D) nominal output raises the interest rate while a fall in real output lowers the interest rate, given the price level. E) real output increase raises the interest rate while a fall in real output lowers the interest rate, given the price level and the money supply. Answer: E Difficulty: Easy AACSB: Application of knowledge 9) What will be the effects of an increase in the money supply on the interest rate? Answer: An increase in the money supply will cause interest rates to decrease. This should increase investment and possibly consumption of durable goods. The reduction in the interest rate will cause a depreciation of the dollar. Difficulty: Moderate AACSB: Application of knowledge 10) What will be the effects of an increase in real output on the interest rate? Answer: An increase in real output will increase the interest rate. If investment depends only on interest rate, this will cause investment to go down. The increased interest rate will cause an appreciation of the dollar. Difficulty: Moderate AACSB: Application of knowledge
10 Copyright © 2022 Pearson Education, Inc.
15.5
The Money Supply and the Exchange Rate in the Short Run
1) An increase in a country's money supply causes A) its currency to appreciate in the foreign exchange market while a reduction in the money supply causes its currency to depreciate. B) its currency to depreciate in the foreign exchange market while a reduction in the money supply causes its currency to appreciate. C) no effect on the values of it currency in international markets. D) its currency to depreciate in the foreign exchange market while a reduction in the money supply causes its currency to further depreciate. E) its currency to depreciate in the domestic market and appreciate in the foreign market. Answer: B Difficulty: Easy AACSB: Application of knowledge 2) A decrease in a country's money supply causes A) its currency to appreciate in the foreign exchange market. B) its currency to depreciate in the foreign exchange market. C) no effect on the values of it currency in international markets. D) its currency to depreciate in the foreign exchange market while an increase in the money supply causes its currency to further depreciate. E) its currency to depreciate in the domestic market and appreciate in the foreign market. Answer: A Difficulty: Easy AACSB: Application of knowledge 3) Different from the short run, the long-run analysis of an economic event allows for A) adjustment of the price level and for full employment of all factors of production. B) no adjustment of the price level. C) no adjustment of factors of production. D) no adjustment of price levels and exchange rate expectations. Answer: A Difficulty: Easy AACSB: Application of knowledge
11 Copyright © 2022 Pearson Education, Inc.
4) Which one of the following statements is the MOST accurate? A) Given PUS, when the money supply rises, the dollar interest rate declines and the dollar depreciates against the euro. B) Given YUS, when the money supply rises, the dollar interest rate declines and the dollar depreciates against the euro. C) Given PUS and YUS, when the money supply decreases, the dollar interest rate declines and the dollar depreciates against the euro. D) Given PUS and YUS, when the money supply rises, the dollar interest rate declines and the dollar appreciates against the euro. E) Given PUS and YUS, when the money supply rises, the dollar interest rate declines and the dollar depreciates against the euro. Answer: E Difficulty: Moderate AACSB: Application of knowledge 5) Given PUS and YUS A) an increase in the European money supply causes the euro to appreciate against the dollar, but it does not disturb the U.S. money market equilibrium. B) an increase in the European money supply causes the euro to appreciate against the dollar, and it creates excess demand for dollars in the U.S. money market. C) an increase in the European money supply causes the euro to depreciate against the dollar, and it creates excess demand for dollars in the U.S. money market. D) an increase in the European money supply causes the euro to depreciate against the dollar, but it does not disturb the U.S. money market equilibrium. E) an increase in the European money supply causes the euro to depreciate against the dollar, and disturbing the U.S. money market equilibrium. Answer: D Difficulty: Moderate AACSB: Application of knowledge 6) Explain how the money markets of two countries are linked through the foreign exchange market. Answer: The monetary policy actions by the Fed affect the U.S. interest rate, changing the dollar/euro exchange rate that clears the foreign exchange market. The European System of Central Banks (ECB) can affect the exchange rate by changing the European money supply and interest rate. Difficulty: Moderate AACSB: Application of knowledge
12 Copyright © 2022 Pearson Education, Inc.
7) Explain the following figure.
Answer: The figure explains how the money markets of two countries are linked through the foreign exchange market. The monetary policy actions by the Fed affect the U.S. interest rate, changing the dollar/euro exchange rate that clears the foreign exchange market. The European System of Central Banks (ESCB) can affect the exchange rate by changing the European money supply and interest rate. Difficulty: Moderate AACSB: Application of knowledge
13 Copyright © 2022 Pearson Education, Inc.
8) Using a figure describing both the U.S. money market and the foreign exchange market, analyze the effects of a temporary increase in the European money supply on the dollar/euro exchange rate. Answer: An increase in the European money supply will reduce the interest rate on the euro and thus will cause the schedule of the expected euro return expressed in dollars to shift down, causing a reduction in the dollar/euro exchange rate, i.e., an appreciation of the U.S. Dollar. The euro depreciates against the dollar. The U.S. money demand and money supply are not going to be affected, and thus the interest rate in the U.S. will remain the same.
Difficulty: Difficult AACSB: Application of knowledge
14 Copyright © 2022 Pearson Education, Inc.
9) Combine a graph showing the interest parity condition and one showing money demand and supply to demonstrate simultaneous equilibrium in the money market and the foreign exchange market. How would an increase in the U.S. money supply affect the dollar/euro exchange rate and the U.S. interest rate? Illustrate your answer graphically and explain. Answer: Above the axis is depicted the foreign exchange market, where changes in the rate of return on the dollar are mapped into changes in the exchange rate. Below the axis is depicted the U.S. money market and shows the relation between the rate of return on the dollar and U.S. real money holdings. The mechanism works as follows. Consider an increase in the U.S. real money holdings. Supply and demand dictate that the demand for money must increase, so the rate of return must lower to equilibrate at point 2. The lower rate of return on the dollar will cause the dollar to depreciate (exchange rate moves to point ).
Difficulty: Difficult AACSB: Application of knowledge
15 Copyright © 2022 Pearson Education, Inc.
10) Analyze the effects of an increase in the European money supply on the dollar/euro exchange rate. Answer: The main points are: An increase in the European money supply will reduce the interest rate on the euro, and thus causes the euro to depreciate against the dollar. The U.S. money demand and money supply are not going to be affected, and thus the interest rate in the U.S. will remain the same. Difficulty: Moderate AACSB: Application of knowledge 11) What would be the effect of an increase in the European money supply in the dollar /euro exchange rate? Answer: An increase in the European money supply lowers the dollar return on euro deposits, i.e. the dollar appreciates against the euro. There is no change in the U.S. money market. Difficulty: Moderate AACSB: Application of knowledge
16 Copyright © 2022 Pearson Education, Inc.
12) Using a figure describing both the U.S. money market and the foreign exchange market, analyze the effects of an increase in the U.S. money supply on the dollar/euro exchange rate. Answer: An increase in the U.S. money supply will cause interest rates to decrease. This should increase investment and possibly consumption of durable goods. The reduction in the interest rate will cause a movement to the left along the schedule depicting the expected euro return expressed in dollars. The result is an increase in E or a depreciation of the dollar.
Difficulty: Difficult AACSB: Application of knowledge
17 Copyright © 2022 Pearson Education, Inc.
15.6
Money, the Price Level, and the Exchange Rate in the Long Run
1) An economy's long-run equilibrium is A) the equilibrium that would occur if prices were perfectly flexible. B) the equilibrium that would occur if prices were perfectly flexible and always adjusted immediately. C) the equilibrium that would occur if prices were perfectly flexible and always adjusted immediately to preserve full employment. D) the equilibrium that would occur if prices were perfectly fixed to preserve full employment. E) the equilibrium that would occur if prices were perfectly fixed at the full employment point. Answer: C Difficulty: Easy AACSB: Application of knowledge 2) Which one of the following statements is the MOST accurate? A) All else equal, an increase in a country's money supply causes a proportional increase in its price level. B) All else equal, an increase in a country's money supply causes a proportional decrease in its price level. C) All else equal, an increase in a country's money supply has no effect on its price level. D) All else equal, an increase in a country's money supply may cause an increase or decrease in its price level. Answer: A Difficulty: Easy AACSB: Application of knowledge 3) A change in the level of the supply of money A) increases the long-run values of the interest rate and real output. B) decreases the long-run values of the interest rate and real output. C) has no effect on the long-run values of the interest rate, but may affect real output. D) has no effect on the long-run values of real output, but may affect the interest rate. E) has no effect on the long-run values of the interest rate or real output. Answer: E Difficulty: Easy AACSB: Application of knowledge
18 Copyright © 2022 Pearson Education, Inc.
4) The long run effects of money supply change has A) an ambiguous effect on the long-run values of the interest rate or real output, a proportional change in the price level's long-run value in the opposite direction. B) a proportional effect on the long-run values of the interest rate or real output, a proportional change in the price level's long-run value in the same direction. C) no effect on the long-run values of the interest rate or real output, a proportional change in the price level's long-run value in the same direction. D) no effect on the long-run values of the interest rate or real output, no change in the price level's long-run value. E) an ambiguous effect on the long-run values of the interest rate or real output, a disproportional change in the price level's long-run value in the same direction. Answer: C Difficulty: Easy AACSB: Application of knowledge 5) Changes in the money supply growth rate A) are neutral in the short run. B) need not be neutral in the short run. C) are neutral in the long run. D) need not be neutral in the long run. E) affect the real output of the economy. Answer: D Difficulty: Easy AACSB: Application of knowledge 6) A sustained change in the monetary growth rate will A) immediately affect equilibrium real money balances by raising the money interest rate. B) eventually affect equilibrium nominal money balances by raising the money interest rate. C) eventually affect equilibrium real money balances by reducing the money interest rate. D) eventually affect equilibrium real money balances by raising the real interest rate. E) eventually affect equilibrium real money balances by raising the money interest rate. Answer: E Difficulty: Easy AACSB: Application of knowledge 7) A permanent increase in a country's money supply A) causes a more than proportional increase in its price level. B) causes a less than proportional increase in its price level. C) causes a proportional increase in its price level. D) leaves its price level constant in long-run equilibrium. E) causes an inversely proportional fall in its price level. Answer: C Difficulty: Easy AACSB: Application of knowledge
19 Copyright © 2022 Pearson Education, Inc.
8) Money demand behavior may A) change as a result of demographic trends or financial innovations such as electronic cashtransfer facilities. B) change only as a result of demographic trends. C) change only as a result of financial innovations such as electronic cash-transfer facilities. D) not change as a result of demographic trends or financial innovations such as electronic cashtransfer facilities. E) change as a result of demographic trends but not as a result of financial innovations such as electronic cash-transfer facilities. Answer: A Difficulty: Easy AACSB: Application of knowledge 9) Using year-by-year data from 1980-2014 shows that A) there is a strong positive relation between average Latin American money-supply growth and inflation. B) there is a strong negative relation between average Latin American money-supply growth and inflation. C) there is a strong positive relation between average Latin American money-supply growth and deflation. D) it is difficult to find a strong positive relation between average Latin American money-supply growth and inflation. E) there is a weak positive relation between average Latin American money-supply growth and inflation. Answer: A Difficulty: Easy AACSB: Application of knowledge 10) Which one of the following statements is the MOST accurate? A) A permanent increase in a country's money supply causes a proportional long-run depreciation of its currency against foreign currencies. B) A temporary increase in a country's money supply causes a proportional long-run depreciation of its currency against foreign currencies. C) A permanent increase in a country's money supply causes a proportional long-run appreciation of its currency against foreign currencies. D) A permanent increase in a country's money supply causes a proportional short-run depreciation of its currency against foreign currencies. E) A permanent increase in a country's money supply causes a proportional short-run appreciation of its currency against foreign currencies. Answer: A Difficulty: Easy AACSB: Application of knowledge
20 Copyright © 2022 Pearson Education, Inc.
11) Which one of the following statements is the MOST accurate? A) A permanent decrease in a country's money supply causes a proportional long-run appreciation of its currency against foreign currencies. B) A temporary increase in a country's money supply causes a proportional long-run depreciation of its currency against foreign currencies. C) A permanent decrease in a country's money supply causes a proportional long-run depreciation of its currency against foreign currencies. D) A permanent increase in a country's money supply causes a proportional short-run depreciation of its currency against foreign currencies. Answer: A Difficulty: Easy AACSB: Application of knowledge 12) Describe the effects of an increase in a country's long-run money supply on: (a) price level (b) values of interest rate and real output (c) exchange rates Answer: All else equal, an increase in a country's long-run money supply (a) causes a proportional increase in its price level. (b) has no effect on the long-run values of the interest rate or real output, a proportional change in the price level's long-run value in the same direction. (c) causes a proportional long-run depreciation of its currency against foreign currencies. Difficulty: Easy AACSB: Application of knowledge 13) Describe the effects of a decrease in a country's long-run money supply on (a) price level (b) values of interest rate and real output (c) exchange rate Answer: All else equal, a decrease in a country's long-run money supply (a) causes a proportional decrease in its price level. (b) has no effect on the long-run values of the interest rate or real output, a proportional change in the price level's long-run value in the same direction. (c) causes a proportional long-run appreciation of its currency against foreign currencies. Difficulty: Easy AACSB: Application of knowledge
21 Copyright © 2022 Pearson Education, Inc.
14) Use Figure 15-10 below to describe the link between average money growth and inflation in Western Hemisphere Developing Countries from 1980 to 2014.
Answer: Even year by year, there is a strong positive relation between average Latin American money supply growth and inflation. On average, years with higher money growth also tended to be years with higher inflation. In addition, the data points cluster around the 45-degree line, along which money supplies and price levels increase in proportion. The data confirm the strong long-run link between national money supplies and national price levels predicted by economic theory. Difficulty: Moderate AACSB: Application of knowledge
22 Copyright © 2022 Pearson Education, Inc.
15.7
Inflation and Exchange Rate Dynamics
1) For main industrial countries such as Japan and the U.S. A) there is much less month-to-month variability of the exchange rate, suggesting that price levels are relatively sticky in the short run. B) there is much more month-to-month variability of the exchange rate, suggesting that price levels are relatively sticky in the short run. C) there is almost the same month-to-month variability of the exchange rate and price levels. D) it is hard to tell whether month-to-month variability of the exchange rate is similar to changes in price levels. E) there is much more month-to-month variability of the exchange rate, suggesting that price levels are relatively sticky in the long run. Answer: B Difficulty: Easy AACSB: Application of knowledge 2) What term means an explosive and seemingly uncontrollable inflation in which money loses value rapidly and may even go out of use? A) superinflation B) stagflation C) hyperinflation D) maginflation E) deflation Answer: C Difficulty: Easy AACSB: Application of knowledge 3) Which one of the following statements is the MOST accurate? A) There is a lively academic debate over the possibility that seemingly sticky wages and prices are in reality quite fixed. B) There is a lively academic debate over the possibility that seemingly sticky wages and prices are in reality much more sticky than theory assumes. C) There is a lively academic debate over the possibility that seemingly sticky wages and prices are in reality quite flexible. D) There is no debate over the possibility that wages and prices are sticky in the long run. E) There is no debate over the possibility that wages and prices are sticky in the short run. Answer: C Difficulty: Easy AACSB: Application of knowledge
23 Copyright © 2022 Pearson Education, Inc.
4) During hyperinflation, exploding inflation causes real money demand to A) fall over time, and this additional monetary change makes money prices rise even more quickly than the money supply itself rises. B) increase over time, and this additional monetary change makes money prices rise even more quickly than the money supply itself rises. C) fall over time, and this additional monetary change makes money prices decrease even more quickly than the money supply itself rises. D) increase over time, and this additional monetary change makes money prices decrease even more quickly than the money supply itself rises. E) fall over time, and this additional monetary change makes money prices decrease even less quickly than the money supply itself rises. Answer: A Difficulty: Easy AACSB: Application of knowledge 5) A change in the money supply creates demand and cost pressures that lead to future increases in the price level from which main sources? I. Excess demand for output and labor II. Inflationary expectations III. Raw materials prices A) I B) II C) II and III D) I and II E) I, II, and III Answer: E Difficulty: Easy AACSB: Application of knowledge 6) The most extreme inflationary conditions occurred A) in Zimbabwe between 2007 and 2009. B) in Chile in 2012. C) in Eastern Europe in the 1990s. D) in Western Europe in the 1980s. E) in Germany in 2013. Answer: A Difficulty: Easy AACSB: Application of knowledge
24 Copyright © 2022 Pearson Education, Inc.
7) In a classic paper, Columbia University economist Phillip Cagan drew the line between inflation and hyperinflation at an inflation rate of A) 50 percent per month. B) 10 percent per month. C) 20 percent per month. D) 5 percent per month. E) 25 percent per month. Answer: A Difficulty: Easy AACSB: Application of knowledge 8) In a classic paper, Columbia University economist Phillip Cagan drew the line between inflation and hyperinflation at an inflation rate of A) more than 120 percent per year. B) more than 100 percent per year. C) more than 200 percent per year. D) more than 12,000 percent per year. E) more than 1,000 percent per year. Answer: D Difficulty: Easy AACSB: Application of knowledge 9) In Zimbabwe, the government stopped the country's hyperinflation by A) reducing domestic monetary growth drastically. B) returning to a gold/silver currency standard. C) switching to foreign currencies that are relatively stable. D) passing a law making price increases illegal. E) implementing a new currency based on diamonds. Answer: C Difficulty: Moderate AACSB: Application of knowledge 10) "Although the price levels appear to display short-run stickiness in many countries, a change in the money supply creates immediate demand and cost pressures that eventually lead to future increase in the price level." Discuss. Answer: The statement is true. The pressures come from three main sources: excess demand for output and labor; inflationary expectations; and, raw material prices. Difficulty: Moderate AACSB: Application of knowledge
25 Copyright © 2022 Pearson Education, Inc.
11) Explain the effects of a permanent increase in the U.S. money supply in the short run and in the long run. Assume that the U.S. real national income is constant. Answer: An increase in the nominal money supply raises the real money supply, lowering the interest rate in the short run. The money supply increase is considered to continue in the future; thus, it will affect the exchange rate expectations. This will make the expected return on the euro more desirable and thus the dollar depreciates. In the case of a permanent increase in the U.S. money supply, the dollar depreciates more than under a temporary increase in the money supply. Now, in the long run, prices will rise until the real money balances are the same as before the permanent increase in the money supply. Since the output level is given, the U.S. interest rate, which decreased before, will start to increase, until it will move back to its original level. The equilibrium interest rate must be the same as its original long run value. This increase in the interest rate must cause the dollar to appreciate against the euro after its sharp depreciation as a result of the permanent increase in the money supply. So a large depreciation is followed by an appreciation of the dollar. Eventually, the dollar depreciates in proportion to the increase in the price level, which in turn increases by the same proportion as the permanent increase in the money supply. Thus, money is neutral, in the sense that it cannot affect, in the long run, real variables, such as output, investment, etc. Difficulty: Difficult AACSB: Application of knowledge 15.8
Permanent Money Supply Changes and the Exchange Rate
1) In a world where the price level could adjust immediately to its new long-run level after a money supply increase A) the dollar interest rate would increase because prices would adjust immediately and prevent the money supply from rising. B) the dollar interest rate would not fall because prices would adjust immediately and prevent the money supply from rising. C) the dollar interest rate would fall because prices would adjust immediately and prevent the money supply from decreasing. D) the dollar interest rate would decrease because prices would adjust immediately and prevent the money supply from decreasing. E) the dollar interest rate would fall because prices would not be able to prevent the money supply from rising. Answer: B Difficulty: Easy AACSB: Application of knowledge 2) The exchange rate is said to overshoot when A) its immediate response to a disturbance is greater than its long-run response. B) its immediate response to a disturbance is less than its long-run response. C) its immediate response to a disturbance is the same as its long-run response. D) its immediate response to a disturbance is not related to its long-run response. Answer: A Difficulty: Easy AACSB: Application of knowledge 26 Copyright © 2022 Pearson Education, Inc.
3) After a permanent increase in the money supply A) the exchange rate overshoots in the short run. B) the exchange rate overshoots in the long run. C) the exchange rate smoothly depreciates in the short run. D) the exchange rate smoothly appreciates in the short run. E) the exchange rate remains the same. Answer: A Difficulty: Easy AACSB: Application of knowledge 4) Michael Woodford says the following is an advantage of interest-rate instruments for central banks. A) Conduct monetary policy without inflation. B) Conduct monetary policy even if checking deposits pay interest at competitive rates. C) Conduct monetary policy without government approval. D) Conduct monetary policy with consumers in mind. E) Conduct monetary policy with workers in mind. Answer: B Difficulty: Easy AACSB: Analytical thinking 5) Inflation targeting was initiated by which central bank in 1989? A) U.S. B) Japan C) Canada D) New Zealand E) U.K. Answer: D Difficulty: Easy AACSB: Application of knowledge 6) Which of the following can help to explain why higher inflation may lead to currency appreciations? A) The interest rate is not the prime target of monetary policy. B) Most central banks adjust their policy interest rates expressly so as to keep inflation in check. C) Central banks increase the money supply leading to overshooting of the exchange rate. D) Inflation will increase the purchasing power of a currency. E) The world market does not adjust their currency trade to reflect inflation. Answer: B Difficulty: Easy AACSB: Application of knowledge
27 Copyright © 2022 Pearson Education, Inc.
7) Which one of the countries below announces inflation targets? A) Japan B) U.S. C) Canada D) Mexico E) Nicaragua Answer: C Difficulty: Easy AACSB: Application of knowledge
28 Copyright © 2022 Pearson Education, Inc.
8) Using figures for both the short run and the long run, show the effects of a permanent increase in the U.S. money supply. Try to line up your figures to the short and long run equilibria side by side. Assume that the U.S. real national income is constant. Answer:
An increase in the nominal money supply raises the real money supply, lowering the interest rate in the short run (the movement from 1 to 2 on the lower left figure). The money supply increase is considered to continue in the future, and thus it will affect the exchange rate expectations. This will make the expected return on the euro more desirable and thus the dollar depreciates. In the case of a permanent increase in the U.S. money supply, the dollar depreciates more than under a temporary increase in the money supply (from point to point in the upper left figure).
29 Copyright © 2022 Pearson Education, Inc.
Now, in the long run, (the right hand side figure), prices will rise until the real money balances are the same as before the permanent increase in the money supply (from point 2 to point 4, in the lower right figure). Since the output level is given, the U.S. interest rate which decreased before, will start to increase, until it will move back to its original level (from Point 2 to 4 in the lower left figure). The equilibrium interest rate must be the same as its original long run value (at point 4 in the lower right figure). This increase in the interest rate must cause the dollar to appreciate against the euro after its sharp depreciation as a result of the permanent increase in the money supply (this process is depicted in the upper right figure from point to ). So a large depreciation (from Point in the left upper figure to pint in both the left and right upper figures) is followed by an appreciation of the dollar (the movement from to point in the upper right hand side figure). Eventually, the dollar depreciates in proportion to the increase in the price level, which in turn increases by the same proportion as the permanent increase in the money supply. Thus, money is neutral, in the sense that it cannot affect, in the long run, real variables, such as output, investment, etc. Note that points and represent the same exchange rate. Difficulty: Difficult AACSB: Application of knowledge 9) Explain the exchange rate over-shooting hypothesis. Answer: Many prices in the economy are written into long-term contracts and cannot be changed immediately when changes in the money supply occur. A permanent increase in M, holding P constant, increases the real money supply (M/P) and lowers the nominal interest rate (R). This shifts the dollar return schedule left. A permanent increase in M also creates the expectation that in the long run all prices, including the exchange rate, would rise. A rise in the expected exchange rate shifts the ERR(DM) schedule right. Therefore, in the short run equilibrium is established at point 2 In the long run the price level adjusts and rises proportionately with the money supply. Therefore, M/P and R return to their initial levels in the long run and the equilibrium exchange rate is determined at point 3. In other words, the exchange rate first overshoots and then returns to its long run level. Therefore, the fluctuations in E are much stronger than those of P. Difficulty: Difficult AACSB: Application of knowledge
30 Copyright © 2022 Pearson Education, Inc.
10) Using 4 different figures, plot the time paths showing the effects of a permanent increase in the United States money supply on: (a) U.S. Money supply (b) The dollar interest rate. (c) The U.S. price level (d) The dollar/euro exchange rate Answer: See below.
Difficulty: Difficult AACSB: Application of knowledge 11) What is inflation targeting? Answer: A central bank can keep inflation from getting too high or too low by raising the interest rate when it learns that inflation is running higher than expected, and lowering it when inflation is running lower. Many central banks now follow formal strategies of inflation targeting, under which they announce a target (or target range) for the inflation rate and adjust the interest rate to keep inflation on target. Difficulty: Moderate AACSB: Application of knowledge
31 Copyright © 2022 Pearson Education, Inc.
12) Can higher inflation lead to currency appreciation? Explain the "paradox" case of Canada when inflation drove Canadian dollar higher. Answer: Yes. Suppose market participants unexpectedly push up prices and borrow to enlarge the money supply. Thus, when the Canadian government releases new price data, the data show a price level higher than what market participants had previously predicted. If the Bank of Canada is expected to raise interest rates quickly so as to push the price level and money supply back on course, there is no reason for the future expected exchange rate to change. But with higher Canadian interest rates, interest parity requires an expected future depreciation of the Canadian dollar, which is consistent with an unchanged future exchange rate only if the Canadian dollar appreciates immediately. Difficulty: Moderate AACSB: Application of knowledge
32 Copyright © 2022 Pearson Education, Inc.
International Economics: Theory and Policy, 12e (Krugman) Chapter 16 Price Levels and the Exchange Rate in the Long Run 16.1
The Law of One Price
1) Which of the following statements is the MOST accurate? A) In the long run, national price levels play a minor role in determining both interest rates and the relative prices at which countries' products are traded. B) In the long run, national price levels play a key role only in determining interest rates. C) In the long run, national price levels play a key role only in determining the relative prices at which countries' products are traded. D) In the long run, national price levels play a key role in determining both interest rates and the relative prices at which countries' products are traded. E) In the long run, national price levels play no role in determining interest rates and the relative prices at which countries' products are traded. Answer: D Difficulty: Easy AACSB: Application of knowledge 2) Which of the following statements is the MOST accurate? The law of one price states A) in competitive markets free of transportation costs and official barriers to trade, identical goods sold in different countries must sell for the same price when their prices are expressed in terms of the same currency. B) in competitive markets free of transportation costs and official barriers to trade, identical goods sold in the same country must sell for the same price when their prices are expressed in terms of the same currency. C) in competitive markets free of transportation costs and official barriers to trade, identical goods sold in different countries must sell for the same price. D) identical goods sold in different countries must sell for the same price when their prices are expressed in terms of the same currency. E) in competitive markets free of official barriers to trade, identical goods are sold at the same price regardless of transportation costs. Answer: A Difficulty: Easy AACSB: Application of knowledge 3) The law of one price states that when trade is open and costless, identical goods must trade at ________ prices regardless of where they are sold. A) the same relative B) higher C) lower D) different Answer: A Difficulty: Easy AACSB: Application of knowledge
1 Copyright © 2022 Pearson Education, Inc.
4) Using the law of one price, if the dollar/pound exchange rate is $1.50 per pound and a sweater sells for $45 in New York must sell for ________ per sweater in London, the same as its price in New York. A) ₤30.00 B) ₤45.00 C) ₤67.50 D) ₤15.00 Answer: A Difficulty: Easy AACSB: Application of knowledge 5) Using the law of one price, if the dollar/euro exchange rate is $1.25 per euro, a Tesla Model 3 car sells for $35,000 in the U.S. market must sell for ________ in Europe. A) ₤28,000 B) ₤43,750 C) ₤35,714 D) ₤52,490 E) ₤54,990 Answer: A Difficulty: Easy AACSB: Application of knowledge 6) Under the law of one price, if the price for one particular good is PUS in the US and PE in Europe, then A) E$/P = PUS/PE. B) E$/€ = PUS/PE C) E$/€ = PUS - PE. D) E$/€ = PE/PES. E) E$/€ = PUS + PE. Answer: B Difficulty: Easy AACSB: Application of knowledge
2 Copyright © 2022 Pearson Education, Inc.
7) Explain the Law of One Price. Give an example. Answer: The law of one price states that in competitive markets free of transportation costs and trade barriers, identical goods sold in different countries must sell for the same price when expressed in terms of the same currency. = (E$/£) × ( E$/£ =
) for good i.
/
If, for example, the price of the same sweater was cheaper in London than in New York, U.S. importers and British exporters would have an incentive to buy sweaters in London and ship them to New York, pushing the London price up and the New York price down, until both were equal. Difficulty: Moderate AACSB: Application of knowledge 8) Fill in the following table, assuming the law of one price prevails.
Answer:
Difficulty: Moderate AACSB: Analytical thinking
3 Copyright © 2022 Pearson Education, Inc.
16.2
Purchasing Power Parity
1) The theory of purchasing power parity states that the exchange rate between two countries' currencies ________ the ratio of the countries' price levels. A) equals B) is higher than C) is lower than D) is different from Answer: A Difficulty: Easy AACSB: Application of knowledge 2) The purchasing power parity theory predicts that a ________ in a currency's domestic purchasing power will be associated with a proportional currency ________ in the foreign exchange market. A) fall; depreciation B) fall; appreciation C) rise; depreciation D) rise; floating Answer: A Difficulty: Easy AACSB: Application of knowledge 3) Under Purchasing Power Parity A) E$/€ = PiUS/PiE. B) E$/€ = PiE/PiUS. C) E$/€ = PUS/PE. D) E$/€ = PE/PES. E) E$/€ = PiE + PiUS/PiE. Answer: C Difficulty: Easy AACSB: Application of knowledge 4) If the reference commodity basket costs $200 in the United States and €160 in Europe, Purchasing Power Parity (PPP) theory predicts a dollar/euro exchange rate of ________ per euro. If the U.S. price level were to triple to $600/basket, PPP would imply an exchange rate of ________ per euro. A) $1.25; $3.75 B) $1.25; $0.42 C) $0.80; $2.40 D) $0.80; $0.27 Answer: A Difficulty: Easy AACSB: Application of knowledge
4 Copyright © 2022 Pearson Education, Inc.
5) Which of the following statements is the MOST accurate? A) The law of one price applies only to the general price level. B) The law of one price applies to the general price level while PPP applies to individual commodities. C) The law of one price applies to individual commodities while PPP applies to both the general price level and to individual commodities. D) PPP applies only to individual commodities. E) The law of one price applies to individual commodities while PPP applies to the general price level. Answer: E Difficulty: Easy AACSB: Application of knowledge 6) Which of the following statements is the MOST accurate? A) If PPP holds true, then the law of one price holds true for every commodity as long as the reference baskets used to reckon different countries' price levels are the same. B) If the law of one price holds true for every commodity, PPP must hold automatically. C) If the law of one price holds true for every commodity, PPP must automatically hold as long as the reference baskets used to reckon different countries' price levels are the same. D) If the law of one price does not hold true for every commodity, PPP cannot be true as long as the reference baskets used to reckon different countries' price levels are the same. E) If PPP holds true, then the law of one price must hold true automatically. Answer: C Difficulty: Easy AACSB: Application of knowledge 7) Which of the following statements is the MOST accurate? A) Absolute PPP does not imply relative PPP. B) Relative PPP implies absolute PPP. C) There is no causality relation between the two. D) Absolute PPP implies relative PPP. E) Absolute PPP is inversely related to relative PPP. Answer: D Difficulty: Easy AACSB: Application of knowledge
5 Copyright © 2022 Pearson Education, Inc.
8) Which of the following statements is the MOST accurate? A) Relative PPP may be valid even when absolute PPP is not, provided the factors causing deviations from absolute PPP are more or less stable over different commodities space. B) Absolute PPP may be valid even when relative PPP is not, provided the factors causing deviations from relative PPP are more or less stable over time. C) Relative PPP may be valid even when absolute PPP is not, provided the factors causing deviations from absolute PPP are more or less stable over time. D) Relative PPP is not valid when absolute PPP is not. E) Relative PPP is only valid when absolute PPP is valid, providing the factors causing deviations from relative PPP are more or less stable over time. Answer: C Difficulty: Moderate AACSB: Application of knowledge 9) Explain Purchasing Power Parity. Answer: PPP states that the exchange rate between two countries' currencies equals the ratio of the countries' price levels. A fall in a currency's domestic purchasing power (i.e. an increase in the domestic price level) will be associated with a proportional currency depreciation in the foreign exchange market and vice versa. = PUS/PE where P is the price of a reference commodity basket. Rearrange: PUS = × (PE) Thus, PPP asserts that all countries' price levels are equal when measured in terms of the same currency. Difficulty: Moderate AACSB: Application of knowledge 10) Discuss the relationship between PPP and the Law of One Price. Answer: The law of one price applies to individual commodities while PPP applies to the general price level. Proponents of PPP argue that its validity in the long run doesn't require the law of one price to hold exactly. When goods and services temporarily become more expensive in one country than in others, the demands for its currency and its products falls, pushing the exchange rate and domestic prices back in line with PPP and vice versa. Difficulty: Moderate AACSB: Application of knowledge 11) Discuss the differences between Absolute PPP and Relative PPP. Answer: Absolute PPP states that the exchange rate between two currencies equals the ratio of their price levels. Relative PPP states that the percentage change in the exchange rate between two currencies over a given period equals the difference between the inflation rates of those two currencies. Difficulty: Moderate AACSB: Application of knowledge
6 Copyright © 2022 Pearson Education, Inc.
12) Explain why Relative PPP is useful when comparing countries that base their price levels on different product baskets. Answer: For Example: If the U.S. price level rises by 10% over a year while Europe's rises by only 5%, relative PPP predicts a 5% depreciation of the dollar against the euro. This just cancels the 5% by which U.S. inflation exceeds European, leaving the relative domestic and foreign purchasing powers of both currencies unchanged. ( )/ = ( )US,t - ( )E,t between dates t and t - 1. Relative PPP is useful when comparing countries that base their price levels on different product baskets. Relative PPP may be valid even when absolute PPP is not. Difficulty: Moderate AACSB: Application of knowledge 13) Suppose Russia's inflation rate is 200% over one year but the inflation rate in Switzerland is only 2%. According to relative PPP, what should happen over the year to the Swiss franc's exchange rate against the Russian ruble? Answer: (Eruble/franc, t - Eruble/franc, t-1)/Eruble/franc, t-1 = 2 - 0.02 = 1.98 So there will be a 198% depreciation of the ruble against the franc or, conversely, a 198% appreciation of the franc against the ruble. Difficulty: Moderate AACSB: Application of knowledge
7 Copyright © 2022 Pearson Education, Inc.
14) Assuming relative PPP, fill in the table below:
Answer: Using (
-
)/
= ΠUS, t - ΠE, t one gets:
Difficulty: Moderate AACSB: Analytical thinking
8 Copyright © 2022 Pearson Education, Inc.
16.3
A Long-Run Exchange Rate Model Based on PPP
1) Which of the following statements is the MOST accurate? In general A) the monetary approach to the exchange rate is a long run theory. B) the monetary approach to the exchange rate is a short run theory. C) the monetary approach to the exchange rate is both a short and long run theory. D) the monetary approach to the exchange rate neither long run nor short run theory. E) the monetary approach to the exchange rate is considered less practical than the law of one price. Answer: A Difficulty: Easy AACSB: Application of knowledge 2) The monetary approach makes the general prediction that A) the exchange rate, which is the relative price of American and European money, is fully determined in the long run by the relative supplies of those monies. B) the exchange rate, which is the relative price of American and European money, is fully determined in the short run by the relative supplies of those monies and the relative demands for them. C) the exchange rate, which is the relative price of American and European money, is fully determined in the short run and long run by the relative supplies of those monies and the relative demands for them. D) the exchange rate, which is the relative price of American and European money, is fully determined in the long run by the relative supplies of those monies and the relative demands for them. E) the money supply in the U.S. will adjust to European monetary equilibrium. Answer: D Difficulty: Easy AACSB: Application of knowledge
9 Copyright © 2022 Pearson Education, Inc.
3) Under the monetary approach to exchange rate theory, money supply growth at a constant rate A) eventually results in ongoing price level deflation at the same rate, but changes in this longrun deflation rate do not affect the full-employment output level or the long-run relative prices of goods and services. B) eventually results in ongoing price level inflation at the same rate, but changes in this longrun inflation rate do affect the full-employment output level and the long-run relative prices of goods and services. C) eventually results in ongoing price level inflation at the same rate, but changes in this longrun inflation rate do not affect the full-employment output level or the long-run relative prices of goods and services. D) eventually results in ongoing price level inflation at the same rate, but changes in this longrun inflation rate do not affect the full-employment output level, only the long-run relative prices of goods and services. E) eventually results in ongoing price level deflation at the same rate, but changes in this longrun deflation rate do not affect the full-employment output level, only the long-run relative prices of goods and services. Answer: C Difficulty: Easy AACSB: Application of knowledge 4) Which of the following statements is the MOST accurate? In general, under the monetary approach to the exchange rate A) the interest rate is not independent of the money supply growth rate in the short run. B) the interest rate is independent of the money supply growth rate in the long run. C) the interest rate is not independent of the money supply growth rate in the long run, but independent in the short run. D) the interest rate is not independent of the money supply growth rate in the long run. E) the interest rate is a factor of the money supply growth rate only in the short term. Answer: D Difficulty: Easy AACSB: Application of knowledge 5) Which of the following statements is the MOST accurate? In general, under the monetary approach to the exchange rate A) while the short-run interest rate does not depend on the absolute level of the money supply, continuing growth in the money supply eventually will affect the interest rate. B) while the long-run interest rate does depend on the absolute level of the money supply, continuing growth in the money supply does not affect the interest rate. C) while the long-run interest rate does not depend on the absolute level of the money supply, continuing growth in the money supply eventually will affect the interest rate. D) the long-run interest rate does not depend on the absolute level of the money supply, and thus continuing growth in the money supply will not affect the interest rate. E) while the short-run interest rate does not depend on the absolute level of the money supply, continuing decline in the money supply eventually will not affect the interest rate. Answer: C Difficulty: Easy AACSB: Application of knowledge 10 Copyright © 2022 Pearson Education, Inc.
6) If people expect relative PPP to hold A) the difference between the interest rates offered by dollar and euro deposits will equal the difference between the inflation rates expected, in the United States and Europe, respectively, over the relevant horizon. B) the difference between the interest rates offered by dollar and euro deposits will equal the difference between the inflation rates expected in Europe and the United States, respectively. C) the difference between the interest rates offered by dollar and euro deposits will equal the difference between the inflation rates expected, over the relevant horizon, in the United States and Europe, respectively, in the short run. D) the difference between the interest rates offered by dollar and euro deposits will be above the difference between the inflation rates expected, over the relevant horizon, in the United States and Europe, respectively. E) the difference between the interest rates offered by dollar and euro deposits will be below the difference between the inflation rates expected, over the relevant horizon, in the United States and Europe, respectively. Answer: A Difficulty: Moderate AACSB: Application of knowledge 7) Who among the following list of people is an early 20th century economist from Yale University who wrote the book The Theory of Interest? A) Gustav Cassel B) Irving Fisher C) David Ricardo D) Paul Krugman E) Israel Kirzner Answer: B Difficulty: Easy AACSB: Application of knowledge 8) Under PPP (and by the Fisher Effect), all else equal A) a rise in a country's expected inflation rate will eventually cause a more-than proportional rise in the interest rate that deposits of its currency offer in order to accommodate for the higher inflation. B) a fall in a country's expected inflation rate will eventually cause an equal rise in the interest rate that deposits of its currency offer. C) a rise in a country's expected inflation rate will eventually cause an equal rise in the interest rate that deposits of its currency offer. D) a rise in a country's expected inflation rate will eventually cause a less than proportional rise in the interest rate that deposits of its currency offer to accommodate the rise in expected inflation. E) a fall in a country's expected inflation rate will eventually cause an inversely proportional rise in the interest rate that deposits of its currency offer to accommodate the rise in expected inflation. Answer: C Difficulty: Easy AACSB: Application of knowledge 11 Copyright © 2022 Pearson Education, Inc.
9) In the short run A) the interest rate can rise when the domestic money supply falls. B) the interest rate can decrease when the domestic money supply falls. C) the interest rate stays constant when the domestic money supply falls. D) the interest rate rises in the same proportion as the domestic money supply falls. E) the interest rate never rises when the domestic money supply falls. Answer: A Difficulty: Easy AACSB: Application of knowledge 10) Under a flexible-price monetary approach to the exchange rate A) when the domestic money supply falls, the price level would eventually fall, increasing the interest rate. B) when the domestic money supply falls, the price level would fall right away, causing a reduction in the interest rate. C) when the domestic money supply falls, the price level would fall right away, causing an increase in the interest rate. D) when the domestic money supply falls, the price level would eventually fall, keeping the interest rate constant. E) when the domestic money supply falls, the price level would fall right away, keeping the interest rate constant. Answer: E Difficulty: Easy AACSB: Application of knowledge 11) Under sticky prices A) a fall in the money supply raises the interest rate to preserve money market equilibrium. B) a fall in the money supply reduces the interest rate to preserve money market equilibrium. C) a fall in the money supply keeps the interest rate intact to preserve money market equilibrium. D) a fall in the money supply does not affect the interest rate in the short run, only in the long run. E) a fall in the money supply raises the interest rate to preserve money market equilibrium in the long run. Answer: A Difficulty: Easy AACSB: Application of knowledge
12 Copyright © 2022 Pearson Education, Inc.
12) Under sticky prices A) an interest rate rise is associated with lower expected deflation and a long-run currency appreciation, so the currency appreciates immediately. B) an interest rate rise is associated with higher expected inflation and a long-run currency appreciation, so the currency appreciates immediately. C) an interest rate rise is associated with lower expected inflation and a long-run currency depreciation, so the currency appreciates immediately. D) an interest rate rise is associated with lower expected inflation and a long-run currency depreciation, so the currency depreciates immediately. E) an interest rate rise is associated with lower expected inflation and a long-run currency appreciation, so the currency appreciates immediately. Answer: E Difficulty: Easy AACSB: Application of knowledge 13) Under the monetary approach to the exchange rate A) an interest rate decrease is associated with higher expected inflation and a currency that will be weaker on all future dates. B) an interest rate increase is associated with higher expected deflation and a currency that will be weaker on all future dates. C) an interest rate increase is associated with higher expected inflation and a currency that will be strengthened on all future dates. D) an interest rate increase is associated with higher expected deflation and a currency that will be strengthened on all future dates. E) an interest rate increase is associated with higher expected inflation and a currency that will be weaker on all future dates. Answer: E Difficulty: Moderate AACSB: Application of knowledge 14) Under the monetary approach to the exchange rate A) a reduction in the money supply will cause immediate currency depreciation. B) a rise in the money supply will cause currency depreciation. C) a rise in the money supply will cause immediate currency appreciation. D) a rise in the money supply will cause depreciation. E) a rise in the money supply will cause immediate currency depreciation. Answer: E Difficulty: Moderate AACSB: Application of knowledge 15) Explain why an exchange rate model based on PPP is a long run theory. Answer: PPP theory is a monetary approach to the exchange rate. It is a long-run theory because it does not allow for price rigidities. It assumes that prices can adjust right away to maintain full employment as well as PPP. Difficulty: Moderate AACSB: Application of knowledge 13 Copyright © 2022 Pearson Education, Inc.
16) Present and explain the Fundamental Equation of the Monetary Approach. Answer: Assume = PUS/PE and that domestic price levels depend on domestic money demands and supplies: PUS = MUSS/L(R$, YUS) PE = MES/L( , YE) Therefore, the exchange rate is fully determined in the long run by the relative supplies of those monies and the relative real demands for them. Shifts in interest rates and output levels affect the exchange rate only through their influence on money demand. Difficulty: Moderate AACSB: Application of knowledge 17) What are the predictions for the long-run equilibrium of the Monetary Approach? Answer: Money supplies: Given the equations, = PUS/PE PUS = MUSS/L(R$, YUS) PE = MES/L( , YE) one can show that an increase in the U.S. money supply MUSS causes a proportional increase in the U.S. price level PUS, which in turn causes a proportional increase in . Thus, an increase in U.S. money supply causes a proportional long-run depreciation of the dollar against the euro and vice versa. Interest rates: A rise in the interest rate R$ lowers U.S. money demand L(R$, YUS) thereby causing a rise in the U.S. price level and a proportional depreciation of the dollar against the euro. Output levels: A rise in U.S. output YUS raises real U.S. money demand leading to a fall in the long-run U.S. price level and an appreciation of the dollar against the euro. Difficulty: Moderate AACSB: Application of knowledge
14 Copyright © 2022 Pearson Education, Inc.
18) Discuss the effects of ongoing inflation based on the PPP theory. Answer: Other things equal, money supply growth at a constant rate eventually results in ongoing price level inflation at the same rate as the money supply growth, but changes in this long-run inflation rate do not affect the full-employment output level or the long-run relative prices of goods and services. The interest rate, however, is affected by continuing growth in the money supply (inflation). This can be shown by combining PPP with the interest parity condition. To show it analytically, recall that the condition of parity between dollar and euro assets is: R$ =
+(
-
)/
And according to relative PPP: ( )/ = ΠUS,t - ΠE,t If people expect relative PPP to hold, the difference between interest rates offered by dollar and euro deposits will equal the difference between the expected inflation rates, over the relative horizon, in the U.S. and Europe. Difficulty: Moderate AACSB: Application of knowledge 19) Describe and explain the relationship between expected inflation rates in two countries and their interest rate differential according to the PPP theory. Answer: Expected inflation is given by the following equation: Πe = (Pe - P)/P where Pe is the expected price level in a country a year from today. If relative PPP is expected to hold then: (
-
)/
-
Combine the expected version of relative PPP with the interest parity condition: R$ =
+(
R$ -
=
-
)/
Rearrange: -
If, as PPP predicts, currency depreciation is expected to offset international inflation difference, the interest rate difference must equal the expected inflation difference. Difficulty: Difficult AACSB: Application of knowledge
15 Copyright © 2022 Pearson Education, Inc.
20) What is the Fisher Effect? Provide an example. Answer: All else equal, a rise in a country's expected inflation rate will eventually cause an equal rise in the interest rate that deposits of its currency offer. Similarly, a fall in the expected inflation rate will eventually cause a fall in the interest rate. Ex: If the expected U.S. inflation were to rise permanently from Π to Π + ΔΠ, current dollar interest rates R$ would eventually catch up to the higher inflation, rising by a value ΔR$ = ΔΠ in accordance with the Monetary Approach that in the long run purely monetary developments should have no effect on an economy's relative prices since the real rate of return on dollar assets would remain unchanged. Difficulty: Moderate AACSB: Application of knowledge 21) Discuss the different effects on the domestic interest rates when prices are assumed flexible and when they are assumed to be sticky. Answer: When prices are flexible, a decrease in the domestic money supply has no effect on the interest rate, because of the immediate decrease in the price level. However, when prices are assumed to be sticky, a decrease in the domestic money supply will cause the interest rate to rise, because the sticky domestic price level leads to an excess demand for real money balances at the initial interest rate. Difficulty: Moderate AACSB: Application of knowledge 16.4
Empirical Evidence on PPP and the Law of One Price
1) Which of the following statements is the MOST accurate? A) All versions of the Purchasing Power Parity theory do badly in explaining the actual data on exchange rates and national price levels. B) Only the absolute Purchasing Power Parity explains well the actual data on exchange rates and national price levels. C) Only the relative Purchasing Power Parity explains well the actual data on exchange rates and national price levels. D) All versions of the Purchasing Power Parity theory explain well the actual data on exchange rates and national price levels. Answer: A Difficulty: Easy AACSB: Application of knowledge 2) In practice A) changes in national price levels often tell us relatively little about exchange rate movements. B) changes in national price levels raise the exchange rate. C) changes in national price levels lower the exchange rate. D) changes in national price levels often tell us about exchange rate movements. E) changes in national price levels match identical changes in the exchange rate. Answer: A Difficulty: Easy AACSB: Application of knowledge 16 Copyright © 2022 Pearson Education, Inc.
3) Which of the following statements is the MOST accurate? A) The prices of identical commodity baskets, when converted to a single currency, are the same across countries. B) The prices of identical commodity baskets, when converted to a single currency, differ substantially across countries. C) The prices of identical commodity baskets, when converted to a single currency, do not differ substantially across countries. D) The prices of identical commodity baskets, when converted to a single currency, are often the same across countries. E) The prices of identical commodity baskets, when converted to a single currency, are the same across countries more than 50% of the time. Answer: B Difficulty: Easy AACSB: Application of knowledge 4) Which of the following statements is the MOST accurate? A) The law of one price does fare well in all recent studies. B) The law of one price does fare well in many recent studies. C) The law of one price sometimes fares well in recent studies. D) The law of one price does not fare well in recent studies. E) The law of one price has not been studied recently. Answer: D Difficulty: Easy AACSB: Application of knowledge 5) Which of the following statements is the MOST accurate? A) Relative PPP is not a reasonable approximation to the data. B) Relative PPP is sometimes a reasonable approximation to the data but often performs poorly. C) Relative PPP is sometimes a reasonable approximation to the data. D) PPP is sometimes a reasonable approximation to the data. E) PPP is sometimes a reasonable approximation to the data but usually performs poorly. Answer: B Difficulty: Easy AACSB: Application of knowledge
17 Copyright © 2022 Pearson Education, Inc.
6) Relative PPP predicts that the yen/dollar exchange rate and the relative Japan-U.S. price levels will move in proportion. In the 1980s, when Japan's price level consistently fell relative to that in the United States, A) there was a steep appreciation of the dollar against the yen, against what relative PPP predicted. B) there was a steep depreciation of the dollar against the yen, same as what relative PPP predicted. C) there was a steep appreciation of the yen against the dollar, against what relative PPP predicted. D) there was a steep appreciation of the yen against the dollar, same as what relative PPP predicted. Answer: A Difficulty: Easy AACSB: Application of knowledge 7) What can explain the failure of relative PPP to hold in reality? Answer: Government measures of the price level differ from country to country. One reason for these differences is that people living in different countries spend their income in different ways. Because of this inherent difference among countries, certain baskets will be affected more by price changes given their consumptions basket. For example, consumers in country, X, eats more fish relative to another country. More than likely, the government, upon determining a commodity basket to reflect preference, will have an overwhelming representation of fish in their basket. Any price level change in the fish market will be felt particularly by country X, and their overall price level will reflect this. Thus, changes in the relative prices of basket components can cause relative PPP to become distorted. Difficulty: Difficult AACSB: Application of knowledge
18 Copyright © 2022 Pearson Education, Inc.
16.5
Explaining the Problems with PPP
1) In order for the condition E$/HK$ = PUS/PHK to hold, what assumptions does the principle of purchasing power parity make? A) Only that there are no transportation costs and restrictions on trade. B) Only that the markets are perfectly competitive, i.e., P = MC. C) The factors of production are identical between countries. D) No arbitrage exists. E) HK and the U.S. are perfectly competitive and there are no transportation costs or restrictions on trade. Answer: E Difficulty: Easy AACSB: Application of knowledge 2) The PPP theory fails in reality for all of the following reasons EXCEPT A) transport costs. B) monopolistic or oligopolistic practices in goods markets. C) the inflation data reported in different countries are based on different commodity baskets. D) restrictions on trade. E) inflation rates are unrelated to money supply growth. Answer: E Difficulty: Easy AACSB: Application of knowledge 3) Which one of the following statements is the MOST accurate? A) The purchasing power of any given country's currency will increase in countries where the prices of non-tradable goods rise. B) The purchasing power of any given country's currency will fall in countries where the prices of non-tradable goods fall. C) The purchasing power of any given country's currency will fall in countries where the prices of non-tradable goods rise. D) The purchasing power of any given country's currency will remain constant in countries where the prices of non-tradable goods rise. E) The purchasing power of any given country's currency will fall in countries where the prices of non-tradable goods remain constant. Answer: C Difficulty: Easy AACSB: Application of knowledge
19 Copyright © 2022 Pearson Education, Inc.
4) Which one of the following statements is the MOST accurate? A) Relative price changes could not lead to PPP violations even if trade were free and costless. B) Relative price changes could lead to PPP violations only if trade were free and costless. C) Relative price changes could lead to PPP violations even if trade were free and costless. D) Price changes could lead to PPP violations even if trade were free and costless. E) Price changes could not lead to PPP violations even if trade were free and costless. Answer: C Difficulty: Moderate AACSB: Application of knowledge 5) Which one of the following statements is the MOST accurate? A) Departures from PPP are similar in both the short run and long run. B) Departures from PPP are even greater in the long run than in the short run. C) Departures from PPP are always greater in the short run than in the long run. D) It is hard to tell whether departures from PPP are greater in the short run than in the long run. E) Departures from PPP may often be greater in the short run than in the long run. Answer: E Difficulty: Easy AACSB: Application of knowledge 6) Floating exchange rates A) systematically lead to much larger but less frequent short-run deviations from the absolute PPP. B) systematically lead to much larger and more frequent short-run deviations from the relative PPP. C) systematically lead to much smaller and less frequent short-run deviations from the relative PPP. D) systematically lead to much smaller but more frequent short-run deviations from the relative PPP. E) systematically lead to much smaller and less frequent short-run deviations from the absolute PPP. Answer: B Difficulty: Easy AACSB: Application of knowledge 7) Which of the following are theories meant to explain "Why price levels are lower in poorer countries"? A) only Bhagwati-Kravis-Lipsey B) only Balassa-Samuelson C) only Goldberg-Knetter D) Bhagwati-Kravis-Lipsey and Balassa-Samuelson E) Bhagwati-Kravis-Lipsey and Goldberg-Knetter Answer: D Difficulty: Easy AACSB: Application of knowledge
20 Copyright © 2022 Pearson Education, Inc.
8) Does the existence of non-tradable goods allow for deviations from Purchasing power Parity? Answer: Yes, the existence of nontradables allows deviations from PPP. This is because the price of a nontradable is determined entirely by its domestic supply and demand curves, and in turn fluctuations in demand and supply for these good will affect the price level. Examples include housing, haircut, services etc. Difficulty: Moderate AACSB: Application of knowledge 9) What effect do non-tradable goods have on PPP? Answer: The effect is quite substantial. In 2006, the services accounted for about 75% of the value of U.S. output. Along with haircuts, non-tradable goods include routine medical treatment, housing etc. For the most part, nontradable goods are comprised of services, and the output of the construction industry. Nontradable help explain much of the wide departure from PPP that is present in empirical data. Difficulty: Moderate AACSB: Application of knowledge 10) Discuss why the empirical support for PPP and the law of one price is weak in recent data. Answer: The failure of these propositions in the real world is related to trade barriers and departures from free competition, factors that can result in pricing to market by exporters. In addition, different definitions of price levels in different countries bedevil attempts to test PPP using the price indexes governments publish. For some products, including many services, international transport costs are so steep that these products become non-tradable (see page 425). Difficulty: Moderate AACSB: Application of knowledge 11) Explain why price levels are lower in poorer countries. Answer: One theory explains the difference in prices on different endowments of capital and labor (Bhagwati, Kravis, and Lipsey). The explanation is as follows: ∙ Rich countries have high capital-labor ratios while poor countries have much more labor relative to capital. ∙ Because rich countries have high capital labor ratios, the MPL is greater and thus they have a higher wage. ∙ Higher wages lead to higher disposable income, and citizens' demand for goods will increase. ∙ Because labor is cheaper in poor countries and is used intensively in producing non-tradable goods; non-tradable goods will be cheaper in the poor countries than in the rich. ∙ The price of non-tradable goods will move with the increase in wage, thus increasing the price level of the good. Rich Countries: Expensive Non-tradable goods vs. Poor Countries: Cheap Non-tradable goods. Difficulty: Moderate AACSB: Application of knowledge
21 Copyright © 2022 Pearson Education, Inc.
16.6
Beyond Purchasing Power Parity: A General Model of Long-Run Exchange Rates
1) Which of the following statements is MOST accurate? A) The United States price level will place a relatively light weight on commodities produced and consumed in America, while the European price level will place a relatively heavy weight on commodities produced and consumed in Europe. B) The United States price level will place a relatively light weight on commodities produced and consumed in America, and the European price level will place a relatively light weight on commodities produced and consumed in Europe. C) The United States price level will place a relatively heavy weight on commodities produced and consumed in America, and the European price level will place a relatively heavy weight on commodities produced and consumed in Europe. D) The United States price level will place a relatively heavy weight on commodities produced and consumed in Europe, and the European price level will place a relatively heavy weight on commodities produced and consumed in America. E) The United States price level will place a relatively light weight on commodities produced and consumed in Europe, and the European price level will place a relatively heavy weight on commodities produced and consumed in America. Answer: C Difficulty: Moderate AACSB: Application of knowledge 2) When the domestic money prices of goods are held constant A) a nominal dollar appreciation makes U.S. goods cheaper compared with foreign goods. B) a nominal dollar depreciation makes U.S. goods less appealing in foreign markets. C) a nominal dollar appreciation does not affect the prices of U.S. goods. D) a nominal dollar depreciation makes U.S. goods more expensive compared with foreign goods. E) a nominal dollar depreciation makes U.S. goods cheaper compared with foreign goods and a nominal dollar appreciation makes U.S. goods more expensive compared with foreign goods. Answer: E Difficulty: Easy AACSB: Application of knowledge 3) An increase in the world relative demand for U.S. output causes A) a short-run real depreciation of the dollar against the euro. B) a long-run real appreciation of the dollar against the euro. C) a long-run real depreciation of the dollar against the euro. D) a short-run real appreciation of the euro against the dollar. E) a long-run real appreciation of the euro against the dollar. Answer: B Difficulty: Easy AACSB: Application of knowledge
22 Copyright © 2022 Pearson Education, Inc.
4) A decrease in the world relative demand for U.S. output causes A) a long-run real depreciation of the dollar against the euro. B) a short-run real appreciation of the euro against the dollar. C) a short-run real depreciation of the dollar against the euro. D) a long-run real appreciation of the euro against the dollar. Answer: A Difficulty: Easy AACSB: Application of knowledge 5) Which of the following statements is MOST accurate? A) A relative expansion of U.S. output causes a long-run depreciation of the dollar against the euro, while a relative expansion of European output causes a long-run real appreciation of the dollar against the euro. B) A relative decline of U.S. output causes a long-run depreciation of the dollar against the euro, while a relative expansion of European output causes a long-run real appreciation of the dollar against the euro. C) A relative expansion of U.S. output causes a long-run appreciation of the dollar against the euro, while a relative expansion of European output causes a long-run real depreciation of the dollar against the euro. D) A relative expansion of U.S. output causes a long-run depreciation of the dollar against the euro, while a relative decline of European output causes a long-run real appreciation of the dollar against the euro. E) A relative decline of U.S. output causes a long-run depreciation of the dollar against the euro, while a relative decline of European output causes a long-run real appreciation of the dollar against the euro. Answer: A Difficulty: Moderate AACSB: Application of knowledge 6) When all variables start out at their long-run equilibrium levels, the most important determinants of long-run swings in nominal exchange rates do NOT include A) a shift in relative money supply levels. B) a shift in relative money supply growth rates. C) a change in relative output demand. D) a change in relative output supply. E) a change in relative inflation rates. Answer: E Difficulty: Difficult AACSB: Application of knowledge
23 Copyright © 2022 Pearson Education, Inc.
7) Which of the following statements is MOST accurate? A) In the output market, an increase in demand for U.S. output leads to an increase in the longrun nominal dollar/euro exchange rate. B) In the output market, an increase in the demand for European output leads to an increase in the long-run nominal dollar/euro exchange rate. C) In the output market, a decrease in demand for U.S. output leads to a decrease in the long-run nominal dollar/euro exchange rate. D) In the output market, an increase in the demand for European output leads to a decrease in the long-run nominal dollar/euro exchange rate. E) In the output market, an increase in the demand for European output leads to an increase in the long-run nominal euro/dollar exchange rate. Answer: B Difficulty: Moderate AACSB: Application of knowledge 8) Which of the following statements is MOST accurate? A) In the money market, an increase in U.S. money supply level leads to a proportional increase in the long-run nominal dollar/euro exchange rate. B) In the money market, an increase in European money supply level leads to a proportional increase in the long-run nominal dollar/euro exchange rate. C) In the money market, an increase in U.S. money supply growth rate leads to a decrease in the long-run nominal dollar/euro exchange rate. D) In the money market, an increase in European money supply growth leads to an increase in the long-run nominal dollar/euro exchange rate. E) In the money market, an increase in U.S. money supply level leads to a proportional decrease in the long-run nominal dollar/euro exchange rate. Answer: A Difficulty: Easy AACSB: Application of knowledge 9) In the long run A) exchange rates obey relative PPP when all disturbances occur in the output markets. B) exchange rates obey absolute PPP when all disturbances occur in the output markets. C) exchange rates are unlikely to obey relative PPP when all disturbances occur in the output markets. D) exchange rates are unlikely to obey relative PPP when all disturbances are monetary in nature. E) exchange rates obey absolute PPP when all disturbances are monetary in nature. Answer: C Difficulty: Moderate AACSB: Application of knowledge
24 Copyright © 2022 Pearson Education, Inc.
10) In the long run A) exchange rates obey absolute PPP when all disturbances are monetary in nature. B) exchange rates obey relative PPP when all disturbances occur in the output markets. C) exchange rates obey absolute PPP when all disturbances occur in the output markets. D) exchange rates are unlikely to obey relative PPP when all disturbances occur in the output markets. E) exchange rates are unlikely to obey relative PPP when all disturbances are monetary in nature. Answer: A Difficulty: Easy AACSB: Application of knowledge 11) Which of the following statements is the MOST accurate about the Law of One Price on Scandinavian ferry lines? A) Due to menu costs, the Law of One Price does not hold. B) To avoid arbitrage opportunities, the Law of One Price must hold. C) Transaction costs of exchanging currency causes the Law of One Price to fail. D) Transportation costs between ferry lines leads to a violation of the Law of One Price. E) The physical distance allowed the Law of One Price to hold. Answer: C Difficulty: Easy AACSB: Application of knowledge 12) What are the predictions of the PPP theory with regards to the real exchange rates? Answer: The real exchange rate between two countries is a broad summary measure of the prices of one country's goods and services relative to the other's. PPP predicts that the real exchange rate never permanently changes, which is different from nominal exchange rates that deals with the relative price of two currencies. Difficulty: Moderate AACSB: Application of knowledge 13) What is the real exchange rate between the dollar and the euro equal to? Answer: Let ∙ Real dollar/euro exchange rate = ∙
Nominal exchange rate =
∙ Price of an unchanging basket in US = Pus ∙ Price of an unchanging basket in Europe = PE Then, =( × PE)/PUS A rise in the real dollar/euro exchange rate is called a real depreciation of the dollar against the euro, a fall in purchasing power of the dollar. A fall in the real dollar/euro exchange rate is called a real appreciation of the dollar against the euro, a rise in purchasing power of the dollar. Difficulty: Difficult AACSB: Application of knowledge 25 Copyright © 2022 Pearson Education, Inc.
14) Define the concept of the real exchange rate and explain how it differs from the nominal exchange rate. Answer: In general, the real exchange rate between two countries' currencies is the price of the second country's commodity basket (in terms of the first country's currency) relative to the price of the first country's commodity basket. For example, in the case of U.S. and Europe, the real dollar/euro exchange rate is the dollar value of Europe's price level divided by the U.S. price level. We can thus denote the real dollar/euro exchange rate ( =(
) as:
× PE)/PUS
where is the nominal dollar/euro exchange rate, PE is Europe's price level, and PUS is the U.S. price level. Unlike the real exchange rate, which is the relative price of two output baskets, the nominal exchange rate is the relative price of two currencies. However, as we can see from the equation above, real exchange rates are defined in terms of nominal exchange rates. Difficulty: Difficult AACSB: Application of knowledge 15) Describe the chain of events leading to exchange rate determination for the following cases: (a) An increase in U.S. money supply (d) Increase in growth rate of U.S. money supply (c) Increase in world relative demand for U.S. products (d) Increase in relative U.S. output supply Answer: Chain of events leading to exchange rate determination: = × (Pus/PE) Increase in U.S. money supply: Pus rises in proportion to the money supply; q remains the same. All dollar prices will rise (including dollar price of euro). Increase in growth rate of U.S. money supply: Inflation rate, dollar interest rate, Pus, E, rises in proportion to Pus. Increase in world relative demand for U.S. products: E falls, and q does as well. Increase in relative U.S. output supply: Dollar depreciates, lowers relative price of U.S. output, rise in q, effect on E is not clear since q and Pus work in opposite directions. Difficulty: Difficult AACSB: Application of knowledge
26 Copyright © 2022 Pearson Education, Inc.
16) Construct a table that will summarize the effects of money market and output market changes on the long-run nominal dollar/euro exchange rate Answer:
Difficulty: Difficult AACSB: Application of knowledge 16.7
International Interest Rate Differences and the Real Exchange Rate
1) Interest rate differences between countries depend on A) differences in expected inflation, but not on expected changes in the real exchange rate. B) differences in expected changes in the real exchange rate, but not on expected inflation. C) neither differences in expected inflation, nor on expected changes in the real exchange rate. D) differences in expected inflation and nothing else. E) differences in expected inflation, and on expected changes in the real exchange rate. Answer: E Difficulty: Easy AACSB: Application of knowledge 2) The expected rate of change in the nominal dollar/euro exchange rate is best described as A) the expected rate of change in the real dollar/euro exchange rate minus the U.S.-Europe expected inflation difference. B) the expected rate of change in the real dollar/euro exchange rate plus the U.S.-Europe real interest rate difference. C) the expected rate of change in the real dollar/euro exchange rate plus the U.S.-Europe expected inflation difference. D) the expected rate of change in the real dollar/euro exchange rate minus the U.S.-Europe real interest rate difference. E) the expected rate of change in the real dollar/euro exchange rate plus the European expected inflation. Answer: C Difficulty: Easy AACSB: Application of knowledge
27 Copyright © 2022 Pearson Education, Inc.
3) The expected change in the real dollar/euro exchange rate is A) the expected change in the nominal dollar/euro exchange rate minus the U.S.-Europe expected inflation difference. B) the expected change in the nominal dollar/euro exchange rate plus the U.S.-Europe expected inflation difference. C) the expected change in the nominal dollar/euro exchange rate minus the real interest rate difference. D) the expected change in the nominal dollar/euro exchange rate plus the real interest rate difference. Answer: A Difficulty: Easy AACSB: Application of knowledge 4) If the expected rate of real dollar depreciation against the euro is 1 percent per year, and the expected inflation difference between the United States and Europe is 5 percent per year, the dollar/euro interest difference will be ________ percent. A) 6 B) 4 C) 3 D) 2 E) 1 Answer: A Difficulty: Easy AACSB: Application of knowledge 16.8
Real Interest Parity
1) Nominal interest rates are A) rates of return measured in monetary terms. B) rates of return measured in terms of a country's output. C) rates of return measured in PPP terms. D) rates of return after taking inflation into account. Answer: A Difficulty: Easy AACSB: Application of knowledge 2) Real interest rates are A) rates of return measured in terms of a country's output. B) rates of return measured in monetary terms. C) rates of return measured in PPP terms. D) rates of return before taking inflation into account. Answer: A Difficulty: Easy AACSB: Application of knowledge
28 Copyright © 2022 Pearson Education, Inc.
3) The expected real interest rate (re) in terms of the nominal interest rate (R) and the expected inflation rate (πe) is given by A) re = πe + R. B) re = 2πe + R2. C) re = πe + R2. D) re = R - πe. E) re = R2 - πe. Answer: D Difficulty: Easy AACSB: Application of knowledge 4) The difference between nominal and real interest rates is that A) nominal interest rates are measured in terms of a country's output, while real interest rates are measured in monetary terms. B) nominal interest rates are measured in monetary terms, while real interest rates are measured in terms of a country's output. C) nominal interest rates can fluctuate, while real interest rates always remain fixed. D) real interest rates can fluctuate, while nominal interest rates always remain fixed. E) real interest rates are the same in every country, while nominal interest rates are different for every country. Answer: B Difficulty: Easy AACSB: Application of knowledge 5) What is the real interest rate parity condition? Answer: Real Interest Parity Condition: (
-
)=(
-
)/
Difficulty: Moderate AACSB: Application of knowledge 6) Is a depreciation of the dollar/euro exchange rate correlated with a decrease in the dollar return on U.S. deposits? Explain your answer. Answer: No. Assume that the Interest Parity is maintained, i.e., R$ =
+(
-
)/
Holding
constant, one would expect a depreciation of the dollar/euro exchange rate (i.e.
increase in
) to be correlated with a decrease in R$, dollar returns on euro deposits.
However, the higher expected inflation in the U.S. implies an increase in the future dollar to euro exchange rate. Thus, the quantity (
-
despite a depreciation in the current dollar to euro exchange rate, Difficulty: Difficult AACSB: Application of knowledge 29 Copyright © 2022 Pearson Education, Inc.
)/
, the expected
goes up and, increases .
16.9 Appendix to Chapter 16: The Fisher Effect, the Interest Rate, and the Exchange Rate Under the Flexible-Price Monetary Approach 1) The monetary approach to interest rates assumes that the prices of goods are ________, which implies that a country's currency will ________, when nominal interest rates ________ because of ________ expected future inflation. A) perfectly flexible; depreciate; increase; higher B) perfectly flexible; appreciate; increase; higher C) immutable; depreciate; increase; higher D) immutable; appreciate; decrease; higher E) absolutely inflexible; depreciate; decrease; higher Answer: A Difficulty: Moderate AACSB: Application of knowledge 2) When the nominal dollar interest rate ________, money demand will ________, and the general price level will ________. A) increases; decrease; increase B) increases; increase; increase C) increases; decrease; decrease D) increases; increase; decrease E) decreases; increase; increase Answer: A Difficulty: Easy AACSB: Application of knowledge
30 Copyright © 2022 Pearson Education, Inc.
3) To answer the following question, please refer to the figure below. Concentrating only at the lower right quadrant, discuss the effects of a change in U.S. expected inflation.
31 Copyright © 2022 Pearson Education, Inc.
Answer: The lower right quadrant shows the equilibrium in the U.S. Money Market, where =
/
A given interest rate
corresponds with a given U.S. real money supply,
/
.
Consider a rise of ΔΠ in the future rate of U.S money supply growth (i.e. an increase in the expected rate of inflation). The Key point: The rise in expected future inflation generates expectations of more rapid currency depreciation in the future. Under PPP the dollar now depreciates at a rate of Π+ΔΠ. Interest parity therefore requires the dollar interest rate to rise where = Note: R$ -
+ ΔΠ (Point 2 in the figure.) =
-
This relation shows a change in the U.S. interest rate due to an increase in expected U.S. inflation has no effect on the euro interest rate. The rise in the interest rate from
to
creates a monetary excess supply of real U.S.
money balances at the prevailing price level P1. However, since under this Monetary Approach, prices are assumed to be flexible, prices will immediately adjust from P1 to P2, thus causing the following two effects: One, reducing real money supply and two, bringing U.S. money market back into equilibrium. Difficulty: Difficult AACSB: Application of knowledge
32 Copyright © 2022 Pearson Education, Inc.
4) To answer the following question, please refer to the figure below. Concentrating only at the lower left quadrant, discuss the relationship between the U.S. real money supply and the dollar/euro exchange rate, E$/E.
33 Copyright © 2022 Pearson Education, Inc.
Answer: The lower left quadrant in the figure described the Purchasing Power Parity (PPP) relationship. The relationship between the U.S. real money supply and the dollar/euro exchange rate, E$/E is negative. is equal to the price level ratio, PUS/ . In this derivation of the relationship, the following variables are assumed constants: and So,
. =
/PUS PUS ↑ →
↑
→ Thus, the purchasing power of the dollar decreases due to the increase in the price level. → i.e., dollar depreciates due to PPP Difficulty: Difficult AACSB: Application of knowledge
34 Copyright © 2022 Pearson Education, Inc.
,
,
5) To answer the following question, please refer to the figure below. Concentrating only at the upper right quadrant, discuss the foreign exchange market equilibrium.
Answer: The upper right quadrant describes the equilibrium in the foreign exchange market. We begin with the Interest Parity Condition. R$ =
+(
-
)/
In general, two effects are present: →
and
→
A rise in the interest rate normally creates an excess demand for dollar deposits and appreciation in the currency market. However, in this case the increase is due to higher expected inflation or higher expected monetary growth in the U.S. which implies a faster expected depreciation of the dollar against the euro, , thus,
goes up and thus reduced the attractiveness of U.S. deposits.
Difficulty: Difficult AACSB: Application of knowledge 35 Copyright © 2022 Pearson Education, Inc.
International Economics: Theory and Policy, 12e (Krugman) Chapter 17 Output and the Exchange Rate in the Short Run 17.1
Determinants of Aggregate Demand in an Open Economy
1) What is the best way to describe aggregate demand? A) quantity required to satisfy equilibrium B) net export demand C) amount of a country's goods and services demanded by households and firms throughout the world D) individual's demand E) domestic demand of foreign imports Answer: C Difficulty: Easy AACSB: Application of knowledge 2) In the short-run, a country's overall output level depends on ________; in the long-run, domestic output depends only on ________. A) aggregate demand for its products; the available domestic supplies of factors of production such as labor and capital B) the available domestic supplies of factors of production such as labor and capital; aggregate demand for its products C) aggregate supply for its products; aggregate demand for its products D) the available domestic supplies of factors of production such as labor and capital; aggregate supply for its products Answer: A Difficulty: Easy AACSB: Application of knowledge 3) Aggregate demand for an open economy's output is the sum of A) consumption demand, investment demand, government demand, net export demand. B) investment demand, government demand, the current account. C) consumption demand, government demand, investment demand. D) current account, government demand, investment demand. Answer: A Difficulty: Easy AACSB: Application of knowledge 4) Disposable income is defined as A) Y - C. B) Y - T. C) C - T. D) I - C. E) Y - I. Answer: B Difficulty: Easy AACSB: Application of knowledge 1 Copyright © 2022 Pearson Education, Inc.
5) Disposable income equals A) national income minus taxes. B) present income plus expected future income. C) national income minus consumption. D) investment minus consumption. E) consumption minus taxes. Answer: A Difficulty: Easy AACSB: Application of knowledge 6) Which one of the following statements is MOST accurate? A) Factors of production can only be over-employed in the short run. B) Factors of production can only be under-employed in the short run. C) Factors of production can be over- or under-employed in the long run. D) Factors of production can be over- or under-employed in the short run. E) Factors of production are fully employed in the short run. Answer: D Difficulty: Easy AACSB: Application of knowledge 7) Why is the economy at full employment in the long run? A) Only wages have the ability to adjust. B) Only price can adjust. C) Prices don't adjust. D) Wages and the price level eventually adjust to full employment equilibrium levels. E) Government policies eventually converge on the full employment strategy. Answer: D Difficulty: Easy AACSB: Application of knowledge 8) Which one of the following statements is MOST accurate? A) In the long run, foreign output depends only on the available domestic supplies of factors of production. B) In the short run, domestic output depends only on the available domestic supplies of factors of production. C) In the long run, domestic output depends only on the available domestic supplies of factors of production. D) In the long run and in the short run, domestic output depends only on the available domestic supplies of factors of production. E) In the long run, domestic output depends only on the real exchange rate. Answer: C Difficulty: Easy AACSB: Application of knowledge
2 Copyright © 2022 Pearson Education, Inc.
9) Consumption level can be written as a function of A) disposable income. B) investment income. C) government income. D) exports income. Answer: A Difficulty: Easy AACSB: Application of knowledge 10) Consumption and disposable income are A) positively related. B) negatively related. C) completely unrelated. D) randomly related. Answer: A Difficulty: Easy AACSB: Application of knowledge 11) Because each consumer demands ________ goods and services as his or her real income ________, consumption will ________ as disposable income ________ at the aggregate level. A) more; rises; increase; increases B) less; rises; decrease; increases C) more; lowers; increase; decreases D) more; lowers; decrease; increases Answer: A Difficulty: Easy AACSB: Application of knowledge 12) The current account balance is A) the supply of a country's exports less the country's own demand for imports. B) the demand for a country's exports plus the country's own demand for imports. C) the country's own demand for imports less the demand for a country's exports. D) the demand for a country's exports less the country's own demand for imports. E) the country's federal reserves minus the national debt. Answer: D Difficulty: Easy AACSB: Application of knowledge 13) The domestic currency price of a representative foreign expenditure basket is A) P, the domestic price level. B) E, the nominal exchange rate. C) P times E, the domestic price level times the domestic price level. D) P , the foreign price level. E) P times E, the foreign price level times the nominal exchange rate. Answer: E Difficulty: Moderate AACSB: Application of knowledge 3 Copyright © 2022 Pearson Education, Inc.
14) The domestic currency price of a representative domestic expenditure basket is A) P, the domestic price level. B) E, the nominal exchange rate. C) P times E, the domestic price level times the domestic price level. D) P , the foreign price level. E) P times E, the foreign price level times the nominal exchange rate. Answer: A Difficulty: Easy AACSB: Application of knowledge 15) The real exchange rate, q, is defined as A) the price of the foreign basket in terms of the domestic one. B) the price of the domestic basket in terms of the foreign one. C) the price of the foreign basket. D) the price of the domestic basket. E) the nominal exchange rate in terms of the domestic basket. Answer: A Difficulty: Easy AACSB: Application of knowledge 16) The real exchange rate is A) how much of a foreign currency you can buy with the domestic currency. B) foreign CPI divided by the domestic CPI. C) the price of foreign goods in terms of domestic goods. D) the price of foreign goods in dollars. E) the domestic currency divided by the price level. Answer: C Difficulty: Easy AACSB: Application of knowledge 17) A country's domestic currency's real exchange rate, q, is best described by A) the price of similar goods in the same market. B) the price of the domestic basket in terms of the foreign one. C) the price of a domestic basket. D) the price of the foreign basket in terms of the domestic basket. E) the price of different goods baskets in the same market. Answer: D Difficulty: Easy AACSB: Application of knowledge
4 Copyright © 2022 Pearson Education, Inc.
18) A country's domestic currency's real exchange rate, q, is defined as A) E. B) E times P. C) E times P . D) (E times P )/P. E) P/(E times P ). Answer: D Difficulty: Easy AACSB: Application of knowledge 19) If the representative basket of European goods and services costs 40 euros, the representative U.S. basket costs $50, and the dollar/euro exchange rate is $0.90 per euro, then the price of the European basket in terms of U.S. basket is A) [(0.9 $/euro) (40 euro per a European basket)]/[(50 $/U.S. basket)]. B) [(0.9 $/euro) (50 $/U.S. basket)]/[(40 euro per a European basket)]. C) [(40 euro per a European basket)]/[(50 $/U.S. basket) (0.9 $/euro)]. D) [(50 $/U.S. basket)]. E) [(0.9 $/euro) (40 euro per a European basket) (50 $ U.S. basket)]. Answer: A Difficulty: Moderate AACSB: Application of knowledge 20) If the representative basket of European goods and services costs 40 euros, the representative U.S. basket costs $50, and the dollar/euro exchange rate is $1.10 per euro, then the price of the European basket in terms of U.S. basket is A) 0.88. B) 0.80. C) 1.38. D) 1.25. Answer: A Difficulty: Easy AACSB: Application of knowledge 21) Current account is given by the equation: A) CA = IM - EX (measured in terms of domestic output). B) CA = IM - EX (measured in terms of foreign output). C) CA = EX - IM (measured in terms of domestic output). D) CA = EX - IM (measured in terms of foreign output). E) CA = EX + IM (measured in terms of domestic output). Answer: C Difficulty: Easy AACSB: Application of knowledge
5 Copyright © 2022 Pearson Education, Inc.
22) A ________ in the price of the foreign basket in terms of domestic baskets will be associated with a ________ in the relative price of foreign output in general relative to domestic output. A) rise; rise B) rise; fall C) fall; rise D) fall; no change E) rise; no change Answer: A Difficulty: Easy AACSB: Application of knowledge 23) How does an increase in the real exchange rate affect exports and imports? A) exports increase; imports decrease B) exports decrease; imports increase C) exports increase; imports change ambiguously D) exports change ambiguously; imports decrease E) exports increase; imports are constant Answer: C Difficulty: Easy AACSB: Application of knowledge 24) When EP /P rises A) IM will rise. B) IM will fall. C) IM may rise or fall. D) IM is not affected. E) IM and P* will both rise. Answer: C Difficulty: Moderate AACSB: Application of knowledge 25) When the real exchange rate rises A) imports measured in terms of domestic output will rise. B) imports measured in terms of domestic output will fall. C) imports measured in terms of domestic output will never be affected. D) imports measured in terms of domestic output may rise or fall. E) imports measured in terms of foreign output will rise. Answer: D Difficulty: Moderate AACSB: Application of knowledge
6 Copyright © 2022 Pearson Education, Inc.
26) An increase in the real exchange rate A) makes imports more expensive. B) makes imports less expensive. C) does not affect import values. D) always makes the number of imports rise. E) makes domestic consumers spend more on only foreign imports. Answer: A Difficulty: Moderate AACSB: Application of knowledge 27) Which of the following compete to determine whether the current account improves or worsens following a rise in the real exchange rate? A) appreciation and depreciation B) crowding out effect and producers effect C) volume effect and value effect D) volume effect and inflation E) producers effect and value effect Answer: C Difficulty: Easy AACSB: Application of knowledge 28) Assuming that the volume effect outweighs the value, the current account will increase if A) the real exchange rate decreases. B) the real exchange rate increases. C) disposable income increases. D) exports fall. E) domestic prices fall. Answer: B Difficulty: Easy AACSB: Application of knowledge 29) What have we assumed when we conclude that a real depreciation of the currency improves the current account? A) The volume effect outweighs the value effect. B) The value effect outweighs the volume effect. C) All else equal and the volume effect outweighs the value effect. D) All else equal and the value effect outweighs the volume effect. E) All else equal and the volume effect equals the value effect. Answer: C Difficulty: Moderate AACSB: Application of knowledge
7 Copyright © 2022 Pearson Education, Inc.
30) What have we assumed when we conclude that a real appreciation of the currency worsens the current account? A) All else equal and the volume effect outweighs the value effect. B) The value effect outweighs the volume effect. C) The volume effect outweighs the value effect. D) All else equal and the value effect outweighs the volume effect. E) All else equal and the volume effect equals the value effect. Answer: A Difficulty: Easy AACSB: Application of knowledge 31) If a dollar depreciation lowers the relative cost of producing in the United States, A) the German auto manufacturer BMW can shift production from Germany to its Spartanburg, South Carolina. B) the German auto manufacturer BMW can shift production from Spartanburg, South Carolina to Germany. C) the German auto manufacturer BMW can shift production from Spartanburg, South Carolina to outside the U.S. D) the German auto manufacturer BMW can shift production from Germany to non-U.S. locations. Answer: A Difficulty: Easy AACSB: Application of knowledge 32) Which one of the following statements is the MOST accurate? A) An increase in disposable income improves the current account. B) An increase in disposable income does not affect the current account. C) An increase in disposable income worsens the current account. D) An increase in income worsens the current account. E) An increase in income improves the current account. Answer: C Difficulty: Moderate AACSB: Application of knowledge
8 Copyright © 2022 Pearson Education, Inc.
33) Which one of the following statements is the MOST accurate? A) An increase in the real exchange rate and an increase in disposable income improve the current account. B) A decrease in the real exchange rate and a decrease in disposable income improve the current account. C) A decrease in the real exchange rate and an increase in disposable income improve the current account. D) An increase in the real exchange rate and a decrease in disposable income improve the current account. E) An increase in the real exchange rate and a decrease in disposable income lowers the current account. Answer: D Difficulty: Moderate AACSB: Application of knowledge 34) Which of the following would cause the current account to decrease? A) an increase in the nominal exchange rate, E B) an appreciation of the home currency C) an increase in disposable income D) an increase in foreign prices, P E) a decrease in domestic prices, P Answer: C Difficulty: Easy AACSB: Application of knowledge 35) Find the real exchange rate for the following case: Assume that the representative basket of European goods and services costs 40 euros and the representative U.S. basket costs $50, and the dollar/euro exchange rate is $0.90 per euro, then the price of the European basket in terms of U.S. basket is ________. Answer: [(0.9 $/euro) (40 euro per a European basket)]/[(50 $/U.S. basket)] = 0.72 U.S. basket/European basket. Difficulty: Moderate AACSB: Application of knowledge 36) Find the real exchange rate for the following case: Assume that the representative basket of European goods costs 150 euros and the representative U.S. basket costs $90, and the dollar/euro exchange rate is $0.80 per euro, then the price of the European basket in terms of U.S. basket is ________. Answer: [(0.80 $/euro) (150 euro per a European basket)]/[(90 $/U.S. basket)] = 1.33 U.S. baskets/European basket. Difficulty: Moderate AACSB: Application of knowledge
9 Copyright © 2022 Pearson Education, Inc.
37) Find the real exchange rate for the following case: Assume that the representative basket of European goods costs 150 euros and the representative U.S. basket costs $200, and the dollar/euro exchange rate is $1.20 per euro, then the price of the European basket in terms of U.S. basket is ________. Answer: [(1.20 $/euro) (150 euro per a European basket)]/[(200 $/U.S. basket)] = 0.9 U.S. baskets/European basket. Difficulty: Moderate AACSB: Application of knowledge 38) Find the real exchange rate for the following case: Assume that the representative basket of European goods costs 100 euros and the representative U.S. basket costs $125, and the dollar/euro exchange rate is $0.75 per euro, then the price of the European basket in terms of U.S. basket is ________. Answer: [(0.75 $/euro) (100 euro per a European basket)]/[(125 $/U.S. basket)] = 0.60 U.S. baskets/European basket. Difficulty: Moderate AACSB: Application of knowledge 39) Fill in the following table.
Answer:
Difficulty: Moderate AACSB: Application of knowledge
10 Copyright © 2022 Pearson Education, Inc.
40) Fill in the following table.
Answer:
Difficulty: Moderate AACSB: Application of knowledge 41) What is the real exchange rate? What is its relationship to the current account? Answer: Defined as: EP /P (the exchange rate multiplied by foreign prices, divided by domestic prices). While the nominal exchange rate measures how much of a foreign currency one can buy with a unit of domestic currency, the real exchange rate measures how many goods and services one could buy. A rise in the real exchange rate (a depreciation of domestic currency) means that domestic goods are cheaper compared to foreign goods, so exports increase and imports may increase or decrease. Difficulty: Moderate AACSB: Application of knowledge
11 Copyright © 2022 Pearson Education, Inc.
42) Explain how does an increase in the real exchange rate affect exports and imports? Answer: When the real exchange rate increases, domestic products are cheaper relative to foreign products. Due to this, exports increase as foreigners demand more of our exports. The change in imports is ambiguous because fewer units of imports are purchased (the volume effect), but each foreign unit is now more expensive (the value effect). Remember: exports and imports are measured in terms of domestic output, i.e. dollar value, not volume of units. Difficulty: Moderate AACSB: Application of knowledge 43) Please discuss the volume effect and the value effect in regards to how the current account will move given a change in the real exchange rate. Answer: The volume effect takes place when consumer spending shifts on export and import quantities, while the value effect results when the domestic output worth of a given amount of foreign imports is changed. It is assumed that the volume effect outweighs the value effect, so that, other things equal, a real depreciation of the currency improves the current account. Difficulty: Difficult AACSB: Application of knowledge 17.2
The Equation of Aggregate Demand
1) Aggregate demand for home output can be written as a function of A) the real exchange rate, disposable income, investment demand, and government spending. B) the nominal exchange rate, disposable income, investment demand, and government spending. C) the real exchange rate, before tax income, investment demand, and government spending. D) the nominal exchange rate, disposable income, consumer demand, and government spending. Answer: A Difficulty: Easy AACSB: Application of knowledge 2) Which of the following statements is TRUE? A) A rise in EP*/P makes domestic goods and services cheaper relative to foreign goods and services and shifts both domestic and foreign spending from foreign goods to domestic goods. B) A fall in EP*/P makes domestic goods and services cheaper relative to foreign goods and services and shifts both domestic and foreign spending from foreign goods to domestic goods. C) A rise in EP*/P makes domestic goods and services more expensive relative to foreign goods and services and shifts both domestic and foreign spending from domestic goods to foreign goods. D) A fall in EP*/P makes domestic goods and services more expensive relative to foreign goods and services and shifts both domestic and foreign spending from domestic goods to foreign goods. Answer: A Difficulty: Easy AACSB: Application of knowledge
12 Copyright © 2022 Pearson Education, Inc.
3) A real ________ of the home currency raises aggregate demand for home output, other things equal; a real ________ lowers aggregate demand for home output. A) depreciation; appreciation B) appreciation; depreciation C) appreciation; appreciation D) depreciation; depreciation Answer: A Difficulty: Easy AACSB: Application of knowledge 4) A real depreciation of the home currency ________ aggregate demand for home output, other things equal; a real appreciation ________ aggregate demand for home output. A) raises; lowers B) lowers; raises C) raises; raises D) lowers; lowers Answer: A Difficulty: Easy AACSB: Application of knowledge 5) With no trade and finance exchange barriers, which of the following statements is TRUE? A) A real depreciation of the home currency raises aggregate demand for home output, other things equal; a real appreciation lowers aggregate demand for home output. B) A real depreciation of the home currency lowers aggregate demand for home output, other things equal; a real appreciation raises aggregate demand for home output. C) A real appreciation of the home currency raises aggregate demand for home output, other things equal; a real depreciation lowers aggregate demand for home output. D) A real appreciation or depreciation of the home currency has no effect on aggregate demand for home output. Answer: A Difficulty: Easy AACSB: Application of knowledge 6) How does a rise in real income affect aggregate demand? A) Y ↑ implies Yd ↑ implies IM ↑ implies CA ↓ implies AD ↓, but Y ↑ implies Yd ↑ implies C ↑ implies AD ↑ by more. B) Y ↑ implies Yd ↑ implies IM ↓ implies CA ↓ implies AD ↓, but Y ↑ implies Yd ↑ implies C ↑ implies AD ↑ by more. C) Y ↑ implies Yd ↑ implies IM ↑ implies CA ↑ implies AD ↑, and Y ↑ implies Yd ↑ implies C ↑ implies AD ↑. D) Y ↑ implies Yd ↑ implies IM ↑ implies CA ↓ implies AD ↓, but Y ↑ implies Yd ↑ implies C ↑ implies AD ↑ by less. E) Y ↑ implies Yd ↑ implies IM ↓ implies CA ↓ implies AD ↓, but Y ↑ implies Yd ↑ implies C ↑ implies AD ↑ by less. Answer: A Difficulty: Moderate AACSB: Application of knowledge 13 Copyright © 2022 Pearson Education, Inc.
7) Which one of the following statements is the MOST accurate? A) A rise in domestic real income raises aggregate demand for home output. B) A rise in domestic real income decreases aggregate demand for home output because of the increase demand for import. C) A rise in domestic real income keeps aggregate demand for home output at the same level. D) It is difficult to tell whether a rise in domestic real income affects positively or negatively aggregate demand for home output. E) A rise in domestic real income decreases aggregate demand for home output because the CA is raised. Answer: A Difficulty: Easy AACSB: Application of knowledge 8) What is an accurate implication resulting from an increase in income? A) an increase in exchange rate B) a decrease in exchange rate C) a decrease in consumption D) a decrease in output E) an increase in consumption Answer: E Difficulty: Easy AACSB: Application of knowledge 9) Which one of the following statements is MOST accurate? A) In general, consumption demand rises by less than disposable income. B) In general, consumption demand rises by more than disposable income. C) In general, consumption demand rises by more than income. D) In general, consumption demand rises by the same amount as disposable income rises. E) In general, consumption demand rises are unrelated to disposable income rises. Answer: A Difficulty: Moderate AACSB: Application of knowledge 10) Explain the difference between the following two expressions: Y = C(Yd) + I + G + CA(EP /P, Yd) and Y = C + I +G + CA Answer: The first one represents a behavioral equation and thus may express equilibrium condition for the output market or the aggregate desired demand for output. The second equation is only an identity that is always true. Difficulty: Moderate AACSB: Application of knowledge
14 Copyright © 2022 Pearson Education, Inc.
11) Explain how a rise in real exchange rate affects aggregate demand. Answer: A rise in EP*/P makes domestic goods and services cheaper relative to foreign goods and services and shifts both domestic and foreign spending from foreign goods to domestic goods. As a result, CA rises and aggregate demand, D, therefore goes up. Difficulty: Easy AACSB: Application of knowledge 12) Explain how does a rise in real income affect aggregate demand? Answer: A rise in domestic real income, Y, leads to a rise in disposable income, Yd. This raises the spending on imports, IM, thus lowering the current account, CA, and reducing aggregate demand, AD. However, the rise in Yd also causes a rise in consumption, C, and raises aggregate demand, AD, by more than the corresponding decrease. Difficulty: Moderate AACSB: Application of knowledge 13) Refer to Figure 17-1, explain why the slope of the aggregate demand function is less than 1. Answer: Aggregate demand is a function of the real exchange rate (EP*/P), disposable income (Y — T), investment demand (I), and government spending (G). If all other factors remain unchanged, a rise in output (real income), Y, increases aggregate demand. As Y rises, consumption rises by a fraction of the increase in income. Part of this increase in consumption, moreover, goes into import spending. The effect of an increase in Y on the aggregate demand for home output is therefore smaller than the accompanying rise in consumption demand, which is smaller, in turn, than the increase in Y. Therefore, the slope of the aggregate demand function is less than 1 (as indicated by its position within the 45-degree angle). Difficulty: Moderate AACSB: Application of knowledge
15 Copyright © 2022 Pearson Education, Inc.
17.3
How Output Is Determined in the Short Run
1) Which of the following equations does NOT state a condition required for equilibrium output? A) Y = C(Yd) + I + G + CA(EP*/P,Yd) B) Y = C(Y - T) + I + G + CA(EP*/P,Y - T) C) Y = D(EP*/P,Y - T,I,G) D) R = R* + (EP/E) E) Y = D(EP*/P,Yd,I,G) Answer: D Difficulty: Difficult AACSB: Application of knowledge 2) The unique equilibrium output level in the short run is found at the intersection of the following curves. A) aggregate demand and aggregate supply B) aggregate demand and 45 degree line C) aggregate supply and 45 degree line D) aggregate demand and short-run aggregate supply E) aggregate supply and long-run demand Answer: B Difficulty: Easy AACSB: Application of knowledge 3) In the short run, we assume that the money prices of goods and services are A) temporarily fixed. B) permanently fixed. C) allowed to fluctuate. D) equal to long-run prices. E) fully employed. Answer: A Difficulty: Easy AACSB: Application of knowledge 4) What would be the best description of what we assume about money prices in the short run? A) Money prices of goods and services vary. B) Money prices of goods and services are not related to each other. C) Money prices of goods are fixed. D) Money prices of services are fixed. E) Money prices of goods and services are only temporarily fixed. Answer: E Difficulty: Easy AACSB: Application of knowledge
16 Copyright © 2022 Pearson Education, Inc.
5) Assume money prices of goods and services are temporarily fixed, explain how national output is determined in the short-run. Answer: The output market is in equilibrium when real domestic output, Y, equals the aggregate demand for domestic output. In Figure 17-2, it is the intersection (at point 1) of the aggregate demand schedule and a 45-degree line drawn from the origin (the equation D = Y) gives us the unique output level Y1 at which aggregate demand equals domestic output. Difficulty: Easy AACSB: Application of knowledge 17.4
Output Market Equilibrium in the Short Run: The DD Schedule
1) The economy as a whole is in equilibrium only A) when both the output market and the asset markets are in equilibrium. B) when the asset markets are in equilibrium. C) when the output market is in equilibrium. D) when neither the output market nor the asset markets are in equilibrium. Answer: A Difficulty: Easy AACSB: Application of knowledge 2) The DD schedule shows all combinations of which 2 variables so that the output market is in equilibrium? A) imports and exports B) exports and the exchange rate C) foreign prices and the exchange rate D) output and the exchange rate E) output and exports Answer: D Difficulty: Easy AACSB: Application of knowledge 3) How would you define a DD schedule? A) the combinations of output and the exchange rate that must hold when the home money market and the foreign exchange market are in equilibrium B) the combinations of output and the exchange rate that must hold when the output market is in short-run equilibrium C) factors of production in the long run D) the aggregate demand in relation to the foreign market value E) the currency depreciation in relation to the exchange rate Answer: B Difficulty: Easy AACSB: Application of knowledge
17 Copyright © 2022 Pearson Education, Inc.
4) In the short run, any rise in the real exchange rate, EP /P, will cause A) an upward shift in the aggregate demand function and a reduction in output. B) an upward shift in the aggregate demand function and an expansion of output. C) a downward shift in the aggregate demand function and an expansion of output. D) an downward shift in the aggregate demand function and a reduction in output. E) an upward shift in the aggregate demand function but leaves output intact. Answer: B Difficulty: Easy AACSB: Application of knowledge 5) In the short run, any fall in EP /P, regardless of its causes, will cause A) an upward shift in the aggregate demand function and an expansion of output. B) an upward shift in the aggregate demand function and a reduction in output. C) a downward shift in the aggregate demand function and an expansion of output. D) an downward shift in the aggregate demand function and a reduction in output. E) an upward shift in the aggregate demand function but leaves output intact. Answer: D Difficulty: Easy AACSB: Application of knowledge 6) Which of the following does NOT affect the position of the DD curve? A) monetary policy B) government spending C) taxes D) export demand E) price levels Answer: A Difficulty: Easy AACSB: Application of knowledge 7) Temporary tax cuts would cause A) the AA-curve to shift downward. B) the AA-curve to shift upward. C) the DD-curve to shift to the left. D) the DD-curve to shift to the right. E) a shift in the AA-curve, although the direction is ambiguous. Answer: D Difficulty: Easy AACSB: Application of knowledge
18 Copyright © 2022 Pearson Education, Inc.
8) A rise in government demand would cause A) the DD-curve to shift to the right. B) the AA-curve to shift downward. C) the AA-curve to shift upward. D) the DD-curve to shift to the left. E) a shift in the AA-curve, although the direction is ambiguous. Answer: A Difficulty: Easy AACSB: Application of knowledge 9) A fall in government demand would cause A) the DD-curve to shift to the left. B) the AA-curve to shift downward. C) the AA-curve to shift upward. D) the DD-curve to shift to the right. Answer: A Difficulty: Easy AACSB: Application of knowledge 10) An increase in taxes would cause A) the DD-curve to shift to the left. B) the AA-curve to shift downward. C) the AA-curve to shift upward. D) the DD-curve to shift to the right. Answer: A Difficulty: Easy AACSB: Application of knowledge 11) An increase in investment demand would cause A) the DD-curve to shift to the right. B) the AA-curve to shift downward. C) the AA-curve to shift upward. D) the DD-curve to shift to the left. Answer: A Difficulty: Easy AACSB: Application of knowledge 12) An increase in domestic price level makes domestic output more expensive relative to foreign output and lowers net export demand, therefore this will shift A) the DD-curve to the left. B) the AA-curve downward. C) the AA-curve upward. D) the DD-curve to the right. Answer: A Difficulty: Easy AACSB: Application of knowledge 19 Copyright © 2022 Pearson Education, Inc.
13) An increase in foreign price P* makes foreign goods and services relatively more expensive, therefore this will shift A) the DD-curve to the right. B) the AA-curve downward. C) the AA-curve upward. D) the DD-curve to the left. Answer: A Difficulty: Easy AACSB: Application of knowledge 14) An autonomous fall in domestic consumption, if it is not entirely due to a fall in import demand, will shift A) the DD-curve to the left. B) the AA-curve downward. C) the AA-curve upward. D) the DD-curve to the right. Answer: A Difficulty: Easy AACSB: Application of knowledge 15) A shift in world demand away from domestic products and toward foreign products causes A) the DD-curve to shift to the left. B) the AA-curve to shift downward. C) the AA-curve to shift upward. D) the DD-curve to shift to the right. Answer: A Difficulty: Easy AACSB: Application of knowledge 16) Which of the following is the MOST accurate? A) Any disturbance that lowers aggregate demand for domestic output shifts the DD schedule to the right. B) Any disturbance that lowers aggregate demand for foreign output shifts the DD schedule to the left. C) Any disturbance that raises aggregate demand for domestic output shifts the DD schedule to the right. D) Any disturbance that raises aggregate demand for domestic output shifts the DD schedule to the left. E) Any disturbance that lowers aggregate demand for domestic output shifts the DD schedule downward. Answer: C Difficulty: Easy AACSB: Application of knowledge
20 Copyright © 2022 Pearson Education, Inc.
17) Which of the following is the MOST accurate? A) Any disturbance that lowers aggregate demand for domestic output shifts the DD schedule to the right. B) Any disturbance that lowers aggregate demand for domestic output shifts the DD schedule to the left. C) Any disturbance that raises aggregate demand for domestic output shifts the DD schedule to the left. D) Any disturbance that lowers aggregate demand for domestic output shifts the DD schedule downward. Answer: B Difficulty: Easy AACSB: Application of knowledge 18) List the main factors affecting the position of the DD schedule. Answer: The level of government demand, taxes, and investment; the domestic and foreign price levels; variations in domestic consumption behavior; and the foreign demand for home output. Difficulty: Easy AACSB: Application of knowledge 19) Give 4 examples of situations that would cause the DD-curve to shift to the left. Answer: Correct answers include any situations that involve: (1) a decrease in government spending (e.g., decrease in military spending) (2) an increase in taxes (3) a fall in Investment demand (4) a price increase, which would lower net export demand (assuming E and P stay constant) (5) a fall in foreign prices (assuming E and P stay constant) (6) an autonomous fall in consumption demand (as long as it is not entirely a change in import demand) (7) a shift to demanding more foreign goods at the expense of domestic good demand Difficulty: Moderate AACSB: Application of knowledge 20) Explain what are the factors that shift the DD Schedule. Answer: A change in government demand, change in taxes, a change in investment, change in domestic prices, change in foreign prices, changes in the consumption function and a demand shift between foreign and domestic goods. Difficulty: Moderate AACSB: Application of knowledge
21 Copyright © 2022 Pearson Education, Inc.
17.5
Asset Market Equilibrium in the Short Run: The AA Schedule
1) The interest parity condition states that the foreign exchange market is in equilibrium only when A) the expected rates of return on domestic and foreign currency deposits are equal. B) the expected rates of return on domestic and foreign currency deposits are different. C) the expected rates of return on domestic currency deposits are higher than those on foreign currency deposits. D) the expected rates of return on foreign currency deposits are higher than those on domestic currency deposits. Answer: A Difficulty: Easy AACSB: Application of knowledge 2) What best describes the slope of the AA schedule? A) It is derived by the schedule of interest rate and output combinations that are consistent with equilibrium in the domestic money market and the foreign exchange market. B) It is derived by the schedule of exchange rate and output combinations that are consistent with equilibrium in the foreign money market and the domestic exchange market. C) It is derived by the schedule of exchange rate and output combinations that are consistent with equilibrium in the domestic money market and the foreign exchange market. D) It is derived by the schedule of exchange rate and output combinations that are consistent with equilibrium in the domestic bond market and the foreign asset market. E) It is derived by the schedule of exchange rate and output combinations that are greater than equilibrium in the foreign money market and the domestic exchange market. Answer: C Difficulty: Easy AACSB: Application of knowledge 3) What best describes the slope of the AA schedule? A) The AA schedule has a positive slope because an increase in output leads to a depreciation of the currency. B) The AA schedule has a negative slope because an increase in output leads to a decrease in the domestic interest rate. C) The AA schedule has a negative slope because an increase in output leads to an increase in the domestic interest rate and a domestic currency appreciation. D) The AA schedule has a positive slope because an increase in the money supply leads to an increase in the domestic interest rate. E) The AA schedule has a positive slope because a decrease in output leads to a depreciation of the currency. Answer: C Difficulty: Easy AACSB: Application of knowledge
22 Copyright © 2022 Pearson Education, Inc.
4) The interest parity condition requires that A) all countries have the same interest rate. B) there is a unique exchange rate for every output level. C) purchasing power parity hold. D) interest rates are fixed in the short run. E) the money supply is held constant. Answer: B Difficulty: Easy AACSB: Application of knowledge 5) Which one of the following statements is the MOST accurate? A) For asset markets to remain in equilibrium, a rise in domestic output must be accompanied by a depreciation of domestic currency, all else equal. B) For asset markets to remain in equilibrium, a fall in domestic output must be accompanied by a depreciation of foreign currency, all else equal. C) For asset markets to remain in equilibrium, a rise in domestic output must be accompanied by an appreciation of domestic currency, all else equal. D) For asset markets to remain in equilibrium, a fall in domestic output must be accompanied by an appreciation of domestic currency, all else equal. E) For asset markets to remain in equilibrium, a fall in domestic output must be accompanied by an appreciation of foreign currency, all else equal. Answer: C Difficulty: Easy AACSB: Application of knowledge 6) Assume the asset market is always in equilibrium. Therefore, a rise in Y would result in A) an appreciation of the home currency. B) higher inflation abroad. C) a decreased demand for domestic products. D) a contraction of the money supply. E) a depreciation of the home currency. Answer: A Difficulty: Moderate AACSB: Application of knowledge 7) Assume the asset market is always in equilibrium. Therefore, a fall in Y would result in A) an depreciation of the home currency. B) higher inflation abroad. C) a decreased demand for domestic products. D) a contraction of the money supply. E) an appreciation of the home currency. Answer: A Difficulty: Moderate AACSB: Application of knowledge
23 Copyright © 2022 Pearson Education, Inc.
8) Assume the asset market is always in equilibrium. An increase in domestic money supply Ms causes A) the AA-curve to shift upward. B) the DD-curve to the right. C) the AA-curve to shift downward. D) the DD-curve to the left. Answer: A Difficulty: Easy AACSB: Application of knowledge 9) Assume the asset market is always in equilibrium. An increase in P causes A) the AA-curve to shift downward. B) the DD-curve to the right. C) the AA-curve to shift upward. D) the DD-curve to the left. Answer: A Difficulty: Easy AACSB: Application of knowledge 10) Assume the asset market is always in equilibrium. A rise in the expected future exchange rate causes A) the AA-curve to shift upward. B) the DD-curve to the right. C) the AA-curve to shift downward. D) the DD-curve to the left. Answer: A Difficulty: Easy AACSB: Application of knowledge 11) Assume the asset market is always in equilibrium. A fall in R* causes A) the AA-curve to shift downward. B) the DD-curve to the right. C) the AA-curve to shift upward. D) the DD-curve to the left. Answer: A Difficulty: Easy AACSB: Application of knowledge 12) Assume the asset market is always in equilibrium. An increase in money demand causes A) the AA-curve to shift upward. B) the DD-curve to the right. C) the AA-curve to shift downward. D) the DD-curve to the left. Answer: A Difficulty: Easy AACSB: Application of knowledge 24 Copyright © 2022 Pearson Education, Inc.
13) Explain how the AA schedule is derived. Answer: For a fixed real money supply, an increase in output leads to an increase in the domestic interest rate. In the foreign exchange market, an increase in the domestic interest rate leads to a lower nominal exchange rate, thus appreciating the currency. Therefore, the relationship between nominal exchange rate and output is negative; this leads to a negative slope of the AA schedule, which has the nominal exchange rate and output on its axes. Difficulty: Moderate AACSB: Application of knowledge 14) List the main factors affecting the position of the AA schedule. Answer: Changes in the domestic money supply; changes in the domestic price level; changes in the expected future exchange rate; changes in the foreign interest rate; and shifts in the aggregate real money demand schedule. Difficulty: Easy AACSB: Application of knowledge 15) What is the AA-curve? Why does it have a negative slope? What factors cause it to shift? Answer: The AA-curve is the specific levels of E and Y under which the money and foreign exchange markets are in equilibrium. The AA-curve has a negative slope because an increase in Y will cause E to fall (a domestic currency appreciation). The factors that affect it are: the money supply, price level, expected exchange rate, foreign interest rates, and the level of real money demand. Difficulty: Moderate AACSB: Application of knowledge 16) Explain what are the factors that shift the AA Schedule? Answer: Changes in the domestic money supply; changes in the domestic price level; changes in the expected future exchange rates; changes in the foreign interest rate and shifts in the aggregate real money demand. Difficulty: Moderate AACSB: Application of knowledge
25 Copyright © 2022 Pearson Education, Inc.
17.6 Short-Run Equilibrium for an Open Economy: Putting the DD and AA Schedules Together 1) Which of the following have to be in equilibrium for the economy to be in equilibrium? A) the money market only B) the goods market only C) the output and asset markets D) the savings and investment markets E) the goods and output markets Answer: C Difficulty: Easy AACSB: Application of knowledge 2) Imagine that the economy is at a point that is above both AA and DD, where both the output and asset markets are out of equilibrium. Which first action is TRUE? A) The economy will stay at this level in the short run. B) The exchange rate will first drop to a point on the AA schedule. C) The exchange rate will first move to a point on the DD schedule. D) The AA-DD equilibrium will shift to the position of the economy. E) The exchange rate will first move left to a position on the AA schedule. Answer: B Difficulty: Moderate AACSB: Application of knowledge 3) Why does an exchange rate-output combination lying above both DD and AA jump first to AA in equilibrium? A) Asset prices can adjust immediately. B) Production plans can adjust immediately. C) to preserve full employment D) Prices are nominal and demand is real. E) Aggregate demand adjusts faster than output. Answer: A Difficulty: Moderate AACSB: Application of knowledge 4) Imagine that the economy is at a point that is below both AA and DD, where both the output and asset markets are out of equilibrium. Which first action is TRUE? A) The economy will stay at this level in the short run. B) The exchange rate will first rise to a point on the AA schedule. C) The exchange rate will first rise to a point on the DD schedule. D) The AA-DD equilibrium will shift to the position of the economy. E) The output level will first increase to a position on the DD schedule. Answer: B Difficulty: Difficult AACSB: Application of knowledge
26 Copyright © 2022 Pearson Education, Inc.
5) Assume the output market adjusts more rapidly than the asset market. A point of disequilibrium that is below both AA and DD will therefore initially result in A) an increase in output. B) a decrease in output. C) a contraction of the money supply. D) a depreciation of the home currency. E) an appreciation of the home currency. Answer: B Difficulty: Difficult AACSB: Application of knowledge 6) Explain how an increase in government spending would affect the DD-AA schedule in the short run. Answer: An increase in government spending will increase aggregate demand, which will shift the DD to the right. If AA remains unchanged, the new equilibrium will be at a higher Y and lower E. Since E is the nominal exchange rate, a lower E is an appreciation of the currency. Difficulty: Moderate AACSB: Application of knowledge 7) Imagine that the economy is at a point on the DD-AA schedule that is above both AA and DD and where both the output and asset markets are out of equilibrium. Explain what will happen next? Answer: Since the asset market adjusts very quickly, the exchange rate drops immediately to a point on the AA schedule. There will be excess demand for the domestic currency because the high expected future appreciation rate of the domestic currency implies that the expected domestic currency return on foreign deposits is below that on domestic deposits. This excess demand leads to an immediate fall in the exchange rate. Difficulty: Moderate AACSB: Application of knowledge
27 Copyright © 2022 Pearson Education, Inc.
8) Use a figure to study the following question: Imagine that the economy is at a point on the DD-AA schedule that is above both AA and DD, where both the output and asset markets are out of equilibrium. Explain what will happen next. Answer: Since the asset market adjusts very quickly, the exchange rate drops immediately to a point on the AA schedule. There will be excess demand for the domestic currency because the high expected future appreciation rate of the domestic currency implies that the expected domestic currency return on foreign deposits is below that on domestic deposits. This excess demand leads to an immediate fall in the exchange rate. The figure:
Difficulty: Moderate AACSB: Application of knowledge 17.7
Temporary Changes in Monetary and Fiscal Policy
1) In the short run, a temporary increase in money supply A) shifts the DD curve to the right, increases output and appreciates the currency. B) shifts the AA curve downward, increases output and depreciates the currency. C) shifts the AA curve downward, decreases output and depreciates the currency. D) shifts the AA curve downward, increases output and appreciates the currency. E) shifts the AA curve upward, increases output and depreciates the currency. Answer: E Difficulty: Moderate AACSB: Application of knowledge 2) In the short run, a temporary decrease in money supply A) shifts the AA curve downward, decreases output and appreciates the currency. B) shifts the AA curve downward, increases output and depreciates the currency. C) shifts the DD curve to the right, increases output and appreciates the currency. D) shifts the AA curve downward, increases output and appreciates the currency. E) shifts the AA curve upward, increases output and depreciates the currency. Answer: A Difficulty: Moderate AACSB: Application of knowledge 28 Copyright © 2022 Pearson Education, Inc.
3) In the short run, with prices fixed, how would an increase in government spending affect the DD-AA equilibrium? A) It will increase output and appreciate the currency. B) It will increase output and depreciate the currency. C) It will decrease output and appreciate the currency. D) It will decrease output and depreciate the currency. E) It will increase output and have no effect on the currency. Answer: A Difficulty: Moderate AACSB: Application of knowledge 4) In the short run, an increase in government purchases will cause A) a shift of the DD curve to the left and an increase in output. B) a shift of the DD curve to the right and a decrease in output. C) a shift of the DD curve to the left and a decrease in output. D) a shift of the DD curve to the right and an increase in output. E) a shift of the DD curve the left and an appreciation of the currency. Answer: D Difficulty: Moderate AACSB: Application of knowledge 5) In the short run, a decrease in government purchases will cause A) a shift of the DD curve to the left and a decrease in output. B) a shift of the DD curve to the right and a decrease in output. C) a shift of the DD curve to the left and an increase in output. D) a shift of the DD curve to the right and an increase in output. E) a shift of the DD curve the left and an appreciation of the currency. Answer: A Difficulty: Moderate AACSB: Application of knowledge 6) In the short run, a tax increase A) shifts the DD curve to the right, increases output and appreciates the currency. B) shifts the AA curve downward, increases output and depreciates the currency. C) shifts the AA curve downward, decreases output and depreciates the currency. D) shifts the AA curve downward, increases output and appreciates the currency. E) shifts the DD curve to the left, decreases output and depreciates the currency. Answer: E Difficulty: Moderate AACSB: Application of knowledge
29 Copyright © 2022 Pearson Education, Inc.
7) In the short run, a tax decrease A) shifts the DD curve to the right, increases output and appreciates the currency. B) shifts the AA curve downward, increases output and depreciates the currency. C) shifts the AA curve downward, decreases output and depreciates the currency. D) shifts the AA curve downward, increases output and appreciates the currency. E) shifts the DD curve to the left, decreases output and depreciates the currency. Answer: A Difficulty: Moderate AACSB: Application of knowledge 8) Using the DD-AA framework, which one of the following statements is the MOST accurate? A) Only monetary policy can bring the economy to full employment. B) Only fiscal policy can bring the economy to full employment. C) Only both monetary and fiscal policies can bring the economy to full employment. D) Both policies are capable of bringing the economy to full employment and low inflation. E) Monetary policy by itself or fiscal policy by itself can bring the economy to full employment. Answer: E Difficulty: Moderate AACSB: Application of knowledge 9) Monetary expansion causes the current account balance to increase in the short run. Discuss. Is the same the case for fiscal expansion? Answer: An increase in the money supply leads to an increase in Y and E (output increases and the currency depreciates, respectively). Because of the currency depreciation, domestic goods are now cheaper compared to foreign goods. Exports increase and imports decrease, therefore the CAB increases. An expansion of fiscal policy actually reduces the CAB: the DD curve is shifted right. Therefore Y rises, but E falls (output rises but the currency appreciates.) Domestic goods are more expensive, and the CAB falls. Difficulty: Difficult AACSB: Application of knowledge 10) What are two ways the government can maintain full employment in an open economy? Also give an example for each. Answer: There are two types of government policy, monetary and fiscal policy. Examples of monetary policy are changes in the money supply. Examples of fiscal policy are changes in government spending or taxes. Difficulty: Moderate AACSB: Application of knowledge
30 Copyright © 2022 Pearson Education, Inc.
11) Using a figure show that under full employment, a temporary fiscal expansion would increase output (over-employment) but cannot increase output in the long run. Answer: A temporarily fiscal expansion will move the economy from DD1 to DD2, and output increases. A permanent fiscal expansion will also shift the AA curve downward. The nominal exchange rate appreciates, i.e. E decreases.
Difficulty: Moderate AACSB: Application of knowledge 17.8
Inflation Bias and Other Problems of Policy Formulation
1) The outcome of the government's expansionary monetary policy that leads to high inflation without average gain in output is called A) inflation bias. B) over inflation. C) hyper inflation. D) discrete inflation. Answer: A Difficulty: Easy AACSB: Application of knowledge 2) A government concerned about the exchange rate effect of its policy response needs to know ________ before it can choose between monetary and fiscal policy. A) the source of the disturbance B) the output market equilibrium C) the asset market equilibrium D) inflation bias Answer: A Difficulty: Easy AACSB: Application of knowledge 31 Copyright © 2022 Pearson Education, Inc.
3) Real-world policy choices are frequently determined by ________ rather than by detailed consideration of whether shocks to the economy originate in the output or asset markets. A) bureaucratic necessities B) political bias C) public consensus D) voting process Answer: A Difficulty: Easy AACSB: Application of knowledge 4) To avoid procedural delays, governments are likely to respond to disturbances by changing ________ even when a shift in ________ would be more appropriate. A) monetary policy; fiscal policy B) fiscal policy; monetary policy C) trade policy; investment policy D) tax policy; trade policy Answer: A Difficulty: Easy AACSB: Application of knowledge 5) What is inflation bias? What measures have governments taken to avoid it? Answer: Inflation bias is caused when a government is expected to use policy tools to create an economic expansion (such as before an election). Because it is expected, wages and therefore prices are increased. If the government did not pursue the expansionary policy then, there would be a recession! Inflation is increased without the advantage of an increase in output. Making the central bank independent of the political government is one answer to avoid inflation bias. Difficulty: Moderate AACSB: Application of knowledge 6) Explain and give some examples of governmental policy problems. Answer: Steady nominal prices give government the power to raise output when it is low. It can also cause them to create a tool that can be used for an economic boom. An example is just before an election. The temptation can be a problem when workers and companies expect it in advance. This will cause a rise in wage demand and prices in the expected expansionary policies. An inflation bias causing high inflation but no average gains in output is also a problem. Others are the difficulty in showing the sources or time of economic changes, and time lags in implementing policies. Impact on the government budget by fiscal policy also causes problems by the way of a tax cut; increase in spending may lead to a government budget deficit that must sooner or later be closed by a fiscal reversal. Policy problems that seem to act quickly have actually a lag time with varying lengths. Difficulty: Moderate AACSB: Application of knowledge
32 Copyright © 2022 Pearson Education, Inc.
7) Discuss the impact of fiscal policy on government budget. Answer: Tax cut or spending increase may lead to a larger government budget deficit, which must sooner or later be closed by a fiscal reversal, as happened following the multibillion-dollar fiscal stimulus package sponsored by the Obama administration in the United States in 2009. Unfortunately, there is no guarantee the government will have the political will to synchronize these actions with the state of the business cycle. Difficulty: Easy AACSB: Application of knowledge 8) Compared with monetary policy, what are the disadvantages of using fiscal policy? Answer: Shifts in fiscal policy often can be made only after lengthy legislative deliberation, while monetary policy is usually exercised expeditiously by the central bank. Another problem with fiscal policy is its impact on the government budget. A tax cut or spending increase may lead to a larger government budget deficit, which must sooner or later be closed by a fiscal reversal. Difficulty: Moderate AACSB: Application of knowledge 9) A naive implication of the DD-AA framework is that either fiscal or monetary policy can lead to full employment. Discuss why this view is naive. Answer: (1) Inflation may arise without any gain in output if the government misuses its power to print money. (2) In practice, it is sometimes hard to be sure whether a disturbance to the economy originates in the output or assets markets. (3) Shifts in fiscal policy often can be made only after lengthy legislative deliberations. Governments are likely to respond to disturbances by changing the monetary policy even when a shift in fiscal policy would be more appropriate. (4) Fiscal policy impacts the government budget and may lead to government budget deficit that must sooner or later be closed by a fiscal reversal. The state of the electoral cycle may be more important. (5) Policies operate in reality with lags of varying length. Difficulty: Difficult AACSB: Application of knowledge
33 Copyright © 2022 Pearson Education, Inc.
17.9
Permanent Shifts in Monetary and Fiscal Policy
1) In long-run equilibrium after a permanent money-supply increase there follows A) an increase in exchange rate, E. B) a decrease in exchange rate, E. C) an increase in output, Y. D) a decrease in output, Y. E) an unchanged exchange rate, E. Answer: A Difficulty: Moderate AACSB: Application of knowledge 2) A permanent increase in the domestic money supply A) must ultimately lead to a proportional decrease in E, and, therefore, the expected future exchange rate must rise proportionally. B) must ultimately lead to a proportional decrease in E, and, therefore, the expected future exchange rate must decrease proportionally. C) must ultimately lead to a proportional rise in E, and, therefore, the expected future exchange rate must rise proportionally. D) must ultimately lead to a proportional rise in E, and, therefore, the expected future exchange rate must rise more than proportionally. E) must ultimately lead to a proportional rise in E, and, therefore, the expected future exchange rate must rise less than proportionally. Answer: C Difficulty: Difficult AACSB: Application of knowledge 3) In the short run, a permanent increase in the domestic money supply causes A) a greater rightward shift in the DD curve than that caused by an equal, but transitory, increase. B) a greater downward shift in the AA curve than that caused by an equal, but transitory, increase. C) a smaller upward shift in the AA curve than that caused by an equal, but transitory, increase. D) a smaller downward shift in the AA curve than that caused by an equal, but transitory, increase. E) a greater upward shift in the AA curve than that caused by an equal, but transitory, increase. Answer: E Difficulty: Difficult AACSB: Application of knowledge
34 Copyright © 2022 Pearson Education, Inc.
4) In the short run, a permanent increase in the domestic money supply A) has stronger effects on the exchange rate and output than an equal temporary increase. B) has stronger effects only on the exchange rate but not on output than an equal temporary increase. C) has weaker effects on the exchange rate and output than an equal temporary increase. D) has stronger effects on output, but lower effect on the exchange rate than an equal temporary increase. E) has weaker effects only on the exchange rate than an equal temporary increase. Answer: A Difficulty: Difficult AACSB: Application of knowledge 5) Which one of the following statements is the MOST accurate? A) Over time, the inflationary pressure that follows a temporary money supply expansion pushes the price level to its long-run value and returns the economy to full employment. B) Over time, the inflationary pressure that follows a permanent money supply expansion pushes the price level to its long-run value and returns the economy to full employment. C) Over time, the inflationary pressure that follows a temporary money supply expansion pushes the price level to its long-run value, but leaves the economy in a state of artificially low employment. D) Over time, the inflationary pressure that follows a permanent money supply expansion pushes the price level to its long-run value, but leaves the economy in a state of artificially low employment. E) Over time, the inflationary pressure that follows a permanent money supply expansion pushes the price level beyond its long-run value and lower the level of employment. Answer: B Difficulty: Moderate AACSB: Application of knowledge 6) Which one of the following statements is the MOST accurate? A) A permanent increase in the money supply cannot have any short-run effects. B) A permanent increase in taxes cannot have any short-run effects. C) A permanent decrease in the money supply cannot have short-run effects. D) A permanent decrease in taxes cannot have short-run effects. E) A permanent increase in money demand can be offset with a permanent increase in the money supply of equal magnitude. Answer: E Difficulty: Moderate AACSB: Application of knowledge
35 Copyright © 2022 Pearson Education, Inc.
7) If the economy starts in long-run equilibrium, a permanent fiscal expansion will cause A) an increase in exchange rate, E. B) a decrease in exchange rate, E. C) an increase in output, Y. D) a decrease in output, Y. E) shifting of the AA curve up and to the right. Answer: B Difficulty: Moderate AACSB: Application of knowledge 8) A permanent fiscal expansion A) shifts the DD to the right and the AA schedule upward, increasing output. B) shifts the DD to the left and the AA schedule upward, decreasing output. C) shifts the DD to the right and the AA schedule downward, leaving output the same. D) shifts the DD to the left and the AA schedule downward, decreasing output. E) shifts the DD to the left and the AA schedule downward, leaving output the same. Answer: C Difficulty: Moderate AACSB: Application of knowledge 9) Show the effects of a permanent increase in the money supply. Answer: (1) AA-shifts upward-increase in Y and E both higher than if money supply change was temporary, rising price level makes AD decrease, DD shifts left (2) rising prices also reduce real money supply, so AA shifts downward (although not all the way back to original position) (3) AA and DD reach short-run equilibrium at an E that is higher than initially, but lower than the short-run effects of the shift. (4) Output returns to initial level because higher prices reversed the effect of the initial depreciation on Aggregate Demand. Difficulty: Moderate AACSB: Application of knowledge
36 Copyright © 2022 Pearson Education, Inc.
10) Explain the following figure:
Answer: The figure depicts the effect of a permanent increase in the money supply starting from full employment equilibrium. After the initial increase in the money supply and the move of the AA curve upward from AA1 to AA2, a steadily increasing price level shifts the AA downward and the DD schedule to the left until a new long-run equilibrium is reached. Note that point 3 is above point 1, because Ee is permanently higher after a permanent increase in the money supply. The expected exchange rate, Ee, has risen by the same percentage as Ms. Notice that along the adjustment path between the initial short-run equilibrium (point 2) and the longrun equilibrium (point 3) the domestic currency actually appreciates (from E2 to E3) following its initial sharp depreciation (from E1 to E2). Difficulty: Difficult AACSB: Application of knowledge
37 Copyright © 2022 Pearson Education, Inc.
11) Using the DD-AA framework, show the phenomenon of overshooting. Use a figure to explain when it is taking place. Answer: The figure below shows the phenomenon of overshooting. A permanent increase in the money supply starting from full employment equilibrium will shift the AA curve upward from AA1 to AA2. Now, a steadily increasing price level shifts the AA downward and the DD schedule to the left until a new long-run equilibrium is reached. Note that point 3 is above point 1, because Ee is permanently higher after a permanent increase in the money supply. The expected exchange rate, Ee, has risen by the same percentage as Ms. Notice that along the adjustment path between the initial short-run equilibrium (point 2) and the long-run equilibrium (point 3) the domestic currency actually appreciates (from E2 to E3) following its initial sharp depreciation (from E1 to E2). This exchange rate behavior is an example of overshooting, in which the exchange rate's initial response to some change is greater than its long-run response.
Difficulty: Difficult AACSB: Application of knowledge 12) Demonstrate how a permanent fiscal expansion will not increase output in the long run. Answer: (1) E on Y-axis, Y on X-axis (2) DD shifts right (3) temporary equilibrium where E lower and Y increased (4) permanent increase in demand caused by increase in G causes currency to appreciate: AA shifts downward (5) therefore Y returns to original levels, E decreases even more RESULT of permanent fiscal expansion: currency appreciation, output does not change. This effect is called "crowding out." Difficulty: Difficult AACSB: Application of knowledge
38 Copyright © 2022 Pearson Education, Inc.
13) Using the DD model, explain what happens to output when Government demands increase. Use a figure to explain when it is taking place. Answer: The figure below shows the G1 to G2 raises output at every level of the exchange rate. The change shifts the DD to the right, which, in turn, increases output to Y2.
Difficulty: Difficult AACSB: Application of knowledge 17.10
Macroeconomic Policies and the Current Account
1) Policy makers are often concerned about the level of the current account. An excessive imbalance in the current account—either a surplus or a deficit— A) may have undesirable long-run effects on national welfare. B) will have no effects on national welfare C) may turn a country into a developed or less developed one D) will have no effects on aggregate demand Answer: A Difficulty: Easy AACSB: Application of knowledge 2) What can large external imbalances generate? A) Political pressures for governments to impose restrictions on trade. B) Political pressures for governments to remove restrictions on trade. C) Political pressures for governments to make the country more open to trade. D) Political pressures for governments to switch to a self-sufficient economy without trade. Answer: A Difficulty: Easy AACSB: Application of knowledge
39 Copyright © 2022 Pearson Education, Inc.
3) Which of the following is TRUE of the current account balance? A) Monetary expansion has no effect on the current account balance. B) Monetary expansion decreases the current account balance. C) Fiscal expansion increases the current account balance. D) Fiscal expansion has no effect on the current account balance. E) Monetary expansion increases the current account balance. Answer: E Difficulty: Moderate AACSB: Application of knowledge 4) Which statement best describes the current account balance in the short run? A) Monetary expansion lowers the current account balance. B) Monetary expansion keeps the current account balance the same. C) Fiscal expansion increases the current account balance. D) Fiscal expansion keeps the current account balance the same. E) Monetary expansion increases the current account balance. Answer: E Difficulty: Moderate AACSB: Application of knowledge 5) In the short run A) monetary expansion causes the CA to increase and fiscal expansion causes the CA to decrease. B) monetary expansion causes the CA to decrease and fiscal expansion causes the CA to decrease. C) monetary expansion causes the CA to increase and fiscal expansion causes the CA to increase. D) monetary expansion causes the CA to decrease and fiscal expansion causes the CA to increase. E) monetary expansion causes the CA to increase and the effects of fiscal expansion are ambiguous. Answer: A Difficulty: Moderate AACSB: Application of knowledge 6) Which statement describes most accurately how fiscal policies affect current account? A) Expansionary fiscal policy reduces the current account balance. B) Expansionary fiscal policy expands the current account balance. C) Expansionary fiscal policy has no effect on the current account balance. D) Expansionary fiscal policy either reduces or expands the current account balance. Answer: A Difficulty: Easy AACSB: Application of knowledge
40 Copyright © 2022 Pearson Education, Inc.
17.11
Gradual Trade Flow Adjustment and Current Account Dynamics
1) The J-curve illustrates which of the following? A) the effects of depreciation on the home country's economy B) the immediate increase in current account caused by a currency depreciation C) the gradual adjustment of home prices to a currency depreciation D) the short-term effects of depreciation on the current account E) the Keynesian view of international trade dynamics Answer: D Difficulty: Easy AACSB: Application of knowledge 2) The J-curve illustrates which of the following? A) the time lag with which a real currency depreciation improves the current account B) the effects of depreciation on the home country's economy C) the immediate increase in current account caused by a currency depreciation D) the gradual adjustment of home prices to a currency depreciation E) the Keynesian view of international trade dynamics Answer: A Difficulty: Easy AACSB: Application of knowledge 3) Empirical evidence indicates for most industrial countries a J-curve lasting more than ________ but less than ________. Then the current account continues to improve afterward. A) six months; a year B) a year; two years C) two years; five years D) three months; six months Answer: A Difficulty: Easy AACSB: Application of knowledge 4) The percent by which import prices rise when the home currency depreciates by 1% is the degree of A) pass-forward from exchange rates to import prices. B) pass-through from exchange rates to import prices. C) pass-on from exchange rates to import prices. D) roll-forward from exchange rates to import prices. E) pass-beyond from exchange rates to import prices. Answer: B Difficulty: Easy AACSB: Application of knowledge
41 Copyright © 2022 Pearson Education, Inc.
5) In practice, many U.S. import prices tend to rise by only around A) 1/4 of a typical dollar depreciation over the following year. B) 1/3 of a typical dollar depreciation over the following year. C) 1/2 of a typical dollar depreciation over the following year. D) 2/3 of a typical dollar depreciation over the following year. E) 2/5 of a typical dollar depreciation over the following year. Answer: C Difficulty: Easy AACSB: Application of knowledge 6) A phenomenon in which different countries produce a portion of the value added in a final product is called A) global value chains. B) international production sharing. C) multinational outsourcing chains. D) intermediation production offshoring. Answer: A Difficulty: Easy AACSB: Application of knowledge 7) Imported value that accounts for a significant portion of the gross value of exports is called ________ linkages. A) backward B) forward C) value added D) intermediate Answer: A Difficulty: Easy AACSB: Application of knowledge 8) A country's exports may go on to be incorporated as inputs into several rounds of exports from other countries, this key aspect of global value chains is called A) forward linkages. B) backward linkages. C) value linkages. D) intermediate linkages. Answer: A Difficulty: Easy AACSB: Application of knowledge
42 Copyright © 2022 Pearson Education, Inc.
9) Which of the following statements is TRUE? A) There is strong evidence that more complex global value chains go hand in hand with larger volumes of both imports and exports. B) There is no evidence that more complex global value chains go hand in hand with larger volumes of both imports and exports. C) There is strong evidence that more complex global value chains go hand in hand with smaller volumes of both imports and exports. D) There is strong evidence that less complex global value chains go hand in hand with larger volumes of both imports and exports. Answer: A Difficulty: Easy AACSB: Application of knowledge 10) A permanent fiscal expansion would cause A) an appreciation of currency and a current account deficit. B) an appreciation of currency and a current account surplus. C) a depreciation of currency and a current account surplus. D) a depreciation of currency and a current account deficit. Answer: A Difficulty: Easy AACSB: Application of knowledge 11) The net foreign wealth of an economy with a ________ is ________ over time. A) deficit; falling B) surplus; falling C) deficit; rising D) surplus; unchanged Answer: A Difficulty: Easy AACSB: Application of knowledge 12) Which currency is not only the key vehicle currency in the global foreign exchange market but also the dominant invoice currency? A) the U.S. dollar B) the Euro C) the Japanese Yen D) the British Pound Answer: A Difficulty: Easy AACSB: Application of knowledge
43 Copyright © 2022 Pearson Education, Inc.
13) Because a country with a current account deficit is transferring wealth to foreigners, domestic consumption is ________ over time and foreign consumption is ________. A) falling; rising B) falling; falling C) rising; falling D) rising; rising Answer: A Difficulty: Easy AACSB: Application of knowledge 14) Describe a J Curve. Answer: The time lag with which real currency depreciation improves the current account. The current account is measured in terms of domestic output, and can drop quickly right after real currency depreciation because most import and export orders are placed several months in advance. In the first few months after the depreciation, export and import volumes therefore may reflect buying decisions that were made on the basis of the old real exchange rate. Difficulty: Moderate AACSB: Application of knowledge 15) Define global value chains and describe its key aspects. Answer: A web of complex global value chains is something in which different countries produce portions of the value added in a final product. Under these arrangements, one country's value added may cross borders many times at successive production stages before reaching its ultimate buyer. Two key aspects of global value chains include backward linkages and forward linkages. Difficulty: Easy AACSB: Application of knowledge
44 Copyright © 2022 Pearson Education, Inc.
16) What are forward and backward linkages? Discuss their implications for the effects of exchange rate changes on export and import prices? Answer: When imported value accounts for a significant portion of the gross value of exports, it is called backward linkages. When a country's exports may go on to be incorporated as inputs into several rounds of exports from other countries, it is called forward linkages. Assume that the pass-through of an exchange rate change to import prices is complete, that is, when a country's currency depreciates, its import prices rise immediately in proportion to the depreciation. If the country has strong backward linkages, however, that assumption means that the price of the intermediate imports used to produce its exports will rise. As a result, the depreciation will have only a partial effect in lowering the export prices it charges to its trading partners. Forward linkages work in the same direction on the import side, though the effect is more roundabout. Because a currency depreciation lowers a country's export prices, it will also lower the prices of imports that incorporate those exports as inputs. Even if the domestic-currency prices of imports rise fully in response to a home currency depreciation, given the foreigncurrency prices of imports, the currency depreciation can lower the foreign-currency import prices themselves in the presence of global value chains. This effect will reduce the exchange rate's impact in shifting demand away from imports and raising net exports. Difficulty: Easy AACSB: Application of knowledge 17) What is the implication of the immediate pass-through to importers of U.S. goods, in terms of their own currencies, close to 1? Answer: The implication is that dollar depreciation may boost U.S. exports immediately but have little impact on U.S. imports. Of course, foreign firms selling into the United States and invoicing in dollars will earn lower profits in terms of their own currencies (can you see why?) and as a result will desire to raise their goods' U.S. prices over time to recoup their losses. Difficulty: Easy AACSB: Application of knowledge
45 Copyright © 2022 Pearson Education, Inc.
17.12
The Liquidity Trap
1) If a country's nominal interest rate is zero, then A) the country's economy is in a liquidity trap. B) exchange rates with other countries are likely to decline. C) exchange rates with other countries are likely to increase. D) monetary policy is likely to be very effective in stimulating the economy. E) the country's economy has achieved monetary equilibrium. Answer: A Difficulty: Easy AACSB: Application of knowledge 2) When an economy is in a liquidity trap A) monetary policy cannot be used to influence the exchange rate. B) monetary policy can be used to drive interest rates down, but not to drive them up. C) there is an excess demand for bonds. D) people and institutions avoid holding cash balances. E) it can escape only by introducing a hard, or illiquid, currency. Answer: A Difficulty: Moderate AACSB: Application of knowledge 3) Which of the following is an example of an "unconventional monetary policy" by a central bank? A) the purchase of specific categories of assets with new money B) the sale of long-term government bonds for foreign exchange C) the purchase of long-term government bonds using foreign exchange D) raising reserve requirements by commercial banks E) selling gold reserves Answer: A Difficulty: Moderate AACSB: Application of knowledge 4) If an economy is in a liquidity trap, then the nominal interest rate is ________ and the only effective policy that can be used to stimulate the economy is ________. A) zero or negative; expansionary fiscal policy B) zero or negative; expansionary monetary policy C) high and rising; contractionary monetary policy D) high and rising; expansionary monetary policy E) high and rising; expansionary fiscal policy Answer: A Difficulty: Moderate AACSB: Application of knowledge
46 Copyright © 2022 Pearson Education, Inc.
5) A government spending multiplier A) measures the size of the increase in output caused by an increase in government spending. B) measures the size of the increase in output caused by a decrease in taxes. C) measures the size of the increase in output caused by a decrease in interest rate. D) measures the size of the increase in output caused by an increase in investment. Answer: A Difficulty: Easy AACSB: Application of knowledge 6) Robert E. Hall of Stanford University suggests that most studies find a multiplier A) between 0.5 and 1.0. B) between 1.0 and 2.0. C) between 2.0 and 3.0. D) between 3.0 and 4.0. Answer: A Difficulty: Easy AACSB: Application of knowledge 7) Alan Auerbach and Yuriy Gorodnichenko of the University of California, Berkeley, analyze data from the mostly wealthy OECD member countries and find that for economies in recession, the multiplier is about A) 2 B) 3.7 C) 1 D) 0 Answer: A Difficulty: Easy AACSB: Application of knowledge 8) What is a liquidity trap? Why economists recommend banks to avoid the zero lower bound (ZLB)? Answer: A liquidity trap is when an economy's nominal interest rate falls to zero and the central bank will face great difficulty in reducing it further by increasing the money supply. While a zero interest rate may please borrowers who can borrow for free, it worries makers of macroeconomic policy, who are trapped in a situation where they may no longer be able to steer the economy through conventional monetary expansion. Economists therefore recommend that if possible, central banks avoid the zero lower bound (ZLB) on the nominal interest rate. Difficulty: Easy AACSB: Application of knowledge 9) What does it mean when the government spending multiplier is equal to 1? Answer: When the government raises its consumption by $1, the resulting increase in output will be at most $1–smaller than the big multipliers of the simplest closed-economy models, but still an effect likely to have a substantial positive impact on employment. Difficulty: Easy AACSB: Application of knowledge 47 Copyright © 2022 Pearson Education, Inc.
17.13
Appendix 1 to Chapter 17: Intertemporal Trade and Consumption Demand
1) If present consumption demand is
and future consumption demand
, present income is
and future income is , and r is the real rate of interest. The intertemporal budget constraint can be written as A) + /(1 + r) = + /(1 + r) B)
-
/(1 + r) =
-
/(1 + r)
C)
+
/(1 - r) =
+
/(1 + r)
D) + = + Answer: A Difficulty: Easy AACSB: Application of knowledge 2) A ________ in lifetime income that is concentrated in present will indeed lead to a ________ in current consumption less than the ________ in current income. A) rise; rise; rise B) rise; fall; rise C) fall; rise; fall D) fall; rise; rise Answer: A Difficulty: Easy AACSB: Application of knowledge 3) An intertemporal budget constraint A) requires the present value of consumption to be equal to the present value of production. B) requires total spending in each period to be equal to total consumption in each period. C) does not take into account the ability to borrow or loan goods domestically. D) categorizes income into permanent and temporary income. E) limits consumption to the amount produced in each time period. Answer: A Difficulty: Moderate AACSB: Application of knowledge 4) If consumers experience an increase in lifetime income, current spending will ________, current saving will ________, and future spending will ________. A) increase; increase; increase B) increase; decrease; decrease C) increase; decrease; increase D) increase; increase; decrease E) decrease; increase; increase Answer: A Difficulty: Moderate AACSB: Application of knowledge
48 Copyright © 2022 Pearson Education, Inc.
5) If consumers experience an decrease in lifetime income, current spending will ________, current saving will ________, and future spending will ________. A) decrease; decrease; decrease B) increase; decrease; decrease C) increase; decrease; increase D) increase; increase; decrease E) decrease; increase; increase Answer: A Difficulty: Moderate AACSB: Application of knowledge 17.14 Appendix 2 to Chapter 17: The Marshall-Lerner Condition and Empirical Estimates of Trade Elasticities 1) The Marshall-Lerner Condition states that, all else equal A) nominal appreciation improves the current account if export and import volumes are sufficiently elastic with respect to the real exchange rate. B) real depreciation improves the current account if export and import volumes are sufficiently inelastic with respect to the real exchange rate. C) real appreciation improves the current account if export and import volumes are sufficiently elastic with respect to the real exchange rate. D) real depreciation improves the current account if export and import volumes are sufficiently elastic with respect to the real exchange rate. E) the sum of import and export elasticities must be equal to one in order for depreciation to occur. Answer: D Difficulty: Easy AACSB: Application of knowledge 2) The Marshall-Lerner condition holds that a country's current account balance will ________ in response to a real ________ in a nation's currency if the________. A) improve; depreciation; sum of the price elasticities of export and import demand exceeds 1 B) worsen; depreciation; sum of the price elasticities of export and import demand exceeds 1 C) improve; appreciation; sum of the price elasticities of export and import demand exceeds 1 D) improve; appreciation; sum of the price elasticities of export and import demand exceeds 0 E) worsen; depreciation; sum of the price elasticities of export and import demand exceeds 0 Answer: A Difficulty: Moderate AACSB: Application of knowledge
49 Copyright © 2022 Pearson Education, Inc.
3) A real depreciation of a nation's currency gives rise to the ________ effect and the ________ effect on the current account. A) volume; value B) depletion; expansion C) surplus; deficit D) output; trade E) price; profit Answer: A Difficulty: Moderate AACSB: Application of knowledge 4) One implication of an empirical investigation of the Marshall-Lerner condition is that, in the ________, a real ________ in a nation's currency is likely to ________ the country's current account balance. A) long run; depreciation; improve B) short run; depreciation; improve C) long run; appreciation; improve D) short run; appreciation; improve E) short run but not the long run; appreciation; improve Answer: A Difficulty: Easy AACSB: Application of knowledge 17.15
Appendix 3 to Chapter 17: The IS-LM Model and the DD-AA Model
1) The IS-LM model assumes that A) investment is negatively related to the expected real interest rate. B) investment is positively related to the expected real interest rate. C) investment is not directly related to the expected real interest rate. D) investment is not related to consumer purchases of durable goods. Answer: A Difficulty: Easy AACSB: Application of knowledge 2) Which of the following statements is TRUE? A) Consumption response to the interest rate is weaker than the investment response. B) Consumption response to the interest rate is stronger than the investment response. C) Consumption response to the interest rate is the same as the investment response. D) Consumption response to the interest rate is always the opposite of the investment response. Answer: A Difficulty: Easy AACSB: Application of knowledge
50 Copyright © 2022 Pearson Education, Inc.
3) The IS-LM model can be used to analyze which of the following? A) the effects of monetary and fiscal policies B) the effects of inflation and unemployment C) the effects of trade barriers and trade wars D) the Keynesian view of international trade dynamics Answer: A Difficulty: Easy AACSB: Application of knowledge 4) A temporary increase in money supply A) shifts LM to the right, lowering the interest rate and expanding output. B) shifts LM to the left, lowering the interest rate and expanding output. C) shifts LM to the right, raising the interest rate and lowering output. D) shifts LM to the left, raising the interest rate and expanding output. Answer: A Difficulty: Easy AACSB: Application of knowledge 5) A permanent increase in money supply A) shifts both LM and IS to the right. B) shifts LM to the right and IS to the left. C) shifts LM to the right and has no effect on IS. D) shifts IS to the left and has no effect on LM. Answer: A Difficulty: Easy AACSB: Application of knowledge 6) A temporary increase in government spending A) shifts IS to the right but has no effect on LM. B) shifts LM to the right and IS to the left. C) shifts LM to the right and does not shift IS. D) shifts IS to the left and does not shift LM. Answer: A Difficulty: Easy AACSB: Application of knowledge 7) A permanent fiscal expansion A) has no effect on output or the home interest rate. B) has positive effect on output. C) has positive effect on interest rate. D) has positive effect on both output and home interest rate. Answer: A Difficulty: Easy AACSB: Application of knowledge
51 Copyright © 2022 Pearson Education, Inc.
8) Explain how a fall in the nominal interest rate R raises aggregate demand. Answer: A fall in the nominal interest rate R raises aggregate demand through two channels: (1) Given the expected future exchange rate, a fall in R causes a domestic currency depreciation that improves the current account. (2) Given expected inflation, a fall in R directly encourages consumption and investment spending that falls only partly on imports. Difficulty: Moderate AACSB: Application of knowledge
52 Copyright © 2022 Pearson Education, Inc.
International Economics: Theory and Policy, 12e (Krugman) Chapter 18 Fixed Exchange Rates and Foreign Exchange Intervention 18.1
Why Study Fixed Exchange Rates?
1) Central banks often intervene in currency markets. This activity is called A) super-pegging. B) flexible floating. C) currency warfare. D) fixing exchange rates. E) managed floating. Answer: E Difficulty: Easy AACSB: Application of knowledge 2) A system of managed floating exchange rates is A) a system in which governments may attempt to moderate exchange rate movements without keeping exchange rates rigidly fixed. B) a system in which governments use flexible exchange rates. C) a system in which governments are forbidden from attempts to moderate exchange rate movements without keeping exchange rates rigidly fixed. D) a system in which governments need to reach a prior agreement among them before they may attempt to moderate exchange rate movements without keeping exchange rates rigidly fixed. E) a system in which governments use extensive fiscal policy to discourage exchange rate movements. Answer: A Difficulty: Easy AACSB: Application of knowledge 3) Which of the following is an example of a regional currency arrangement? A) most-favored nation status B) free-trade zones C) exchange rate union D) currency cartel associations E) agreement on commercial trade Answer: C Difficulty: Easy AACSB: Application of knowledge
1 Copyright © 2022 Pearson Education, Inc.
4) Organizations whose members agree to fix their mutual exchange rates while allowing their currencies to fluctuate in value against the currencies of nonmember countries are called A) exchange rate unions. B) mutual rate unions. C) pegging rate unions. D) common rate unions. Answer: A Difficulty: Easy AACSB: Application of knowledge 5) Industrialized countries typically ________ their floating exchange rates. Developing countries often ________ their floating exchange rates. A) fix; allow markets to determine B) manage; peg C) allow markets to determine; fix D) fix; manage E) peg; manage Answer: B Difficulty: Easy AACSB: Application of knowledge 6) Why is it important to understand fixed exchange rates in the modern global economy? Answer: Fixed rates continue to be important for four reasons: 1. Managed floating: Central banks intervene in foreign exchange markets. 2. Regional currency arrangements: Some countries peg their currency to another currency. 3. Developing countries and countries in transition: These countries often attempt to peg their currency to another currency. 4. Lessons of the past: Fixed exchange rates could have a resurgence. Difficulty: Moderate AACSB: Application of knowledge 7) What is the difference between clean and dirty float exchange rates? Answer: The system of partially floating dollar exchange rates is sometimes referred to as a dirty float, to distinguish it from a clean float in which governments make no direct attempts to influence foreign currency values. Difficulty: Easy AACSB: Application of knowledge
2 Copyright © 2022 Pearson Education, Inc.
18.2
Central Bank Intervention and the Money Supply
1) The central bank balance sheet records A) the assets held by the central bank and its liabilities. B) the purchase and sale of goods and services. C) the imports and exports payments. D) the key transactions among big banks. Answer: A Difficulty: Easy AACSB: Application of knowledge 2) A central bank's international reserves consist of its holdings of A) gold. B) silver and gold. C) foreign assets and gold. D) domestic assets and precious metals. E) foreign and domestic currency holdings. Answer: C Difficulty: Easy AACSB: Application of knowledge 3) The liabilities side of a central bank's accounts consists of A) deposits held by private banks. B) currency in circulation. C) deposits held by private banks and currency in circulation. D) deposits held by foreign banks, domestic assets, and currency in circulation. E) foreign assets and domestic assets. Answer: C Difficulty: Easy AACSB: Application of knowledge 4) When the central bank buys an asset from the public, ________ in central bank liabilities associated with the asset purchase causes the money supply to ________. When the central bank sells an asset to the public, ________ in central bank liabilities causes the money supply to ________. A) an increase; expand; a decrease; shrink B) a decrease; shrink; an increase; expand C) an increase; shrink; a decrease; expand D) a decrease; expand; an increase; shrink Answer: A Difficulty: Easy AACSB: Application of knowledge
3 Copyright © 2022 Pearson Education, Inc.
5) Which one of the following statements is most correct? A) Any central bank purchase of assets automatically results in an increase in the domestic money supply, while any central bank sale of assets automatically causes the money supply to decline. B) Any central bank purchase of assets results in an increase in the domestic money supply, while any central bank sale of assets causes the money supply to decline. C) Any central bank purchase of assets automatically results in a decrease in the domestic money supply, while any central bank sale of assets automatically causes the money supply to decline. D) Any central bank purchase of assets automatically results in a decrease in the domestic money supply, while any central bank sale of assets automatically causes the money supply to increase. E) Any central bank purchase of assets automatically results in an increase in the domestic money supply, while any central bank sale of assets does not necessarily affect the money supply. Answer: A Difficulty: Easy AACSB: Application of knowledge 6) Central banks sometimes carry out equal foreign and domestic asset transactions in opposite directions to nullify the impact of their foreign exchange operations on the domestic money supply. This type of policy is called A) sterilized foreign exchange intervention. B) nullified foreign exchange intervention. C) balancing foreign exchange intervention. D) offsetting foreign exchange intervention. Answer: A Difficulty: Easy AACSB: Application of knowledge
4 Copyright © 2022 Pearson Education, Inc.
7) Which one of the following statements is the most correct? A) If central banks are not sterilizing and the home country has a balance of payments surplus, any associated increase in the home central bank's foreign assets implies an increased home money supply. B) If central banks are not sterilizing and the home country has a balance of payments surplus, any associated increase in the home central bank's foreign assets implies a decreased home money supply. C) If central banks are not sterilizing and the home country has a balance of payments surplus, any associated increase in the home central bank's foreign assets implies an increased home money demand. D) If central banks are not sterilizing and the home country has a balance of payments surplus, any associated decrease in the home central bank's foreign assets implies an increased home money supply. E) If central banks are not sterilizing and the home country has a balance of payments shortage, any associated decrease in the home central bank's foreign assets implies an increased home money supply. Answer: A Difficulty: Easy AACSB: Application of knowledge 8) Which one of the following statements is most correct? A) If central banks are not sterilizing and the home country has a balance of payments surplus, any associated increase in a foreign central bank's claims on the home country implies a decreased foreign money supply. B) If central banks are not sterilizing and the home country has a balance of payments surplus, any associated decrease in a foreign central bank's claims on the home country implies a decreased foreign money demand. C) If central banks are not sterilizing and the home country has a balance of payments surplus, any associated decrease in a foreign central bank's claims on the home country implies a decreased foreign money supply. D) If central banks are not sterilizing and the home country has a balance of payments shortage, any associated decrease in a foreign central bank's claims on the home country implies a decreased foreign money supply. E) If central banks are not sterilizing and the home country has a balance of payments shortage, any associated decrease in a foreign central bank's claims on the home country implies an increased domestic money supply. Answer: C Difficulty: Easy AACSB: Application of knowledge
5 Copyright © 2022 Pearson Education, Inc.
9) What do a central bank' foreign and domestic assets consist of? Answer: Foreign assets consist mainly of foreign currency bonds owned by the central bank. These foreign assets make up the central bank's official international reserves, and their level changes when the central bank intervenes in the foreign exchange market by buying or selling foreign exchange. A central bank's international reserves also include any gold that it owns. Domestic assets are central bank holdings of claims to future payments by its own citizens and domestic institutions. These claims usually take the form of domestic government bonds and loans to domestic private banks. Difficulty: Easy AACSB: Application of knowledge 10) A balance sheet for the central bank of Pecunia is shown below: Central Bank Balance Sheet Assets Foreign assets $1,000 Domestic assets $1,500
Liabilities Deposits held by private banks Currency in circulation
$500 $2,000
Please write the new balance sheet if the bank sells $100 worth of foreign bonds for domestic currency. Answer: Central Bank Balance Sheet Assets Liabilities Foreign assets $900 Deposits held by private banks $500 Domestic assets $1,500 Currency in circulation $1,900 Difficulty: Moderate AACSB: Application of knowledge 11) A balance sheet for the central bank of Pecunia is shown below: Central Bank Balance Sheet Assets Foreign assets $1,000 Domestic assets $1,500
Liabilities Deposits held by private banks Currency in circulation
$500 $2,000
Please write the new balance sheet if the bank purchased $100 in foreign bonds by writing a check on itself. Answer: Central Bank Balance Sheet Assets Liabilities Foreign assets $1,100 Deposits held by private banks $600 Domestic assets $1,500 Currency in circulation $2,000 Difficulty: Moderate AACSB: Application of knowledge
6 Copyright © 2022 Pearson Education, Inc.
12) A balance sheet for the central bank of Pecunia is shown below: Central Bank Balance Sheet Assets Foreign assets $1,000 Domestic assets $1,500
Liabilities Deposits held by private banks Currency in circulation
$500 $2,000
Please write the new balance sheet if the bank makes a sterilized transaction by selling $100 of foreign assets for domestic currency and then purchasing $100 of domestic assets by writing a check on itself. Answer: Central Bank Balance Sheet Assets Liabilities Foreign assets $900 Deposits held by private banks $600 Domestic assets $1,600 Currency in circulation $1,900 Difficulty: Moderate AACSB: Application of knowledge 13) Please define and give an example of sterilized foreign exchange intervention. Answer: Sterilized foreign exchange intervention occurs when a central bank carries out equal foreign and domestic asset transactions in opposite directions to nullify the impact on the domestic money supply. An example is a central bank purchasing $100 of domestic assets but selling $100 of foreign bonds. Difficulty: Moderate AACSB: Application of knowledge
7 Copyright © 2022 Pearson Education, Inc.
18.3
How the Central Bank Fixes the Exchange Rate
1) To hold the exchange rate constant, A) a central bank must always be willing to trade currencies at the fixed exchange rate with the private actors in the foreign exchange market. B) a central bank cannot trade currencies at the fixed exchange rate with the private actors in the foreign exchange market. C) a central bank does not intervene to remove excess supplies or demands in the foreign exchange market. D) a central bank lets the foreign exchange and asset markets adjust itself to equilibrium. Answer: A Difficulty: Easy AACSB: Application of knowledge 2) Under fixed exchange rate, the foreign exchange market is in equilibrium when the interest parity condition holds, that is A) the domestic and foreign interest rates are equal, R = R . B) R = R + (Ee - E)/E. C) the foreign and domestic interest rates are unequal. D) the expected rate of domestic currency depreciation is high. E) the expected rate of currency depreciation is one. Answer: A Difficulty: Easy AACSB: Application of knowledge 3) Under fixed exchange rate, in general which one of the following statements is the MOST accurate? A) The following condition should hold for domestic money market equilibrium: Ms/P = L(R , Y). B) The following condition should hold for domestic money market equilibrium: Md/P = L(R , Y). C) The following condition should hold for domestic money market equilibrium: Ms = L(R , Y). D) The following condition should hold for domestic money market equilibrium: P = L(R , Y). E) The following condition should hold for domestic money market equilibrium: R*Md/P = L(Y). Answer: A Difficulty: Easy AACSB: Application of knowledge 4) What are the factors affecting the demand for foreign currency? Answer: Three factors affect the demand for foreign currency. They are expected return, risk, and liquidity. Difficulty: Moderate AACSB: Application of knowledge 8 Copyright © 2022 Pearson Education, Inc.
5) If the central bank does not purchase foreign assets when output increases but instead holds the money stock constant, can it still keep the exchange rate fixed at ? Please explain. Answer: No, the rise in output leads to an excess demand for money. If the central bank does not increase supply to meet this demand, the domestic interest rate would rise above the foreign rate, R*. This higher rate of return (and given expectations in the foreign exchange market) would cause the exchange rate to fall below . Difficulty: Difficult AACSB: Application of knowledge 6) If the central bank does not purchase foreign assets when output increases but instead holds the money stock constant, can it still keep the exchange rate fixed at ? Please explain with the aid of a figure. Answer:
No, the rise in output leads to an excess demand for money. If the central bank does not increase supply to meet this demand, the domestic interest rate would rise above the foreign rate, R*. This higher rate of return (and given expectations in the foreign exchange market) would cause the exchange rate to fall below . Difficulty: Moderate AACSB: Application of knowledge 9 Copyright © 2022 Pearson Education, Inc.
7) Please draw a figure illustrating the actions the central bank must take to maintain a fixed exchange rate following an increase in output. Answer:
A rise in output from to will increase the real money demand, so the central bank must purchase foreign assets and raise the money supply from to , in order to maintain a fixed exchange rate . Difficulty: Difficult AACSB: Application of knowledge
10 Copyright © 2022 Pearson Education, Inc.
18.4
Stabilization Policies with a Fixed Exchange Rate
1) By fixing the exchange rate, the central bank gives up its ability to A) adjust taxes. B) increase government spending. C) influence the economy through fiscal policy. D) depreciate the domestic currency. E) influence the economy through monetary policy. Answer: E Difficulty: Easy AACSB: Application of knowledge 2) Which one of the following statements is the MOST accurate? A) Under a fixed exchange rate, central bank monetary tools are powerless to affect the economy's money supply. B) Under a flexible exchange rate, central bank monetary tools are powerless to affect the economy's money supply or its output. C) Under a fixed exchange rate, fiscal policy tools are powerless to affect the economy's money supply or its output. D) Under a fixed exchange rate, central bank monetary tools are powerless to affect the economy's money supply or its output. E) Under a dirty float exchange rate, central bank monetary tools are powerless to affect the economy's money supply or its output. Answer: D Difficulty: Easy AACSB: Application of knowledge 3) Under fixed rates, which one of the following statements is the MOST accurate? A) Monetary policy can affect only output. B) Monetary policy can affect only employment. C) Monetary policy can affect only international reserves. D) Monetary policy can not affect international reserves. E) Monetary policy can only affect money supply. Answer: C Difficulty: Easy AACSB: Application of knowledge 4) Fiscal expansion under fixed exchange rates will have what temporary effect? A) the money supply will decrease B) output will decrease C) the exchange rate will increase D) the exchange rate will decrease E) there will be no effect Answer: D Difficulty: Easy AACSB: Application of knowledge 11 Copyright © 2022 Pearson Education, Inc.
5) Under fixed rates, which one of the following statements is the MOST accurate? A) Fiscal policy can affect output, employment and international reserves at the same time. B) Fiscal policy can affect only employment. C) Fiscal policy can affect only international reserves. D) Fiscal policy can affect only output and employment. E) Fiscal employment can affect only output and international reserves. Answer: A Difficulty: Easy AACSB: Application of knowledge 6) Which one of the following statements is the MOST accurate? A) Fiscal policy has the same effect on employment under fixed and flexible exchange rate regimes. B) Fiscal policy affects employment less under fixed than under flexible exchange rate regimes. C) Fiscal policy affects employment more under fixed than under flexible exchange rate regimes. D) Fiscal policy cannot affect employment under fixed exchange rate but does affect output under flexible exchange rate regimes. E) Fiscal policy can affect employment under fixed exchange rate regimes, but does not affect output under flexible exchange rate regimes. Answer: C Difficulty: Easy AACSB: Application of knowledge 7) Which one of the following statements is the MOST accurate? A) Fiscal policy has the same effect on output under fixed and flexible exchange rate regimes. B) Fiscal policy affects output more under fixed than under flexible exchange rate regimes. C) Fiscal policy affects output less under fixed than under flexible exchange rate regimes. D) Fiscal policy cannot affect output under fixed exchange rate but does affect output under flexible exchange rate regimes. E) Fiscal policy can affect output under fixed exchange rate but does not affect output under flexible exchange rate regimes. Answer: B Difficulty: Easy AACSB: Application of knowledge
12 Copyright © 2022 Pearson Education, Inc.
8) Which one of the following statements is the MOST accurate? A) A devaluation occurs when the central bank lowers the domestic currency price of foreign currency, E, and a revaluation occurs when the central bank raises E. B) A devaluation occurs when the central bank raises the domestic currency price of foreign currency, E, and a revaluation occurs when the central bank lowers E. C) Devaluation occurs when the domestic currency price of foreign currency, E, raises and a revaluation occurs when E is lowered. D) A devaluation occurs when the central bank of the foreign country raises the domestic currency price of foreign currency, E, and a revaluation occurs when the central bank of the foreign country lowers E. E) A devaluation occurs when the central bank raises the foreign currency price of domestic currency, E, and a revaluation occurs when the central bank lowers E. Answer: B Difficulty: Easy AACSB: Application of knowledge 9) Which one of the following statements is the MOST accurate? A) Depreciation is a rise in E when the exchange rate is fixed while devaluation is a rise in E when the exchange rate floats. B) Depreciation is a decrease in E when the exchange rate floats while devaluation is a rise in E when the exchange rate is fixed. C) Depreciation is a rise in E when the exchange rate floats while devaluation is a rise in E when the exchange rate is fixed. D) Depreciation is a rise in E when the exchange rate floats while devaluation is a decrease in E when the exchange rate is fixed. E) Depreciation is a fall in E when the exchange rate is fixed while devaluation is a fall in E when the exchange rate floats. Answer: C Difficulty: Easy AACSB: Application of knowledge 10) Which one of the following statements is the MOST accurate? A) Appreciation is a rise in E when the exchange rate floats while revaluation is a fall in E when the exchange rate is fixed. B) Appreciation is a fall in E when the exchange rate floats while revaluation is a fall in E when the exchange rate is fixed. C) Appreciation is a fall in E when the exchange rate is fixed while revaluation is a fall in E when the exchange rate is flexible. D) Appreciation is a fall in E when the exchange rate floats while revaluation is a rise in E when the exchange rate is fixed. E) Appreciation is a rise in E when the exchange rate floats while revaluation is a rise in E when the exchange rate is fixed. Answer: B Difficulty: Easy AACSB: Application of knowledge
13 Copyright © 2022 Pearson Education, Inc.
11) Which one of the following statements is the MOST accurate? A) Devaluation reflects a deliberate government decision. B) Depreciation reflects a deliberate government decision. C) Devaluation reflects a deliberate government decision while depreciation is an outcome of government actions and market forces acting together. D) Depreciation reflects a deliberate government decision while devaluation is an outcome of government actions and market forces acting together. E) Devaluation and depreciation have the same meaning and the same causes. Answer: C Difficulty: Easy AACSB: Application of knowledge 12) Which one of the following statements is the MOST accurate? A) Revaluation reflects an outcome of government actions and market forces acting together while appreciation reflects a deliberate government decision. B) Revaluation reflects a deliberate government decision while appreciation is an outcome of government actions and market forces acting together. C) Revaluation reflects a deliberate government decision while appreciation is an outcome of government actions. D) Revaluation and appreciation have the same meaning and the same causes. E) Appreciation reflects a deliberate government decision while revaluation is an outcome of government actions and market forces acting together. Answer: B Difficulty: Easy AACSB: Application of knowledge 13) Under fixed exchange rates, which one of the following statements is the MOST accurate? A) Keeping other things constant, devaluation causes a decrease in output, a decrease in official reserves, and a contraction of the money supply. B) Keeping other things constant, devaluation causes a rise in output, a rise in official reserves, and an expansion of the money supply. C) Keeping other things constant, devaluation causes a rise in output and a rise in official reserves. D) Keeping other things constant, devaluation causes a rise in output and an expansion of the money supply. E) Keeping other things constant, devaluation causes a rise in official reserves, and an expansion of the money supply. Answer: B Difficulty: Easy AACSB: Application of knowledge
14 Copyright © 2022 Pearson Education, Inc.
14) Under fixed exchange rates, which one of the following statements is the MOST accurate? A) Devaluation causes a rise in output. B) Devaluation causes a decrease in output. C) Devaluation has no effect on output. D) Devaluation causes a rise in output and a decrease in official reserves. E) Devaluation causes a decrease in output and in official reserves. Answer: A Difficulty: Easy AACSB: Application of knowledge 15) Under fixed exchange rates, which one of the following statements is the MOST accurate? A) Devaluation causes a reduction of the money supply. B) Devaluation has no effect on the stock of money. C) Devaluation causes an expansion of the money supply. D) Devaluation causes a reduction in output. E) Devaluation causes a reduction in official reserves. Answer: C Difficulty: Easy AACSB: Application of knowledge 16) Which of the following best describes a deliberate government decision to lower the exchange rate, E? A) appreciation B) depreciation C) revaluation D) devaluation E) accumulation Answer: C Difficulty: Easy AACSB: Application of knowledge 17) When a country's currency is devalued A) output decreases. B) output increases and the money supply decreases. C) the money supply decreases. D) output decreases and the money supply increases. E) both the output and the money supply increases. Answer: E Difficulty: Easy AACSB: Application of knowledge
15 Copyright © 2022 Pearson Education, Inc.
18) The main reason(s) why governments sometimes choose to devalue their currencies is (are) A) devaluation makes domestic goods more expensive in relation to foreign goods. B) devaluation makes domestic services more expensive in relation to foreign services. C) devaluation increases foreign reserves held by the central bank. D) devaluation improves the current account and increases foreign reserves held by the central bank. E) devaluation hurts foreign currencies. Answer: D Difficulty: Easy AACSB: Application of knowledge 19) Which of the following statements is TRUE? A) A devaluation under a fixed rate has the same long-run effect as a proportional increase in the money supply under a floating rate. B) A devaluation under a fixed rate has a stronger long-run effect in the money supply than under a floating rate. C) A devaluation under a fixed rate has a weaker long-run effect in the money supply than under a floating rate. D) A devaluation under a fixed rate has no long-run effect in the money supply. Answer: A Difficulty: Easy AACSB: Application of knowledge
16 Copyright © 2022 Pearson Education, Inc.
20) Use a figure to illustrate the ineffectiveness of monetary policy to spur on an economy under a fixed exchange rate. Answer:
The initial equilibrium rests at point 1. If the central bank wishes to use monetary policy to increase output from to , then they might buy domestic assets and shift the AA curve outward. However, the central bank must maintain a fixed exchange rate , so would have to sell foreign assets for domestic currency, returning the economy to point 1. Difficulty: Difficult AACSB: Application of knowledge
17 Copyright © 2022 Pearson Education, Inc.
21) Use a figure to explain the potential effectiveness of fiscal policy to spur on the economy under a fixed exchange rate. Answer:
With an aim toward increasing output, the government could use fiscal policy to shift the DD curve outward. The central bank will have to take steps to maintain a fixed exchange rate , among the options is buying foreign assets with money, to shift the AA schedule outward until the equilibrium at point 3 is reached. Difficulty: Difficult AACSB: Application of knowledge 22) Please discuss the difference between devaluation and depreciation. Answer: Depreciation is a rise in the exchange rate E when the exchange rate floats, while devaluation is a rise in E when the exchange rate is fixed. Devaluation reflects a deliberate government decision, while depreciation is an outcome of government actions and market forces ("the invisible hand") acting together. Difficulty: Moderate AACSB: Application of knowledge
18 Copyright © 2022 Pearson Education, Inc.
23) Define devaluation and use a figure to show the effect of a currency devaluation on the economy. Answer:
A devaluation occurs when the central bank raises the domestic currency price of foreign currency. In the figure, the domestic currency is devalued from to . Since nothing in the DD schedule has changed, the new equilibrium at point 2 must be reached by an expansion of the money supply (AA curve shifts outward). Notice also that output has increased from to . Difficulty: Difficult AACSB: Application of knowledge 24) Why do governments sometimes choose to devalue their currencies? Answer: First, devaluation allows the government to fight domestic unemployment despite the lack of effective monetary policy. If government spending and budget deficits are politically unpopular, for example, or if the legislative process is slow, a government may opt for devaluation as the most convenient way of boosting aggregate demand. A second reason for devaluing is the resulting improvement in the current account, a development the government may believe to be desirable. The third motive behind devaluations, one we mentioned at the start of this subsection, is their effect on the central bank's foreign reserves. If the central bank is running low on reserves, a sudden, one-time devaluation (one that nobody expects to be repeated) can be used to draw in more reserves. Difficulty: Easy AACSB: Application of knowledge
19 Copyright © 2022 Pearson Education, Inc.
25) Please use a figure to discuss whether or not a devaluation under a fixed exchange rate has the same long-run effect as a proportional increase in the money supply under a floating rate. Answer:
A currency devaluation shifts the AA schedule outward from equilibrium point 1 to equilibrium point 2. The devaluation does not change long-run demand or supply conditions in the output market. Thus, the increase in the long-run price level will exactly offset the increase in exchange rate. Thus, a devaluation is neutral in the long run and this is the exact same scenario as for an increase in the money supply under a floating exchange rate. Difficulty: Difficult AACSB: Application of knowledge
20 Copyright © 2022 Pearson Education, Inc.
18.5
Balance of Payments Crises and Capital Flight
1) A balance of payments crisis is best described as A) a sharp change in interest rates sparked by a change in expectations about the level of imports. B) a sharp change in foreign reserves sparked by a change in expectations about the future exchange rate. C) a sharp change in interest rates sparked by a change in expectations about the level of exports. D) a sharp change in foreign reserves sparked by a change in expectations about the level of imports. E) a sharp change in foreign reserves sparked by a change in expectations about domestic production. Answer: B Difficulty: Easy AACSB: Application of knowledge 2) The expectation of future devaluation causes a balance of payments crisis marked by A) a sharp rise in reserves and a fall in the home interest rate below the world interest rate. B) a sharp fall in reserves and an even bigger fall in the home interest rate below the world interest rate. C) a sharp fall in reserves and a rise in the home interest rate above the world interest rate. D) a sharp rise in reserves and an even greater rise in the home interest rate above the world interest. E) a sharp rise in reserves and a rise in the home interest rate to the level of the world interest. Answer: C Difficulty: Easy AACSB: Application of knowledge 3) The expectation of future revaluation causes a balance of payments crisis marked by A) a sharp rise in reserves and a fall in the home interest rate below the world interest rate. B) a sharp fall in reserves and an even bigger fall in the home interest rate below the world interest rate. C) a sharp fall in reserves and a rise in the home interest rate above the world interest rate. D) a sharp rise in reserves and an even greater rise in the home interest rate above the world interest. E) a sharp fall in reserves and an unchanged home interest rate. Answer: A Difficulty: Easy AACSB: Application of knowledge
21 Copyright © 2022 Pearson Education, Inc.
4) Capital flight A) increases reserves. B) is never associated with the expectation of devaluation. C) may undo expected devaluation. D) reduces losses during a devaluation scare. E) decreases reserves and may induce devaluation. Answer: E Difficulty: Easy AACSB: Application of knowledge 5) The reserve loss accompanying a devaluation scare is often labeled A) capital flight. B) reserve scarcity. C) capital flee. D) devaluation fear. Answer: A Difficulty: Easy AACSB: Application of knowledge 6) Currency crises may result from A) central bank balance sheets with higher liabilities than assets. B) political upheaval leading to lowering exports. C) a reconfiguration of central bank balance sheets. D) speculative attacks on the currency or central banks purchasing excessive amounts of government bonds. E) depreciation of foreign reserves. Answer: D Difficulty: Easy AACSB: Application of knowledge 7) ________ is a situation when an economy can be vulnerable to currency speculation not directly caused by the unhealthy economic fundamental conditions or improper government policies. A) Self-fulfilling currency crisis B) Self-fulfilling debt crisis C) Self-fulfilling financial crisis D) Self-fulfilling prophecy crisis Answer: A Difficulty: Easy AACSB: Application of knowledge
22 Copyright © 2022 Pearson Education, Inc.
8) What is capital flight? Answer: The reserve loss accompanying a devaluation scare is often labeled capital flight. Residents flee the domestic currency by selling it to the central bank for foreign exchange; they then invest the foreign currency abroad. At the same time, foreigners convert holdings of home assets into their own currencies and repatriate the proceeds. When fears of devaluation arise because the central bank's reserves are low to begin with, capital flight is of particular concern to the government. By pushing reserves even lower, capital flight may force the central bank to devalue sooner and by a larger amount than planned. Difficulty: Easy AACSB: Application of knowledge 9) What causes currency crises? How can they be prevented? Answer: A government is following policies that are not consistent with maintaining a fixed exchange rate over the longer term. Once market expectations take those policies into account, the country's interest rates inevitably are forced up. A country's central bank may be buying bonds from the domestic government to allow the government to run continuing fiscal deficits. Since these central bank purchases of domestic assets cause ongoing losses of central bank foreign exchange reserves, reserves will fall toward a point at which the central bank may find itself without the means to support the exchange rate. As the possibility of a collapse rises over time, so will domestic interest rates, until the central bank indeed runs out of foreign reserves and the fixed exchange rate is abandoned. The only way for the central bank to avoid this fate is to stop bankrolling the government deficit, hopefully forcing the government to live within its means. Difficulty: Moderate AACSB: Application of knowledge
23 Copyright © 2022 Pearson Education, Inc.
18.6
Managed Floating and Sterilized Intervention
1) Which of the following statements is TRUE? A) Under managed floating, monetary policy is influenced by exchange rate changes without being completely subordinate to the requirements of a fixed rate. B) Under managed floating, monetary policy cannot be influenced by exchange rate changes due to the requirements of a fixed rate. C) Under managed floating, the central bank manages to keep a fixed foreign exchange rate all the time. D) Under managed floating, the central bank does not intervene with foreign exchange rate at all. Answer: A Difficulty: Easy AACSB: Application of knowledge 2) An action taken by a country's central bank to counter the effects on the money supply with opposite domestic asset transactions is called A) sterilized foreign exchange intervention. B) unsterilized foreign exchange intervention. C) counter foreign exchange intervention. D) balancing foreign exchange intervention. Answer: A Difficulty: Easy AACSB: Application of knowledge 3) Perfect asset substitutability is the assumption that A) the foreign exchange market is in equilibrium only when expected returns on domestic assets are greater than returns on foreign currency bonds. B) the foreign exchange market is in equilibrium only when expected returns on foreign currency bonds are greater than returns on domestic assets. C) the foreign exchange market is in equilibrium only when expected returns on all assets are negative. D) the foreign exchange market is in equilibrium only when expected returns on domestic assets are equal to returns on foreign currency bonds. E) the foreign exchange market is in equilibrium only when domestic assets are risk-free. Answer: D Difficulty: Easy AACSB: Application of knowledge 4) Imperfect asset substitutability assumes A) the returns on foreign and domestic currency bonds are identical. B) the returns on foreign and domestic currency are unrelated. C) the risks of holding foreign and domestic currency are identical. D) the risks of holding foreign and domestic currency are unrelated to returns. E) the returns on foreign and domestic currency differ and are influenced by risk. Answer: E Difficulty: Easy AACSB: Application of knowledge 24 Copyright © 2022 Pearson Education, Inc.
5) The global financial crisis of 2007-2008 resulted in a(n) ________ of the Swiss franc as foreign currency flowed ________ the country. As a result, Swiss products became ________ competitive in world markets. A) depreciation; out of; more B) depreciation; into; more C) appreciation; out of; less D) depreciation; out of; less E) appreciation; into; less Answer: E Difficulty: Easy AACSB: Application of knowledge 6) The global financial crisis of 2007-2008 resulted in a(n) ________ of the Swiss franc. In 2011, the Swiss central bank intervened in order to cause a(n) ________ of the franc. A) appreciation; appreciation B) depreciation; depreciation C) appreciation; revaluation D) depreciation; appreciation E) appreciation; depreciation Answer: E Difficulty: Easy AACSB: Application of knowledge 7) Imperfect asset substitutability exists A) when it is possible for the expected returns on two assets to be different. B) when the expected returns on two assets are the same. C) only when one asset is foreign and the other is domestic. D) when there is risk in the foreign exchange market. E) when assets are liquid. Answer: D Difficulty: Easy AACSB: Application of knowledge 8) The interest parity condition can be written as A) R = R - (Ee - E)/E. B) R = R + (Ee - E)/E. C) R = R2 - (Ee - E)/E. D) R = R /(Ee - E). E) R = R + (Ee + E)/E. Answer: B Difficulty: Easy AACSB: Application of knowledge
25 Copyright © 2022 Pearson Education, Inc.
9) When domestic and foreign currency bonds are imperfect substitutes, the domestic interest rate (R) can be written as A) R = R - (Ee - E)/E + ρ. B) R = R - (Ee - E)/E. C) R = R + (Ee - E)/E + ρ. D) R = R - (Ee + E)/E + ρ. E) R = R - (Ee - E)ρ. Answer: C Difficulty: Easy AACSB: Application of knowledge 10) In the interest rate parity condition with imperfect substitutes and a risk premium of ρ A) an increased stock of domestic government debt will raise the difference between the expected returns on domestic and foreign currency bonds. B) a decreased stock of domestic government debt will raise the difference between the expected returns on domestic and foreign currency bonds. C) an increased stock of domestic government debt will reduce the difference between the expected returns on domestic and foreign currency bonds. D) an increased stock of domestic government debt will have no effect on the difference between the expected returns on domestic and foreign currency bonds. E) a decreased stock of domestic government debt will have no effect on the difference between the expected returns on domestic and foreign currency bonds. Answer: A Difficulty: Easy AACSB: Application of knowledge 11) With imperfect asset substitutability, sterilized purchases of foreign exchange can cause the home currency to ________, and sterilized sales of foreign exchange cause the home currency to ________. A) depreciate; appreciate B) appreciate; depreciate C) depreciate; depreciate D) appreciate; appreciate Answer: A Difficulty: Easy AACSB: Application of knowledge
26 Copyright © 2022 Pearson Education, Inc.
12) The signaling effect of foreign exchange intervention A) never has any effect on exchange rates. B) can alter the market's view of exchange rates independent from the stance of monetary and fiscal policies. C) cannot cause an immediate exchange rate change when bonds denominated in different currencies are perfect substitutes. D) never leads to actual changes in monetary or fiscal policy. E) can alter the market's view of future monetary policies and cause an immediate exchange rate change. Answer: E Difficulty: Easy AACSB: Application of knowledge 13) Please describe in detail a self-fulfilling currency crisis. Answer: Consider an economy in which domestic commercial banks' liabilities are mainly short-term deposits, and in which many of the banks' loans to businesses are likely to go unpaid in the event of a recession. If the market suspects there will be devaluation, interest rates will rise, banks' borrowing costs go up, and a banks' assets have lower value if a recession hits. To prevent financial collapse, the central bank will lend money to banks and no longer be able to keep the exchange rate from rising. Thus, the emergence of devaluation expectations eventually leads to a devaluation of currency (self-fulfilling). Difficulty: Moderate AACSB: Application of knowledge 14) Sterilization will be fruitless under a fixed exchange rate. Explain. Answer: To hold the exchange rate constant when fiscal policy becomes more expansive, the central bank must buy foreign assets and expand the home money supply. The policy raises output, but it eventually also causes inflation, which the central bank may try to avoid by sterilizing the increase in the money supply that its fiscal policy has induced. As quickly as the central bank sells domestic assets to reduce the money supply, however, it will have to buy more foreign assets to keep the exchange rate fixed. The ineffectiveness of monetary policy under a fixed exchange rate implies that sterilization is a self-defeating policy. Difficulty: Moderate AACSB: Application of knowledge 15) Describe the effect of the 2008-2009 global financial crisis on the Swiss franc and the central bank's efforts to respond to the resulting problems. Answer: The 2008-2009 global financial crisis resulted in appreciation of the franc as currency traders purchased the franc as a safe haven currency. The Swiss economy consequently suffered as its products became less competitive with imports. The Swiss responded by committing to currency intervention designed to control appreciation of the franc and restore the country's competitiveness in global markets. Difficulty: Moderate AACSB: Application of knowledge
27 Copyright © 2022 Pearson Education, Inc.
16) Use a figure to explain how a balance of payments crisis occurs and its hand in capital flight. Answer:
Suppose the foreign exchange market expects the government to devalue the currency in the future and adopt a new fixed exchange rate > . This leads to a rightward shift in the curve that measures the expected domestic currency return on foreign currency deposits. Since the exchange rate remains fixed at , the domestic interest rate must rise to R* + ( - )/ . The central bank must sell foreign reserves and shrink the money supply in response. This reserve loss accompanying a devaluation scare is labeled capital flight. Difficulty: Difficult AACSB: Application of knowledge
28 Copyright © 2022 Pearson Education, Inc.
17) Assuming perfect asset substitutability, can sterilized intervention by the central bank be effective? Please discuss. Answer: No, a sterilized foreign exchange intervention by the central bank leaves the domestic money supply unchanged. Under floating exchange rates, a change in the interest rate is needed to affect the exchange rate, but the interest rate won't change if the money supply does not. Under a fixed exchange rate, an expansive policy needs to be offset by an increase in the domestic money supply. To avoid inflation, the central bank sterilizes this increase in the money supply by selling domestic assets. However, with a fixed exchange rate, this means buying foreign assets. If foreign assets are perfect substitutes for domestic assets, this sterilization is not effective. Difficulty: Moderate AACSB: Application of knowledge 18) Use a figure to show the effect of a sterilized central bank purchase of foreign assets under the imperfect asset substitutability assumption. Answer:
The interest parity condition is given by R = R* + ( - E)/E + Suppose that the domestic assets of the central bank fall from to through a sterilized purchase of foreign assets. Then the risk-adjusted return increases and the exchange rate increases. Difficulty: Difficult AACSB: Application of knowledge 29 Copyright © 2022 Pearson Education, Inc.
19) Assume that initially, the risk premium, ρ = 0 and that the domestic and foreign interest rates are given by R = .06, R* = .05. Suppose that the risk premium depends linearly on the difference between domestic government debt, B, and domestic assets of the central bank, A, i.e., ρ= Find the new domestic interest rate if a sterilized purchase of foreign assets adjusts A s.t. (a) B - A = -.01/ (b) B - A = .01/ (c) B - A = .03/ Answer: (a) R = .05 + .01 + (-.01) = .05 (b) R = .05 + .01 + (.01) = .07 (c) R = .05 + .01 + (.03) = .09 Difficulty: Difficult AACSB: Application of knowledge 20) Assume that initially, the risk premium, ρ = 0 and that the domestic and foreign interest rates are given by R = .06, R* = .05. Suppose that the risk premium depends linearly on the difference between domestic government debt, B, and domestic assets of the central bank, A, i.e., ρ= How much will the central bank have to reduce domestic assets A s.t. the domestic interest rate will increase by (a) 1% (b) 4%? Answer: (a) ρ = .01 = ΔA =
- (B -
)
- (B -
)
(b) ρ = .04 = ΔA =
Difficulty: Difficult AACSB: Application of knowledge 21) Does the signaling effect of foreign exchange intervention support or refute the claim that assets cannot be perfect substitutes if sterilized intervention is going to have any effect? Please explain. Answer: The signaling effect refutes the claim. Even with the assumption of perfect asset substitutability, if the market is unsure of the future direction of policy, then sterilized intervention can fix a market's expectations about the exchange rate in the future. From post discussion, a change in the expected exchange rate will lead to a change in the exchange rate today. Difficulty: Moderate AACSB: Application of knowledge
30 Copyright © 2022 Pearson Education, Inc.
18.7
Reserve Currencies in the World Monetary System
1) What is a reverse currency? A) the currency central banks hold in their international reserves B) the currency central banks use for invoices C) the currency central banks use for investment D) the currency central banks hold to pay public debt Answer: A Difficulty: Easy AACSB: Application of knowledge 2) Between the end of World War II and 1973, the main reverse currency that almost every country pegged the exchange rate of its money was the A) U.S. dollar. B) Euro. C) British pound. D) Japanese yen. E) Swiss franc. Answer: A Difficulty: Easy AACSB: Application of knowledge 3) What is the gold standard? Answer: Under a gold standard, central banks peg the prices of their currencies in terms of gold and hold gold as official international reserves. Difficulty: Easy AACSB: Application of knowledge 4) Briefly describe two systems for fixing the exchange rates of all currencies against each other and the time periods in which they were used. Answer: The first is to single cut one country's currency as the reserve currency. The other countries hold this reserve currency and fix their interest rate to it by standing ready to exchange domestic currency for the reserve currency. The U.S. dollar was the reserve currency from 1945 to 1973. The second is the gold standard in which central banks peg the prices of their currencies in terms of gold and hold gold as official international reserves. This was used between 1870 and 1914. Difficulty: Moderate AACSB: Application of knowledge
31 Copyright © 2022 Pearson Education, Inc.
5) This question concerns the mechanism of a reserve currency standard. Two countries, X and Y, have two currencies, x and y, fixed to the reserve currency, the U.S. dollar. Suppose the exchange rate between x and the U.S. dollar is 3x per dollar. Suppose the exchange rate between y and the U.S. dollar is 5y per dollar. Explain (using numbers) the mechanism if the x-y exchange rate was 0.5 x per y. Answer: At this exchange rate, an investor can make an arbitrage profit by selling $100 to the central bank of X (receiving 300 x), then selling your 300 x to the foreign exchange market for 300 x/(0.5 x per y) = 600 y, then buying U.S. dollars in the amount of $120 from the central bank of Y. Thus the foreign exchange market will bid the x-y exchange rate up to 0.6 x per y. Difficulty: Difficult AACSB: Application of knowledge 6) This question concerns the mechanism of a reserve currency standard. Two countries, X and Y, have two currencies, x and y, fixed to the reserve currency, the U.S. dollar. Suppose the exchange rate between x and the U.S. dollar is 3x per dollar. Suppose the exchange rate between y and the U.S. dollar is 5y per dollar. Explain (using numbers) the mechanism if the x-y exchange rate was 0.8 x per y. Answer: At this exchange rate, an investor can make an arbitrage profit by selling $100 to the central bank of Y (receiving 500 y), then selling this 500 y to the foreign exchange market for 500 y/(0.8 x per y) = 400 x, then buying $133.33 U.S. dollars from the central bank of X with this 400 x. Thus the foreign exchange market will bid the x-y exchange rate down to 0.6. Difficulty: Difficult AACSB: Application of knowledge 7) Explain how a country whose currency is the reserve currency can use monetary policy for macroeconomic stabilization. In particular, explain the result if that country doubled its domestic money supply. Answer: The immediate result of the doubling of the money supply in the reserve currency's country will be able to increase the exchange rate between the reserve currency and all other currencies. However, all other countries must fix their exchange rate to the reserve currency, so they will purchase the reserve currency and hold it as official international reserves (thus increase their own money supply) until the exchange rate has returned to normal. Thus, the reserve country has the power to affect its own economy and all other countries must adjust in response. Difficulty: Moderate AACSB: Application of knowledge
32 Copyright © 2022 Pearson Education, Inc.
18.8
The Gold Standard
1) Which of the following statements is TRUE? A) Under a gold standard, each country fixes the price of its currency in terms of gold by standing ready to trade domestic currency for gold whenever necessary to defend the official price. B) Under a gold standard, each country is not responsible for pegging its currency's price in terms of the official international reserve asset, gold. C) Under a gold standard, countries with limited gold reserves cannot participate. D) Under a gold standard, all countries sets the same price of its currency in terms of gold. Answer: A Difficulty: Moderate AACSB: Application of knowledge 2) The gold standard, like a reserve currency system, results in A) fixed exchange rates between all currencies. B) volatile exchange rates between all currencies. C) unpredictable exchange rates between all currencies. D) variable exchange rates between all currencies. Answer: A Difficulty: Easy AACSB: Application of knowledge 3) If the dollar price of gold is pegged at $35 per ounce by the Federal Reserve while the pound price of gold is pegged at £14.58 per ounce by the Bank of England, the dollar/pound exchange rate must be constant at A) $2.40 per pound. B) $0.42 per pound. C) $0.24 per pound. D) $4.20 per pound. Answer: A Difficulty: Easy AACSB: Application of knowledge 4) Under a gold standard, whenever a country is losing reserves and seeing its money supply ________ as a consequence, foreign countries are gaining reserves and seeing their money supplies ________. A) shrink; expand B) expand; shrink C) shrink; shrink D) expand; expand Answer: A Difficulty: Easy AACSB: Application of knowledge
33 Copyright © 2022 Pearson Education, Inc.
5) The gold standard offers many potential benefits EXCEPT A) more influence on world macroeconomic conditions by countries with potentially large gold production. B) symmetric nature of international monetary adjustment. C) automatic limit on central banks' money creation. D) certainty with a fixed pattern of exchanged rates. Answer: A Difficulty: Easy AACSB: Application of knowledge 6) The gold standard has some drawbacks EXCEPT A) symmetric nature of international monetary adjustment. B) overall price level stability depends on whether the relative price of gold and all other goods and services is stable. C) central bank cannot increase holdings of international reserves as its economy grows unless new gold is discovered. D) gold-producing nations gain unfair advantage. Answer: A Difficulty: Easy AACSB: Application of knowledge 7) From 1837 and up until the Civil War, the United States adhered to a A) gold standard. B) silver standard. C) bimetallic standard. D) bronze standard. E) copper standard. Answer: C Difficulty: Easy AACSB: Application of knowledge 8) From the Civil War up to 1914, the United States adhered to a A) gold standard. B) silver standard. C) bimetallic standard. D) bronze standard. E) copper standard. Answer: A Difficulty: Easy AACSB: Application of knowledge
34 Copyright © 2022 Pearson Education, Inc.
9) From 1837 until the Civil War, the U.S. adhered to a bimetallic standard, in which the currency was based on both A) silver and gold. B) silver and platinum. C) gold and palladium. D) platinum and palladium. Answer: A Difficulty: Easy AACSB: Application of knowledge 10) Halfway between the gold standard and a pure reserve currency standard where central banks' reserves consist of gold and currencies whose prices in terms of gold are fixed, is the A) gold exchange standard. B) mixed exchange standard. C) currency exchange standard. D) reserve currency standard. Answer: A Difficulty: Easy AACSB: Application of knowledge 11) The top four major currencies in countries' international reserve holdings include A) U.S. dollar, euro, British pound, and Japanese yen. B) U.S. dollar, euro, Chinese renminbi, and Japanese yen. C) U.S. dollar, euro, Japanese yen, and Swiss franc. D) U.S. dollar, euro, Chinese renminbi, and British pound. Answer: A Difficulty: Easy AACSB: Application of knowledge 12) Under the gold standard, if the dollar price of gold is pegged at $35 per ounce and the euro price of gold is pegged at 12 euro per ounce, what is the dollar/euro exchange rate? Answer: The dollar/euro exchange rate must be constant and equal to ($35 per ounce) / (12 euro per ounce) = $2.92 per euro. Difficulty: Moderate AACSB: Application of knowledge 13) Under the gold standard, if the dollar price of gold is pegged at $35 per ounce and the dollar/euro exchange rate is set at $2.40 per euro, what must the euro price of gold be pegged at? Answer: The euro price of gold is constant and equal to ($35 per ounce) / ($2.40 per euro) = 14.58 euro per ounce. Difficulty: Moderate AACSB: Application of knowledge
35 Copyright © 2022 Pearson Education, Inc.
14) Describe the mechanism which would take place if the Bank of England decides to increase its money supply by purchasing domestic assets under the gold standard. Answer: The increase in Britain's money supply would push interest rates down and make foreign currency assets more attractive than domestic ones. Holders of pound deposits will attempt to sell them for foreign deposits. To accomplish this, they sell pound deposits to the Bank of England for gold and then use this gold to purchase foreign deposits. England loses foreign reserves since it is selling gold and foreign countries are gaining reserves. Equilibrium is re-established after Britain's money supply has fallen enough to force the British interest rate up until it is equally as attractive as the interest rate on foreign currency. Difficulty: Difficult AACSB: Application of knowledge 15) List the drawbacks of the gold standard. Answer: 1. Undesirable constraints on the use of monetary policy to fight unemployment. 2. A stable overall price level is achieved only if the relative price of gold and all other goods and services is stable. 3. A central bank cannot increase holdings of international reserves as its economy grows unless new gold is discovered. 4. Unfair advantage to gold-producing nations. Difficulty: Moderate AACSB: Application of knowledge 16) Briefly discuss the main advantage of the bimetallic standard over the gold standard. Answer: The advantage of bimetallism was that it might reduce the price level instability resulting from the use of gold alone. Were gold to become scarce and expensive, cheaper and relatively abundant silver would become the predominant form of money, thereby mitigating the deflation that a pure gold standard would imply. Difficulty: Moderate AACSB: Application of knowledge 17) Please briefly describe what is meant by a gold exchange standard. Answer: Under a gold exchange standard, central banks' reserves consist of gold and currencies whose price in terms of gold are fixed, and each central bank fixes its exchange rate to a currency with a fixed gold price. The post-WWII currency system was supposed to be a gold exchange standard with the U.S. responsible for fixing the price of gold at $35 per ounce. Difficulty: Moderate AACSB: Application of knowledge
36 Copyright © 2022 Pearson Education, Inc.
18) From the figure below, please provide an explanation for the slower pace of accumulation of international reserves held by developing countries starting with the crisis years of 2008-2009.
Answer: The growth of global capital markets has increased the potential variability of financial flaws across borders, and especially across the borders of developing countries. The sharp decline in developing country reserve growth started from 2008-2009 due to an international debt crisis. Difficulty: Difficult AACSB: Application of knowledge
37 Copyright © 2022 Pearson Education, Inc.
18.9 Appendix 1 to Chapter 18: Equilibrium in the Foreign Exchange Market with Imperfect Asset Substitutability 1) If assets are imperfect substitutes, then an increase in the amount of domestic currency bonds held by the public will ________ the risk premium and ________ the amount of domestic currency bonds held by the central bank. A) increase; leave unchanged B) increase; decrease C) increase; increase D) decrease; decrease E) leave unchanged; decrease Answer: A Difficulty: Moderate AACSB: Application of knowledge 2) If assets are imperfect substitutes, then a decrease in the amount of domestic currency bonds held by the public will ________ the risk premium and ________ the amount of domestic currency bonds held by the central bank. A) decrease; leave unchanged B) increase; decrease C) increase; increase D) decrease; decrease E) leave unchanged; decrease Answer: A Difficulty: Moderate AACSB: Application of knowledge
38 Copyright © 2022 Pearson Education, Inc.
18.10
Appendix 2 to Chapter 18: The Timing of Balance of Payments Crises
1) Balance of payments crises under fixed exchange rates occur because of A) government policies that are inconsistent with fixed exchange rates. B) punitive currency wars. C) global inflation and trade imbalances due to war. D) excessive exports and imports that overload the global system. E) monotonic expansion in global currency volume. Answer: A Difficulty: Easy AACSB: Application of knowledge 2) A balance of payments crises under fixed exchange rates occurs when A) marginal returns on foreign exchange investments approach zero. B) a country runs out of foreign reserves. C) a country is in a liquidity trap. D) forward currency markets undergo high volatility. E) exports and imports expand beyond some point. Answer: B Difficulty: Easy AACSB: Application of knowledge 18.11
Appendix 3 to Chapter 18: The Monetary Approach to the Balance of Payments
1) The fluctuations in central bank reserves can be thought of as the result of changes in the money market. This method of analyzing the balance of payments is called the A) monetary approach to the balance of payments. B) fiscal approach to the balance of payments. C) fluctuation approach to the balance of payments. D) reserve approach to the balance of payments. Answer: A Difficulty: Easy AACSB: Application of knowledge 2) The monetary approach was developed in the 1950s and 1960s by A) IMF researchers Polak, Johnson, Mundell, and their students at the University of Chicago. B) Polak, Johnson, and Mundell. C) students at the University of Chicago. D) Krugman, Obstfeld, and Melitz. Answer: A Difficulty: Easy AACSB: Application of knowledge
39 Copyright © 2022 Pearson Education, Inc.
International Economics: Theory and Policy, 12e (Krugman) Chapter 19 International Monetary Systems: An Historical Overview 19.1
Macroeconomic Policy Goals in an Open Economy
1) What made it sometimes more difficult for governments to achieve such policy goals as full employment and price level stability? A) The interdependence of open national economies. B) The independence of closed national economies. C) The independence of open national economies. D) The interdependence of closed national economies. Answer: A Difficulty: Easy AACSB: Application of knowledge 2) In an open economy, macroeconomic policy has two basic goals: A) internal and external balance. B) internal and foreign balance. C) external and domestic balance. D) domestic and foreign balance. Answer: A Difficulty: Easy AACSB: Application of knowledge 3) By internal balance, most economists mean A) full employment. B) price stability. C) full employment and domestic price level stability. D) full employment and moderate increase in prices. E) full employment and high disposable income. Answer: C Difficulty: Easy AACSB: Application of knowledge 4) By external balance, most economists mean A) avoiding excessive imbalances in international payments. B) balance between exports and imports. C) balance between the trade and service accounts. D) what amounts to fixed exchange rates. E) imbalance in internal transactions. Answer: A Difficulty: Easy AACSB: Application of knowledge
1 Copyright © 2022 Pearson Education, Inc.
5) ________ can lead to general price level movements that reduce the economy's efficiency by making the real value of the monetary unit less certain and thus a less useful guide for economic decisions. A) Under- and overemployment B) Full employment C) Cyclical employment D) Seasonal employment Answer: A Difficulty: Easy AACSB: Application of knowledge 6) Which one of the following statements is TRUE? A) Inflation but not deflation can occur even under conditions of full employment. B) Deflation but not inflation can occur even under conditions of full employment. C) Inflation or deflation can occur even under conditions of full employment. D) Inflation can occur even under conditions of full employment only in the long run. E) Inflation does not coincide with periods of high unemployment levels. Answer: C Difficulty: Easy AACSB: Application of knowledge 7) A country seeking to maintain internal balance would be concerned A) only with attaining low levels of unemployment. B) primarily with ensuring that saving is weighted more towards domestic investment than the current account. C) with large fluctuations in output or wage prices. D) with maintaining an adequate stock of gold reserves. E) with stabilizing employment levels globally. Answer: C Difficulty: Easy AACSB: Application of knowledge 8) Inflation can occur under conditions of full employment A) only if the central bank continues to inject money into the economy and the agents' expectations of inflation are supported by the bank's activities. B) only if the central bank continues to inject money into the economy. C) only if the central bank continues to withdraw money from the economy. D) only if the central bank continues to inject money into the economy and all agents expect that inflation will not occur. E) only if the central bank fails to inject money into the economy. Answer: A Difficulty: Easy AACSB: Application of knowledge
2 Copyright © 2022 Pearson Education, Inc.
9) A sudden increase in the U.S. price level A) makes those with dollar debts worse off. B) makes those with dollar debts better off. C) does not affect those with dollar debts. D) makes those with foreign debts better off. E) increases all dollar debts. Answer: B Difficulty: Easy AACSB: Application of knowledge 10) A sudden increase in the U.S. price level A) makes creditors in dollars better off. B) makes creditors in dollars worse off. C) does not affect creditors in dollars. D) makes creditors in DM worse off. E) makes lenders worse off. Answer: B Difficulty: Easy AACSB: Application of knowledge 11) A sudden decrease in the U.S. price level A) makes those with dollar debts worse off. B) makes those with dollar debts better off. C) does not affect those with dollar debts. D) makes those with DM worse off. E) makes creditors worse off. Answer: A Difficulty: Easy AACSB: Application of knowledge 12) A sudden decrease in the U.S. price level A) makes creditors in dollars better off. B) makes creditors in dollars worse off. C) does not affect creditors in dollars. D) makes creditors in DM better off. E) makes those with dollar debts better off. Answer: A Difficulty: Easy AACSB: Application of knowledge
3 Copyright © 2022 Pearson Education, Inc.
13) The costs of inflation have been most apparent in the post-war period in countries like A) Argentina. B) Belgium. C) the United States. D) Canada. E) Japan. Answer: A Difficulty: Easy AACSB: Application of knowledge 14) The costs of inflation have been most apparent in the post-war period in countries like A) Brazil. B) Belgium. C) the United States. D) Canada. E) Japan. Answer: A Difficulty: Easy AACSB: Application of knowledge 15) The costs of inflation have been most apparent in the post-war period in countries like A) Serbia. B) Belgium. C) the United States. D) Canada. E) Japan. Answer: A Difficulty: Easy AACSB: Application of knowledge 16) The costs of inflation have been most apparent in the post-war period in countries like A) Zimbabwe. B) Belgium. C) the United States. D) Canada. E) Japan. Answer: A Difficulty: Easy AACSB: Application of knowledge
4 Copyright © 2022 Pearson Education, Inc.
17) External balance as the balance in a country's current account A) is appropriate in some circumstances, and not appropriate as a general rule. B) is always appropriate as a general rule. C) is inappropriate in all circumstances. D) is appropriate in all circumstances. Answer: A Difficulty: Easy AACSB: Application of knowledge 18) Trade of consumption over time is called A) intertemporal trade. B) future trade. C) consumption trade. D) time trade. Answer: A Difficulty: Easy AACSB: Application of knowledge 19) A current account surplus A) poses a problem if domestic savings are being invested more profitably abroad than they would be at home. B) may pose no problem if domestic savings are being invested more profitably abroad than they would be at home. C) may pose no problem if domestic savings are being invested less profitably abroad than they would be at home. D) there is no relation between current account surplus and between savings and investment. E) poses a problem if domestic savings are being invested less profitably abroad than they would be at home. Answer: B Difficulty: Easy AACSB: Application of knowledge 20) A current account deficit A) will not pose a problem, especially if it is accompanied by an expansionary fiscal policy. B) may pose no problem if the borrowed funds are channeled into productive domestic investment projects that pay for themselves with the revenue they generate in the future. C) may still pose a problem, even if the borrowed funds are channeled into productive domestic investment projects. D) there is no relation between current account surplus and between savings and investment. E) will pose a problem because the country is borrowing funds from the rest of the world that it won't be able to pay back later. Answer: B Difficulty: Easy AACSB: Application of knowledge
5 Copyright © 2022 Pearson Education, Inc.
21) Which one of the following statements is TRUE? A) Countries with strong investment opportunities should invest little at home and channel their savings into more productive investment activity abroad. B) Countries with weak investment opportunities should invest little at home and channel their savings into more productive investment activity abroad. C) Countries with weak investment opportunities should invest more at home. D) Countries with weak investment opportunities should invest little abroad. E) Countries with weak investment opportunities should invest little abroad and channel their savings into more productive investment activity domestically. Answer: B Difficulty: Easy AACSB: Application of knowledge 22) Countries with A) strong investment opportunities should invest little at home and channel their savings into more productive investment activity abroad. B) strong investment opportunities should invest more at home and less abroad. C) weak investment opportunities should invest more at home. D) weak investment opportunities should invest little abroad. E) countries with productive investment should invest exclusively at home. Answer: B Difficulty: Easy AACSB: Application of knowledge 23) Countries where investment is relatively A) productive should be net exporters of currently available output. B) unproductive should be net importers of currently available output. C) unproductive should be net exporters of currently available output. D) unproductive should be net exporters of future available output. E) unproductive should focus on their internal balance. Answer: C Difficulty: Easy AACSB: Application of knowledge 24) Countries where investment is relatively A) unproductive should have current account deficits. B) unproductive should have current account surpluses. C) productive should have current account surpluses. D) productive should have low outputs. Answer: B Difficulty: Easy AACSB: Application of knowledge
6 Copyright © 2022 Pearson Education, Inc.
25) Which one of the following statements is TRUE? A) Countries where investment is relatively productive should be net importers of current output. B) Countries where investment is relatively unproductive should be net importers of current output. C) Countries where investment is relatively productive should be net exporters of current output. D) Countries where investment is relatively productive should not export or import current output. E) Countries where investment is relatively unproductive should invest at home. Answer: A Difficulty: Easy AACSB: Application of knowledge 26) Countries where investment is A) relatively unproductive should have current account deficits. B) relatively productive should have current account surpluses. C) relatively productive should have current account deficits. D) relatively productive should have balanced current accounts. E) relatively unproductive should have balanced current accounts. Answer: C Difficulty: Easy AACSB: Application of knowledge 27) A large current account deficit caused by an expansionary fiscal policy that does not simultaneously make domestic investment opportunities more profitable may signal a need for the government to A) restore external balance by changing its economic course. B) restore internal balance by lowering taxes even more. C) restore internal balance by increasing government spending even more. D) restore external balance by promoting more imports. Answer: A Difficulty: Easy AACSB: Application of knowledge 28) Every open economy faces ________ that limits its spending over time to levels that allow it to pay the interest and principal on its foreign debts. A) an intertemporal budget constraint B) no budget constraint C) no international or national regulation D) an international annual budget agreement Answer: A Difficulty: Easy AACSB: Application of knowledge
7 Copyright © 2022 Pearson Education, Inc.
29) When countries begin to have trouble meeting their payments on past foreign loans, foreign creditors become reluctant to lend them new funds and may even demand immediate repayment of the earlier loans. Economists refer to such an event as a sudden ________ in foreign lending. A) stop B) penalty C) punishment D) pause Answer: A Difficulty: Easy AACSB: Application of knowledge 30) Governments prefer to avoid excessive current account surpluses because A) the returns to domestic savings are more difficult to tax than those on assets abroad. B) an addition to the home capital stock may increase domestic unemployment and therefore lead to higher national income. C) foreign investment in one firm may have beneficial technological spillover effects on other foreign producers that the investing firm does not capture. D) an addition to the home capital stock may reduce domestic unemployment and therefore lead to higher national income. E) domestic savings increase with more investment abroad. Answer: D Difficulty: Easy AACSB: Application of knowledge 31) Governments prefer to avoid excessive current account surpluses because A) the returns to domestic savings are easier to tax than those on assets abroad. B) the returns to domestic savings are more difficult to tax than those on assets abroad. C) an addition to the home capital stock may increase domestic unemployment and therefore lead to higher national income. D) foreign investment in one firm may have beneficial technological spillover effects on other foreign producers that the investing firm does not capture. E) domestic savings increase with more investment abroad. Answer: A Difficulty: Easy AACSB: Application of knowledge 32) The case of New Zealand, described in the text, is concerned with the country's A) prospects for long term growth. B) ability to sustain current account deficits. C) unproductive industrial sector and its prospects for long-run growth. D) labor productivity. E) exchange rate volatility relative to other currencies. Answer: B Difficulty: Easy AACSB: Application of knowledge
8 Copyright © 2022 Pearson Education, Inc.
33) New Zealand's net debt to foreign lenders stands at around ________ percent of its national output. A) 50 B) 40 C) 30 D) 20 E) 10 Answer: A Difficulty: Easy AACSB: Application of knowledge 34) "The line distinguishing external from internal goals can be fuzzy." Discuss. Answer: This statement is true. For example, employment target for export industries when export growth influences the ability of the economy to repay its foreign debts. Difficulty: Moderate AACSB: Application of knowledge 35) Why do governments prefer to avoid current account deficits that are too large? Answer: A current account deficit may pose no problem if the borrowed funds are channeled into productive domestic investment projects that pay for themselves with the revenue they generate in the future. However, sometimes, large current account deficits represent temporarily high consumption resulting from misguided government policies or some other malfunctioning of the economy. Sometimes the investment projects that draw on foreign funds may be badly planned and based on overoptimistic expectations about future profitability. In such cases, the government might wish to reduce the current account deficit immediately rather than face problems in repaying its foreign debt in the future. A large current account deficit can undermine foreign investors' confidence and contribute to a sudden stop. In the event of a sudden stop, moreover, the larger the initial deficit, the larger and more painful the fall in domestic spending that is needed to make the economy live strictly within its means. Difficulty: Moderate AACSB: Application of knowledge 36) What is a sudden stop in foreign lending and its purpose? Answer: A sudden stop in foreign lending happens when countries begin to have trouble meeting their payments on past foreign loans, foreign creditors become reluctant to lend them new funds and may demand immediate repayment of the earlier loans. In such cases, the purpose is for home government to take severe action to reduce the country's desired borrowing from foreigners to feasible levels as well as to repay maturing loans that foreigners are unwilling to renew. Difficulty: Easy AACSB: Application of knowledge
9 Copyright © 2022 Pearson Education, Inc.
37) Why do governments prefer to avoid excessive current account surpluses? Or, why are growing domestic claims to foreign wealth ever a problem? Answer: For a given level of national saving, an increased current account surplus implies lower investment in domestic plant and equipment. A few reasons why: first, the returns to domestic savings may be easier to tax than those on assets abroad; second, an addition to the home capital stock may reduce domestic unemployment and therefore lead to higher national income; third, domestic investment by one firm may have beneficial technological spillover effects on other domestic producers that the investing firm does not capture. In addition, the country may in the future find itself unable to collect the money it is owed. Furthermore, countries with large surpluses can become targets for discriminatory protectionist measures by trading partners with external deficits. Difficulty: Moderate AACSB: Application of knowledge 38) Using an equation, explain why governments prefer to avoid excessive current account surpluses? Answer: This follows from the national income identity, S = CA + I, which says that total domestic savings, S, is divided between foreign asset accumulation, CA, and domestic investment, I. Difficulty: Moderate AACSB: Application of knowledge 39) The case of New Zealand, described in the text, asks what question about the country's international debt position? Answer: Fundamentally, the question is whether or not a country can sustain a current account deficit indefinitely. The answer is that, under certain conditions, yes it can. Difficulty: Moderate AACSB: Reflective thinking 40) The case of New Zealand, as described in the text, draws what simple conclusion regarding the country's international debt position? Answer: Fundamentally, the question is whether or not a country can sustain a current account deficit indefinitely. The conclusion is that, under certain conditions, yes it can. Difficulty: Moderate AACSB: Reflective thinking 41) The case of New Zealand, described in the text, draws what technical conclusion regarding the country's international debt position? Answer: Fundamentally, the question is whether or not a country can sustain a current account deficit indefinitely. The conclusion depends upon the country's future prospects. If net exports are expected to rise at a rate sufficient to counteract the effects of debt (including interest payments) over the relevant time period, then deficits could go on for an extended period. Difficulty: Moderate AACSB: Reflective thinking
10 Copyright © 2022 Pearson Education, Inc.
19.2
Classifying Monetary Systems: The Open-Economy Monetary Trilemma
1) Which of the following is one component of the "trilemma" that is faced by policy makers in choosing monetary arrangements? A) exchange rate stability B) restrictions on international capital movements C) tariffs and subsidies D) restrictions on the migration of labor E) global inflation Answer: A Difficulty: Easy AACSB: Application of knowledge 2) Which of the following is one component of the "trilemma" that is faced by policy makers in choosing monetary arrangements? A) freedom of international capital movements B) exchange rate instability C) tariffs and subsidies D) restrictions on the migration of labor E) global inflation Answer: A Difficulty: Easy AACSB: Application of knowledge 3) Which of the following is one component of the "trilemma" that is faced by policy makers in choosing monetary arrangements? A) monetary policy oriented towards domestic goals B) exchange rate instability C) tariffs and subsidies D) restrictions on the migration of labor E) global inflation Answer: A Difficulty: Easy AACSB: Application of knowledge
11 Copyright © 2022 Pearson Education, Inc.
4) What is the nature of the trilemma that is encountered when choosing monetary arrangements? A) Only two of the three aspects of internal and external balance can be accommodated simultaneously. B) Only three of the four aspects of internal and external balance can be accommodated simultaneously. C) Only one of the three aspects of internal and external balance can be accommodated simultaneously. D) Only two of the four aspects of internal and external balance can be accommodated simultaneously. E) Only one of the four aspects of internal and external balance can be accommodated simultaneously. Answer: A Difficulty: Easy AACSB: Application of knowledge 5) What are the components of the trilemma that is encountered when a country chooses its monetary policy and what is the meaning of the term? Answer: The components are (1) exchange rates, (2) domestic goals, and (3) international capital movements. The monetary trilemma (a three-part dilemma) exists because only two of the three components can be influenced by monetary policy. Difficulty: Moderate AACSB: Application of knowledge 19.3
International Macroeconomic Policy under the Gold Standard, 1870-1914
1) The gold standard had its origin in the use of gold coins as A) a medium of exchange, unit of account, and store of value. B) an input for fine jewelry. C) a medium of exchange. D) unit of account. E) store of value. Answer: A Difficulty: Easy AACSB: Application of knowledge 2) In which country was the gold standard established as a legal institution in 1819? A) Great Britain B) India C) China D) Australia Answer: A Difficulty: Easy AACSB: Application of knowledge
12 Copyright © 2022 Pearson Education, Inc.
3) When did the United States, Germany, and Japan adopt the gold standard? A) later in the 19th century B) later in the 18th century C) later in the 20th century D) later in the 21st century Answer: A Difficulty: Easy AACSB: Application of knowledge 4) Under the gold standard era of 1870-1914 A) Tokyo was the center of the international monetary system. B) Paris was the center of the international monetary system. C) Berlin was the center of the international monetary system. D) New York was the center of the international monetary system. E) London was the center of the international monetary system. Answer: E Difficulty: Easy AACSB: Application of knowledge 5) Under the gold standard, the primary responsibility of a central bank was to A) fix the exchange rate between its currency and gold. B) manage the flow of gold in and out of the country. C) float the exchange rate between its currency and gold. D) keep gold in national reserves safe. Answer: A Difficulty: Easy AACSB: Application of knowledge 6) Under the gold standard era of 1870-1914 A) central banks tried to have sharp fluctuations in the balance of payments. B) central banks tried to avoid sharp fluctuations in the current account of the balance of payments. C) central banks tried to avoid sharp fluctuations in the trade account of the balance of payments. D) central banks tried to avoid sharp fluctuations in the capital account of the balance of payments. E) central banks tried to avoid sharp fluctuations in the balance of payments. Answer: E Difficulty: Easy AACSB: Application of knowledge
13 Copyright © 2022 Pearson Education, Inc.
7) A country's balance of payment is A) the difference between the current plus capital account balances and the balance of net nonreserved financial flows abroad. B) the difference between export and import payments. C) the difference between a country's domestic assets and foreign liabilities. D) the difference between the outflow and inflow of gold shipments between the central banks. Answer: A Difficulty: Easy AACSB: Application of knowledge 8) A country is said to be in balance of payments equilibrium, when the sum of its current and its A) non-reserved capital accounts equals zero. B) reserved capital accounts equals zero. C) non-reserved capital accounts equals to the surplus in the capital account. D) non-reserved capital accounts equals to the deficit in the capital account. E) non-reserved capital accounts' balance is higher than the total capital account balance. Answer: A Difficulty: Easy AACSB: Application of knowledge 9) Under the Gold standard, a country is said to be in balance of payments equilibrium when the current account balance is A) financed entirely by international lending without reserve movements. B) financed by international lending and with reserve movements. C) equal to zero. D) financed entirely by international lending and past gold reserves. E) financed entirely by gold reserves. Answer: A Difficulty: Easy AACSB: Application of knowledge 10) The price-specie-flow mechanism A) is an automatic mechanism for assuring external balance under floating exchange rates. B) is an automatic mechanism for assuring external balance under the gold standard. C) is an automatic mechanism for assuring internal balance under floating exchange rates. D) is an automatic mechanism for assuring internal balance under the gold standard. E) is an automatic mechanism for assuring internal balance under mercantilism. Answer: B Difficulty: Moderate AACSB: Application of knowledge
14 Copyright © 2022 Pearson Education, Inc.
11) The price-specie-flow mechanism is a model developed by ________ to recognize the powerful automatic mechanism that contributes to the simultaneous achievement of balance of payment by all countries. A) David Hume B) Keynes C) Frank Baum D) Jeffrey Frieden Answer: A Difficulty: Easy AACSB: Application of knowledge 12) Under the price-specie-flow mechanism, what happens when, say, Great Britain's current plus capital account surplus is greater than its non-reserve financial account balance? A) Great Britain's loans will finance all foreign net imports. B) An automatic drop in Great Britain's domestic prices and rise in foreign prices. C) Gold reserves will flow into Great Britain. D) Gold reserves will flow out of Great Britain. E) Great Britain will experience a deficit. Answer: C Difficulty: Moderate AACSB: Application of knowledge 13) The "rules of the game" under the gold standard can best be described as which of the following? A) selling domestic assets in a deficit and buying assets in a surplus B) slowing down the automatic adjustments processes inherent in the gold standard C) selling domestic assets in order to accumulate gold D) selling foreign assets in a deficit and buying foreign assets in a surplus E) selling domestic assets in a surplus Answer: A Difficulty: Easy AACSB: Application of knowledge 14) Until the United States Civil War, The Unites States had a A) gold-based monetary standard. B) silver-based monetary standard. C) bimetallic monetary standard consisting of silver and gold. D) bimetallic monetary standard consisting of copper and silver. E) bimetallic monetary standard consisting of copper and gold. Answer: C Difficulty: Easy AACSB: Application of knowledge
15 Copyright © 2022 Pearson Education, Inc.
15) Which of the following statements is TRUE? A) The gold standard allowed high degrees of exchange rate stability and international financial capital mobility but did not allow monetary policy to pursue internal policy goals. B) The gold standard allowed high degrees of exchange rate stability, international financial capital mobility and monetary policy to pursue internal policy goals. C) The gold did not allow high degrees of exchange rate stability, international financial capital mobility, and monetary policy to pursue internal policy goals. D) The gold standard allowed monetary policy to pursue internal policy goals, but did not allow high degrees of exchange rate stability and international financial capital mobility. Answer: A Difficulty: Easy AACSB: Application of knowledge 16) Until the Civil War, the United States had A) bimetallic monetary standard consisting of silver and gold. B) gold standard. C) silver standard. D) paper currency, called the "greenback." Answer: A Difficulty: Easy AACSB: Application of knowledge 17) Once the United States Civil War broke out, the United States moved to a A) gold standard. B) silver standard. C) bimetallic monetary standard consisting of silver and gold. D) bimetallic monetary standard consisting of copper and gold. E) paper currency, called the "greenback." Answer: E Difficulty: Easy AACSB: Application of knowledge 18) L. Frank Baum's classic 1900 children's book, The Wonderful Wizard of Oz, is A) an allegorical rendition of the U.S. political struggle over silver. B) an allegorical rendition of the U.S. political struggle over copper. C) an allegorical rendition of the U.S. political struggle over both silver and gold. D) an allegorical rendition of the U.S. political struggle over indebted farmers. E) an allegorical rendition of the U.S. political struggle over gold. Answer: E Difficulty: Easy AACSB: Application of knowledge
16 Copyright © 2022 Pearson Education, Inc.
19) In L. Frank Baum's classic 1900 children's book, The Wonderful Wizard of Oz, the name "oz" is a reference to A) an ounce (oz.) of gold. B) an ounce (oz.) of silver. C) an ounce (oz.) of copper. D) an ounce (oz.) of gold or silver. E) an ounce (oz.) of wheat. Answer: A Difficulty: Easy AACSB: Application of knowledge 20) How did the international monetary system influence macroeconomic policy-making and performance during the gold standard era (1870-1914)? Answer: London was the center of the international monetary system. The primary responsibility of the central bank was to preserve the official parity between its currency and gold. To maintain this price, the central bank needed an adequate stock of gold reserves. Central banks tried to avoid sharp fluctuations in the balance of payments. Difficulty: Moderate AACSB: Application of knowledge 21) It is claimed that L. Frank Baum's classic 1900 children's book, The Wonderful Wizard of Oz, is an allegorical rendition of the U.S. political struggle over gold. Answer: This statement is true. In L. Frank Baum's classic 1900 children's book, The Wonderful Wizard of Oz, the name "oz" is a reference to an ounce (oz.) of gold, and the yellow brick road represents the false promise of gold. The story represents the struggle of farmers in the western U.S. who were heavily indebted. Difficulty: Easy AACSB: Application of knowledge 22) Explain how the price-specie-flow mechanism works under the gold standard. Answer: When a country with a gold standard had a positive balance of trade, gold would flow into the country in the amount that the value of exports exceeds the value of imports. Conversely, when such a country had a negative balance of trade, gold would flow out of the country in the amount that the value of imports exceeds the value of exports. These gold flows automatically influence the domestic and foreign money supply, change domestic and foreign price of goods and services, therefore reducing the domestic trade surplus/deficit so that all countries reach balance of payment equilibrium. Difficulty: Moderate AACSB: Application of knowledge
17 Copyright © 2022 Pearson Education, Inc.
23) Why "the rules of the game" of the gold standard were frequently violated before 1914? Answer: The incentives to obey the rules applied with greater force to deficit than to surplus countries, so in practice it was the deficit countries that bore the burden of bringing the payments balances of all countries into equilibrium. By not always taking action to reduce gold inflows, the surplus countries worsened a problem of international policy coordination inherent in the system: Deficit countries competing for a limited supply of gold reserves might adopt overly contractionary monetary policies that harmed employment while doing little to improve their reserve positions. Difficulty: Moderate AACSB: Application of knowledge 19.4
The Interwar Years, 1918-1939
1) The gold standard period was A) up until the first world war. B) between the first and second world wars. C) following the second world war until 1970. D) between 1954 and 1970. E) between 1814 and 1865. Answer: A Difficulty: Easy AACSB: Application of knowledge 2) Which of the following statements is TRUE? A) Governments effectively suspended the gold standard during World War I and financed part of their massive military expenditures by printing money. B) Governments continued with the gold standard during World War I and used gold to finance their massive military expenditures. C) Governments effectively suspended the gold standard during World War I and financed part of their massive military expenditures by borrowing money from other countries. D) Governments continued with the gold standard during World War I and increased gold reserves to finance reconstruction process. Answer: A Difficulty: Easy AACSB: Application of knowledge 3) Britain returned to the gold standard in ________ then left again in ________ when foreign holders of sterling lost confidence in Brian's promise to maintain its currency's value and began converting their sterling to gold. A) 1925; 1931 B) 1919; 1922 C) 1922; 1925 D) 1925; 1929 Answer: A Difficulty: Easy AACSB: Application of knowledge 18 Copyright © 2022 Pearson Education, Inc.
4) The U.S. left the gold standard in ________ then returned in ________. A) 1933; 1934 B) 1925; 1931 C) 1919; 1933 D) 1925; 1929 Answer: A Difficulty: Easy AACSB: Application of knowledge 5) A policy of "beggar-thy-neighbor" is a policy that A) often benefits the home country in the long run. B) often benefits the foreign country in the long run. C) often benefits the foreign country in the short run. D) does not often benefit any country in the long run. E) benefits the home country's neighbors in the long run. Answer: D Difficulty: Easy AACSB: Application of knowledge 6) A measure that raises domestic welfare is called a ________ when it benefits the home country at the cost of worsening economic conditions abroad. A) beggar-thy-neighbor policy B) price-specie-flow mechanism C) prosper-thy-neighbor policy D) the wizard of oz mechanism Answer: A Difficulty: Easy AACSB: Application of knowledge 7) The world economy ________ into increasingly ________ national units in the early 1930s. A) disintegrated; autarkic B) integrated; interdependent C) disintegrated; dependent D) integrated; self-sufficient Answer: A Difficulty: Easy AACSB: Application of knowledge
19 Copyright © 2022 Pearson Education, Inc.
8) Which of the following statements is TRUE? A) All countries would have been better off in a world with freer international trade, provided international cooperation had helped each country preserve its external balance and financial stability without sacrificing internal policy goals. B) All countries would have been worse off in a world with freer international trade, provided international cooperation had helped each country preserve its external balance and financial stability without sacrificing internal policy goals. C) All countries would have been better off in a world with higher trade barriers and less international cooperation. D) All countries would have been worse off in a world with freer international trade and more international cooperation. Answer: A Difficulty: Easy AACSB: Application of knowledge 9) The Great Depression that started in 1929 was A) confined only to the United States. B) confined only to the United States and Britain. C) confined only to the United States and Europe. D) a global phenomenon. E) confined only to the Americas. Answer: D Difficulty: Easy AACSB: Application of knowledge 10) Which one of the following statements is the MOST accurate? By the year 1932, the United States A) and Canada alone held more than 70 percent of the world's monetary gold. B) and Germany alone held more than 70 percent of the world's monetary gold. C) and Britain alone held more than 70 percent of the world's monetary gold. D) Britain, and France alone held more than 70 percent of the world's monetary gold. E) and France alone held more than 70 percent of the world's monetary gold. Answer: E Difficulty: Easy AACSB: Application of knowledge 11) Countries that abandoned the gold standard freed themselves to adopt more expansionary monetary policies that limited (or prevented) both A) domestic deflation and output contraction. B) domestic inflation and output expansion. C) domestic deflation and output expansion. D) domestic inflation and output contraction. Answer: A Difficulty: Easy AACSB: Application of knowledge
20 Copyright © 2022 Pearson Education, Inc.
12) Countries with the A) biggest deflations and output contractions are countries which were never on the gold standard until 1936. B) biggest inflations and output contractions are countries which were on the gold standard until 1936. C) lowest deflations and output contractions are countries which were on the gold standard until 1936. D) biggest deflations and output increases are countries which were on the gold standard until 1936. E) biggest deflations and output contractions are countries which stayed on the gold standard until 1936. Answer: E Difficulty: Easy AACSB: Application of knowledge 13) How did the international monetary system influence macroeconomic policy-making and performance during the interwar period (1918-1939)? Answer: Governments effectively suspended the gold standard during World War I and financed part of their massive military expenditures by printing money. Further, labor forces and productivity capacity had been reduced sharply through war losses. As a result, price levels were higher everywhere at the conclusion of the war in 1918. Of special note is the German hyperinflation that occurred when prices in Germany increased by a factor of 481.5 billion! The United States returned to gold in 1919. In 1922, at a conference in Genoa, Italy, a group of countries including Britain, France, Italy and Japan agreed on a program of a partial gold exchange standard in which smaller countries could hold as reserves the currencies of several large countries whose own international reserves would consist entirely of gold. In 1925, Britain returned to the gold standard by pegging the pound to gold at the prewar price. Thus, the Bank of England was therefore forced to follow contractionary monetary policies that contributed to severe unemployment and to the decline of London as the leading financial center. The world economy disintegrated into increasingly autarkic (self-sufficient) national units in the early 1930s. Difficulty: Difficult AACSB: Application of knowledge 14) Describe the effects of the Smoot-Hawley tariff imposed by the United States in 1930. Answer: It had a damaging effect on employment abroad. The foreign response involved retaliatory trade restrictions and preferential trading arrangements among a group of countries. This is an example of "beggar-thy-neighbor" policy, meaning a policy that benefits the home country only because it worsens economic conditions abroad. Difficulty: Moderate AACSB: Application of knowledge 15) What explains the nearly universal scope of the Great Depression? Answer: The international gold standard played a central role in starting, deepening, and spreading the Great Depression. Difficulty: Moderate AACSB: Application of knowledge 21 Copyright © 2022 Pearson Education, Inc.
19.5
The Bretton Woods System and the International Monetary Fund
1) What was signed by representatives of 44 countries in Bretton Woods, New Hampshire in July 1944? A) the Articles of Agreement of the International Monetary Fund (IMF) B) the Articles of Agreement of the World Bank (WB) C) the Articles of Agreement of the World Trade Organization (WTO) D) the General Agreement on Tariffs and Trade (GATT) Answer: A Difficulty: Easy AACSB: Application of knowledge 2) Countries singed the Articles of Agreement in Bretton Woods with a hope to A) design an international monetary system that would foster full employment and price stability while allowing individual countries to attain external balance without restrictions on international trade. B) design an international system that would enforce stricter restrictions on international trade. C) design an international tax system that would increase taxes on imports and exports. D) design an international monetary system that would allow floating exchange rates. Answer: A Difficulty: Easy AACSB: Application of knowledge 3) The IMF's Articles of Agreement forced the U.S. to exchange gold for dollars at what price? A) $25/ ounce B) $35/ ounce C) $45/ ounce D) $55/ ounce E) $20/ ounce Answer: B Difficulty: Easy AACSB: Application of knowledge 4) Which of the two features of the IMF Articles of Agreement helped promote flexibility in external adjustment? A) IMF members helped countries maintain full employment. B) IMF allowed countries to attain internal balance. C) New countries would enter the agreement if they fixed their exchange rate. D) IMF members contributed their currency to form a pool of resources that IMF could lend to countries in need and parities in the exchange rate against the dollar could be adjusted with agreement of IMF. E) IMF members argued against the use of floating exchange rates. Answer: D Difficulty: Easy AACSB: Application of knowledge
22 Copyright © 2022 Pearson Education, Inc.
5) The IMF Articles of Agreement, through a mixture of discipline and flexibility, hoped to A) avoid a repetition of the turbulent interwar experience. B) enforce stricter restrictions on international trade. C) increase taxes on imports and exports. D) promote floating exchange rates. Answer: A Difficulty: Easy AACSB: Application of knowledge 6) A convertible currency is a currency that may be freely exchanged for A) domestic assets. B) only silver. C) only copper. D) national currency. E) foreign currencies. Answer: E Difficulty: Easy AACSB: Application of knowledge 7) The dollar of the United States became the postwar world's key currency because of all EXCEPT A) the early convertibility of the U.S. dollar in 1945. B) the special position of the dollar under the Bretton Woods system. C) the strength of the American economy relative to the devastated economies of Europe and Japan. D) central banks naturally found it advantageous to hold their international reserves in the form of interest-bearing dollar assets. E) the ease of transporting U.S. dollars compared with other currencies. Answer: E Difficulty: Moderate AACSB: Application of knowledge 8) With growing opportunities to move funds across borders after the restoration of the convertibility, national interest rates became more A) linked. B) isolated. C) disconnected. D) independent. Answer: A Difficulty: Easy AACSB: Application of knowledge
23 Copyright © 2022 Pearson Education, Inc.
9) A person holding dollar deposits during the devaluation of the dollar would A) enjoy a monetary gain. B) see the foreign currency value of dollar assets increase by the amount of the exchange rate change. C) shift their wealth into domestic investments. D) suffer a monetary loss and see the foreign currency value of dollar assets decrease by the amount of the exchange rate change. E) see no change in their investments. Answer: D Difficulty: Moderate AACSB: Application of knowledge 10) Countries with large current account surpluses might be viewed by the market as candidates for A) devaluation. B) revaluation. C) bankruptcy. D) depreciation. E) investment. Answer: B Difficulty: Easy AACSB: Application of knowledge 11) Countries with large and persistent current account deficits might be viewed by the market as candidates for A) devaluation. B) revaluation. C) bankruptcy. D) depreciation. E) investment. Answer: A Difficulty: Easy AACSB: Application of knowledge 12) The Bretton Woods structure of fixed exchange rates was brought down eventually because of A) the balance of payments crises. B) the border crises. C) the financial crisis. D) The debt crisis. Answer: A Difficulty: Easy AACSB: Application of knowledge
24 Copyright © 2022 Pearson Education, Inc.
13) How did the international monetary system influence macroeconomic policy-making and performance during the post-World War II years during which exchange rates were fixed under the Bretton Woods agreement (1946-1973)? Answer: In July 1944, representatives of 44 countries met in Bretton Woods, New Hampshire, and drafted and signed the Articles of Agreement of the International Monetary Fund (IMF) and of the World Bank. The agreement established fixed exchange rates against the U.S. dollar and an unvarying dollar price of gold-$35 an ounce. The dollar of the United States became the postwar world's key currency because of a few factors: (1) The early convertibility of the U.S. dollar in 1945 (2) The special position of the dollar under the Bretton Woods system (3) The strength of the American economy relative to the devastated economies of Europe and Japan (4) Central banks naturally found it advantageous to hold their international reserves in the form of interest-bearing dollar assets. The Marshall Plan, a program of dollar grants from the United States to European countries, was initiated in 1948. Most countries in Europe did not restore convertibility until the end of 1958, with Japan following in 1964. Difficulty: Easy AACSB: Application of knowledge 14) How did the Bretton Woods system resolve the trilemma? Answer: The system was based on the presumption that movements of private financial capital could be restricted, allowing some degree of independence for domestically oriented monetary policies. The new system thus was diametrically opposed to the gold standard's subordination of monetary policy to external considerations such as freedom of financial flows. After the experience of high interwar unemployment, the architects of the Bretton Woods system hoped to ensure that countries would not be forced to adopt contractionary monetary policies for balance of payments reasons in the face of an economic downturn. Difficulty: Moderate AACSB: Application of knowledge 15) What is a convertible currency? Answer: A convertible currency is a currency that may be freely exchanged for foreign currencies. Difficulty: Easy AACSB: Application of knowledge
25 Copyright © 2022 Pearson Education, Inc.
16) Explain why the United States dollar became the postwar world's key currency. Answer: (1) The early convertibility of the U.S. dollar in 1945. (2) The special position of the dollar under the Bretton Woods system. (3) The strength of the American economy relative to the devastated economies of Europe and Japan. (4) Central banks naturally found it advantageous to hold their international reserves in the form of interest-bearing dollar assets. Difficulty: Easy AACSB: Application of knowledge 17) How did the international monetary system created at Bretton Woods in 1944 allow its members to reconcile their external commitments with their internal goals of full employment and price stability? Answer: The system set up by the Bretton Woods Agreement called for fixed exchange rates against the U.S. dollar and an unvarying dollar price of gold–$35 an ounce. Member countries held their official international reserves largely in the form of gold or dollar assets and had the right to sell dollars to the Federal Reserve for gold at the official price.Therefore, it provided a stable exchange rate environment that nurtured the reconstruction of the world economy and the growth of international trade and finance. Difficulty: Moderate AACSB: Application of knowledge 18) Discuss the impact of the restoration of convertibility in 1958. Answer: As foreign exchange trading expanded, financial markets in different countries became more tightly integrated, an important step towards the creation of today's worldwide foreign exchange market. With growing opportunities to move funds across borders, national interest rates became more closely linked and the speed with which policy changes might cause a country to lose or gain international reserves increased. Difficulty: Moderate AACSB: Application of knowledge 19) Explain how a country with a current account deficit is a ripe candidate for currency devaluation. Answer: If, for example, Great Britain had a current account deficit, the holders of pounds would become nervous and shift their wealth into other currencies. In order to hold the pound's exchange rate against the dollar pegged, the Bank of England would have to buy pounds and supply the foreign assets that market participants wished to hold. This resulting loss in foreign reserves, if large enough, would most likely force a devaluation by leaving the Bank of England without enough reserves to prop up the exchange rate. Difficulty: Moderate AACSB: Application of knowledge
26 Copyright © 2022 Pearson Education, Inc.
20) Explain how a country with a current account surplus is a ripe candidate for currency revaluation. Answer: If a country like Germany had a current account surplus, it would sell its currency in the foreign exchange market in order to keep it from appreciating. The German central banks would thus find themselves swamped with official reserves, and Germany would face the problem of having its money supply grow uncontrollably, a trend that would most likely drive up the domestic price levels and upset internal balance. Revaluation of the currency would thus be a viable solution to this problem. Difficulty: Moderate AACSB: Application of knowledge 19.6
Analyzing Policy Options for Reaching Internal and External Balance
1) Internal balance requires that A) aggregate demand equal the full employment output level. B) aggregate demand equal aggregate supply. C) domestic interest rate equal foreign interest rate. D) import supply equal export demand. Answer: A Difficulty: Easy AACSB: Application of knowledge 2) When the exchange rate, E, and the foreign price level, P*, is fixed, domestic inflation depends primarily on A) amount of aggregate demand. B) home price level set by IMF. C) current account balance. D) government tax policy. E) foreign interest rates. Answer: A Difficulty: Moderate AACSB: Application of knowledge 3) The current account surplus is A) an increasing function of disposable income and an increasing function of the real exchange rate. B) a decreasing function of disposable income and a decreasing function of the real exchange rate. C) a decreasing function of disposable income and an increasing function of the real exchange rate. D) only a decreasing function of disposable income. E) only an increasing function of the real exchange rate. Answer: C Difficulty: Easy AACSB: Application of knowledge
27 Copyright © 2022 Pearson Education, Inc.
4) Under fixed exchange rates A) monetary policy is not an effective policy. B) fiscal policy is not an effective policy. C) monetary policy and fiscal policy are not effective. D) both monetary and fiscal policies are effective. E) monetary policy has an unpredictable effect on the domestic money supply. Answer: A Difficulty: Easy AACSB: Application of knowledge 5) Fiscal expansion A) stimulates aggregate demand and causes output to decline. B) decreases aggregate demand and causes output to decline. C) stimulates aggregate demand and causes output to rise. D) decreases aggregate demand and causes output to rise. E) decreases government expenditures. Answer: C Difficulty: Easy AACSB: Application of knowledge 6) A devaluation of the home currency A) makes foreign goods and services cheaper relative to those sold at home. B) makes domestic goods and services more expensive relative to those sold abroad. C) decreases demand and output. D) increases demand for domestic goods and services. E) increases output and makes domestic goods and services cheaper relative to those sold abroad. Answer: E Difficulty: Easy AACSB: Application of knowledge 7) An attempt by a central bank to alter the money supply by buying or selling domestic assets A) will leave both domestic money supply and foreign reserves unchanged. B) will cause an offsetting change in aggregate demand. C) will lead to a rise in domestic employment and output. D) will lead to a decrease in domestic employment and output. E) will cause an offsetting change in foreign reserves and leave the domestic money supply unchanged. Answer: E Difficulty: Easy AACSB: Application of knowledge
28 Copyright © 2022 Pearson Education, Inc.
8) The condition of internal balance (full-employment output equals aggregate demand) can NOT be written as A) Yf = C + I + G. B) Yf = C + I + G + CA(EP*/P, A). C) Yf = A + CA(EP*/P, A). D) Yf = A + CA. Answer: A Difficulty: Moderate AACSB: Application of knowledge 9) Given P and P*, a rise in E makes A) domestic goods cheaper and improves the current account. B) domestic goods more expensive and worsens the current account. C) domestic goods cheaper and worsens the current account. D) foreign goods cheaper and improves the current account. Answer: A Difficulty: Moderate AACSB: Application of knowledge 10) Given P and P*, a rise in E ________ net exports and ________ the current account. A) raises; improves B) raises; worsens C) lowers; improves D) lowers; worsens Answer: A Difficulty: Moderate AACSB: Application of knowledge 11) Given P and P*, a fall in E makes A) domestic goods more expensive and worsens the current account. B) domestic goods cheaper and improves the current account. C) domestic goods cheaper and worsens the current account. D) foreign goods more expensive and worsens the current account. Answer: A Difficulty: Moderate AACSB: Application of knowledge 12) Given P and P*, a fall in E ________ net exports and ________ the current account. A) lowers; worsens B) raises; improves C) raises; worsens D) lowers; improves Answer: A Difficulty: Moderate AACSB: Application of knowledge 29 Copyright © 2022 Pearson Education, Inc.
13) The XX schedule shows how much A) fiscal expansion is needed to hold the current account surplus at X as the currency is devalued by a given amount. B) monetary expansion is needed to hold the current account surplus at X as the currency is devalued by a given amount. C) fiscal expansion is needed to hold the current account surplus at X as the currency is evaluated by a given amount. D) fiscal and monetary expansions are needed to hold the current account surplus at X as the currency is devalued by a given amount. E) foreign funding is needed to hold the current account surplus at X as the currency is devalued by a given amount. Answer: A Difficulty: Easy AACSB: Application of knowledge 14) Under fixed exchange rates, domestic asset transactions by the central bank A) can be used to alter the level of foreign reserves but not to affect the state of employment and output. B) cannot be used to alter the level of foreign reserves but only to affect the state of employment and output. C) can be used to alter the level of foreign reserves and to affect the state of employment and output. D) can be used to alter the domestic money supply and the level of foreign reserves. E) can raise output to full-employment level. Answer: A Difficulty: Easy AACSB: Application of knowledge 15) An expenditure-changing policy A) alters the direction of the economy's total demand for goods and services. B) alters the level of the economy's total demand for goods and services. C) has no effect on aggregate demand. D) is the same thing as an expenditure-switching policy. E) affects aggregate supply but not aggregate demand. Answer: B Difficulty: Easy AACSB: Application of knowledge
30 Copyright © 2022 Pearson Education, Inc.
16) An expenditure-switching policy A) changes the direction of demand, shifting it between domestic output and imports with exchange rate adjustment. B) changes the level of the economy's total demand for goods and services. C) has no effect on aggregate demand. D) is the same thing as an expenditure-changing policy. E) affects aggregate supply but not aggregate demand. Answer: A Difficulty: Easy AACSB: Application of knowledge 17) By raising the real exchange rate EP*/P, A) devaluation improves the current account and aggregate demand. B) devaluation worsens the current account and aggregate demand. C) revaluation improves the current account and aggregate demand. D) devaluation has no effect on current account and aggregate demand. Answer: A Difficulty: Easy AACSB: Application of knowledge 18) The alteration of exchange rates to move the economy to internal and external balance may lead to all EXCEPT A) a balance of payments crisis. B) changes in the terms of trade. C) changes in the level of imports or exports. D) changes in interest rates. E) a guaranteed unilateral improvement in economic wealth. Answer: E Difficulty: Moderate AACSB: Application of knowledge 19) "A monetary policy is not a policy tool under fixed exchange rates." Discuss. Answer: True, under fixed exchange rates, domestic asset transactions by the central bank can be used to alter the level of foreign reserves but not to affect the state of employment and output. Difficulty: Moderate AACSB: Application of knowledge 20) "Fiscal policy, is generally insufficient to attain the two goals of internal and external balance." Discuss. Answer: Only if the economy had been displaced horizontally from point 1 would fiscal policy be able to do the job alone. In addition, fiscal policy is an unwieldy tool, since it often cannot be implemented without legislative approval. Another drawback is that a fiscal expansion, for example, might have to be reversed after some time if it leads to chronic government budget deficits. Difficulty: Easy AACSB: Application of knowledge 31 Copyright © 2022 Pearson Education, Inc.
21) What is the difference between an expenditure-changing policy and an expenditureswitching policy? Answer: An expenditure-changing policy alters the level of the economy's total demand for goods and services. An expenditure-switching policy, on the other hand, induces an exchange rate adjustment and thus changes the direction of demand, shifting it between domestic output and imports. Difficulty: Moderate AACSB: Application of knowledge 22) (a) Assume that R denotes the domestic interest rate and R denotes the foreign interest rate. Under a fixed exchange rate what is the relation between R and R (b) Assume E denotes the domestic currency price of the dollar for a country which is not the United States. If one wants to analyze only the short-run effects of a policy, what does one assume about the Home and Foreign price levels, P and P , respectively. (c) Assume that there is no ongoing balance of payment crisis. What does this assumption really assume? (d) Assume a fixed exchange rate system. What does this tell you about E? (e) Under the above assumptions what are the conditions for internal balance? (f) How would your answer to Part D above change if P is unstable due to foreign inflation. (g) Given the definitions above, how would one define the real exchange rate? (h) Write the condition for internal balance. (i) Define the variable not defined before in Part G above. (j) Using the equation for internal balance derived above, given our assumptions analyze the effects of a fiscal expansion. (k) What would happen if the government of that country, which is not the United States under Bretton Woods, decides to devaluate its currency? (l) What would happen if the government of that country, which is not the United States under Bretton Woods, decides to use monetary policy rather than fiscal policy? (m) Given all of the above, what is the relation between the exchange rate, E, and fiscal ease, i.e., an increase in G or a reduction in T? (n) Assume that the economy is at internal balance. What will happen if G goes up for a given level of E? (o) Assume that the economy is at internal balance. What will happen if G goes down for a given level of E?
32 Copyright © 2022 Pearson Education, Inc.
Answer: (a) R = R (b) Constant prices. (c) That Ee, the expected exchange rate, is equal to the exchange rate today, E. In other words, E = Ee. (d) E is constant, i.e., E = E0. (e) Since P and E are fixed, the expected price is fixed; thus, no inflation is expected. Then, internal balance will require only full employment, aggregate demand equaling the fullemployment level, Yf. (f) In this case, full employment alone will not guarantee price stability under a fixed exchange rate. (g) The real exchange rate is equal to EP /P. (h) Yf = C(Yf - T) + I + G + CA(EP /P, Yf - T) (i) C = consumption, I = Investment, (Yf - T) = disposable income, T = taxes. (j) An increase in G or a reduction in T will increase aggregate demand and will cause output to rise in the short run. (k) A rise in E makes domestic goods and services cheaper relative to those sold abroad and thus increases demand for output. (l) A monetary policy is not a policy tool under fixed exchange rates. Under fixed exchange rates, domestic asset transactions by the central bank can be used to alter the level of foreign reserves but not to affect the state of employment and output. (m) Negative relation. (n) Over-employment. (o) Under employment. Difficulty: Moderate AACSB: Application of knowledge
33 Copyright © 2022 Pearson Education, Inc.
23) Assume that the government has a target value, X, for the current account surplus. (a) What is the goal of external balance? (b) Assume that we are dealing with only the short run, what are the values of P and P∗? (c) Given fixed P and P , what would happen if E rises? (d) Given P and P , what would happen if T decreases, i.e., an expansionary fiscal policy? (e) Given P and P , what would happen if G increases, i.e., an expansionary fiscal policy? (f) Given all of the above, what is the relation between the exchange rate, E, and fiscal ease, i.e., an increase in G or a reduction in T? (g) Assume that the economy is in external balance. What will happen if the government maintains its current account at X, but devaluates the domestic currency? (h) Assume that the economy is at external balance. What will happen if the government raises E? (i) Assume that the economy is at external balance. What will happen if the government lowers E? Answer: (a) The goal of external balance requires the government to manage fiscal policy and the exchange rate so that the following equation is satisfied: CA(EP /P, Y - T) = X. (b) Constant prices. (c) An increase in E makes domestic goods cheaper, thus improving the current account. (d) A fall in T raises output, Y. The resulting increase in output increases disposable income and thus leads to increased home spending on foreign goods, worsening the current account. (e) Similar to the answer above. A rise in G causes CA to fall by increasing Y. (f) Positive relationship. (g) The government raises E; thus, either G should go up or T should go down to maintain the external balance. (h) An increase in E raises net exports and thus leads to a surplus in the current account higher than the target level of X. This will represent a point above the XX schedule. (i) A decrease in E reduces net exports and thus leads to a deficit in the current account lower than the target level of X. This will represent a point below the XX schedule. Difficulty: Moderate AACSB: Application of knowledge
34 Copyright © 2022 Pearson Education, Inc.
24) Use a figure below to describe the four zones of economic discomfort.
Answer: The answer is given in the following figure.
Difficulty: Moderate AACSB: Application of knowledge
35 Copyright © 2022 Pearson Education, Inc.
25) Using the II-XX framework, show using a figure that fiscal policies by themselves cannot bring the economy to both internal and external balances. Answer: Starting at point 2, fiscal policy is shown as only horizontal movements. This means that the economy can reach either point 3 (internal balance) or point 4 (external balance) in the figure, but not both internal and external balances. Only a devaluation of the currency accompanied with an increase in fiscal ease, namely increasing government expenditures or decreasing taxes, will move the economy to both internal and external balances at point 1 in the figure.
Difficulty: Moderate AACSB: Application of knowledge
36 Copyright © 2022 Pearson Education, Inc.
19.7
The External Balance Problem of the United States Under Bretton Woods
1) The confidence problem of the Bretton Woods systems articulated by Robert Triffin refers to A) the unwillingness of central banks to accumulate currency for fear of not being able to convert it to gold in case a run on the banks occurs. B) consumer fear of stock market instability. C) producer fear of rising wages. D) the lack of convertibility of gold into silver. E) low consumer spending because of balance of payment crises. Answer: A Difficulty: Easy AACSB: Application of knowledge 2) The acceleration of American inflation in the late 1960s was A) a worldwide phenomenon. B) a national problem of the US alone. C) a regional problem. D) a good problem to have. Answer: A Difficulty: Easy AACSB: Application of knowledge 3) In order to bring about a real depreciation of the dollar, the U.S. can hope for A) a rise in the U.S. price level. B) a fall in foreign price levels. C) a rise in the dollar's nominal value in terms of foreign currencies. D) a rise in foreign price levels or a fall in the dollar's nominal value in terms of foreign currencies. E) increased output and full employment. Answer: D Difficulty: Easy AACSB: Application of knowledge 4) The collapse of the Bretton Woods system marked A) the end of floating exchange rates and a move to fixed exchange rates. B) the end of fixed exchange rates and a move to floating exchange rates. C) the beginning of the gold standard. D) a plunge in the price of gold. E) the elimination of paper currencies. Answer: B Difficulty: Easy AACSB: Application of knowledge
37 Copyright © 2022 Pearson Education, Inc.
5) Which of the following statements is MOST accurate? A) A revaluation restores internal and external balance immediately, without causing domestic inflation. B) A devaluation restores internal and external balance immediately, without causing domestic inflation. C) A revaluation restores internal and external balance immediately, but also causes domestic inflation. D) A devaluation restores internal and external balance immediately, but also causes domestic inflation. E) A devaluation restores external balance in the long run, without causing immediate domestic inflation. Answer: A Difficulty: Easy AACSB: Application of knowledge 6) Which of the following is the MOST accurate? A) U.S. macroeconomic policies in the late 1960s helped cause the breakdown of the Bretton Woods system by early 1973. B) U.S. macroeconomic policies in the late 1970s helped cause the breakdown of the Bretton Woods system by early 1983. C) U.S. macroeconomic policies in the late 1980s helped cause the breakdown of the Bretton Woods system by early 1993. D) U.S. macroeconomic policies in the late 1950s helped cause the breakdown of the Bretton Woods system by early 1963. E) U.S. macroeconomic policies in the late 1960s delayed the breakdown of the Bretton Woods system to early 1973. Answer: A Difficulty: Easy AACSB: Application of knowledge
38 Copyright © 2022 Pearson Education, Inc.
7) Refer to the graph below, which shows the effect of ________ on the home economy.
A) foreign inflation B) domestic inflation C) foreign deflation D) domestic recession E) foreign recession Answer: A Difficulty: Moderate AACSB: Application of knowledge
39 Copyright © 2022 Pearson Education, Inc.
8) Refer to the graph below. The movement from point 1 to point 2 is stimulated by a disequilibrium in which there is domestic ________ and ________.
A) overemployment; trade surplus B) unemployment; trade surplus C) overemployment; trade deficit D) unemployment; trade deficit E) inflation; unemployment Answer: A Difficulty: Moderate AACSB: Application of knowledge
40 Copyright © 2022 Pearson Education, Inc.
9) Explain the confidence problem under Bretton Woods system. Is there any possible solution? Answer: As central banks' international reserve needs grew over time, their holdings of dollars would necessarily grow until they exceeded the U.S. gold stock. Since the United States had promised to redeem these dollars at $35 an ounce, it would no longer have the ability to meet its obligations should all dollar holders simultaneously try to convert their dollars into gold. Central banks, knowing that their dollars were no longer "as good as gold," might become unwilling to accumulate more dollars and might even bring down the system by attempting to cash in the dollars they already held. One possible solution was an increase in the official price of gold in terms of the dollar and all other currencies. But such an increase would have been inflationary and would have had the politically unattractive consequence of enriching the main gold-supplying countries. Further, an increase in gold's price would have caused central banks to expect further decreases in the gold value of their dollar reserve holdings in the future, thereby possibly worsening the confidence problem rather than solving it! Difficulty: Moderate AACSB: Application of knowledge 10) How can inflation be imported from abroad under Bretton Woods system? Answer: A country faced with a rise in the foreign price level will be thrown out of balance and ultimately will import the foreign inflation if it holds its exchange rate fixed. Difficulty: Moderate AACSB: Application of knowledge 19.8
The Case for Floating Exchange Rates
1) In a system of floating exchange rates, A) central banks do not intervene in the foreign exchange market to fix rates. B) central banks always intervene in the foreign exchange market to fix rates. C) central banks sometimes intervene in the foreign exchange market to fix rates. D) central banks may intervene in the foreign exchange market to fix rates. Answer: A Difficulty: Easy AACSB: Application of knowledge 2) Advocates of floating rate suggested it is favorable for economies for all of the following reasons EXCEPT A) it automatically matches the domestic inflation with ongoing foreign inflation. B) it would deliver necessary exchange rate flexibility. C) it would produce benefits for the world economy. D) it is a healthy development in the evolution of the world monetary. E) it would put markets at center stage in determining exchange rates. Answer: A Difficulty: Easy AACSB: Application of knowledge
41 Copyright © 2022 Pearson Education, Inc.
3) Advocates of floating rate suggested it is favorable for economies for all of the following reasons EXCEPT A) it discourages attack from foreign exchange speculators because of the fact that exchange rate adjustment is immediate. B) it helps stabilize the shock effect on unemployment in case of economic changes such as a fall in export demand. C) it automatically matches the domestic inflation with ongoing foreign inflation. D) it gives every country the opportunity to guide its own monetary conditions at home. E) it brings the LR exchange rate to the level predicted by PPP without government policy decisions. Answer: C Difficulty: Moderate AACSB: Application of knowledge 4) If central banks were no longer obliged to intervene in currency markets to fix exchange rates, governments would be able to use monetary policy to reach A) internal balance. B) external balance. C) internal and external balance. D) internal but not external balance. E) external but not internal balance. Answer: C Difficulty: Easy AACSB: Application of knowledge 5) Advocates of flexible exchange rates claim that under flexible exchange rates A) no country would be forced to import only inflation from abroad. B) no country would be forced to import only deflation from abroad. C) no country would be forced to import inflation and deflation from abroad. D) flexible exchange rates are not able to halt importing inflation from abroad. E) flexible exchange rates are not able to halt importing deflation from abroad. Answer: C Difficulty: Easy AACSB: Application of knowledge 6) Advocates of flexible exchange rates claim that under flexible exchange rates A) the United States would now be able to set world monetary conditions all by itself. B) Germany would no longer be able to set world monetary conditions all by itself. C) the United Kingdom would no longer be able to set world monetary conditions all by itself. D) the United States would no longer be able to set world monetary conditions all by itself. E) Germany would now be able set world monetary conditions all by itself. Answer: D Difficulty: Easy AACSB: Application of knowledge
42 Copyright © 2022 Pearson Education, Inc.
7) Advocates of flexible exchange rates claim that under flexible exchange rates A) the United States would no longer have the same opportunity as other countries to influence its exchange rate against foreign currencies. B) the United States would have the same opportunity as other countries to influence its exchange rate against foreign currencies. C) the United Kingdom would not have the same opportunity as other countries to influence its exchange rate against foreign currencies. D) Germany would not have the same opportunity as other countries to influence its exchange rate against foreign currencies. E) China would have the same opportunity as other countries to influence its exchange rate against foreign currencies. Answer: B Difficulty: Easy AACSB: Application of knowledge 8) Some claim that the long and agonizing periods of speculation preceding exchange rate realignments would A) not occur under a fixed exchange rate regime. B) not occur under floating. C) become more severe under currency board. D) become less severe under floating. E) be prolonged under floating. Answer: D Difficulty: Easy AACSB: Application of knowledge 9) Advocates of flexible exchange rates claim that under flexible exchange rates, marketdetermined exchange rates would move automatically so as A) to prevent the emergence of big current account deficits and surpluses. B) countries have little scope to use monetary policy to attain internal and external balance. C) countries keep domestic interest rate in line with that of the United States. D) to solve the problem of monetary trilemma. Answer: A Difficulty: Easy AACSB: Application of knowledge 10) Under the fixed rate regime foreign countries could hold their dollar exchange rates constant by A) using tight monetary policy. B) using expansionary fiscal policy. C) negotiating with the central bank of the United States. D) setting their domestic interest rate equal to the U.S. interest rate. E) holding their exchange rates constantly pegged to the euro and yen. Answer: D Difficulty: Moderate AACSB: Application of knowledge 43 Copyright © 2022 Pearson Education, Inc.
11) Advocates of floating rates pointed out that A) removal of the obligation to peg currency values would restore monetary control to central banks. B) imposing of the obligation to peg currency values would restore monetary control to central banks. C) removing of the obligation to peg currency values would restore fiscal control. D) imposing of the obligation to peg currency values would restore fiscal control. E) imposing of the obligation to peg currency would restore monetary control to the consumer. Answer: A Difficulty: Easy AACSB: Application of knowledge 12) Advocates of flexible exchange rates claim that under flexible exchange rates, if the central bank faced unemployment A) and thus wished to decrease its money supply, there would no longer be any legal barrier to the currency depreciation this would cause. B) and thus wished to expand its money supply, there would no longer be any legal barrier to the currency depreciation this would cause. C) and wished to expand its money supply, there would no longer be any legal barrier to the currency appreciation this would cause. D) and wished to decrease its money supply, there now would be a legal barrier to the currency depreciation this would cause. E) and wished to increase output, there would no longer be a legal barrier to the currency appreciation this would cause. Answer: B Difficulty: Easy AACSB: Application of knowledge 13) Advocates of flexible exchange rates claim that under flexible exchange rates, a currency A) appreciation caused by increasing the money supply would reduce unemployment by lowering the relative price of domestic products. B) depreciation caused by increasing the money supply would increase unemployment by lowering the relative price of domestic products. C) depreciation caused by increasing the money supply would reduce unemployment by lowering the relative price of domestic products. D) depreciation caused by increasing the money supply would reduce unemployment by increasing the relative price of domestic products. E) depreciation cause by decreasing the money supply would not effect unemployment, but would increase the relative price of domestic products. Answer: C Difficulty: Easy AACSB: Application of knowledge
44 Copyright © 2022 Pearson Education, Inc.
14) Advocates of flexible exchange rates claim that under flexible exchange rates, a currency A) depreciation caused by increasing the money supply would reduce unemployment by increasing world demand for them. B) appreciation caused by increasing the money supply would reduce unemployment by increasing world demand for them. C) appreciation caused by decreasing the money supply would reduce unemployment by increasing world demand for them. D) appreciation caused by increasing the money supply would increase unemployment by increasing world demand for them. E) appreciation caused by increasing the money supply would increase unemployment by decreasing world demand for them. Answer: A Difficulty: Easy AACSB: Application of knowledge 15) Advocates of flexible exchange rates claim that under flexible exchange rates, a currency A) depreciation caused by increasing the money supply would reduce unemployment by lowering the relative price of domestic products and increasing the world demand for them. B) appreciation caused by increasing the money supply would reduce unemployment by lowering the relative price of domestic products and increasing world demand for them. C) appreciation caused by decreasing the money supply would reduce unemployment by lowering the relative price of domestic products and increasing world demand for them. D) appreciation caused by increasing the money supply would increase unemployment by lowering the relative price of domestic products and increasing world demand for them. E) appreciation caused by increasing the money supply would increase unemployment by lowering the relative price of domestic products and by decreasing world demand for them. Answer: A Difficulty: Easy AACSB: Application of knowledge 16) Advocates of flexible exchange rates claim that under flexible exchange rates, the central bank of A) an overheated economy could cool down activity by increasing the money supply without worrying that undesired reserve inflow would undermine its stabilization effort. B) a cooled economy could cool down activity by contracting the money supply without worrying that undesired reserve inflow would undermine its stabilization effort. C) an overheated economy could cool down activity by contracting the money supply without worrying that undesired reserve inflow would undermine its stabilization effort. D) an overheated economy could cool down activity by contracting the money supply without worrying that undesired reserve outflow would undermine its stabilization effort. E) an overheated economy could cool down activity by decreasing employment and increasing output without worrying that this would undermine its stabilization effort. Answer: C Difficulty: Easy AACSB: Application of knowledge
45 Copyright © 2022 Pearson Education, Inc.
17) Advocates of flexible exchange rates claim that under flexible exchange rates A) enhanced control over fiscal policy would allow countries to dismantle their distorting barriers to international payments. B) reduced control over monetary policy would allow countries to dismantle their distorting barriers to international payments. C) enhanced control over monetary policy would allow countries to increase their distorting barriers to international payments. D) enhanced control over monetary policy would allow countries to dismantle their distorting barriers to international payments. E) enhanced control over monetary policy would destabilize exchange rates. Answer: D Difficulty: Easy AACSB: Application of knowledge 18) By the end of the 1960s, many countries felt that they were importing inflation from A) the United States. B) Germany. C) France. D) Japan. E) the United Kingdom. Answer: A Difficulty: Easy AACSB: Application of knowledge 19) Which one of the following statements is TRUE? A) By devaluing its currency, that is, by lowering the domestic currency price of foreign currency, a country can insulate itself completely from an inflationary increase in foreign prices. B) By revaluing its currency, that is, by increasing the domestic currency price of foreign currency, a country can insulate itself completely from an inflationary increase in foreign prices. C) By revaluing its currency, that is, by lowering the domestic currency price of foreign currency, a country cannot insulate itself completely from an inflationary increase in foreign prices. D) By revaluing its currency, that is, by lowering the domestic currency price of foreign currency, a country can insulate itself completely from an inflationary increase in foreign prices. E) By revaluing its currency, that is, by lowering the domestic currency price of foreign currency, a country cannot insulate itself completely from a harmful decrease in foreign prices. Answer: D Difficulty: Easy AACSB: Application of knowledge
46 Copyright © 2022 Pearson Education, Inc.
20) When all changes in the world are due to A) fiscal policy, purchasing power parity holds true in the long run. B) monetary policy, purchasing power parity does not hold true in the long run. C) monetary policy, purchasing power parity holds true in the long run. D) monetary policy, purchasing power parity holds true even in the short run. E) fiscal and monetary policy, purchasing power parity holds true in the long run. Answer: C Difficulty: Easy AACSB: Application of knowledge 21) Under purchasing power parity (PPP), if U.S. monetary growth leads to a long run doubling of the U.S. price level, while Europe's price level remains constant, PPP predicts that the A) long-run euro price of the dollar will be doubled. B) long-run euro price of the dollar will be halved. C) long-run euro price of the dollar will remain the same. D) short-run euro price of the dollar will be halved. E) short-run euro price of the dollar will be doubled. Answer: B Difficulty: Easy AACSB: Application of knowledge 22) Under flexible exchange rate regime, a money-induced A) decrease in U.S. prices causes an immediate appreciation of the foreign currencies against the dollar. B) increase in U.S. prices causes an immediate appreciation of the foreign currencies against the dollar. C) increase in U.S. prices causes an eventual appreciation of the foreign currencies against the dollar. D) increase in U.S. prices causes an eventual depreciation of the foreign currencies against the dollar. E) decrease in U.S. prices causes no change in foreign exchange rate. Answer: B Difficulty: Easy AACSB: Application of knowledge
47 Copyright © 2022 Pearson Education, Inc.
23) Under Bretton Woods, A) any foreign country cannot devalue its currency against the dollar in conditions of "fundamental disequilibrium." B) any foreign country could devalue its currency against the dollar in conditions of "fundamental disequilibrium," but the system's rules did not give the United States the option of devaluing against foreign currencies. C) any foreign country could devalue its currency against the dollar in conditions of "fundamental disequilibrium," and the system's rules did give the United States the same option of devaluing against foreign currencies. D) the U.S. could devalue its currency against the foreign currencies in conditions of "fundamental disequilibrium." E) any foreign country can revalue its currency against the dollar in conditions of "fundamental disequilibrium." Answer: B Difficulty: Easy AACSB: Application of knowledge 24) The DD schedule shows A) interest rate and output pairs for which aggregate demand equals aggregate output. B) exchange rate and output pairs for which aggregate demand equals aggregate output. C) exchange rate and output pairs for which aggregate supply equals aggregate output. D) interest rate and output pairs for which aggregate supply equals aggregate output. E) exchange rate and output pairs for which aggregate demand is greater than aggregate output. Answer: B Difficulty: Easy AACSB: Application of knowledge 25) The AA schedule shows A) interest rate and output pairs at which the foreign exchange market and the domestic money market are in equilibrium. B) exchange rate and output pairs at which the foreign exchange market and the domestic money market are in equilibrium. C) interest rate and output pairs at which only the foreign exchange market is in equilibrium. D) exchange rate and output pairs at which only the foreign exchange market is in equilibrium. E) exchange rate and output pairs at which only the domestic money market are in equilibrium. Answer: B Difficulty: Easy AACSB: Application of knowledge
48 Copyright © 2022 Pearson Education, Inc.
26) Under flexible exchange rate, the response of an economy to a temporary fall in foreign demand for its exports is A) the currency appreciates, and output falls. B) the currency depreciates, and output falls. C) the currency depreciates, and output increases. D) the currency depreciates, and output remains constant. E) the currency appreciates, and output increases. Answer: B Difficulty: Moderate AACSB: Application of knowledge 27) Under fixed exchange rate, the response of an economy to a temporary fall in foreign demand for its exports is A) the currency appreciates, and output falls. B) the currency depreciates, and output falls. C) the currency remains the same, and output decreases. D) the currency depreciates, and output remains constant. E) the currency appreciates, and output remains the same. Answer: C Difficulty: Moderate AACSB: Application of knowledge 28) Which of the following is NOT a result of a temporary fall in foreign demand on one country's exports under floating exchange rate? A) The DD curve shifts to the left due to reduction of aggregate demand. B) The AA curve shifts downwards due to reduction of money supply. C) a fall in aggregate output D) depreciation in home country's currency E) a fall in the home interest rate Answer: B Difficulty: Moderate AACSB: Application of knowledge 29) Which of the following is NOT a result of a permanent fall in foreign demand on one country's exports under floating exchange rate? A) The DD curve shifts to the left due to reduction of aggregate demand. B) The AA curve shifts upwards due to the increased expected long-run exchange rate. C) a reduction in output by a smaller degree compared to temporary fall in demand D) depreciation in home country's currency E) a raised level of unemployment Answer: E Difficulty: Moderate AACSB: Application of knowledge
49 Copyright © 2022 Pearson Education, Inc.
30) Comparing fixed to flexible exchange rate, the response of an economy to a temporary fall in foreign demand for its exports is A) output actually falls less under fixed rate than under floating rate. B) output actually falls more under fixed rate than under floating rate. C) output actually remains the same under fixed rate than under floating rate. D) the currency value grows in a fixed rate system and falls in a flexible system. E) output grows in a fixed rate system and falls in a flexible system. Answer: B Difficulty: Moderate AACSB: Application of knowledge 31) The effects of a decrease in export demand A) is a powerful argument in favor of fixed rates. B) is a powerful argument in favor of flexible rates. C) shows the difficulties in determining which exchange rate is better. D) is a powerful argument in favor of fixed rates only in the short run. E) is a powerful argument in favor of fixed rates only in the long run. Answer: B Difficulty: Moderate AACSB: Application of knowledge 32) The reason that the claim that floating exchange rates result in greater economic autonomy for individual countries may not be entirely accurate is that A) empirical research finds no supporting data. B) policy makers are influenced by the effect of domestic policies on the exchange rate. C) there is no generally satisfactory method for measuring economic autonomy. D) it is based on the assumption of a gold standard. E) countries that run large trade deficits must increase exports to balance trade. Answer: B Difficulty: Moderate AACSB: Application of knowledge 33) Under a flexible exchange rate regime, an increase in real money demand A) moves the AA curve upward. B) moves the AA curve downward. C) leaves the AA curve unchanged. D) moves the DD curve to the right. E) moves the DD curve to the left. Answer: B Difficulty: Easy AACSB: Application of knowledge
50 Copyright © 2022 Pearson Education, Inc.
34) Under the flexible exchange rate, lowering the price of a foreign currency will A) allow the expansion of monetary policy without causing inflation. B) decrease the foreign country's output. C) prevent a foreign price increase from causing deflation at home. D) cause a home price increase to be exported to the foreign markets. E) cause a "beggar-thy-neighbor" effect. Answer: A Difficulty: Moderate AACSB: Application of knowledge 35) What caused inflation to accelerate in most countries in 1974? A) the oil shock B) the real estate market shock C) the climate change shock D) the floating exchange rate shock Answer: A Difficulty: Easy AACSB: Application of knowledge 36) A combination of stagnating output and high inflation is called A) stagflation. B) hyperinflation. C) stagnation. D) inflagnation. Answer: A Difficulty: Easy AACSB: Application of knowledge 37) Federal Reserve Chairman Volcker's policy to fight inflation A) led to the 1981-1983 recession, but was ultimately successful. B) led to the 1981-1983 recession, but did not end high inflation due to beggar-thy-neighbor effects. C) was perfectly complemented by Reagan's decrease in fiscal spending. D) led to the 1981-1983 recession and foretold the economic downturn in the mid-1990s. E) led to an immediate depreciation of the dollar. Answer: A Difficulty: Easy AACSB: Application of knowledge
51 Copyright © 2022 Pearson Education, Inc.
38) The Plaza Accord of 1985 announces that the A) G-5 countries will intervene in the foreign exchange market to bring about a dollar appreciation. B) G-7 countries will intervene in the foreign exchange market to bring about a dollar depreciation. C) G-5 countries will intervene in the foreign exchange market to bring about a dollar depreciation. D) G-7 countries will intervene in the foreign exchange market to bring about a euro depreciation. E) G-5 countries will not intervene in the foreign exchange market unless the dollar needs to appreciate. Answer: C Difficulty: Easy AACSB: Application of knowledge 39) Supporters of a floating exchange rate cited all of the following as advantages over the Bretton Woods system EXCEPT A) each country would be able to choose its own long-run inflation rate. B) parity changes and speculative attacks would no longer be possible. C) countries would be forced to work cooperatively in deciding monetary policy. D) exchange rates would be set symmetrically in foreign markets rather than by government decision. E) governments would not need to export inflation to decrease domestic unemployment. Answer: C Difficulty: Easy AACSB: Application of knowledge 40) If the demand for Home exports decreased abroad, the Home fall in output would be greatest A) if the decrease was temporary and the exchange rate was fixed. B) if the decrease was temporary and the exchange rate was floating. C) if the decrease was permanent and the exchange rate was fixed. D) if the decrease was permanent and the exchange rate was floating. E) if the decrease was permanent and the exchange rate was high. Answer: C Difficulty: Moderate AACSB: Application of knowledge 41) If most of the shocks that buffet the economy come from the output market shocks, then A) fixed exchange rates are better than flexible exchange rates. B) flexible exchange rates are better than fixed exchange rates. C) which system is chosen is not important. D) fixed exchange rates are better than flexible exchange rates only in the short run. E) flexible exchange rates are better than fixed exchange rates only in the short-run. Answer: B Difficulty: Moderate AACSB: Application of knowledge 52 Copyright © 2022 Pearson Education, Inc.
42) Present the case for floating exchange rates. Answer: (1) Monetary policy autonomy — Governments would be able to use monetary policy to reach internal and external balance. No country would be forced to import inflation and deflation from abroad. (2) Symmetry — The United States would no longer be able to set world monetary conditions all by itself. The United States would have the same opportunity as other countries to influence its exchange rate against foreign currencies. (3) Exchange rates as automatic stabilizers — The long and agonizing periods of speculation preceding exchange rate realignments would not occur under floating. Difficulty: Moderate AACSB: Application of knowledge 43) "No central bank can be indifferent to its currency's value in the foreign exchange market." Discuss. Answer: — despite the "Monetary Policy Autonomy" theory of the original supporters of floating exchange rates — exchange rate's role in inflation — prices are sticky in the short run, so foreign developments can affect real interest rates and real exchange rates at home — don't want their exchange rate to be too volatile as it affects the demand for their domestic products — appreciation or depreciation can cause inflation that is difficult to counter — banks intervene on a discretionary basis so it is still necessary for them to continue to hold foreign reserves — "dirty floats" stabilize output and price level after shocks that affect exchange rates — empirically: after 1973 countries have continued to intervene to affect exchange rates Difficulty: Moderate AACSB: Application of knowledge
53 Copyright © 2022 Pearson Education, Inc.
44) Refer to the above figure. Use the DD-AA model to examine and compare the response of an economy under fixed and floating exchange rate to a temporary fall in foreign demand for its exports. Answer: The DD curve shifts to the left. When the exchange rate floats, because the demand shift is assumed to be temporary, it does not change the long-run expected exchange rate and so does not move the asset market equilibrium schedule AA. Thus, E rises, i.e. the currency depreciates and output falls. Under fixed exchange rate, the central bank must prevent the currency depreciation that occurs under a floating rate; thus, it buys domestic money with foreign currency, reducing the domestic money supply and shifting the AA to the left and down. E will remain constant and output will fall. Difficulty: Moderate AACSB: Application of knowledge 54 Copyright © 2022 Pearson Education, Inc.
45) Refer to the above figure. Use the DD-AA model to examine and compare the response of an economy under fixed and floating exchange rate to a permanent fall in foreign demand for its exports. Answer: The DD curve shifts to the left. Under flexible exchange rate, the expected exchange rate Ee also rises and AA shifts upward and to the right. Thus, a permanent shock causes a greater depreciation than a temporary one. Under fixed exchange rate, the central bank must prevent the currency depreciation that occurs under a floating rate; thus, it buys domestic money with foreign currency, reducing the domestic money supply and shifting the AA to the left and down. E will remain constant and output will fall. Under fixed exchange rate, a fall in export demand, if permanent, has led to a situation of "fundamental disequilibrium" calling for a devaluation of the currency or a long period of domestic unemployment as export prices fell. Uncertainty about the government's intention would have encouraged speculative capital outflows, further worsening the situation by depleting central bank reserves and contracting the domestic money supply at a time of unemployment. Difficulty: Moderate AACSB: Application of knowledge 46) Use the DD-AA model to compare the domestic economic response under flexible and fixed exchange rate regimes to a temporary rise in export demand from foreign countries. Answer: Under floating rate: The DD curve shifts right. AA does not change because the temporary increase will not affect the long run expected exchange rate. Output rises and E falls (depreciates). Under fixed rate: The DD curve shifts right. The central bank intervenes to prevent a change in the exchange rate. By selling domestic currency they expand the domestic supply and the AA curve shifts right, keeping E constant. Output, however, will rise due to the new equilibrium of the DD and AA curves to the right of its former location. Difficulty: Moderate AACSB: Application of knowledge
55 Copyright © 2022 Pearson Education, Inc.
19.9
Macroeconomic Interdependence under a Floating Rate
1) Due to macroeconomics interdependence between large countries, the effect of a permanent monetary policy expansion by Home is as follows: Home output A) rises, Home's currency depreciates, and Foreign output may rise or fall. B) falls, Home's currency depreciates, and Foreign output may rise or fall. C) rises, Home's currency appreciates, and Foreign output may rise or fall. D) rises, Home's currency depreciates, and Foreign output rises. E) falls, Home's currency appreciates, and Foreign output may rise or fall. Answer: A Difficulty: Easy AACSB: Application of knowledge 2) Due to macroeconomics interdependence between large countries, the effect of a permanent fiscal expansion by Home is as follows: Home output A) falls, Home's currency appreciates, Foreign output rises. B) rises, Home's currency appreciates, Foreign output rises. C) rises, Home's currency depreciates, Foreign output rises. D) rises, Home's currency appreciates, Foreign output decreases. E) falls, Homes currency depreciates, Foreign output rises. Answer: B Difficulty: Easy AACSB: Application of knowledge 3) Which of the following does NOT occur if Home starts a policy of permanent fiscal expansion? A) Home's exchange rate increases. B) Foreign's interest rate rises. C) Home output rises. D) Foreign output rises. E) Current Account Balance increases. Answer: E Difficulty: Moderate AACSB: Application of knowledge 4) Where did crisis happen between 1990 and 1999? A) Europe and Asia B) America and Australia C) Africa and Europe D) Australia and Asia Answer: A Difficulty: Easy AACSB: Application of knowledge
56 Copyright © 2022 Pearson Education, Inc.
5) What happened to Germany's general price level after the reunification of West and East Germany on July 1, 1990? A) rising inflation B) rising deflation C) volatile price fluctuation D) stable price level Answer: A Difficulty: Easy AACSB: Application of knowledge 6) Japan's loose monetary policy from 1986 to 1988 did NOT lead to A) loss in Japan's stock price index value. B) skyrocketing prices for Japanese real estate. C) rising stock prices. D) rising inflation. Answer: A Difficulty: Easy AACSB: Application of knowledge 7) Problems of the Japanese economy in 1997-1998 spilled over to A) East Asia. B) Europe. C) Sub-Saharan Africa. D) Latin America. Answer: A Difficulty: Easy AACSB: Application of knowledge 8) The U.S. stock market soared in the late 1990s as money flooded into high-tech, "dot-com" stocks and then began to collapse in 2000, creating the so-called A) dot-com crash. B) high-tech crumble. C) dot-com failure. D) Internet downfall. Answer: A Difficulty: Easy AACSB: Application of knowledge
57 Copyright © 2022 Pearson Education, Inc.
9) During the first half of the 2000s, the large increase in the U.S. current account deficit was matched by increases in the A) surpluses of other countries, notably Asian countries including China. B) deficits of other countries, notably Asian countries including China. C) deficits of other countries, notably newly industrialized countries such as Singapore and Taiwan. D) surpluses of other countries, notably South American countries. Answer: A Difficulty: Easy AACSB: Application of knowledge 10) The global financial crisis in 2007 started as a result of A) the U.S. subprime mortgage crisis. B) the long-term high mortgage interest rate. C) the U.S. government's budget deficit. D) the significant barriers in world trade. Answer: A Difficulty: Easy AACSB: Application of knowledge 11) The election of Donald Trump as U.S. President had A) mixed consequences for global economy. B) no impacts on the global economy. C) promoted global rate of real GDP growth between 2018 and 2019. D) decreased protectionist actions against a range of U.S. trade partners, notably China and the European Union. Answer: A Difficulty: Easy AACSB: Application of knowledge 12) Early in 2020, global output level declined sharply because of A) the global pandemic crisis. B) the aggressive protectionism actions around the world. C) the high global public debt and deficits. D) the lack of global cooperation. Answer: A Difficulty: Easy AACSB: Application of knowledge
58 Copyright © 2022 Pearson Education, Inc.
13) Which of the following is NOT among the quantitative indicators for the U.S. Department of the Treasury to judge whether it should name a country a currency manipulator? A) having inflation rate of more than 10% a year B) having a bilateral trade surplus of more than $20 billion with the United States C) having an overall (multilateral) current account surplus of more than 2 percent of GDP D) official net intervention purchases of foreign currency totaling more than 2 percent of GDP, taking place in at least six of the past twelve months Answer: A Difficulty: Easy AACSB: Application of knowledge 14) Dangers of deflation do NOT include A) increased purchasing power. B) debt deflation. C) decreased economic resilience. D) more frequent liquidity traps. E) loss of inflation-target-credibility. Answer: A Difficulty: Easy AACSB: Analytical thinking
59 Copyright © 2022 Pearson Education, Inc.
15) Imagine a world with two large countries, Home and Foreign. Evaluate how Home's macroeconomic policies affect Foreign. Compare the small and the large country cases; consider both permanent monetary and fiscal policies. Answer: Note that since the two countries are large, neither country can be thought of any longer as facing a fixed external interest rate or a fixed level of foreign export demand. Consider only permanent shifts. A permanent monetary expansion by Home, in the small country's case, would lead to currency depreciation and increase in output, interest rates also falling. When the Home economy is large, the same would happen, but now the rest of the world is affected too. Because Home is facing real currency depreciation, Foreign must be experiencing a real currency appreciation. This makes foreign goods relatively expensive and thus reduces its output. However, this increases Homes output, since Home's imports will rise. Thus, it is not clear what will happen to Foreign output. Note that Foreign output can rise only if the Foreign nominal interest rate rises too and can fall only if Foreign nominal interest rate falls. This is because the foreign market equilibrium is: M /P = L(R , Y ). Because in this exercise M is not changing and P is sticky by assumption and thus fixed in the short run. Consider now, a permanent expansionary fiscal policy in Home. In the small country case, a permanent fiscal expansion would cause a real currency appreciation and a current account deterioration that would fully nullify any positive effect on aggregate demand. In effect, the expansionary impact of the Home fiscal ease would leak entirely abroad. This is because the counterpart of Home's lower current account balance must be higher current account balance abroad. In the large country case, Foreign output still rises since Foreign's exports become relatively cheaper when Home's currency appreciates. In addition, now some of Foreign's increased spending increases Home exports, so Home's output actually increases along with the output of Foreign. Home's nominal interest rate must rise and Foreign's interest rate rises at the same time as well. Difficulty: Difficult AACSB: Application of knowledge 16) "Even under flexible exchange rate regime, governments could not be indifferent to the behavior of exchange rates and inevitably surrendered some of their policy autonomy in other areas to prevent exchange rate movements they viewed as harmful to their economies." Discuss. Answer: True. One example is Volcker in October 1979 decreasing the U.S. money supply to halt further weakening the dollar. Difficulty: Moderate AACSB: Application of knowledge
60 Copyright © 2022 Pearson Education, Inc.
17) Which system of interest rates is theoretically worst for policy coordination among the industrial countries of the world? How has this played out since the 1980s? Answer: — pro-fixed rate theorists predicted that with floating rates, countries would only make policies that helped themselves at the expense of the world economy (although it has been empirically proven that in the short run policy decisions are exported, requiring the "dirty float") — pro-floaters rebutted that the fixed rate system provided coordination only by giving the U.S. a dominant position — during the 1980s industrial countries could have collectively reduced the effects of recession by coordinating their policies much more effectively, that is floating rates have not provided more coordination as predicted. Difficulty: Moderate AACSB: Application of knowledge 19.10
What Has Been Learned Since 1973?
1) Economic theory and experience since 1973 indicate that, under floating exchange rates, a country's fiscal and monetary policies in the short run and the long run can A) have both domestic and foreign economic impact. B) have domestic or foreign economic impact, but not both. C) have domestic but not foreign economic impact. D) have foreign but not domestic economic impact. E) have neither domestic nor foreign economic impact. Answer: A Difficulty: Moderate AACSB: Application of knowledge 2) Economic theory and experience since 1973 indicate that floating exchange rates allow A) a much larger international divergence in inflation. B) a smaller international divergence in inflation. C) proportional exchange rate and relative price level changes. D) central banks with less control of their money supplies. Answer: A Difficulty: Easy AACSB: Application of knowledge 3) The mechanism behind the inflation insulation provided by a floating exchange rate is A) Purchasing Power Parity. B) a fixed AA curve. C) market speculation. D) tight monetary policy. E) symmetry. Answer: A Difficulty: Easy AACSB: Application of knowledge
61 Copyright © 2022 Pearson Education, Inc.
4) Economic experience since 1973 indicates that, under floating exchange rates, symmetry A) was not attained. B) was attained almost immediately. C) was attained over time as central banks held more U.S. dollars as a reserve currency. D) has been difficult to measure and no consensus has emerged. E) has been attained in foreign countries, but not domestically. Answer: A Difficulty: Easy AACSB: Application of knowledge 5) Economic experience since 1973 indicates that, under floating exchange rates, A) many countries were able to relax the capital controls, which spurred the rapid growth of a global financial industry and allowed greater gains from intertemporal trade and from trade in assets. B) all economic sectors can be cushioned from real disturbances such as shifts in aggregate demand. C) countries always become less vulnerable to shocks arising from the foreign exchange market itself, such as fluctuations in the currency risk premium. D) many countries were still not able to relax the capital controls. Answer: A Difficulty: Easy AACSB: Application of knowledge 6) Economic experience since 1973 indicates that, under floating exchange rates A) large and persistent departures from external balance were not prevented. B) large and persistent departures from external balance were prevented. C) changes in exchange rates failed to act as automatic stabilizers. D) reduced monetary policy autonomy. E) monetary policy autonomy was protected. Answer: A Difficulty: Easy AACSB: Application of knowledge 7) Economic experience since 1973 indicates that, under floating exchange rates, A) problems of international policy coordination clearly have not disappeared. B) problems of international policy coordination have disappeared. C) countries experienced more convergent rates of recovery after the 2007-2009 crisis. D) policy coordination has become increasingly easy. Answer: A Difficulty: Easy AACSB: Application of knowledge
62 Copyright © 2022 Pearson Education, Inc.
8) What has been learned since 1973 with regards to the experience with floating exchange rate regime? Answer: (1) Monetary policy autonomy: Yes, however, floating rate did not insulate countries completely from foreign policy shock. In addition, no central bank can be indifferent to its currency's value in the foreign exchange market, thus the name "dirty float" rather than "clean float." (2) Symmetry: No, the dollar remains an important currency; the DM and the yen have gained importance; the British pound declines in importance. (3) The exchange rate as an automatic stabilizer: Good performance of the flexible regimes; many believe that, otherwise, major realignments of exchange rates should have taken place. However, some sectors suffered, such as agriculture. (4) Discipline: Did countries abuse the autonomy afforded by floating rates? Inflation rates did accelerate after 1973. (5) Destabilizing Speculation: Floating exchange rates have exhibited much more day-to-day volatility than the early advocates of floating would have predicted. However, exchange rates are assets prices and so considerable volatility is to be expected. Over the long run, they do not seem to support the notion of destabilizing speculation. (6) International trade and investment: Critics of floating claimed that international trade and investment would suffer as a result of the increased uncertainty. This prediction was certainly wrong. The use of forward markets and other derivatives expanded dramatically. Still, some economists disagree about the benefit to international trade. (7) Policy coordination: Floating exchange rates have not promoted policy coordination. Difficulty: Moderate AACSB: Application of knowledge
63 Copyright © 2022 Pearson Education, Inc.
19.11
Are Fixed Exchange Rates Even an Option for Most Countries?
1) Maintaining a fixed exchange rate over the long run is today A) virtually impossible. B) more vulnerable to speculative attacks than in the past. C) preferable. D) possible only in special cases when countries are willing to maintain controls over capital movements. E) aided by technology which allows instant movement of money between financial markets in different countries. Answer: D Difficulty: Easy AACSB: Application of knowledge 2) A country following prudent monetary and fiscal policies A) is not safe from speculative attacks on its fixed exchange rate. B) is safe from speculative attacks on its fixed exchange rate. C) is able to peg currency values for long while maintaining open capital markets and national policy sovereignty. D) is able to solve the monetary trilemma. Answer: A Difficulty: Easy AACSB: Application of knowledge 3) "Fixed exchange rates are not even an option for most countries." Discuss. Answer: Durable fixed exchange rate arrangements may not even be possible unless countries are willing to maintain strict controls over capital movements (as China does), or, at the other extreme, move to a shared single currency with their monetary partners (as in Europe). Even a country following prudent monetary and fiscal policies is not safe from speculative attacks on its fixed exchange rate. Difficulty: Moderate AACSB: Application of knowledge
64 Copyright © 2022 Pearson Education, Inc.
19.12
Appendix to Chapter 19: International Policy Coordination Failures
1) Coordination of economic policies among nations is a prisoner's ________ because all countries will be better off if they ________. A) dilemma; cooperate B) conundrum; cooperate C) sentence; compete D) screed; compete E) quandary; collude Answer: A Difficulty: Easy AACSB: Application of knowledge 2) Coordination of economic policies among nations is a prisoner's ________ because if all countries go it alone, they will choose to ________. A) dilemma; compete B) conundrum; cooperate C) sentence; compete D) dilemma; cooperate E) quandary; collude Answer: A Difficulty: Easy AACSB: Application of knowledge 3) Refer to the payoff matrix below, which is a prisoner's dilemma. If both countries go it alone, Home will choose Policy ________ and Foreign will choose Policy ________.
A) 1; A B) 2; B C) 1; B D) 2; A Answer: A Difficulty: Moderate AACSB: Application of knowledge
65 Copyright © 2022 Pearson Education, Inc.
4) Refer to the payoff matrix below, which is a prisoner's dilemma. If both countries cooperate, Home will choose Policy ________ and Foreign will choose Policy ________.
A) 1; A B) 2; B C) 1; B D) 2; A Answer: B Difficulty: Moderate AACSB: Application of knowledge
66 Copyright © 2022 Pearson Education, Inc.
5) Use the following table to illustrate the importance of macroeconomic policy coordination. Show that the two governments would have been happier if the two of them had adopted looser monetary policies, but given the policies that the other government did adopt, it is not in the interest of any individual government to change its course. Assume that each country wishes to get the biggest reduction in inflation rate at the lowest cost in terms of unemployment. This means that each country maximizes-ΔΠ/ΔU, the inflation reduction per point of increased unemployment.
67 Copyright © 2022 Pearson Education, Inc.
Answer: One needs to translate the outcomes of the table above into policy payoffs. Assume that each country wishes to get the biggest reduction in inflation rate at the lowest cost in terms of unemployment. This means that each country maximizes -ΔΠ/ΔU, the inflation reduction per point of increased unemployment. This leads to the following table.
Home finds itself better off with a very restrictive policy regardless of what Foreign does. In a symmetric position, Foreign finds itself better off with a very restrictive policy regardless of what Home does. The result is that both countries will choose very restrictive monetary policies, and each will get a payoff of . However, both countries are actually better off if they simultaneously adopt the somewhat restrictive policies. The resulting payoff for each is 1, which is greater than . Under this last policy configuration, inflation falls less in the two countries, but the rise in unemployment is less than under very restrictive policies. Difficulty: Moderate AACSB: Application of knowledge
68 Copyright © 2022 Pearson Education, Inc.
6) Assume that each country wishes to get the biggest reduction in inflation rate at the lowest cost in terms of unemployment. This means that each country maximizes-ΔΠ/ΔU, the inflation reduction per point of increased unemployment. What are the outcomes of the following games, assuming the max-min criteria is used?
Answer: There are two equilibria in this game, in which one regime uses a "somewhat restrictive" policy, while the other uses a "very restrictive" policy. Difficulty: Difficult AACSB: Application of knowledge
69 Copyright © 2022 Pearson Education, Inc.
International Economics: Theory and Policy, 12e (Krugman) Chapter 20 Financial Globalization: Opportunity and Crisis 20.1
The International Capital Market and the Gains from Trade
1) What would best describe the international capital markets? A) the market of exchange of bonds B) the market of exchange of stocks C) the market of exchange of real estate D) the market in which residents of different countries trade assets E) the currency market Answer: D Difficulty: Easy AACSB: Application of knowledge 2) The international capital market is A) the international currency exchange. B) a market in which capital assets are exchanged for services. C) the market that is subject to intense regulation and must file a report to the Basel committee on a biannual basis. D) not really a single market, but a group of closely interconnected markets in which asset exchanges with some international dimension take place. E) an organization of fiscal policies that dictate international trade. Answer: D Difficulty: Easy AACSB: Application of knowledge 3) Main actors in the international capital markets do NOT include A) casinos and gamblers. B) commercial banks. C) corporations. D) nonbank financial institutions. E) central banks and other government agencies. Answer: A Difficulty: Easy AACSB: Application of knowledge 4) International currency trades take place in the ________ market, which is an important part of the international market. A) foreign exchange B) money C) stock D) financial Answer: A Difficulty: Easy AACSB: Application of knowledge 1 Copyright © 2022 Pearson Education, Inc.
5) International capital market's activities take place in A) a network of world financial centers linked by sophisticated communication systems. B) the stock exchange worldwide. C) only online platforms that allow lenders and borrowers to meet. D) governments' regulated financial centers. Answer: A Difficulty: Easy AACSB: Application of knowledge 6) The assets traded in the capital market do NOT include A) durable goods and non-tradable services. B) stocks. C) bonds. D) bank deposits denominated in different currencies. Answer: A Difficulty: Easy AACSB: Application of knowledge 7) Describe three types of gains from trades? A) trades of exchange rates for goods or services, trades of goods or services for property, and trades of gold for textiles B) trades of goods or services for goods or services, trades of goods or services for assets, and trades of assets for assets C) trades of imports for exports, trades of exports for imports, and trades of natural resources for financial assets D) trades of services for goods, trades of currency for services, and trades of one type of currency for another E) trades of current goods for future services, trades of currency for gold, and trades of one type of currency for another Answer: B Difficulty: Easy AACSB: Application of knowledge 8) Which of the following statements is the MOST accurate? A) By providing a worldwide payments system that lowers transaction costs, banks active in the international capital market enlarge the trade gains that result from such exchanges. B) By providing a worldwide payments system that raises transaction costs, banks active in the international capital market enlarge the trade losses that result from such costs. C) By providing a worldwide cryptocurrency payments system, banks active in the international capital market maximize customers' gains from trade exchanges. D) By providing a worldwide risk-free payments system of digital currencies, banks active in the international capital market minimize customers' losses from trade exchanges. Answer: A Difficulty: Easy AACSB: Application of knowledge
2 Copyright © 2022 Pearson Education, Inc.
9) Intertemporal trade is A) the exchange of goods but not services for claims to future goods. B) the exchange of services but not goods for claims to future services. C) the exchange of goods and services for claims to future goods and services. D) the exchange of domestic goods and services for foreign goods and services. E) the type of trade that the U.S. government focuses most upon. Answer: C Difficulty: Easy AACSB: Application of knowledge 10) The importers' ability to obtain foreign goods today in return for a promise to repay soon after they have sold the goods to domestic customers is an example of A) intertemporal trade. B) trade of assets for assets. C) risk-free trade. D) pure asset swap. Answer: A Difficulty: Easy AACSB: Application of knowledge 11) Trade in assets between countries, like trades involving goods and services, A) can yield benefits to all the countries involved. B) will always make all countries worse off. C) usually benefits one country and harms the other. D) never bring benefits to any country involved. Answer: A Difficulty: Easy AACSB: Application of knowledge 12) When selecting assets, people make decisions based on the riskiness of each asset's return. People dislike risk and economists call this property of people's preferences A) risk averse. B) risk dislike. C) risk neutral. D) risk loving. Answer: A Difficulty: Easy AACSB: Application of knowledge
3 Copyright © 2022 Pearson Education, Inc.
13) What is the basic motive for asset trade? A) the belief that large risks will lead to large returns B) restoration of the balance of payments C) portfolio unification D) economic stability E) increase expected returns and reduced risk Answer: E Difficulty: Easy AACSB: Application of knowledge 14) Using international asset trade, countries can A) never really eliminate all risk. B) eliminate all risk. C) actually increase their risk in some cases. D) eliminate all their risk except for emerging markets. E) never really diversify their holdings. Answer: A Difficulty: Easy AACSB: Application of knowledge 15) If you are offered a gamble in which you win 500 dollars 3/8 of the time and you lose 500 dollars 5/8 of the time, what is your expected payoff and your behavior given that you are a risklover? A) $500, take the gamble B) -$125, take the gamble C) -$125, it is unclear what you would do without further information D) $500, decline the gamble E) -$125, decline the gamble Answer: C Difficulty: Easy AACSB: Application of knowledge 16) Suppose one is offered a gamble in which you win $1,000 half the time but lose $1,000 half the time. Since in this case one is as likely to win as to lose the $1,000, the average payoff on this gamble—its expected value—is: 0.5 ∗ $1,000 + 0.5 ∗ (-$1,000) = 0. Under such circumstances A) no one will take the gamble. B) risk averse individuals will take the gamble. C) risk lovers individuals will not take the gamble. D) risk neutral individuals will not take the gamble. E) risk lovers and risk neutral individuals may take the gamble. Answer: E Difficulty: Easy AACSB: Application of knowledge 4 Copyright © 2022 Pearson Education, Inc.
17) For most practical matters, economists assume that A) individuals are risk neutral. B) individuals are risk lovers. C) individuals are risk averse. D) most individuals are risk lovers. E) most individuals are risk neutral. Answer: C Difficulty: Easy AACSB: Application of knowledge 18) People who are risk averse A) value a collection of assets only on the basis of its expected returns. B) value a collection of assets only on the basis of the risk of that return. C) value a collection of assets not only on the basis of its expected returns but also on the basis of the risk of that return. D) are less likely to invest in life insurance. E) are less likely to have a diverse portfolio. Answer: C Difficulty: Easy AACSB: Application of knowledge 19) The idea of risk aversion A) is at odds with the idea of insurance. B) helps explain the profitability of insurance companies. C) has nothing to do with insurance companies. D) helps explain the losses suffered by the insurance industry. E) helps explain why insurance companies in the long run are zero profit companies. Answer: B Difficulty: Moderate AACSB: Application of knowledge 20) Risk averse people A) will never hold bonds denominated in several different currencies because of transaction costs. B) will always hold bonds denominated in several different currencies because of transaction costs. C) may hold bonds denominated in several different currencies. D) may hold bonds denominated in several different currencies only if satisfying the well known interest party condition. E) will hold only domestic bonds because of the home bias effect. Answer: C Difficulty: Easy AACSB: Application of knowledge
5 Copyright © 2022 Pearson Education, Inc.
21) Which of the following statements is the MOST accurate? A) International trade in assets can make both parties to the trade better off by allowing them to reduce the riskiness of return by portfolio diversification. B) International trade in assets can make both parties to the trade worse off by allowing them to increase the riskiness of return by portfolio diversification. C) International trade in assets can make both parties to the trade worse off by allowing them to eliminate all risk by portfolio unification. D) International trade in assets can make both parties to the trade better off by allowing them to eliminate all risk by portfolio unification. Answer: A Difficulty: Easy AACSB: Application of knowledge 22) International trade reduces the risk by allowing both parties to trade to divide their wealth among a wide spectrum of assets. This is described as A) portfolio diversification. B) portfolio unification. C) risk aversion. D) asset swap. Answer: A Difficulty: Easy AACSB: Application of knowledge 23) Imagine that there are two countries, Home and Far Far Away, and that residents of each own only one asset, domestic land yielding an annual harvest of mangoes. Assume that the yield on the land is uncertain. Half the time, Home's land yields a harvest of 5,000 tons of mangoes at the same time as Far Far Away's land yields a harvest of 2,500 tons. The other half of the time the outcomes are reversed. The average for each country mango harvest is A) 2500. B) 2750. C) 3500. D) 3750. E) 3000. Answer: D Difficulty: Easy AACSB: Application of knowledge
6 Copyright © 2022 Pearson Education, Inc.
24) The two types of trade, intertemporal and pure asset swap ________ perfect substitutes, because ________. A) are; they both offer considerable payoff and are equal in the long run B) are; they both involve the smoothing out of now and future consumption C) are not; asset swapping is immediate and involves only assets, while intertemporal trade takes two time periods and involves both assets and goods/services D) could possibly be; different economic states occur at different points in time E) are not; asset swapping never relates to intertemporal trade Answer: D Difficulty: Easy AACSB: Application of knowledge 25) Equity Instruments include A) stocks. B) bonds. C) banks deposits. D) receipts. E) bank statements. Answer: A Difficulty: Easy AACSB: Application of knowledge 26) Asset trades that deal with debt instruments are best described as A) share of stock. B) exchange rate. C) receipts. D) factors. E) bonds or bank deposits. Answer: E Difficulty: Easy AACSB: Application of knowledge 27) Asset trades that deal with equity instruments are best described as A) share of stock. B) exchange rate. C) bonds. D) bank deposits. E) factors. Answer: A Difficulty: Easy AACSB: Application of knowledge
7 Copyright © 2022 Pearson Education, Inc.
28) Who are the main actors in the international capital market? Answer: (1) Commercial banks (2) Corporations (3) Non-bank financial institutions, such as insurance companies, pension funds, and mutual funds. This includes investment banks, which specialize in underwriting sales of stocks and bonds by corporations and in some cases governments. (4) Central banks and other government agencies Difficulty: Moderate AACSB: Application of knowledge 29) Why is the foreign exchange market an important part of the international capital market? Answer: The foreign exchange market is the central component of the international capital market. The exchange rates it sets help determine the profitability of international transactions of all types. The exchange rates show valuable economic signals to households and firms involved in international trade and investment. Difficulty: Moderate AACSB: Application of knowledge 30) What are the three types of transactions between the residents of different countries? Answer: The three types are: (1) Trades of goods and services for goods or services (2) Trades of goods and services for assets, and (3) Trades of assets for assets. Difficulty: Easy AACSB: Application of knowledge 31) How can international trade in assets make both countries better off? Answer: By allowing them to reduce the riskiness of the return on their wealth and by allowing the two parties to diversify their portfolios, i.e., to divide their wealth among a wider spectrum of assets, and thus reduce the amount of money placed on one specific asset. Difficulty: Moderate AACSB: Application of knowledge 32) Explain Tobin's idea of "Don't put all your eggs in one basket." Answer: Trade in assets can make both parties better off by allowing them to reduce the riskiness of the return on their wealth. Portfolio diversification occurs when the wealth is divided among a wide spectrum of assets, so the amount of money riding on each individual asset is reduced. Obviously, this diversification needs to be contrasted against the expected returns of the new assets that you are acquiring. Difficulty: Moderate AACSB: Application of knowledge
8 Copyright © 2022 Pearson Education, Inc.
33) Why is it useful to make a distinction between debt and equity instruments? Answer: Debt instruments such as bonds and bank deposits are repaid regardless of economic circumstances. Equity instruments, like a share of stock, have a payoff that is linked to economic performances. However, remember the possibility of bankruptcy and the real return, which is subject to domestic currency fluctuations. Difficulty: Moderate AACSB: Application of knowledge 34) Suppose you are offered a gamble in which you win $1,000 half the time but lose $1,000 half the time. If you are risk averter will you take the gamble? Answer: Since you are as likely to win as to lose the $1,000, the average payoff on this gamble—its expected value— is: 0.5 ∗ $1,000 + 0.5 ∗ (-$1,000) = 0. If you are risk averse, you will not take the gamble because, for you, the possibility of losing $1000 outweighs the possibility that you will win, although both outcomes are equally likely. Difficulty: Moderate AACSB: Application of knowledge 35) Suppose you are offered a gamble in which you win $1,000 1/3 of the time but lose $800 2/3 of the time. If you are risk lover will you take the gamble? What will your expected payoff be? Answer: The expected payoff would be: 1/3 (+$1,000) + 2/3 (-$800) = -$200. From this calculation, we know that risk-neutral individuals would not take the gamble, but it is not clear what a risk-loving individual would do. Difficulty: Moderate AACSB: Application of knowledge 36) Calculate the expected payoff for the following cases, where q1 and q2 are the probabilities of state 1 and 2, respectively.
Answer:
Difficulty: Easy AACSB: Analytical thinking
9 Copyright © 2022 Pearson Education, Inc.
37) Calculate the expected payoff for the following cases, where q1 and q2 are the probabilities of state 1 and 2, respectively.
Answer:
Difficulty: Easy AACSB: Analytical thinking 38) Complete the following table.
Answer:
Difficulty: Moderate AACSB: Analytical thinking
10 Copyright © 2022 Pearson Education, Inc.
39) Complete the following table.
Answer:
Difficulty: Moderate AACSB: Analytical thinking
11 Copyright © 2022 Pearson Education, Inc.
40) Calculate the expected payoff for the following cases with the formula: (P1) * (payoff if state 1) + (P2) * (payoff if state 2), where P1 and P2 are the probabilities of state 1 and 2, respectively.
Answer:
Difficulty: Easy AACSB: Analytical thinking
12 Copyright © 2022 Pearson Education, Inc.
41) Complete the following table with the formula (P1) * (payoff if state 1) + (P2)* (payoff if state 2), where P1 and P2 are the probabilities of state 1 and 2, respectively.
Answer:
Difficulty: Moderate AACSB: Analytical thinking 42) Define risk aversion and give an example of a risk-averse person? Answer: Risk aversion is a characteristic of a person that dislikes risk. An example of this is someone who takes out rental or home insurance to reduce the risk of some kind of catastrophe from happening. Difficulty: Moderate AACSB: Application of knowledge The following simple two-country question illustrates how countries are made better off by trade in assets. Imagine that there are two countries, Home and Foreign, and that residents of each own only one asset, domestic land yielding an annual harvest of kiwi fruit. Assume that the yield on the land is uncertain. Half the time, Home's land yields a harvest of 100 tons of kiwi fruit at the same time as Foreign's land yields a harvest of 50 tons. The other half of the time the outcomes are reversed. The Foreign's harvest is 100 tons, but the Home harvest is only 50. 43) Calculate the average, for each country of kiwi harvest. Answer: The average for each country of kiwi harvest is: 0.5 ∗ 100 + 0.5 ∗ 50 = 75 tons of kiwi fruit. However, note that the inhabitants of the two countries never know in advance whether the next year will bring feast or famine. Difficulty: Moderate AACSB: Application of knowledge 13 Copyright © 2022 Pearson Education, Inc.
44) Suppose the two countries can trade shares in the ownership of their perspective assets. Further, assume that a Home owner owns a 10 percent share in Foreign land. He will receive 10 percent share in Foreign land, and thus receives 10 percent of the annual Foreign kiwi fruit harvest. Further assume that a Foreign owner of a 10 percent share in Home land is permitted. In this case, a Foreigner is entitled to 10 percent of the Home harvest. Calculate the expected value of kiwi fruit for each investor. Is the investor better off? Answer: Good year at Home: the farmer will get 90 toms of kiwi from home and 5 tons of kiwi from Foreign. Bad years at home: he will get 45 tons of kiwi from his Home and 10 tons from the Foreign country, namely 55 tons. The probability for a good or a bad year is 0.5. Thus the expected returns will now be: 0.5 ∗ 95 + 0.5 ∗ 55 = 75 It is not clear whether the investor is better off or not. If he likes to smooth his consumption, he may be better off. Otherwise, it is impossible to tell without a particular utility function. Difficulty: Difficult AACSB: Application of knowledge 45) Suppose the two countries can trade shares in the ownership of their perspective assets. Further assume that a Home owner owns a 25 percent share in Foreign land. He will receive 25 percent share in Foreign land and thus receives 25 percent of the annual Foreign kiwi fruit harvest. Further assume also that a Foreign owner of a 25 percent share in Home land is permitted. In this case, a Foreigner is entitled to 25 percent of the Home harvest. Calculate the expected value of kiwi fruit for each investor. Answer: Good year at Home: 75 + 12.5 = 87.5 tons with probability 0.5 Bad year at Home: 37.5 from Home and 25 from Foreign = 62.5 with probability 0.5. The expected return is: 0.5 ∗ 87.5 + 0.5 ∗ 62.5 = 75. Difficulty: Difficult AACSB: Application of knowledge
14 Copyright © 2022 Pearson Education, Inc.
46) Suppose the two countries can trade shares in the ownership of their perspective assets without any restrictions. Assume that the consumers in both countries would like to totally smooth their consumption. Describe the outcomes. Answer: In this case, Home residents will buy a 50 percent share of the land in Foreign, and they will pay for it by giving Foreign residents a 50 percent share in Home land. Explain why. To understand why, think about the returns to the Home and Foreign portfolios when both are equally divided between titles to Home and Foreign land. When times are good at Home (and therefore bad in Foreign), each country earns the same return harvest, which is 75 every year with certainty. Half of the Home harvest (100 ton of kiwi fruit) plus half of the Foreign harvest (50 tons of kiwi fruit) or 75 tons of fruit. 0.5 ∗ 100 + 0.5 ∗ 500 = 75. In the opposite case-bad times in Home, good time in Foreign-each country still earns 75 tons of fruit. Thus, we have shown that if the countries hold portfolios equally divided between the two assets, each country earns a certain return of 75 tons of kiwi fruit. This certain return is exactly the same as the average harvest each faced before international asset trade was allowed. This trade completely eliminates the risk faced by both countries without changing average returns. Assuming risk-averse individuals in both countries, the two countries are better off as a result of asset trade. Difficulty: Difficult AACSB: Application of knowledge 47) Suppose that trade in asset is not allowed but the two countries sign a treaty that guarantees the sending of 25 tons of kiwi in good time by the high output country in that season. What will the outcome be of such a treaty? Explain why. Answer: The outcomes will be exactly the same as in Case D above. In other words, rather than signing a treaty, just leaving the financial markets to function will lead us to the desirable results. Difficulty: Difficult AACSB: Application of knowledge 48) Why is portfolio diversification so important in international trade? Answer: Portfolio diversification is important in international trade because parties of the trade are better off by allowing themselves to reduce the risk of the return on their wealth. Traders can divide their wealth among wider amounts of assets in turn reducing the amount of money they have riding on each individual asset. Difficulty: Moderate AACSB: Application of knowledge 49) What is the difference between equity instruments and debt instruments? Answer: Equity instruments are shares of stocks. It is defined as a claim to the firm's profit. Debt instruments are bonds and bank deposits. They specify that the issuer of the instrument must repay a fixed value. Difficulty: Moderate AACSB: Application of knowledge
15 Copyright © 2022 Pearson Education, Inc.
20.2
International Banking and the International Capital Market
1) As a country begins to liberalize its capital account, what would you expect to happen to the difference between the interest rates for similar assets in this country and another country with open capital markets? A) get larger B) get smaller C) stay the same D) It depends on the existing exchange rate. E) exponential divergence Answer: B Difficulty: Moderate AACSB: Application of knowledge 2) What structures make up the international capital markets? A) stock market, IFM, and the World bank B) bond market, foreign exchange rates, IFM, and the World bank C) commercial banks, corporations, non-bank financial institutions, the central banks, and other government agencies D) commercial banks and corporations E) the central banks and non-bank financial institutions Answer: C Difficulty: Easy AACSB: Application of knowledge 3) ________ are at the center of the international capital market. They run international payments mechanism and a broad range of financial activities. A) Commercial banks B) Corporations C) Nonbank financial institutions D) Central banks E) Government agencies Answer: A Difficulty: Easy AACSB: Application of knowledge 4) Corporations routinely finance their investments to obtain foreign sources of funds in the international capital market including A) selling shares of stocks and debt financing. B) opening subsidiaries in foreign countries. C) increasing the domestic and foreign market shares. D) hiring more foreign employees. Answer: A Difficulty: Easy AACSB: Application of knowledge
16 Copyright © 2022 Pearson Education, Inc.
5) Investment banks in the U.S. are A) regular banks specializing in investment projects. B) not banks at all but institutions which specialize in underwriting sales of stocks and bonds. C) special arm of the U.S. government for U.S. banks operating outside the U.S. D) regular banks specializing in investment projects, but allowed to offer limited domestic transactions. E) international banks that are heavily invested in the U.S. Answer: B Difficulty: Easy AACSB: Application of knowledge 6) Credit Suisse, Goldman Sachs, and Lazard Freres are examples of A) commercial banks. B) corporations. C) non-bank financial institutions, such as insurance companies, pension funds, and mutual funds. D) central banks and other government agencies. E) non-profit organizations. Answer: C Difficulty: Easy AACSB: Application of knowledge 7) Which type of main institution in the international capital market most often is involved in foreign exchange intervention? A) central banks B) non-bank financial institutions C) insurance companies D) corporations E) commercial banks Answer: A Difficulty: Easy AACSB: Application of knowledge 8) The scale of transactions in the international capital market has A) grown more quickly than world GDP since the early 1970s. B) grown less quickly than world GDP since the early 1970s. C) grown about the same rate as the world GDP since the early 1970s. D) been fixed by international regulations. E) decreased more quickly than world GDP since the early 1970s. Answer: A Difficulty: Easy AACSB: Application of knowledge
17 Copyright © 2022 Pearson Education, Inc.
9) A business's use of a bank located outside of the home country is called A) offshore banking. B) international banking. C) domestic banking. D) international swapping. Answer: A Difficulty: Easy AACSB: Application of knowledge 10) What are the types of institution banks used to conduct foreign business? A) corporations B) central banks C) commercial banks D) agency offices, subsidiary banks, and foreign branches E) state-owned enterprises Answer: D Difficulty: Easy AACSB: Application of knowledge 11) Offshore banking can take place at which institution? A) agency office only B) subsidiary bank only C) foreign bank only D) subsidiary bank and foreign bank E) agency office, subsidiary bank, and foreign branch Answer: E Difficulty: Easy AACSB: Application of knowledge 12) The difference between an agency office located abroad and a subsidiary bank located abroad is A) an agency office is just a home bank in another country, while a subsidiary bank is controlled by a foreign bank and subject to the same regulations as local banks. B) an agency office is just a home bank in another country, while a subsidiary bank arranges loans and transfers funds but does not accept deposits. C) an agency office arranges loans and transfers funds but does not accept deposits, while a subsidiary bank is controlled by a foreign bank and subject to the same regulations as local banks. D) an agency office arranges loans and transfers funds but does not accept deposits, while a subsidiary bank is just a home bank in a foreign country. E) an agency office is controlled by a foreign bank and subject to the same regulations as local banks, while a subsidiary bank arranges loans and transfers funds but does not accept deposits. Answer: C Difficulty: Easy AACSB: Application of knowledge
18 Copyright © 2022 Pearson Education, Inc.
13) A bank deposit denominated in a currency other than that of the country in which the bank resides is called A) an offshore deposit. B) an onshore deposit. C) an overseas deposit. D) a foreign deposit. Answer: A Difficulty: Easy AACSB: Application of knowledge 14) Which one of the following possibilities is TRUE? A) Much of eurocurrency trading occurs in Europe. B) Much of eurocurrency trading occurs in the United States. C) Eurocurrencies trading occurs everywhere except the United States. D) Eurocurrencies trading occurs everywhere except Europe. E) Eurocurrencies trading occurs everywhere except China. Answer: C Difficulty: Moderate AACSB: Application of knowledge 15) Eurodollars are A) dollar deposits located in the United States. B) dollar deposits located in Europe. C) dollar deposits located outside Europe. D) dollar deposits located outside the United States. E) dollar deposits located outside both Europe and the United States. Answer: D Difficulty: Easy AACSB: Application of knowledge 16) Eurobanks are A) all European Banks. B) all non American banks. C) banks that accept deposits denominated in Eurocurrencies excluding Eurodollars. D) banks that accept deposits denominated in Eurocurrencies including Eurodollars. E) banks that do not take U.S. dollars. Answer: D Difficulty: Easy AACSB: Application of knowledge
19 Copyright © 2022 Pearson Education, Inc.
17) The leading center of Eurocurrency trading is A) New York City. B) Chicago. C) London. D) Paris. E) Frankfurt. Answer: C Difficulty: Easy AACSB: Application of knowledge 18) Besides world trade growth, what can explain the growth of international banking since the 1960s? A) war in the Middle East B) government focus on banking regulation C) an increase in world travel D) the emergence of developing countries like China E) desire of depositors to hold currencies outside the jurisdiction of the countries that issue them Answer: E Difficulty: Easy AACSB: Application of knowledge 19) Regulatory asymmetries can explain why the following places have become main Eurocurrency centers A) the United States. B) Germany. C) Zurich, Somalia, and Mozambique. D) London, Luxembourg, and The United States. E) London, Luxembourg, and Hong Kong. Answer: E Difficulty: Moderate AACSB: Application of knowledge 20) Discuss why commercial banks are at the center of the international capital market. Answer: Commercial banks are at the center of the international capital market, not only because they run the international payments mechanism but also because of the broad range of financial activities they undertake. Bank liabilities consist chiefly of deposits of various maturities, as well as debt and short-term borrowing from other financial institutions, while their assets consist largely of loans (to corporations and governments), deposits at other banks (interbank deposits), and various securities including bonds. Multinational banks are also heavily involved in other types of asset transaction. For example, banks may underwrite issues of corporate stocks and bonds by agreeing, for a fee, to find buyers for those securities at a guaranteed price. Difficulty: Moderate AACSB: Application of knowledge
20 Copyright © 2022 Pearson Education, Inc.
21) Give an example of regulatory asymmetry in international banking? How did it affect the growth of international banking over the past 50 years? Answer: Commercial banks are often free to pursue activities abroad that they could not legally pursue in their home countries. This type of regulatory asymmetry has spurred the growth of international banking over the past 50 years. Another example of regulatory asymmetry is the shadow banking system (money market mutual funds, investment banks, etc). The shadow banking system has grown dramatically and taken up many of the same functions as traditional banking. They have been minimally regulated compared to banks. Difficulty: Easy AACSB: Application of knowledge 22) Describe the role of offshore banking and of offshore currency (eurocurrencies) trading. Answer: Both have mushroomed due to increased international trade, increased multinational corporations and globalization. Students should also emphasize banks' desire to escape domestic government regulations and taxes on financial activities and political concerns from holding deposits in the country that issued them, which increases default concerns on the part of the foreign investors, etc. Difficulty: Moderate AACSB: Application of knowledge 23) Describe at least one major factor that contributed to the growth of international capital market since the early 1970s. Answer: In the industrial world, countries have progressively dismantled barriers to private capital flows across their borders. The widespread adoption of flexible exchange rates since the early 1970s has allowed countries to reconcile open capital markets with domestic monetary autonomy. Difficulty: Easy AACSB: Application of knowledge 24) Explain the growth of international banking since the 1960s. Answer: Besides world trade growth, another factor is the banks' desire to escape domestic government regulations on financial activity (and sometimes taxes) by shifting some of their operations abroad and into foreign currencies. A further factor is in part political: the desire by some depositors to hold currencies outside the jurisdictions of the countries that issue them. Difficulty: Easy AACSB: Application of knowledge 25) Explain why a London Eurobank has a competitive advantage over a bank in New York in attracting dollar deposits. Answer: It can pay more because the London bank is not subject to any reserve requirements (neither American nor British), whereas the New York bank is subject to U.S. reserve requirements. Difficulty: Moderate AACSB: Application of knowledge
21 Copyright © 2022 Pearson Education, Inc.
26) Explain what Eurocurrencies are and why they are significant. Answer: Eurocurrencies are offshore currency deposits denominated in a currency other than that of the country in which the bank resides. This currency is significant because of the rapid expansion of international trade and increased multinational nature of corporations' activities. Difficulty: Moderate AACSB: Application of knowledge 20.3
Banking and Financial Fragility
1) Which of the following statements is NOT true regarding a bank failure? A) A bank fails when its profit is much lower and it is unable to compete with other local banks. B) A bank fails when it is unable to meet its obligations to its depositors and other creditors. C) A bank fails when it is unable to repay its short-term liabilities, including demand deposits without notice. D) A bank fails when some of the bank's borrowers are unable to repay their loans or the bank's assets decline in value for some other reason. E) A bank fails when faced with a large and sudden loss of deposits — a bank run. Answer: A Difficulty: Easy AACSB: Application of knowledge 2) A bank faced with a large and sudden loss of deposits is likely to shut down despite a fundamentally sound balance sheet. Why could this be? A) Banks have accountants that are too optimistic. B) Banks purposely lie about their balance sheets in order to attract more clients. C) Many bank assets are illiquid and cannot be sold quickly to meet deposit obligations without substantial loss to the bank. D) Many banks operate on a budget that exceeds their actual reserves. E) Many banks will shut down to preserve their interest profits. Answer: C Difficulty: Moderate AACSB: Application of knowledge 3) Which of the following statements is TRUE? A) Bank failure is limited to banks that have mismanaged their assets. B) Bank failure is limited to banks that have invested in real estate. C) Bank failure is limited to banks that have invested in government bonds. D) Bank failure is limited to a few banks. E) Bank failure is NOT limited to banks that have mismanaged their assets. Answer: E Difficulty: Easy AACSB: Application of knowledge
22 Copyright © 2022 Pearson Education, Inc.
4) The difference between assets and liabilities or the amount the bank could lose on its assets before it becomes unable to pay off its debts by selling its assets is called A) bank capital. B) bank failure. C) bank run. D) bank balance. Answer: A Difficulty: Easy AACSB: Application of knowledge 5) The ________ a bank's capital, the ________ chance it becomes insolvent due to losses in asset values. A) lower; higher B) lower; lower C) higher; higher D) higher; same Answer: A Difficulty: Easy AACSB: Application of knowledge 6) Which of the following statements is TRUE? A) Bank failures inflict serious financial harm on individual depositors. B) Bank failures do not inflict serious financial harm on individual depositors. C) Bank failures inflict not only serious financial harm on individual depositors, but also harm the macroeconomic stability of the economy. D) Bank failures inflict serious financial harm on individual depositors, but fortunately do not harm the macroeconomic stability of the economy. E) Bank failures only inflict serious financial harm on the macroeconomic stability of the economy. Answer: C Difficulty: Easy AACSB: Application of knowledge 7) In the United States, which of the following safety precautions has the government NOT taken to reduce Bank failures? A) implemented deposits insurance B) bank reserve requirements C) capital requirements and asset restrictions D) required bank examination E) forcibly closing poorly run banks Answer: E Difficulty: Moderate AACSB: Application of knowledge
23 Copyright © 2022 Pearson Education, Inc.
8) Which of the following statements is TRUE for the U.S.? A) The Federal Deposit Insurance Corporation (FDIC) insures bank deposits against losses up to $250,000. B) The Federal Deposit Insurance Corporation (FDIC) insures bank deposits against losses up to $100,000. C) The Federal Deposit Insurance Corporation (FDIC) insures bank deposits against losses up to $10,000. D) The Federal Deposit Insurance Corporation (FDIC) insures bank deposits against natural disaster up to $100,000. E) The Federal Deposit Insurance Corporation (FDIC) insures bank deposits against floods up to $100,000. Answer: A Difficulty: Easy AACSB: Application of knowledge 9) Which of the following statements is TRUE for the U.S.? A) The FDIC does not provide insurance for deposits for Savings and Loans (S&L) associations. B) The FDIC does provide insurance for deposits for Savings and Loans (S&L) associations, but only up to $50,000. C) The FDIC does provide insurance for deposits for Savings and Loans (S&L) associations up to $250,000. D) The FDIC does provide insurance for deposits for Savings and Loans (S&L) associations up to $150,000. E) The FDIC does provide insurance for deposits for Savings and Loans (S&L) associations up to $100,000. Answer: C Difficulty: Easy AACSB: Application of knowledge 10) Banks in the U.S. A) face rules against lending too large a fraction of their assets to a single private customer only. B) face rules against lending too large a fraction of their assets to a single private customer or to a single foreign government borrower. C) face rules against lending too large a fraction of their assets to a single foreign government borrower only. D) face rules against lending to too many foreign organizations and corporations. E) face rules against lending to other banks. Answer: B Difficulty: Easy AACSB: Application of knowledge
24 Copyright © 2022 Pearson Education, Inc.
11) Banks in the U.S. A) cannot hold common stocks. B) can hold common stocks. C) cannot hold common stocks of companies they do business with. D) cannot hold common stocks of companies that have their headquarters in the same state. E) can hold risky assets. Answer: A Difficulty: Easy AACSB: Application of knowledge 12) Banks in the U.S. A) are prevented from holding assets that are "too risky." B) are not prevented from holding assets that are "too risky." C) are encouraged not to hold assets that are "too risky." D) are not encouraged not to hold assets that are "too risky." E) are encouraged to lend to a single private customer. Answer: A Difficulty: Easy AACSB: Application of knowledge 13) Government supervisors such as a central bank or a separate financial supervision authority A) have the right to examine a bank's books to ensure compliance with bank capital standards and other regulations. B) pay off loans that banks are unable to pay. C) help manage bank over the crisis. D) insure bank depositors against losses. Answer: A Difficulty: Easy AACSB: Application of knowledge 14) In the U.S., banks A) cannot be forced to sell assets that the bank examiner deems too risky. B) can be forced to sell assets that the bank examiner deems too risky. C) can be forced to sell assets that the bank examiner deems too risky only after a court order. D) can be forced to sell assets that the bank examiner deems too risky only after both examiners from the Fed and from the FDIC agree. E) can be forced to trade assets that the bank examiner deems too risky. Answer: B Difficulty: Easy AACSB: Application of knowledge
25 Copyright © 2022 Pearson Education, Inc.
15) In the U.S., banks A) may not be forced by bank examiner to adjust their balance sheets by writing off loans the examiner thinks will not be repaid. B) may be forced by bank examiner to adjust their balance sheets by writing off loans the examiner thinks will not be repaid. C) may be forced by bank examiner to adjust their balance sheets by writing off loans the examiner thinks will not be repaid only if the Fed and the FDIC examiners agree. D) may be forced by bank examiner to adjust their balance sheets by writing off loans the examiner thinks will not be repaid only if the Fed and the Office of the Comptroller of the Currency examiners agree. E) may be forced by bank examiner to adjust their balance sheets by paying off loans the examiner thinks will not be repaid. Answer: B Difficulty: Easy AACSB: Application of knowledge 16) U.S. reserve requirements A) are rejected by half the banks operating in the United States. B) show how regulatory asymmetries can operate to enhance the profitability of Eurocurrency trading. C) tend to harm the bank's business and decrease monetary aggregates. D) force banks to hold a portion of its assets in a liquid form easily mobilized to meet sudden deposit outflows. E) remain in place, but capital requirements have begun defaulting. Answer: D Difficulty: Easy AACSB: Application of knowledge 17) When a central bank lends to banks facing massive deposit outflows as much as they need to satisfy their depositors' claims, it is acting as a ________ to safeguard against financial panic conditional on the bank's sound management. A) lender of last resort B) life saver C) problem solver D) risk taker Answer: A Difficulty: Easy AACSB: Application of knowledge
26 Copyright © 2022 Pearson Education, Inc.
18) The case where people purposely act in a careless way, for example, driving recklessly because they are insured, is called A) asymmetric information. B) risk aversion. C) moral hazard. D) bounded rationality. E) thrill-seeking. Answer: C Difficulty: Easy AACSB: Application of knowledge 19) The ________ that results from a combination of perceived government guarantees and weak regulations of the guaranteed institutions has helped fuel excessively speculative investment in many economies. A) moral hazard B) risk aversion C) asymmetric information D) bounded rationality E) thrill-seeking Answer: A Difficulty: Easy AACSB: Application of knowledge 20) What is "too big to fail" policy? A) It is a government's policy to protect big banks when they get into trouble because their failures may set off a chain reaction that throws the entire financial system into crisis. B) It is a government's policy to prevent small and medium sized banks from becoming too big to get into trouble. C) It is a government's policy to make banks become more interconnected so that their creditors can be fully protected. D) It is a government's policy to avoid a vicious circle of moral hazard to stabilize the financial system. Answer: A Difficulty: Easy AACSB: Application of knowledge 21) Many observers believe that the largely unregulated nature of global banking activity leaves the world financial system vulnerable to bank failure on a massive scale. Is this a real threat? If so, what measures have governments taken to reduce it? Answer: Yes, this is a real threat. A high level of inter-bank depositing implies that problems affecting a single bank could be highly contagious and could spread quickly to banks with which it is thought to do business. In response to this threat, the Basel Committee was created in 1974 to coordinate the surveillance and regulation among bank regulators from different countries. Difficulty: Moderate AACSB: Application of knowledge
27 Copyright © 2022 Pearson Education, Inc.
22) "Bank failure may not be limited to banks that have mismanaged their assets." Explain why? Answer: A sound bank faced with the wholesale loss of deposits is likely to close its door even if the asset side of its balance sheet is fundamentally sound because many bank assets are illiquid and cannot be sold quickly to meet deposit obligations without substantial loss to the bank. Students should stress the atmosphere of financial panic, which will lead to self-fulfilling expectations. Difficulty: Moderate AACSB: Application of knowledge 23) "It is in the interest of each depositor to withdraw her money from a bank if all other depositors are doing the same, even when the bank's assets are sound." Discuss. As part of your answer clearly state whether the statement is true or false. Answer: True. Students should elaborate on bank run and stress the atmosphere of financial panic, which will lead to self-fulfilling expectations. Difficulty: Moderate AACSB: Application of knowledge 24) "There is evidence that the string of U.S. bank closings in the early 1930s helped start and worsen the Great Depression." Discuss. Answer: Bank failure can harm the economy's macroeconomic stability (keep in mind that safeguards intended to reduce the risk of bank failure were put in place after the Great Depression). One bank's problems may spread to sounder banks if they are suspected of having lent to the bank in trouble. This leads to a drastic reduction in the banking system's ability to finance investment and consumer-durable expenditure, so the economy slips into recession (or worse in the case of the Great Depression). Difficulty: Moderate AACSB: Application of knowledge 25) Describe the extensive "safety net" that has been set up in the United States in order to reduce the risk of bank failure. Answer: (1) Deposit insurance (2) Reserve requirements (3) Capital requirements and asset restrictions (4) Bank examination (5) Lender of last resort facilities Difficulty: Moderate AACSB: Application of knowledge 26) What is moral hazard and how to avoid it? Answer: The possibility that someone who is insured against an accident will take less care to prevent it is called moral hazard. Domestic bank supervision and balance-sheet restrictions are necessary to limit the moral hazard resulting from deposit insurance and access to the lender of last resort, which otherwise would lead banks to make excessively risky loans and inadequate provision for their possible failure. Difficulty: Easy AACSB: Application of knowledge 28 Copyright © 2022 Pearson Education, Inc.
27) Explain why the FDIC is following a "too-big-to-fail" policy of fully protecting all depositors at the largest banks. Answer: A tricky question. The FDIC does that although, officially, it still applies only to the $250,000 limit. Probably, the idea is that the cost to one depositor is unbearable relative to the small cost we all need to pay as a collective. Also, there are the issues of political pressure and avoiding blaming the government for mismanaging the safeguards. Difficulty: Moderate AACSB: Application of knowledge 28) Why are economists increasingly in favor of curbs on the size of financial firms instead of "too big to fail" policy? Answer: When a financial institution is "too big to fail" or "too interconnected to fail"–its managers and creditors expect that the government will have no choice but to support it in case it gets into trouble. The resulting moral hazard sets off a vicious circle of firms getting bigger, more profit, more growth, and more moral hazard; therefore makes the entire financial system becomes less stable. For this reason, economists are increasingly in favor of curbs on the size of financial firms despite the possible sacrifice of scale efficiencies. They favor forcing large complex banks and shadow banks to be closed and wound down, in case of insolvency, with minimal disruption and minimal cost to taxpayers. The credible threat of bank closure is necessary for limiting moral hazard–bank managers need to know they can be put out of business if they misbehave–but devising concrete procedures is not easy, especially in an international context. Difficulty: Easy AACSB: Application of knowledge 20.4
How Well Have International Financial Markets Allocated Capital and Risk?
1) What is a difficulty encountered in regulating international banking? A) excessive deposit insurance rates on international banks B) absence of overseas reserve requirements C) oppressive regulatory controls that reduce competitiveness D) lack of funds and incentive to secure payments E) variability in exchange rates Answer: B Difficulty: Moderate AACSB: Application of knowledge 2) The process of banks shifting risky business that home regulators might question to regulatory jurisdictions where fewer questions are asked is called A) regulatory arbitrage. B) regulatory asymmetry. C) regulatory shifting. D) regulatory capture. Answer: A Difficulty: Easy AACSB: Application of knowledge 29 Copyright © 2022 Pearson Education, Inc.
3) A financial trilemma constrains what policy makers in an open economy can achieve. At most two goals from the following list of three are simultaneously feasible A) financial stability, national control over financial safeguard policy, freedom of international capital movements. B) financial security, fiscal decentralization, system scalability. C) national sovereignty, democracy, and globalization economic integration. D) independent monetary policy, fixed exchange rate, freedom of international capital movements. Answer: A Difficulty: Easy AACSB: Application of knowledge 4) The purpose of the Basel Committee was to A) achieve a better coordination of the surveillance exercised by national authorities over the international banking system. B) achieve a better coordination of domestic banking systems. C) achieve a better coordination between brokers and investment bankers. D) achieve a better coordination between bond holder and bond issuers. E) manipulate bank rates for more leverage profits. Answer: A Difficulty: Moderate AACSB: Application of knowledge 5) The Basel Committee was set up in 1974 to A) strengthen the regulation, supervision and practices of banks worldwide with the purpose of enhancing financial stability. B) achieve a better coordination of domestic banking systems. C) strengthen coordination between brokers and investment bankers. D) achieve a better coordination between bond holder and bond issuers. E) manipulate bank rates for more leverage profits. Answer: A Difficulty: Easy AACSB: Application of knowledge 6) The Basel committee A) takes advantages of loopholes in multinational banks. B) does not support regulatory agencies that monitor the assets of banks' foreign subsidiaries. C) submitted its Concordat in 1975 and was then disbanded. D) continues to be the major forum for cooperation in the regulation of international banking. E) met for the first time in 1975. Answer: D Difficulty: Easy AACSB: Application of knowledge
30 Copyright © 2022 Pearson Education, Inc.
7) Capital markets of poor developing countries that liberalized their financial systems to allow private asset trade with foreigners are called A) direct foreign markets. B) foreign exchange markets. C) stock & bond markets. D) emerging markets. E) fledgling financial markets. Answer: D Difficulty: Easy AACSB: Application of knowledge 8) Which statement is NOT true regarding emerging markets? A) Emerging market financial institutions have generally proven to be weaker than those in industrialized countries. B) Emerging markets are the capital markets of poorer, developing countries that have liberalized their financial system to allow private asset trade with foreigners. C) Countries with emerging markets include Brazil, Mexico, and Thailand. D) Countries with emerging markets have been unable to liberalize their financial systems to allow private trade with foreigners. E) Emerging market financial institutions contributed to the financial crisis of 1997-1999. Answer: D Difficulty: Easy AACSB: Application of knowledge 9) What is an appropriate definition for "securitization"? A) the repackaging of bank assets into readily marketable forms B) the promise of a secure return on deposits in FDIC banks C) the simplification of interest-bearing assets into their simplest derivative form D) the unloading of derivative securities in response to a bank run E) the reinforcement of an asset's worth through official certification Answer: A Difficulty: Easy AACSB: Application of knowledge 10) What caused a major economic shock in August 2007? A) U.S. mortgage market B) war in Iraq C) U.S. bond market D) technology stocks E) misreporting from Asian markets Answer: A Difficulty: Easy AACSB: Application of knowledge
31 Copyright © 2022 Pearson Education, Inc.
11) The severity and breadth of the 2007—2009 crisis have led to initiatives to reform both national financial systems and the international system in order to A) fill gaps in existing regulatory frameworks while also paying more attention to the macroeconomic causes and consequences of banking problems. B) avoid financial crisis permanently in the future. C) make it easy for banks to get around capital requirements. D) to set up an omniscient global financial authority responsible for monitoring and regulating the systems. Answer: A Difficulty: Easy AACSB: Application of knowledge 12) ________ was proposed in 2010 as a tougher set of capital standards and regulatory safeguards for international banks. A) Basel III B) Basel II C) Basel I D) FSB Answer: A Difficulty: Easy AACSB: Application of knowledge 13) In April 2009, Financial Stability Board was established to A) monitor the global financial system and make recommendations for global policy coordination and reform. B) implement Base III recommendations. C) regulate systemically important nonbank financial institutions. D) eliminate the problem of "too big to fail." Answer: A Difficulty: Easy AACSB: Application of knowledge 14) The macroprudential perspective on financial regulation seeks A) to ensure the financial system as a whole is sound. B) to ensure that each individual financial institution is sound and that is enough. C) to make each individual institution more resilient without worrying about the whole system. D) to reduce moral hazard in the financial markets. Answer: A Difficulty: Easy AACSB: Application of knowledge
32 Copyright © 2022 Pearson Education, Inc.
15) National financial regulators often face fierce lobbying from their home financial institutions, which argue that A) stricter rules would put them at a disadvantage relative to foreign rivals. B) stricter rules would give them an advantage relative to foreign rivals. C) stricter rules would have no effect on their disadvantage relative to foreign rivals. D) stricter rules would put home institutions and foreign rivals on an equal and fair playing field. Answer: A Difficulty: Easy AACSB: Application of knowledge 16) Describe the financial trilemma for international policy makers. Answer: The difficulties in regulating international financial institutions show that a financial trilemma constrains what policy makers in an open economy can achieve. At most two goals from the following list of three are simultaneously feasible: 1. Financial stability. 2. National control over financial safeguard policy. 3. Freedom of international capital movements. Difficulty: Easy AACSB: Application of knowledge 17) Explain why an international banking system is harder to regulate than a national system. Answer: Students can look at how the effectiveness of the U.S. safeguards is reduced owing to offshore banking activities covered in the textbook and during the lecture. (1) No deposit insurance, in particular, inter-bank deposits are unprotected (2) Absence of reserve requirements (3) Bank examination more difficult to enforce (4) Not clear which group of regulators has responsibility for monitoring a given bank's assets (5) Not clear which central bank, if any, is responsible for providing Lender of Last Resort assistance. Difficulty: Moderate AACSB: Application of knowledge 18) "The internationalization of banking has weakened national safeguards against banking collapse, but at the same time it has made the need for effective safeguards more urgent." Discuss. Answer: True. Students should elaborate on: (1) No deposit insurance, in particular, inter-bank deposits are unprotected (2) Absence of reserve requirements (3) Bank examination more difficult to enforce (4) Not clear which group of regulators has responsibility for monitoring a given bank's assets (5) Not clear which central bank, if any, is responsible for providing Lender of Last Resort assistance. Difficulty: Moderate AACSB: Application of knowledge
33 Copyright © 2022 Pearson Education, Inc.
19) Explain the difficulties in regulating international banking. Answer: (1) No deposit insurance, in particular, inter-bank deposits are unprotected (2) Absence of reserve requirements (3) bank examination more difficult to enforce (4) not clear which group of regulators has responsibility for monitoring a given bank's assets (5) not clear which central bank, if any, is responsible for providing Lender of Last Resort assistance. Difficulty: Moderate AACSB: Application of knowledge 20) Explain the issues involved with the Fed acting as a Lender of Last Resort (LLR). Answer: On the one hand, LLR enables the Fed to avoid panic and disturbance to proper functioning of financial markets. On the other hand, using the policy may cause problems of moral hazard. Difficulty: Moderate AACSB: Application of knowledge 21) Who is the Basel Committee? Discuss both their involvement in the Concordat as well as the role of the Concordat in international banking. Answer: In 1975, the Basel Committee reached an agreement, called the Concordat, which allocated responsibility for supervising multinational banking establishments. In addition, the Concordat called for the sharing of information about banks and the granting of permission to inspect any such banks. Difficulty: Moderate AACSB: Application of knowledge 22) What is securitization? Answer: The term refers to financial instrument in which bank assets are repackaged in readily marketable forms. This kind of "derivatives," although useful for the international investors and the banks that underwrite them, causes huge problems in government abilities to monitor bank assets and independently assess their risk to the soundness of the international banking system. Difficulty: Moderate AACSB: Application of knowledge 23) Explain the source of the global financial crisis of 2007-2009? Answer: Students can answer based on the case study covered in the textbook and during the lecture. Difficulty: Easy AACSB: Application of knowledge
34 Copyright © 2022 Pearson Education, Inc.
20.5
Metrics for International Capital Market Performance
1) The present structure of the international capital market involves risks of financial instability that can be reduced through all of the following EXCEPT A) enhancement of trade barriers. B) close cooperation of bank and financial supervisors in many countries. C) portfolio diversification when residents of different countries trade risky assets. D) innovative way around national regulations that also provide important gains for customers. E) rapid flow of information about investment opportunities. Answer: A Difficulty: Easy AACSB: Application of knowledge 2) What are three things to measure for in evaluating the performance of the capital markets? A) level of intertemporal trade, international trade, portfolio diversification B) level of portfolio diversification, balanced capital accounts, global inflation C) level of portfolio diversification, intertemporal trade, efficiency of foreign exchange D) onshore-offshore interest rate parity, level of portfolio diversification, stability of eurocurrency market E) onshore-offshore interest rate parity, interest parity and foreign exchange, balanced capital accounts Answer: C Difficulty: Easy AACSB: Application of knowledge 3) Investors much prefer to invest in domestic stocks rather than diversifying abroad. This situation is called A) home bias. B) foreign bias. C) domestic bias. D) national bias. Answer: A Difficulty: Easy AACSB: Application of knowledge 4) Why might a country's savings rate have a high positive correlation to its investment rate? A) A country's gains from intertemporal trade may have been large. B) governments' regulation to avoid inflation C) A country's savings rate and investment rate are generally not positively correlated but rather have negative correlation. D) governments' regulation to avoid large current account balances E) A government has not practiced sufficient fiscal regulation. Answer: D Difficulty: Easy AACSB: Application of knowledge
35 Copyright © 2022 Pearson Education, Inc.
5) From 1990 to 2019, OECD countries' saving and investment ratios to output tend to be A) positively related. B) negatively related. C) completely unrelated. D) randomly related. Answer: A Difficulty: Easy AACSB: Application of knowledge 6) Large differences in interest rates between countries would indicate that A) the global market is thriving. B) there is good communication between countries about potential global investment opportunities. C) there are unrealized gains from trade. D) the market is in danger of collapse. E) the supply growth exceeds the aggregate demand. Answer: C Difficulty: Easy AACSB: Application of knowledge 7) Statistical studies of the relationship between interest rates and later depreciation rates show that A) the interest difference has been a very bad predictor in the large swings of exchange rates. B) the interest difference has been an accurate predictor in the large swings of exchange rates. C) the interest difference has correctly predicted the direction in which exchange rates would change. D) the interest difference has not yet been studied as a predictor in the large swings of exchange rates. E) the interest difference is unrelated to the large swings of exchange rates. Answer: A Difficulty: Easy AACSB: Application of knowledge 8) In the Interest Parity Condition, Rt - R t = ( rate differential and (
- Et)/Et + xt, where Rt - R t is the interest
- Et)/Et is the expected change in the exchange rate, what does xt
stand for if it potentially is a market efficient difference between the two? A) market inefficiency B) risk premium C) forecast error D) tracking error E) excessive volatility Answer: B Difficulty: Easy AACSB: Application of knowledge 36 Copyright © 2022 Pearson Education, Inc.
9) Departures from interest parity A) can be explained using theories of risk premium. B) cannot be explained using theories of risk premium. C) may or may not be able to be explained using theories of risk premium, more research is needed. D) are completely unrelated to risk premium. E) occur when risk premium is over calculated. Answer: C Difficulty: Moderate AACSB: Application of knowledge 10) A random walk model can more accurately predict exchange rates as compared to a sophisticated forecast A) always. B) for forecasts up to a year away. C) for forecasts longer than a year away. D) never. E) because of the predictability of exchange rates. Answer: B Difficulty: Easy AACSB: Application of knowledge 11) The exchange rate disconnect puzzle refers to A) the remarkable weak short-term relationship between the exchange rate and the rest of the economy. B) the remarkable strong short-term relationship between the exchange rate and the rest of the economy. C) the remarkable weak long-term relationship between the exchange rate and the rest of the economy. D) the remarkable strong long-term relationship between the exchange rate and the rest of the economy. Answer: A Difficulty: Easy AACSB: Application of knowledge 12) Which of the following is TRUE about exchange rates? A) They should not be volatile because they will determine the economic climate. B) They are generally more volatile than stock prices. C) They are more volatile than several underlying factors that move them such as money supplies and fiscal variables. D) They should be volatile because to correct price signals they adjust quickly in response to economic news, but they are generally less volatile than stock prices. E) They never overreact to economic news. Answer: D Difficulty: Moderate AACSB: Application of knowledge 37 Copyright © 2022 Pearson Education, Inc.
13) Explain why, according to Feldstein and Horioka, one should expect that domestic investment rates diverge widely from saving rates. Answer: The decisions of corporations to invest and for households to save are different. Thus globalization means more and different ways to save and invest and increasing the differences between the two ratios. Difficulty: Moderate AACSB: Application of knowledge 14) Explain why large interest rate differences would be strong evidence of unrealized gains from trade. Answer: The difference between onshore and offshore interest rates on similar assets denominated in the same currency should be small if information about global investment opportunities is transmitted efficiently. Data show that this is the case for the U.S., Germany, Japan and the Netherlands. Difficulty: Moderate AACSB: Application of knowledge 15) Discuss studies based on the interest parity conditions. Answer: In general, the formula does not hold and is not a good predictor of future devaluations. Even worse, it failed to predict correctly even the direction in which the spot exchange rate would change. Difficulty: Moderate AACSB: Application of knowledge 16) Explain the forecast error, ut+1, in terms of: (1) Its equation (what it is equal to) (2) How it is used (3) Its accuracy Answer: (Et+1 - Et)/Et - (
- Et)/Et this equation represents actual minus expected
depreciation, where ut+1 is the forecast error made in predicting future depreciation. Under interest parity the equation ((Et+1 - Et)/Et - (Rt - Rt ) would also be correct, this equation represents the actual currency depreciation minus the interest difference. Statistical methods have been used to see if the forecast error is predictable through the use of past information. Indeed, a number of researchers have found that ut+1 CAN be predicted. Difficulty: Difficult AACSB: Application of knowledge
38 Copyright © 2022 Pearson Education, Inc.
17) Does the interest rate parity hold in each of the following cases? In case it does not hold, calculate the risk premium. A. Rt = 5,
= 3, Et = 5,
=7
B. Rt = 6,
= 3, Et = 5,
=7
C. Rt = 7,
= 3, Et = 5,
=7
D. Rt = 8,
= 3, Et = 5,
=7
E. Rt = 5,
= 4, Et = 5,
=7
F. Rt = 5,
= 5, Et = 5,
=7
G. Rt = 5,
= 6, Et = 5,
=7
H. Rt = 5,
= 3, Et = 6,
=7
I.
Rt = 5,
= 3, Et = 7,
=7
J.
Rt = 5,
= 3, Et = 8,
=7
K. Rt = 5,
= 3, Et = 5,
=8
L. Rt = 5,
= 3, Et = 5,
=9
M. Rt = 5,
= 3, Et = 5,
= 10
39 Copyright © 2022 Pearson Education, Inc.
Answer:
Difficulty: Moderate AACSB: Application of knowledge
40 Copyright © 2022 Pearson Education, Inc.
18) Does the interest rate parity hold in each of the following cases? In case it does not hold, calculate the risk premium. N. Rt = 8,
= 3, Et = 5,
=7
O. Rt = 7,
= 4, Et = 6,
=8
P. Rt = 6,
= 5, Et = 5,
=7
Q. Rt = 5,
= 4, Et = 6,
=8
R. Rt = 8,
= 6, Et = 5,
=7
S. Rt = 7,
= 5, Et = 6,
=8
T. Rt = 6,
= 4, Et = 5,
=7
U. Rt = 5,
= 3, Et = 6,
=8
V. Rt = 8,
= 5, Et = 6,
=8
W. Rt = 7,
= 3, Et = 8,
=9
X. Rt = 6,
= 7, Et = 7,
=8
Y. Rt = 5,
= 6, Et = 5,
=9
Z. Rt = 4,
= 3, Et = 5,
= 10
41 Copyright © 2022 Pearson Education, Inc.
Answer:
Difficulty: Difficult AACSB: Application of knowledge 19) Assume interest parity holds. Calculate
for each of the following cases.
Answer:
Difficulty: Moderate AACSB: Application of knowledge
42 Copyright © 2022 Pearson Education, Inc.
20) Assume interest parity holds. Calculate
for each of the following cases.
Answer:
Difficulty: Moderate AACSB: Application of knowledge
43 Copyright © 2022 Pearson Education, Inc.
21) Does the interest rate parity hold in each of the following cases by using the formula Rt - Rt = (Ee+1 - Et)/Et ? In case it does not hold, calculate the risk premium. 1. Rt = 5,
= 3, Et = 5,
= 17
2. Rt = 6,
= 4, Et = 5,
=5
3. Rt = 7,
= 5, Et = 5,
=8
4. Rt = 8,
= 3, Et = 5,
= 15
5. Rt = 5,
= 4, Et = 6,
=6
6. Rt = 5,
= 5, Et = 3,
= 10
7. Rt = 5,
= 6, Et = 2,
=8
8. Rt = 5,
= 3, Et = 0,
= 20
9. Rt = 5,
= 9, Et = 7,
=3
10. Rt = 5,
= 10, Et = 8,
=7
44 Copyright © 2022 Pearson Education, Inc.
Answer:
Difficulty: Difficult AACSB: Application of knowledge 22) How well has the international capital market performed? Answer: International portfolio diversification has increased. Difficulty: Moderate AACSB: Application of knowledge 23) What does the ambiguous evidence on the foreign exchange market's performance suggest? Answer: The ambiguous evidence on the foreign exchange market's performance suggests an open-minded view. A judgment that the market is doing its job well would support a laissez-faire attitude by governments and a continuation of the present trend toward increased cross-border financial integration in the industrial world. A judgment of market failure, on the other hand, might imply a need for increased foreign exchange intervention by central banks and a reversal of the global trend toward external financial liberalization. The stakes are high, and we need more research and experience before reaching a firm conclusion. Difficulty: Easy AACSB: Application of knowledge
45 Copyright © 2022 Pearson Education, Inc.
International Economics: Theory and Policy, 12e (Krugman) Chapter 21 Optimum Currency Areas and the Euro 21.1
How the European Single Currency Evolved
1) On January 1, 1999, ________ member countries of the European Union (EU) adopted a common currency, the euro. A) 11 B) 8 C) 19 D) 20 Answer: A Difficulty: Easy AACSB: Application of knowledge 2) The EMU created a currency area with more than A) 200 million consumers. B) 250 million consumers. C) about a billion. D) 500 million consumers. E) 340 million consumers. Answer: E Difficulty: Easy AACSB: Application of knowledge 3) Which of the following is TRUE? A) All European countries are part of the EMU. B) All Western European countries are part of the EMU. C) Originally, 20 countries joined the EMU on January 1999. D) No Western European countries are part of the EMU. E) Not all Western European countries are part of the EMU. Answer: E Difficulty: Easy AACSB: Application of knowledge 4) The birth of the Euro A) resulted in fixed exchange rates between all EMU member countries. B) resulted in flexible exchange rates between all EMU member countries. C) resulted in crawling-peg exchange rates between all EMU member countries. D) resulted in non currency board exchange rates between all EMU member countries. E) resulted in floating exchange rates between all EMU member countries. Answer: A Difficulty: Easy AACSB: Application of knowledge
1 Copyright © 2022 Pearson Education, Inc.
5) How many countries are in the EMU as of January 1, 2019? A) 9 B) 15 C) 17 D) 19 E) 25 Answer: D Difficulty: Easy AACSB: Application of knowledge 6) The EU countries were prompted to seek closer coordination of monetary policies and greater exchange rate stability in order A) to enhance Europe's role in the world monetary system. B) to turn the European Union into a truly unified market. C) both to enhance Europe's role in the world monetary system and to turn the European Union into a truly unified market. D) both to turn the European Union into a truly unified market and to counter the rise of Japan in international financial markets. E) to homogenize all European cultures. Answer: C Difficulty: Easy AACSB: Application of knowledge 7) Which of the following statements is TRUE? A) The 1957 Treaty of Rome founded the EU and created a custom union. B) The 1957 Treaty of Rome founded the EU. C) The 1957 Treaty of Rome founded the euro. D) The 1957 Treaty of Rome founded the European Central Bank. E) The 1957 Treaty of Rome founded the Stability and Growth Pact known as SGP. Answer: A Difficulty: Easy AACSB: Application of knowledge 8) Did the 1957 Treaty of Rome turn the EU into a truly unified market? A) Yes, it paved the way for the current EMU. B) No, although it established a customs union, it failed to remove barriers to the movement of goods and factors within Europe. C) No, it was only after the German unification and locating the ECB in Frankfurt that unity was achieved. D) No, since the Northern members of the EU had larger endowments of capital and skilled labor. E) No, the Treaty of Rome created more trade barriers between European countries. Answer: B Difficulty: Easy AACSB: Application of knowledge
2 Copyright © 2022 Pearson Education, Inc.
9) Many European leaders agreed with the following EXCEPT A) raising official barriers to the movements of goods and services among its member countries will turn EU into a unified market on the model of the United States. B) economic cooperation and integration among the former belligerents would be the best guarantee against a repetition of the 20th century's two devastating wars. C) by speaking with a single voice on monetary issues, EU countries can defend more effectively their own economic interests in the face of an increasingly self-absorbed United States. D) exchange rate uncertainty, like official trade barriers, was a major factor reducing trade within Europe. Answer: A Difficulty: Easy AACSB: Application of knowledge 10) On June 23, 2016, nearly ________ percent of British voters chose "Leave" on the referendum question "Should the United Kingdom remain a member of the European Union or leave the European Union?" A) 52 B) 45 C) 42 D) 35 E) 12 Answer: A Difficulty: Easy AACSB: Application of knowledge 11) Which of the following statements is TRUE? A) After more than four decades of membership, Brexit caused many by surprise. B) After more than four decades of membership, Brexit was not a surprise. C) After more than four decades of membership, 70 percent of firms had contingency plan in the event of Brexit. D) After more than four decades of membership, Britain's political leaders were prepared for Brexit. Answer: A Difficulty: Easy AACSB: Application of knowledge 12) EMS stands for A) European Monetary System. B) European Mechanism of Stability. C) European Marketing Supervision. D) European Management System. Answer: A Difficulty: Easy AACSB: Application of knowledge
3 Copyright © 2022 Pearson Education, Inc.
13) In March 1979, original participants in the EMS's exchange rate mechanism does NOT include A) Portugal. B) France. C) Germany. D) Italy. E) Belgium. Answer: A Difficulty: Easy AACSB: Application of knowledge 14) The result of the reunification of eastern and western Germany in 1990 A) was a boom in Germany and higher inflation, with no effect on nearby countries. B) was a recession in Germany and lower inflation, with no effect on nearby countries. C) was a boom in Germany and higher inflation, and, with other EMS countries' commitment to fixed exchange rates, a deep recession in nearby countries. D) was a recession in Germany and lower inflation, and, with other EMS countries' commitment to fixed exchange rates, a deep recession in nearby countries. E) was a recession in Germany and lower inflation, causing a boom in nearby countries. Answer: C Difficulty: Easy AACSB: Application of knowledge 15) The German central bank in the European Monetary System, 1979-1998 A) was very inflation-averse. B) was moderately inflation-averse. C) was willing to accept inflation. D) lacked control over inflation since it had fixed its exchange rate. E) lacked sufficient reserves. Answer: A Difficulty: Easy AACSB: Application of knowledge 16) The credibility theory of the EMS implies in effect that the political costs of violating international exchange rate agreements A) cannot restrain governments from depreciating their currency. B) can restrain governments from depreciating their currency. C) cannot restrain governments from depreciating their currency in the short run. D) cannot restrain governments from depreciating their currency in the long run. E) can control the political policies of member nations. Answer: B Difficulty: Easy AACSB: Application of knowledge
4 Copyright © 2022 Pearson Education, Inc.
17) The credibility theory of the EMS implies in effect that the political costs of violating international exchange rate agreements A) cannot restrain governments from depreciating their currency to gain the short-term advantage of an economic boom at the long-term cost of higher inflation. B) can restrain governments from depreciating their currency to gain the short-term advantage of an economic boom at the long-term cost of higher inflation. C) cannot restrain governments from depreciating their currency in the short run. D) cannot restrain governments from depreciating their currency in the long run. E) cannot restrain governments from depreciating their currency to gain the long-term advantage of an economic boom. Answer: B Difficulty: Easy AACSB: Application of knowledge 18) Under the EMS, Germany set the system's A) monetary policy while the other European countries pegged their currencies to the DM. B) fiscal policy while the other European countries pegged their currencies to the DM. C) monetary policy while the other European countries kept their currencies fluctuating relative to the DM. D) fiscal policy while the other European countries kept their currencies fluctuating relative to the DM. E) monetary policy, while other European countries maintained their traditional policies. Answer: A Difficulty: Easy AACSB: Application of knowledge 19) During the period from 1978-2019, the difference between annual inflation rates of EU countries and the German inflation rate A) grew at an accelerating rate. B) remained fairly constant. C) largely disappeared. D) went through periods of hyperinflation. E) trended upward at a declining rate. Answer: C Difficulty: Easy AACSB: Application of knowledge
5 Copyright © 2022 Pearson Education, Inc.
20) The credibility theory of EMS had as an effect A) the inflation rates of member countries converging to the low German levels, a result that was not matched by similar countries who did not fix their exchange rates. B) the inflation rates of member countries failing to converge to the low German levels. C) the inflation rates of member countries converging to the low German levels, but other countries including U.S. and Britain also reduced inflation in this time period without fixing exchange rates. D) the inflation rate in Germany rose to match the inflation rates of other member countries. E) the inflation rate in the U.S. dropped to the low German levels. Answer: C Difficulty: Easy AACSB: Application of knowledge 21) An inflation-prone country A) gains from vesting its monetary policy decisions with a "conservative" central bank. B) loses from vesting its monetary policy decisions with a "conservative" central bank. C) gains from vesting its fiscal policy decisions with a "conservative" central bank. D) loses from vesting its fiscal policy decisions with a "conservative" central bank. E) remains constant when vesting its fiscal policy decisions with a "conservative" central bank. Answer: A Difficulty: Easy AACSB: Application of knowledge 22) The EU countries have tried to achieve greater internal economic unity through direct measures to A) encourage the free flow of goods, services, and factors of production. B) discourage the free flow of goods, services, and factors of production. C) raise trade barriers within the European Union. D) discourage the free flow of financial capital within the European Union and between the European Union and outside jurisdictions. Answer: A Difficulty: Easy AACSB: Application of knowledge 23) The most important feature of the Single European Act of 1986, which amended the founding Treaty of Rome, was dropping the requirement of A) unanimous consent for measures related to market completion and making it a decision that only Germany and France agreed about. B) unanimous consent for measures related to market completion. C) majority consent for measures related to market completion and making it a decision that only Germany and France agreed about. D) unanimous consent for measures related to agricultural policies only. E) unanimous consent for measures related only to fiscal policies. Answer: B Difficulty: Easy AACSB: Application of knowledge 6 Copyright © 2022 Pearson Education, Inc.
24) The European Economic and Monetary Union (EMU) A) set up a single currency and sole bank for European economic monetary policy. B) eliminated all barriers to trade such as tax differentials between borders. C) produced a single government for handling European affairs. D) created the Common Agricultural Pact. E) eliminated all local currencies in Western Europe. Answer: A Difficulty: Easy AACSB: Application of knowledge 25) The 1991 Maastricht Treaty can be best described as A) a peace treaty between Europe and the United States. B) an agreement for the accession of the Netherlands into the EU. C) an agreement for the creation of a free trade area. D) a provision for the introduction of a single European currency and European central bank. E) the beginning of a floating exchange rate European monetary system. Answer: D Difficulty: Easy AACSB: Application of knowledge 26) Why did British voters decide to leave the EU? Answer: The reasons are complex, but a number of factors stand out. First was the desire to limit immigration from Eastern Europe and the benefits, such as medical care, the U.K. government must provide to immigrants under EU law. Another factor was the panoply of regulations from EU headquarters in Brussels, felt by many to be intrusive, overly detailed, and unnecessary to promote intra-EU trade. Difficulty: Easy AACSB: Application of knowledge 27) How and why did Europe set up its single currency? Answer: The why part is because large fluctuations in the exchange rates among the European countries disturbed trade. Also, one of the main reasons was to design a way to prevent future world war. The how part of the question is related to the collapse of Bretton Woods and the European Currency reform of 1969-1978. The Werner Report of 1971 establishes a three-phase program to lead to the EMU. Difficulty: Moderate AACSB: Application of knowledge 28) How did the European single currency evolve? Answer: The answer is related to the collapse of Bretton Woods and the European Currency reform of 1969-1978. The Werner Report of 1971 establishes three-phase program to lead to the EMU. Difficulty: Moderate AACSB: Application of knowledge
7 Copyright © 2022 Pearson Education, Inc.
29) What prompted the EU countries to seek closer coordination of monetary policies and greater exchange rate stability in the late 1960s? Answer: (1) To enhance Europe's role in the world monetary system (2) To turn the European Union into a truly unified market Difficulty: Moderate AACSB: Application of knowledge 30) Discuss the effects of the reunification of eastern and western Germany in 1990 on both Germany and its neighboring European countries. Answer: Germany: boom, high interest rates to fight inflation. Other European countries: France, Italy and UK in recession, trying to match the high German interest rates to hold their currencies fixed against Germany's, thereby pushing their economies into deep recession. Other European countries tried to continue the fixed exchange rate in order not to lose the credibility they had built up since 1985. The policy conflict between Germany and the other European countries led to a series of fierce speculative attacks on the EMS exchange parities starting in September 1992. By august 1993, the EMS was forced to retreat to very wide (± 10 percent) bands, which is kept in force until the introduction of the euro in 1993. Difficulty: Difficult AACSB: Application of knowledge 31) Explain why the EMS countries decided to fix their exchange rates against the German DM. Answer: In this way, the other EMS countries in effect imported the credibility of the German central bank in fighting inflation, thus discouraging the development of inflationary pressures at home. Difficulty: Moderate AACSB: Application of knowledge 32) Explain the credibility theory of the EMS. Answer: In this way, the other EMS countries in effect imported the credibility of the German central bank in fighting inflation, thus discouraging the development of inflationary pressures at home. This is known as the credibility theory of the EMS. Difficulty: Moderate AACSB: Application of knowledge 33) Explain how the German Bundesbank gained its low-inflation reputation. Answer: Mainly, Germany's experience with hyperinflation in the 1920s and again after World War II left the German electorate with a deeply rooted fear of inflation. The law establishing the Bundesbank singled out the defense of the DM's real value as the primary goal of the German central bank. Difficulty: Moderate AACSB: Application of knowledge
8 Copyright © 2022 Pearson Education, Inc.
34) Describe the effects of the reunification of eastern and western Germany in 1990 on both Germany and its neighboring European countries using the AA-DD framework. Answer: As for Germany a period of boom with high interest rates to fight inflation. Other European countries: France, Italy and UK in recession, trying to match the high German interest rates to hold their currencies fixed against Germany's, thereby pushing their economies into deep recession. Other European countries tried to continue the fixed exchange rate in order not to lose the credibility they had build up since 1985. The policy conflict between Germany and the other European countries led to a series of fierce speculative attacks on the EMS exchange parities starting in September 1992. By August 1993, the EMS was forced to retreat to very wide (± 10 percent) bands, which was kept in force until the introduction of the euro in 1993. Difficulty: Moderate AACSB: Application of knowledge 35) What is the purpose of the following figure?
Answer: The purpose of the figure is to show the inflation convergence within the six original EMS members. The figure shows the difference between domestic inflation and German inflation for six of the original EMS members. As of 2019 all national inflation rates were very close to the German levels. Difficulty: Moderate AACSB: Application of knowledge 9 Copyright © 2022 Pearson Education, Inc.
36) Describe the main provisions of the Maastricht Treaty of 1991. Answer: Called for a single currency by January 1, 1999, harmonizing social security policy within the European Union, and centralizing foreign and defense policy decisions. Difficulty: Easy AACSB: Application of knowledge 37) Why did the EU countries move away from the EMS toward the goal of a single shared currency? Answer: (1) To produce a greater degree of European market integration by removing the threat of EMS currency realignments. (2) Reduce German dominance of the EMS monetary policy. (3) Given the move to complete freedom of capital movements within the EU, fixed but adjustable currency parities, may lead to ferociously speculative attacks, as in 1992-1993. (4) To guarantee the political stability of Europe. Difficulty: Moderate AACSB: Application of knowledge 21.2
The Euro and Economic Policy in the Euro Zone
1) To join the EMU, a country should have no more than A) 1.5 percent inflation rate above the average of the three EU member states with the highest inflation. B) 3 percent inflation rate above the average of the three EU member states with the lowest inflation. C) 4 percent inflation rate above the average of the three EU member states with the lowest inflation. D) 1.5 percent inflation rate above the average of the three EU member states with the lowest inflation. E) 2 percent inflation rate above the average of the three EU member states with the lowest inflation. Answer: D Difficulty: Easy AACSB: Application of knowledge 2) To join the EMU, a country must have A) maintained a stable exchange rate within the ERM without devaluing on its own initiative. B) devalued its currency on its own initiative. C) a volatile exchange rate within the ERM. D) an unstable exchange rate within the ERM with devaluations on its own initiative. Answer: A Difficulty: Easy AACSB: Application of knowledge
10 Copyright © 2022 Pearson Education, Inc.
3) To join the EMU, a country must have A) a public-sector deficit no higher than 3 percent of its GDP in general. B) a public-sector deficit no higher than 2 percent of its GDP in general. C) a public-sector deficit no higher than 1 percent of its GDP in general. D) a zero public-sector deficit. E) a public-sector deficit no higher than 4 percent of its GDP in general. Answer: A Difficulty: Easy AACSB: Application of knowledge 4) To join the EMU, a country must have a public debt below or approaching a reference level of A) 50 percent of its GDP. B) 10 percent of its GDP. C) 60 percent of its GDP. D) 100 percent of its GDP. E) 5 percent of its GDP. Answer: C Difficulty: Easy AACSB: Application of knowledge 5) The main function of the 1997 Stability and Growth Pact (SGP) was to A) exclude a highly indebted EMU country. B) enhance cooperation between France and Germany. C) tighten the fiscal constraints. D) distribute the Euro banknote among European central banks and to create a timetable for the imposition of financial penalties on countries that fail to correct situations of "excessive" deficits and debt promptly enough. E) determine specialized penalties for each member nation. Answer: C Difficulty: Easy AACSB: Application of knowledge 6) The European Central Bank has its headquarter in A) London. B) Berlin. C) Frankfurt. D) Paris. E) Brussels. Answer: C Difficulty: Easy AACSB: Application of knowledge
11 Copyright © 2022 Pearson Education, Inc.
7) The Eurosystem makes decisions by A) votes of the governing council of the European Central Bank (ECB). B) the President of the European Central Bank. C) votes approved by legislatures or voters in every member country of the EU. D) the governors of central banks of non-euro area countries. Answer: A Difficulty: Easy AACSB: Application of knowledge 8) Euro and non-euro central banks in the European System of Central Banks (ESCB) are committed to A) pursue domestic price stability as well as various forms of cooperation with the Eurosystem. B) pursue low inflation under political influences. C) pursue various forms of cooperation with the Eurosystem. D) pursue peace without conflict among member countries. Answer: A Difficulty: Easy AACSB: Application of knowledge 9) Unlike any other central bank in the world, the European Central Bank (ECB) operates A) beyond the reach of any single national government. B) under complete control by the European Parliament. C) by approval from voters in every member country of the EU. D) under control by legislatures in all member countries. Answer: A Difficulty: Easy AACSB: Application of knowledge 10) A revised exchange rate mechanism, referred to as ERM 2, defines broad exchange rate zones against the euro of ± ________ percent. A) 15 B) 10 C) 5 D) 3 E) 1.5 Answer: A Difficulty: Easy AACSB: Application of knowledge
12 Copyright © 2022 Pearson Education, Inc.
11) Under ERM 2 rules, the national central bank of an EU member with its own currency can suspend euro intervention operations A) if there is a civil war. B) if they result in money supply changes that threaten to destabilize the domestic price level. C) if there is a current account deficit. D) if there is a current account surplus. E) if they result in a weakened current account. Answer: B Difficulty: Easy AACSB: Application of knowledge 12) How were the initial members of EMU chosen? How will new members be admitted? What is the structure of the complex of financial and political institutions that govern economic policy in the euro zone? Answer: EU countries should satisfy: (1) Low inflation rate (no more than 1.5 percent above the average of the three EU member states with the lowest inflation). (2) A stable exchange rate within the ERM. (3) Public-sector deficit no higher than 3 percent of its GDP in general. (4) A public debt below or approaching a reference level of 60 percent of its GDP. Difficulty: Difficult AACSB: Application of knowledge 13) What is the purpose of the Stability and Growth Pact (SGP) negotiated by European leaders in 1997? Answer: It tightened the fiscal constraints. The SGP set out "the medium-term budgetary objective of positions close to balance or in surplus." It also set a timetable for the imposition of financial penalties on countries that fail to correct situations of "excessive" deficits and debt promptly enough. Difficulty: Easy AACSB: Application of knowledge 14) What does the Eurosystem do and how are decisions made? Answer: The Eurosystem conducts monetary policy for the euro zone and consists of the European Central Bank (ECB) in Frankfurt plus the 19 national central banks of the euro area. The Eurosystem makes decisions by votes of the governing council of the ECB, consisting of the six-member ECB executive board (including the president of the ECB) and the heads of the national central banks of the euro area. Difficulty: Easy AACSB: Application of knowledge
13 Copyright © 2022 Pearson Education, Inc.
15) What is the revised exchange rate mechanism? Answer: For EU countries that are not yet members of EMU, a revised exchange rate mechanism–referred to as ERM 2–defines broad exchange rate zones against the euro (±15 percent) and specifies reciprocal intervention arrangements to support these target zones. The EU viewed ERM 2 as necessary to discourage competitive devaluations against the euro by EU members outside the euro zone and to give would-be EMU entrants a way of satisfying the Maastricht Treaty's exchange rate stability convergence criterion. Difficulty: Easy AACSB: Application of knowledge 21.3
The Theory of Optimum Currency Areas
1) The European monetary integration process has helped advance the political goals of its founders by giving the European Union A) a stronger position in international affairs. B) a weaker position in international affairs. C) a consensus agreement in all economic and monetary policies. D) full protection from financial and monetary crisis. Answer: A Difficulty: Easy AACSB: Application of knowledge 2) The survival and future development of the European monetary experiment depend on its ability to A) help countries reach their economic goals. B) help non-euro countries to converge in terms of economic growth. C) make its all citizens happy and prosperous. D) cope with economic shocks including Covid-19 pandemic. Answer: A Difficulty: Easy AACSB: Application of knowledge 3) Exchange rate flexibility can have potentially harmful effects EXCEPT A) stabilizing domestic price level. B) making prices less predictable. C) undermining the government's resolve to keep inflation in check. D) cushioning the disruptive impact of various economic shocks. Answer: A Difficulty: Easy AACSB: Application of knowledge
14 Copyright © 2022 Pearson Education, Inc.
4) The theory of optimum currency areas predicts that A) floating exchange rates are most appropriate for areas closely integrated through international trade and factor movements. B) fixed exchange rates are most appropriate for areas that are loosely integrated through international trade and factor movements. C) fixed exchange rates are most appropriate for areas closely integrated through international trade and factor movements. D) floating exchange rates are most appropriate for all countries in Europe. E) fixed exchange rates are most appropriate for all countries in Europe. Answer: C Difficulty: Easy AACSB: Application of knowledge 5) A major economic A) benefit of fixed exchange rates is that they simplify economic calculations and provide a more predictable basis for decisions that involve international transactions than do floating rates. B) benefit of floating exchange rates is that they simplify economic calculations and provide a more predictable basis for decisions that involve international transactions than do fixed rates. C) cost of fixed exchange rates is that they simplify economic calculations and provide a more predictable basis for decisions that involve international transactions than do currency board rates. D) benefit of flexible exchange rates is that they simplify economic calculations and provide a more predictable basis for decisions that involve international transactions than do crawling peg rates. E) benefit of fixed exchange rates is that the value of goods will remain constant across a large region of consumers. Answer: A Difficulty: Moderate AACSB: Application of knowledge 6) The monetary efficiency A) gain from joining the fixed exchange rate system equals the joiner's savings from avoiding the uncertainty, confusion, and calculation and transaction costs that arise when exchange rates float. B) gain from joining the fixed exchange rate system equals the joiner's savings from estimated costs of future predicted exchange rate fluctuation. C) loss from joining the fixed exchange rate system equals the joiner's savings from transaction costs that arise when exchange rates float. D) loss from joining the fixed exchange rate system equals the joiner's savings from uncertainty and confusion when exchange rates float. Answer: A Difficulty: Easy AACSB: Application of knowledge
15 Copyright © 2022 Pearson Education, Inc.
7) The efficiency A) gain from a fixed exchange rate with the euro is smaller when trade between say, Norway and the euro zone, is extensive than when it is small. B) gain from a fixed exchange rate with euro is greater when trade between say, Norway and the euro zone, is extensive than when it is small. C) loss from a fixed exchange rate with the euro is smaller when trade between say, Norway and the euro zone, is extensive than when it is small. D) gain from a fixed exchange rate with euro is the same as when trade between say, Norway and the euro zone, is extensive than when it is small. E) gain from a fixed exchange rate with euro is the same as when trade between say, Norway and the euro zone, is small than when it is small. Answer: B Difficulty: Moderate AACSB: Application of knowledge 8) Other things equal, if Norway's trade with the euro zone amounts to 50 percent of its GNP while its trade with the United States amounts to only 5 percent of GNP, A) a fixed krone/euro exchange rate yields a greater monetary efficiency gain to Norwegian traders than a fixed krone/dollar rate. B) a fixed krone/dollar exchange rate yields a greater monetary efficiency gain to Norwegian traders than a fixed krone/euro rate. C) a fixed krone/euro exchange rate yields a smaller monetary efficiency gain to Norwegian traders than a fixed krone/dollar rate. D) a fixed krone/euro exchange rate yields the same monetary efficiency gain to Norwegian traders as a fixed krone/dollar rate. Answer: A Difficulty: Moderate AACSB: Application of knowledge 9) The monetary efficiency A) loss from pegging the Norwegian krone to the euro (for example) will be higher if factors of production can migrate freely between Norway and the euro area. B) gain from pegging the Norwegian krone to the euro (for example) will be lower if factors of production can migrate freely between Norway and the euro area. C) gain from pegging the Norwegian krone to the euro (for example) will be higher if factors of production can not migrate freely between Norway and the euro area. D) gain from pegging say the Norwegian krone to the euro (for example) will be higher if factors of production can migrate freely between Norway and the euro area. E) gain or loss from pegging the Norwegian krone to the Euro cannot be predicted using the available information. Answer: D Difficulty: Moderate AACSB: Application of knowledge
16 Copyright © 2022 Pearson Education, Inc.
10) A high degree of economic integration between a country and a fixed exchange rate area A) magnifies the monetary efficiency gain the country reaps when it fixes its exchange rate against the area's currencies. B) reduces the monetary efficiency gain the country reaps when it fixes its exchange rate against the area's currencies. C) has no effect on the monetary efficiency gain the country reaps when it fixes its exchange rate against the area's currencies. D) magnifies the monetary efficiency loss the country incurs when it fixes its exchange rate against the area's currencies. Answer: A Difficulty: Easy AACSB: Application of knowledge 11) Which of the following statements is MOST accurate? A) A high degree of economic integration between a country and a fixed exchange rate area that it joins magnifies the monetary efficiency gain the country reaps when it fixes its exchange rate against the area's currencies. B) A low degree of economic integration between a country and a fixed exchange rate area that it joins magnifies the monetary efficiency gain the country reaps when it fixes its exchange rate against the area's currencies. C) A high degree of economic integration between a country and a fixed exchange rate area that it joins reduces the monetary efficiency gain the country reaps when it fixes its exchange rate against the area's currencies. D) A low degree of economic integration between a country and a fixed exchange rate area that it joins has no effect on the monetary efficiency gain the country reaps when it fixes its exchange rate against the area's currencies. Answer: A Difficulty: Easy AACSB: Application of knowledge 12) Which one of the following statements is TRUE? A) The less extensive are cross-border trade and factor movements, the greater is the gain from a fixed cross-border exchange rate. B) The more extensive are cross-border trade and factor movements, the greater is the loss from a fixed cross-border exchange rate. C) The more extensive are cross-border trade and factor movements, the greater is the gain from a fixed cross-border exchange rate. D) The more extensive are cross-border trade, the greater is the loss from a fixed cross-border exchange rate. E) The more extensive are factor movements, the greater is the loss from a fixed cross-border exchange rate. Answer: C Difficulty: Moderate AACSB: Application of knowledge
17 Copyright © 2022 Pearson Education, Inc.
13) The GG schedule shows the relationship between A) a country's degree of economic integration with a fixed exchange rate area and the monetary efficiency gain to the country from joining the area. B) a country's degree of economic integration with a fixed exchange rate area and the monetary efficiency loss to the country from joining the area. C) a country's degree of economic integration with a floating exchange rate area and the monetary efficiency gain to the country from joining the area. D) a country's degree of economic integration with a floating exchange rate area and the monetary efficiency loss to the country from joining the area. Answer: A Difficulty: Easy AACSB: Application of knowledge 14) Why does the GG schedule have a positive slope? A) The monetary efficiency gain a country gets by joining a fixed exchange rate area falls as its economic integration with the area increases. B) The monetary efficiency gain a country gets by joining a fixed exchange rate area rises as its economic integration with the area decreases. C) The monetary efficiency gain a country gets by joining a fixed exchange rate area rises as its economic integration with the area increases. D) The monetary efficiency gain a country gets by joining a floating exchange rate area rises as its economic integration with the area increases. E) The monetary efficiency gain a country gets by joining a fixed exchange rate area is constant after their integration into the area. Answer: C Difficulty: Easy AACSB: Application of knowledge 15) If the euro zone's price level is stable and Norway's exchange rate commitment is firm, A) when Norway pegs to the euro, its efficiency gain from the stability of its currency against the euro is greater the more closely tied are Norway's markets with euro zone markets. B) when Norway pegs to the euro, its efficiency gain from the stability of its currency against the euro is reduced the more closely tied are Norway's markets with euro zone markets. C) when Norway pegs to the euro, there is no impact on its monetary efficiency gain. D) when Norway pegs to the euro, its efficiency loss is greater the more closely tied are Norway's markets with euro zone markets. Answer: A Difficulty: Easy AACSB: Application of knowledge
18 Copyright © 2022 Pearson Education, Inc.
16) When the economy of the pegging country is well integrated with that of the low-inflation area, low domestic inflation is A) easier to achieve. B) harder to achieve. C) impossible to achieve. D) highly uncertain. Answer: A Difficulty: Easy AACSB: Application of knowledge 17) A country that joins an exchange rate area A) gives up its ability to use the exchange rate for the purpose of stabilizing output and employment. B) does not give up its ability to use the exchange rate and monetary policy for the purpose of stabilizing output and employment. C) gives up its ability to use the exchange rate and monetary policy for the purpose of stabilizing output and employment. D) gives up its ability to use only monetary policy for the purpose of stabilizing output and employment. E) does not gives up its ability to use only monetary policy for the purpose of stabilizing output and employment. Answer: C Difficulty: Moderate AACSB: Application of knowledge 18) The LL schedule shows the relationship between A) a country's degree of economic integration with a fixed exchange rate area and the economic stability loss to the country from joining the area. B) a country's degree of economic integration with a fixed exchange rate area and the economic stability gain to the country from joining the area. C) a country's degree of economic integration with a floating exchange rate area and the economic stability loss to the country from joining the area. D) a country's degree of economic integration with a floating exchange rate area and the economic stability gain to the country from joining the area. Answer: A Difficulty: Easy AACSB: Application of knowledge 19) When the economy is disturbed by a change in the output market A) a fixed exchange rate has an advantage over a flexible rate. B) a floating exchange rate has an advantage over a fixed rate. C) a crawling peg exchange rate has an advantage over a flexible rate. D) a floating exchange rate has the same effect as a fixed rate. E) a flexible exchange rate is not as effective as a fixed exchange rate. Answer: B Difficulty: Moderate AACSB: Application of knowledge 19 Copyright © 2022 Pearson Education, Inc.
20) Which one of the following statements is TRUE? A) A fixed exchange rate automatically cushions the economy's output and employment by allowing an immediate change in the relative price of domestic and foreign goods. B) A flexible exchange rate does not automatically cushion the economy's output and employment by allowing an immediate change in the relative price of domestic and foreign goods. C) A flexible exchange rate automatically cushions the economy's output and employment by allowing an immediate change in the relative price of domestic and foreign goods. D) A flexible exchange rate automatically cushions the economy's output and employment by allowing an immediate change in the absolute price of domestic and foreign goods. E) A fixed exchange rate automatically cushions the economy's output and employment by allowing an immediate change in the absolute price of domestic and foreign goods. Answer: C Difficulty: Moderate AACSB: Application of knowledge 21) When the exchange rate is A) flexible, purposeful stabilization is more difficult because monetary policy has no power at all to affect domestic output and employment. B) fixed, purposeful stabilization is less difficult because monetary policy has no power at all to affect domestic output and employment. C) fixed, purposeful stabilization is more difficult because monetary policy has no power at all to affect domestic output and employment. D) a crawling peg, rather than fixed, purposeful stabilization is more difficult because monetary policy has no power at all to affect domestic output and employment. E) fixed rather than crawling peg purposeful stabilization is more difficult because fiscal policy has no power at all to affect domestic output and employment. Answer: C Difficulty: Moderate AACSB: Application of knowledge 22) Economic stability loss is A) the extra instability caused by a country's joining a fixed exchange rate area. B) the extra instability caused by a country's joining a floating exchange rate area. C) the extra instability caused by the monetary authority. D) the extra instability caused by the change in prices of domestic and foreign goods. Answer: A Difficulty: Easy AACSB: Application of knowledge
20 Copyright © 2022 Pearson Education, Inc.
23) When Norway unilaterally fixes its exchange rate against the euro and leaves the krone A) free to float against the non-euro currencies, it is able to keep at least some monetary independence. B) free to float against the non-euro currencies, it is unable to keep at least some monetary independence. C) free to float against the non-euro currencies, it is able to keep its monetary independence. D) run by crawling peg against the non-euro currencies, it is able to keep at least some monetary independence. E) fixed against the non-euro currencies, it is unable to keep its monetary independence. Answer: B Difficulty: Moderate AACSB: Application of knowledge 24) After Norway unilaterally pegs the krone to the euro, domestic money market disturbances will A) no longer affect domestic output despite the continuation of float-rate regime against noneuro currencies. B) now have major effect on domestic output despite the continuation of float-rate regime against non-euro currencies. C) have some effect on domestic output despite the continuation of float-rate regime against noneuro currencies. D) have major effect on domestic employment despite the continuation of float-rate regime against non-euro currencies. E) no longer affect foreign imports despite the continuation of float-rate regime against non-euro currencies. Answer: A Difficulty: Moderate AACSB: Application of knowledge 25) A krone/euro peg alone is A) not enough to provide automatic stability in the face of any monetary shocks that shift the AA schedule. B) enough to provide automatic stability in the face of any monetary shocks that shift the AA schedule. C) not enough to provide automatic stability in the face of any monetary shocks that shift the AA schedule, provided fiscal policy will be used as well. D) enough to provide automatic stability in the face of any monetary shocks that shift the AA schedule, provided the government runs a budget deficit. E) enough to provide partial stability in the face of smaller monetary shocks that shift the AA schedule. Answer: B Difficulty: Moderate AACSB: Application of knowledge
21 Copyright © 2022 Pearson Education, Inc.
26) Since Norway has close trading links with the euro zone A) a small reduction in its price will lead to an increase in euro zone demand for Norwegian goods that is large relative to Norway's output. Thus, full employment can be restored fairly quickly. B) a small reduction in its price will lead to a decrease in euro zone demand for Norwegian goods that is large relative to Norway's output. Thus, full employment can be restored fairly quickly. C) a small reduction in its price will lead to an increase in euro zone demand for Norwegian goods that is small relative to Norway's output. Thus, full employment can be restored fairly quickly. D) a big reduction in its price will lead to an increase in euro zone demand for Norwegian goods that is large relative to Norway's output. Thus, full employment can be restored fairly quickly. E) a big reduction in its price will lead to a decrease in euro zone demand for Norwegian goods that is small relative to Norway's output. Thus, full employment can be restored fairly quickly. Answer: A Difficulty: Moderate AACSB: Application of knowledge 27) If Norway's labor and capital markets are highly correlated with those of its euro zone neighbors A) unemployed workers can easily move abroad to find work and domestic capital can be shifted to more profitable uses in other countries. B) unemployed workers cannot easily move abroad to find work and domestic capital cannot be shifted to more profitable uses in other countries. C) while unemployed workers can easily move abroad to find work, domestic capital cannot be shifted to more profitable uses in other countries. D) while capital can easily move abroad to be put to a more profitable use, unemployed workers cannot easily move abroad to find work. E) unemployment will rise, thanks to competition from foreign labor. Answer: A Difficulty: Moderate AACSB: Application of knowledge 28) The ability of factors to migrate abroad A) reduces the severity of unemployment and the fall in the rate of return available to investors. B) increases the severity of unemployment and the fall in the rate of return available to investors. C) reduces the severity of unemployment but increases the fall in the rate of return available to investors. D) cannot change the severity of unemployment and the constant rate of return available to investors. E) reduces the migration of highly-skilled workers. Answer: A Difficulty: Moderate AACSB: Application of knowledge
22 Copyright © 2022 Pearson Education, Inc.
29) Which one of the following statements is TRUE for Norway, a non-euro country? A) Of course, owners of capital that cannot be moved cannot avoid more of the economic stability loss due to fixed exchange rates when Norway's economy is open to capital flows. B) Even owners of capital that cannot be moved can avoid more of the economic stability loss due to fixed exchange rates when Norway's economy is open to capital flows. C) Owners of capital that cannot be moved can avoid more of the economic stability loss due to fixed exchange rates when Norway's economy is closed to capital flows. D) Even owners of capital that can be moved can avoid more of the economic stability loss due to fixed exchange rates when Norway's economy is closed to capital flows. E) Only owners of capital that can be moved can avoid more of the economic stability loss due to fixed exchange rates when Norway's economy is open to capital flows. Answer: B Difficulty: Moderate AACSB: Application of knowledge 30) Which of the following statements is MOST accurate? A) A low degree of economic integration between a country and the fixed exchange rate area that it joins reduces the resulting economic stability loss due to output market disturbances. B) A high degree of economic integration between a country and the fixed exchange rate area that it joins reduces the resulting economic stability loss due to output market disturbances. C) A high degree of economic integration between a country and the fixed exchange rate area that it joins increases the resulting economic stability loss due to output market disturbances. D) A complete lack of economic integration between a country and the fixed exchange rate area that it joins reduces the resulting economic stability loss due to output market disturbances. E) A low degree of economic integration between a country and the fixed exchange rate area that it joins increases the resulting economic stability loss due to output market disturbances. Answer: B Difficulty: Easy AACSB: Application of knowledge 31) Why does the LL schedule have a negative slope? A) The economic stability loss from pegging to the area's currencies rises as the degree of economic interdependence rises. B) The economic stability loss from pegging to the area's currencies falls as the degree of economic interdependence rises. C) The economic stability loss from pegging to the area's currencies falls as the degree of economic interdependence falls. D) The economic stability loss from pegging to the area's currencies rises as the degree of economic activity increases. E) The economic stability loss from pegging to the area's currencies is constant, even as the degree of economic activity increases. Answer: B Difficulty: Easy AACSB: Application of knowledge
23 Copyright © 2022 Pearson Education, Inc.
32) The intersection of GG and LL determines A) the optimal level of integration desired by Norway. B) the maximum integration level desired by Norway. C) the minimum level of integration that will cause Norway to join the fixed exchange rate regime. D) the maximum level of integration that will cause Norway to join the fixed exchange rate regime. E) the maximum level of integration that can aid Norway if it joins the fixed exchange rate regime. Answer: C Difficulty: Easy AACSB: Application of knowledge 33) The GG-LL framework has important implications about A) how changes in a country's economic environment affect its willingness to peg its currency to an outside currency area. B) how changes in a country's economic environment have no effect on its willingness to peg its currency to an outside currency area. C) how changes in a country's economic environment always decrease its willingness to peg its currency to an outside currency area. D) how changes in a country's economic environment always increase its willingness to peg its currency to an outside currency area. Answer: A Difficulty: Easy AACSB: Application of knowledge 34) Which of the following best defines an optimum currency area? A) a group of nations sharing the same currency B) a group of regions in close proximity to each other C) a group of regions who operate under similar economic policies D) a group of regions with economies closely linked by factor mobility and by trade in goods and services E) a group of nations that engage in free trade with each other Answer: D Difficulty: Easy AACSB: Application of knowledge 35) A fixed exchange rate area will best serve the economic interests of each of its members if A) the degree of output and factor trade among the included economies is high. B) the degree of output and factor trade among the included economies is low. C) the degree of output and factor trade among the included economies is volatile. D) the degree of output and factor trade among the included economies is uncertain. Answer: A Difficulty: Easy AACSB: Application of knowledge
24 Copyright © 2022 Pearson Education, Inc.
36) Which of the following statements is MOST accurate? A) A rise in the size and frequency of country-specific disturbances to the joining country's product markets raises the critical level of economic integration at which the exchange rate area is joined. B) A rise in the size and frequency of country-specific disturbances to the joining country's product markets lowers the critical level of economic integration at which the exchange rate area is joined. C) A decline in the size and frequency of country-specific disturbances to the joining country's product markets raises the critical level of economic integration at which the exchange rate area is joined. D) A rise in the size and frequency of country-specific disturbances to the joining country's product markets has no effect on the critical level of economic integration at which the exchange rate area is joined. E) A decline in the size and frequency of country-specific disturbances to the joining country's product markets does not affect the level of economic integration at which the exchange rate area is joined. Answer: A Difficulty: Easy AACSB: Application of knowledge 37) Which of the following statements is MOST accurate? A) The countries of southern Europe are better endowed with capital and skilled labor than the countries of northern Europe. B) The countries of northern Europe are better endowed with capital and skilled labor than the countries of southern Europe. C) EU products that make intensive use of high-skill labor are most likely to come from Portugal. D) EU products that make intensive use of low-skill labor are most likely to come from Great Britain. E) The countries of eastern Europe are better endowed with capital and skilled labor than the countries of western Europe. Answer: B Difficulty: Easy AACSB: Application of knowledge 38) Fiscal federalism in the EU refers to A) one nation's control of the monetary policy of all the other nations. B) freedom of member countries to leave the EU at any time. C) the transfer of economic resources from members with healthy economies to those suffering economic setbacks. D) one nation's freedom to abandon the Euro and use its own currency. E) the transfer of economic resources between members with healthy economies. Answer: C Difficulty: Easy AACSB: Application of knowledge
25 Copyright © 2022 Pearson Education, Inc.
39) The level of fiscal federalism in the European Union is A) too big to cushion member countries from adverse economic events. B) too small to cushion member countries from adverse economic events. C) appropriate to cushion member countries from adverse economic events. D) too big relative to the one in the U.S. E) similar in its level to that of the U.S. Answer: B Difficulty: Easy AACSB: Application of knowledge 40) A good measure of a country's level of economic integration with a currency area is A) the intersection of DD and GG. B) the country's price level. C) the compatibility of economic policies. D) the intersection of AA and GG. E) the extent of trade between the joining country and the currency area and the ease with which labor and capital can migrate between the joining country and the currency area. Answer: E Difficulty: Moderate AACSB: Application of knowledge 41) Which of the following statements is the MOST accurate? A) Trade of EU countries with other EU countries has increased since the euro was introduced. B) Trade of EU countries with other EU countries has decreased since the euro was introduced. C) Trade of EU countries with other EU countries has increased in some years and decreased in other years since the euro was introduced. D) Trade of EU countries with other EU countries can no longer be measured since both trading parties have the same currency. E) Trade of EU countries with North American countries has decreased since the euro was introduced. Answer: A Difficulty: Easy AACSB: Application of knowledge 42) Compared with inter-regional trade in the he United States, intra-EU trade A) is far greater. B) is greater. C) is about the same. D) is less. E) is far less. Answer: D Difficulty: Easy AACSB: Application of knowledge
26 Copyright © 2022 Pearson Education, Inc.
43) Richard Baldwin's estimate was that the euro increased the trade level of its users by A) only 5 percent. B) only 9 percent. C) over 30 percent. D) over 50 percent. E) only 12 percent. Answer: B Difficulty: Easy AACSB: Application of knowledge 44) A recent study by Andrew Rose of the University of California showed that, on average, two countries that are members of the same currency union A) trade three times as much with each other as countries that do not share a currency. B) trade twenty times as much with each other as countries that do not share a currency. C) trade ten times as much with each other as countries that do not share a currency. D) trade six times as much with each other as countries that do not share a currency. E) trade twice as much with each other as countries that do not share a currency. Answer: A Difficulty: Easy AACSB: Application of knowledge 45) When did the UK decide to adopt the Euro? A) 1999 B) 2001 C) 2008 D) The UK never adopted the Euro. E) 2010 Answer: D Difficulty: Easy AACSB: Application of knowledge 46) A key barrier to labor mobility within Europe is A) the laziness of workers. B) full employment in most European countries. C) differences in language and culture. D) lack of transportation. E) the physical barriers in the landscape. Answer: C Difficulty: Moderate AACSB: Application of knowledge
27 Copyright © 2022 Pearson Education, Inc.
47) What are the biggest advantages the U.S. has over the EU in terms of being an Optimum Currency Area? A) low mobility of labor, higher labor productivity, lower level of intra-regional trade B) high unionization of U.S. labor force C) high mobility of labor force, more transfer payments between regions D) higher uniformity of population's taste in consumption E) more specialized labor force and natural resource advantages Answer: C Difficulty: Moderate AACSB: Application of knowledge 48) "The costs and benefits for a country from joining a fixed-exchange rate area such as the EMS depend on how well-integrated its economy is with those of its potential partners." Discuss. Answer: We will expand on this idea, which is roughly the theory of an optimum currency area as developed by Mundell. A major economic benefit of fixed exchange rates is that they simplify economic calculations and provide a more predictable basis for decisions that involve international transactions than do floating rates. This gain in monetary efficiency would be even higher if the factors of production could migrate freely. The costs of joining a fixed-exchange rate area is that a country gives up the ability to use the exchange rate and monetary policy to stabilize the domestic economy. So, if a country has a well-integrated economy with those in the fixed-exchange rate area, then the benefits would likely outweigh the costs. Difficulty: Moderate AACSB: Application of knowledge 49) Discuss the benefits and costs of joining a fixed-exchange area. Answer: Benefits: In general, gains from the stability of the area and reduced uncertainty. The efficiency gain from a fixed exchange rate with euro is greater when trade between, say Norway and the euro zone, is extensive than when it is small. A major economic benefit of fixed exchange rates it that they simplify economic calculations and provide a more predictable basis for decisions that involve international transactions than do floating rates. The monetary efficiency gain from pegging, say the Norwegian krone to the euro, will be higher if factors of production can migrate freely between Norway and the euro area. The more extensive are crossborder trade and factor movements, the greater is the gain from a fixed cross-border exchange rate. Costs: A country that joins an exchange rate area gives up its ability to use the exchange rate and monetary policy for the purpose of stabilizing output and employment. When the economy is disturbed by a change in the output market, a floating exchange rate has an advantage over a fixed rate. A flexible exchange rate automatically cushions the economy's output and employment by allowing an immediate change in the relative price of domestic and foreign goods. When the exchange rate is fixed, purposeful stabilization is more difficult because monetary policy has no power at all to affect domestic output and employment. Difficulty: Difficult AACSB: Application of knowledge
28 Copyright © 2022 Pearson Education, Inc.
50) Explain what the GG-LL model tells us about the benefits of extensive trade between EU member states and comment on the significance of similarity of economic structure in this framework. Answer: The GG-LL model shows that extensive trade with the rest of the euro zone makes it easier for a member to adjust to output market disturbances that affect it and its currency partners differently. A key element in minimizing such disturbances is similarity in economic structure, especially in the types of products produced. Euro zone countries are, in fact, not entirely dissimilar in the manufacturing structure, as evidenced by the very high volume of intraindustry trade. The hope is that any difference in EU member country factor endowments will be minimized by the completion of a single European market and the redistribution of capital and labor across Europe. This will bring about the desired similarity of economic structure. Difficulty: Difficult AACSB: Application of knowledge 51) Explain the theory of optimum currency areas. Answer: The theory implies that countries will wish to join fixed exchange rate areas closely linked to their own economies through trade and factor mobility. This decision to join is, in turn, determined by the difference between the monetary efficiency gain from joining and the economic stability loss from joining. These factors are both related to the degree of economic integration between the joining country and the larger fixed exchange rate zone. Only when economic integration passes a critical level is it beneficial to join. Difficulty: Difficult AACSB: Application of knowledge 52) Explain why when Norway unilaterally fixes its exchange rate against the euro but leaves the krone free to float against the non-euro currencies, it is unable to keep at least some monetary independence. Answer: Any independent money supply change in Norway would put pressure on krone interest rates and thus on the krone/euro exchange rate. So by pegging the krone even to a single foreign currency, Norway completely surrenders its domestic monetary control. Difficulty: Moderate AACSB: Application of knowledge 53) Explain why after, say Norway unilaterally pegs the krone to the euro, domestic money market disturbances will no longer affect domestic output despite the continuation of float-rate regime against non-euro currencies. Answer: Because Norway's interest rate must equal the euro interest rate, any pure shifts in the AA curve (see chapter 19) will result in immediate reserve inflows or outflows that leave Norway's interest rate unchanged. Difficulty: Moderate AACSB: Application of knowledge
29 Copyright © 2022 Pearson Education, Inc.
54) Explain why even owners of capital that cannot be moved can avoid more of the economic stability loss due to fixed exchange rates when Norway's economy is open to capital flows. Answer: The main reason is through diversification. If Norway's capital market is integrated with those of the EMU neighbors, Norwegians will invest some of their wealth in other countries while at the same time part of Norway's capital stock will be owned by foreigners. Difficulty: Moderate AACSB: Application of knowledge 55) Explain why the oil price shocks after 1973 made countries unwilling to revive the Bretton Woods system of fixed exchange rates. See also Chapter 19. Answer: Using the GG-LL framework will help solve this question. The oil price shock of 1973 pushes the LL curve upward and to the right. Thus, the level of economic integration at which it becomes worthwhile to join the currency rises. In general, increased variability in the product markets makes countries less willing to enter fixed exchange rate areas. This prediction helps explain why the oil price shocks after 1973 made countries unwilling to revive the Bretton Woods system of fixed exchange rates. Difficulty: Moderate AACSB: Application of knowledge 56) Explain why it may make sense for the United States, Japan, and Europe to allow their mutual exchange rate to float? Answer: Even though these regions trade with each other, the extent of that trade is modest compared with regional GDPs and interregional labor mobility is low. Difficulty: Moderate AACSB: Application of knowledge 57) What is one way to offset the economic stability loss due to fixed exchange rates? Answer: Fiscal federalism is one solution in which the EU transfers economic resources from members with healthy economies to those suffering economic setbacks. These transfer payments come in the form of welfare benefits, and they are usually financed by the taxes that other member states pay. Ultimately, the extent of fiscal federalism is limited by the EU's restricted taxation powers. Difficulty: Moderate AACSB: Application of knowledge 58) Using the GG-LL framework, analyze the effect of an increase in the size and frequency of sudden shifts in the demand for a country's exports. Answer: Such a change pushes LL upward and to the right. Thus, the level of economic integration at which it becomes worthwhile to join the currency rises. In general, increased variability in the product markets makes countries less willing to enter fixed exchange rate areas. This prediction helps explain why the oil price shocks after 1973 made countries unwilling to revive the Bretton Woods system of fixed exchange rates. See also Chapter 19. Difficulty: Difficult AACSB: Application of knowledge
30 Copyright © 2022 Pearson Education, Inc.
59) Is Europe an optimum currency area? Answer: An open question. I think yes, the area's economy is closely integrated with its own: most EU members export from 10 to 20 percent of their output to other EU members. But still, the law of one price does not apply to many products such as the auto market. In 1998, prices for the BMW 520i varied by as much as 29.5 percent between the United Kingdom and the Netherlands. Prices for the Ford Fiesta varied as much as 43.5 percent between the United Kingdom and Portugal! The text is more critical, claiming: "The extent of intra-European trade is not large enough, however, to give us an overwhelming reason for believing the European Union itself is an optimum currency area." Also, the text cites the fact that maybe price convergence has more to do with Internet marketing than optimum currency area. The book concludes that on balance, it seems doubtful that the 1992 measures have yet pushed Europe dramatically closer to being an optimum currency area. Difficulty: Difficult AACSB: Application of knowledge 60) How mobile is Europe's labor force? Answer: Differences in language and culture discourage labor movements between European countries. Differences in regional unemployment rates are smaller and less persistent in the United States than are differences between national unemployment rates in the European Union. Even, within European countries, labor mobility appears limited, partly because of government regulations. For example, the requirement in some countries that workers establish residence before receiving unemployment benefits makes it harder for unemployed workers to seek jobs in regions that are far from their current homes. Difficulty: Moderate AACSB: Application of knowledge 61) How much trade do currency unions create? Answer: The main result is that currency unions promote trade. One study found that on average, two countries that are members of the same currency union trade three times as much with each other as countries that do not share a currency. Even if the euro were to raise trade within the euro zone by 50 percent, the positive effect on people's welfare could be immense, as another study has shown. However, some challenge the conclusion. Some claim the results would not be duplicated when applied to large countries such as the members of the EU; another study found out that leaving a common currency area as Ireland did has not caused reduction in UK-Ireland trade. Difficulty: Moderate AACSB: Application of knowledge
31 Copyright © 2022 Pearson Education, Inc.
62) Draw the graph of the GG and LL schedules and explain the logic behind the slopes of each of the schedules. Answer: The correct graph has "degree of economic integration between the joining country and the exchange rate area" on the x-axis, and "gains and losses for the joining country" on the yaxis. The GG curve has a positive slope since the monetary efficiency gain a country gets by joining a fixed exchange rate area rises as its economic integration with the area increases. The LL curve has a negative slope because the economic stability loss from pegging to the area's currencies falls as the degree of economic interdependence rises. The two curves cross at a point that determines the critical level of economic integration (1 between the fixed exchange rate area and the country considering joining). In other words, it is the minimum integration level at which the country will join. (See Figure 21-5 in the text.)
Difficulty: Difficult AACSB: Application of knowledge
32 Copyright © 2022 Pearson Education, Inc.
21.4
The Euro Crisis and the Future of EMU
1) Which one of the following countries was the "spark" that ignited the 2009 euro crisis? A) China B) Greece C) England D) Spain E) Germany Answer: B Difficulty: Easy AACSB: Application of knowledge 2) Which one of the following unexpected events ignited the 2009 euro crisis? A) accelerating hyperinflation and political upheaval B) the prospect of a sovereign default by one or more euro zone countries C) rising oil prices D) revolutions in Switzerland and Belgium E) a Chinese boycott of European products Answer: B Difficulty: Easy AACSB: Application of knowledge 3) A sovereign default occurs when A) the debtor, a country's government, does not make the debt payments it has promised to creditors. B) the debtor, a country's government, makes the debt payments it has promised to creditors. C) the debtor pays off its debts and can still keep the country's sovereignty. D) the debtor negotiates debt payments with its creditors. Answer: A Difficulty: Easy AACSB: Application of knowledge 4) After entry into the euro, real appreciation set in for peripheral euro zone countries, most noticeably the two with massive housing booms, A) Ireland and Spain. B) Germany and France. C) Italy and Portugal. D) Finland and the Netherlands. Answer: A Difficulty: Easy AACSB: Application of knowledge
33 Copyright © 2022 Pearson Education, Inc.
5) As the 1999 launch date for the euro approached, nominal long-term bond rates in prospective member countries converged, leading to ________ real interest rates in those countries with relatively ________ inflation. A) lower; high B) higher; low C) lower; low D) higher; high Answer: A Difficulty: Easy AACSB: Application of knowledge 6) What was the spark that ignited the tinder of overextended banks and uncompetitive, indebted EU economies? A) Greece's intractable fiscal problems B) Germany's low inflation rate C) U.S. real estate market D) European housing market booms Answer: A Difficulty: Easy AACSB: Application of knowledge 7) In mid-April 2010, euro zone countries, working with IMF, agreed on a ________ loan package for Greece. A) €110 billion B) €78 billion C) €67 billion D) €5 billion Answer: A Difficulty: Easy AACSB: Application of knowledge 8) Late in 2010, Ireland negotiated a ________ European Financial Stability Facility (EFSF) troika loan package. In May 2011, Portugal negotiated a ________ troika loan. A) €67.5 billion; €78 billion B) €27.5 billion; €38 billion C) €15 billion; €18 billion D) €7.5 billion; €8 billion Answer: A Difficulty: Easy AACSB: Application of knowledge
34 Copyright © 2022 Pearson Education, Inc.
9) The term troika which came into widespread use during the euro crisis, refers to the loan package to bailout member states since 2010 from A) the European Commission, the European Central Bank, and the IMF. B) the U.S. Treasury. C) the World Bank. D) the Bank of Russia. Answer: A Difficulty: Easy AACSB: Application of knowledge 10) What is a doom loop? A) It is a two-way feedback loop in which each action makes the situation worse, from bank distress to government borrowing problems. B) It is a phenomenon in which each action makes the situation slightly better for banks and the government. C) It is a situation where banks and government can take actions without any impacts on one another. D) It is a loop in which government support banks and banks help to stabilize the government. Answer: A Difficulty: Easy AACSB: Application of knowledge 11) Public debts in the euro area ________ after 2007, in part due to the need for governments to support their weak banks. A) increased rapidly B) decreased rapidly C) remained constant D) fluctuated slightly Answer: A Difficulty: Easy AACSB: Application of knowledge 12) During the euro crisis, due to fiscal cutbacks and the credit squeeze in troubling countries, output ________ and unemployment ________. A) plummeted; soared B) soared; plummeted C) stayed the same; stayed the same D) plummeted; plummeted E) soared; soared Answer: A Difficulty: Easy AACSB: Application of knowledge
35 Copyright © 2022 Pearson Education, Inc.
13) In June 2012, euro area finance ministers extended to Spain an ESM loan potentially as big as ________ to cover recapitalization of its ailing banks. But the country remained in turmoil. A) €100 billion B) €10 billion C) €110 billion D) €1 billion Answer: A Difficulty: Easy AACSB: Application of knowledge 14) In 2012, two key initiatives launched by euro zone leaders during the euro crisis included A) Fiscal Stability Treaty and Single Supervisory Mechanism. B) Stability Pact And Growth Pact. C) European Financial Stability Facility and Troika. D) European Recovery Program and Single Resolution Mechanism. Answer: A Difficulty: Easy AACSB: Application of knowledge 15) What does OMT stand for? A) Outright Monetary Transactions B) Overseas Money Transitions C) Overspending Monetary Transactions D) Occupancy Management Training Answer: A Difficulty: Easy AACSB: Application of knowledge 16) A program under which ECB would purchase sovereign bonds, potentially without limit, to prevent their interest rates from rising too far is called A) Outright Monetary Transactions. B) European Recovery Program. C) Stability and Growth Program. D) Open Market Operations. Answer: A Difficulty: Easy AACSB: Application of knowledge 17) EU countries went into lockdowns to halt the spread of the COVID-19 virus in the first quarter of 2020, A) resulting in sharp falls in economic activity across the continent. B) resulting in sharp increases in economic activity across the continent. C) resulting in no change in economic activity across the continent. D) resulting in a halt in joint economic activity across the continent. Answer: A Difficulty: Easy AACSB: Application of knowledge 36 Copyright © 2022 Pearson Education, Inc.
18) A European Commission initiative supported EU members' short time work programs A) in which businesses reduce workers' hours rather than fire them, but continue to pay them normal or near-normal wages. B) in which businesses hire more workers to work on additional short-term projects for income. C) in which businesses fire workers and offer them loans while they are searching for short term temporary jobs. D) in which businesses pay workers above normal wages to cope with the inflation. Answer: A Difficulty: Easy AACSB: Application of knowledge 19) In response to Covid 19 pandemic, the ECB rolled out a Pandemic Emergency Purchase Program (PEPP) worth ________ to purchase euro area assets. A) €750 billion B) €75 billion C) €50 billion D) €7 billion Answer: A Difficulty: Easy AACSB: Application of knowledge 20) Which of the following statement is NOT true? A) Europe is an optimum currency area. B) If EMU succeeds, it will promote European political as well as economic integration, fostering peace and prosperity in a region. C) If the euro project fails, its driving force, the goal of European political unification, will be set back. D) EMU must overcome difficult challenges, if it is to survive its current crisis and prosper. Answer: A Difficulty: Easy AACSB: Application of knowledge 21) Which of the following statements is TRUE? A) Asymmetric economic developments within different countries of the euro zone will remain hard to handle through monetary policy. B) Asymmetric economic developments within different countries of the euro zone will make it easy to handle through monetary policy. C) Symmetric economic developments within different countries of the euro zone will make it easy to handle through monetary policy. D) Symmetric economic developments within different countries of the euro zone will no longer remain a problem to handle through monetary policy. Answer: A Difficulty: Easy AACSB: Application of knowledge
37 Copyright © 2022 Pearson Education, Inc.
22) In most EU countries, A) labor markets remain rigid and subject to employment taxes and regulations that impede labor mobility between industries and regions. B) labor markets are open and flexible, promoting labor mobility between industries and regions. C) labor markets are not subject to employment taxes and regulations. D) labor markets subject to employment taxes and regulations that promote labor mobility between industries and regions. Answer: A Difficulty: Easy AACSB: Application of knowledge 23) For the euro zone to achieve comparable economic success, it will have to make progress in various areas EXCEPT A) raising taxes and tightening regulations in each member country. B) creating more flexible labor and product markets. C) reforming its fiscal and financial regulatory systems. D) deepening its political union. Answer: A Difficulty: Easy AACSB: Application of knowledge 24) Discuss the problems that the EMU will continue to experience in the coming years. Answer: (1) Europe is not an optimum currency area; therefore asymmetric economic developments within different countries of the euro zone that call for different interest rates cannot be implemented. (2) The political part of the unification is much weaker and may limit the political legitimacy of the economic unification. (3) On the one hand, labor markets remain highly unionized and subject to high government unemployment taxes and other regulations impeding labor mobility. On the other hand, capital has high incentive to migrate to the EMU countries with the lowest wages. (4) Constraints on national fiscal policy are likely to be painful due to the absence of substantial fiscal federalism (fiscal transfer of resources from the rich to the less rich countries within the European Union) within the EU. (5) The EU is considering a largescale expansion of its membership into Eastern Europe and the Mediterranean. This will cause many coordination costs and also the issue of representation of small and big countries. Difficulty: Difficult AACSB: Application of knowledge 25) What behavior by central and private banks in euro zone countries created the conditions for the 2009 euro crisis? Answer: Assets were accumulated by the banks through the purchase of U.S. financial products and through lending to other euro zone countries. Easy credit led to a European housing boom. Following the global financial crisis and the consequent recession, some European countries such as Greece, Ireland, Portugal, Italy and Spain were found to have unsustainable levels of debt relative to national GDP. This raised the specter of a sovereign default by one or more euro zone countries and the crisis was on. Difficulty: Moderate AACSB: Application of knowledge 38 Copyright © 2022 Pearson Education, Inc.
26) During the 2009 euro crisis, a number of countries had private banks that had become too "big to save." Explain. Answer: A private bank is too big to save if the resources available to the home government through the central bank are insufficient to prevent bank failure. Essentially, saving the private bank would lead to a sovereign default by a countries' government and so was not feasible. Difficulty: Moderate AACSB: Application of knowledge 27) What led to the over-extension of credit by some private banks and central banks in the euro zone prior to the 2009 euro crisis? Answer: Differences in interest rates in euro zone countries did not accurately represent differences in risk and inflation. Consequently banks were able to borrow at very low real interest rates while assuming relatively high levels of default risk. Once it became apparent that sovereign default and private bank failures were not only possible but likely in some countries, borrowing costs skyrocketed in those countries. In some cases, credit was cut off entirely. Difficulty: Moderate AACSB: Application of knowledge 28) What event in 2009 ignited the euro crisis? Answer: Greece elected a new government in 2009 which found that the previous government had been misreporting economic statistics for years and the public debt amounted to more than 100% of GDP. It became apparent that Greece would experience a sovereign default unless bailed out by the European Central Bank or some other source of credit. Difficulty: Moderate AACSB: Application of knowledge 29) Is the United States in danger of a sovereign default because, like countries in the euro zone, it has high current account deficits and levels of public debt? Answer: No. The United States has a central bank that can print money as needed to service its debt. It is in no danger of default. Euro zone countries, on the other hand, do not have individual central banks capable of printing money to pay off debt. Instead the European Central Bank and all member countries must collectively agree on monetary policy. Countries that have managed their affairs prudently are understandably unwilling to support the profligate behavior of other countries. Difficulty: Easy AACSB: Application of knowledge
39 Copyright © 2022 Pearson Education, Inc.
30) How do constraints on monetary policy in the United States differ from those experienced by euro zone countries? Answer: United States monetary policy constraints are quite different from those encountered by countries in the euro zone. The U.S. can print dollars to pay off debts as necessary and, while the result might be a higher rate of inflation, it makes it virtually impossible for the U.S. to default on its public debts. Euro countries have a European Central Bank that must make decisions regarding monetary policy that will affect all of the euro zone countries. One possibility that has been discussed is to make it possible to conditionally remove countries from the euro zone to prevent the destruction of the monetary union. Note that this would be equivalent to the United States deciding to remove a state from the union if the state threatened bankruptcy. Difficulty: Moderate AACSB: Application of knowledge 31) What is the "doom loop" responsible for the rapid development and severity of the 2009 euro crisis? Answer: The "doom loop" refers to the feedback loop that runs from private bank distress to central bank distress to further private bank distress and so on, increasing in magnitude as it goes. During the euro crisis this process was evident in several euro zone countries. Difficulty: Moderate AACSB: Application of knowledge 32) What is the "three-pronged approach" to organizing a banking union? Answer: Centralize financial supervision, create a deposit insurance fund, and provide mechanisms for the resolution of insolvent banks within the euro area. Difficulty: Moderate AACSB: Application of knowledge 33) What is fiscal federalism? Answer: Fiscal federalism in the euro zone involves establishing a larger centralized budget managed by a central fiscal authority with the capability to tax, spend, and issue euro bonds. Difficulty: Moderate AACSB: Application of knowledge 34) Describe the single supervisory mechanism or SSM proposed by EU leaders in June of 2012. Answer: The SSM involved granting the European Central Bank the power to police banks throughout the euro zone and the ability to re-capitalize banks directly if needed. Difficulty: Moderate AACSB: Application of knowledge 35) Describe the policy of "outright monetary transactions" or 0MT presented by the president of the European Central Bank in 2012. Answer: The ECB is now committed to purchase sovereign bonds of euro zone countries, potentially without limit, in order to control interest rates in the euro zone. Difficulty: Moderate AACSB: Application of knowledge 40 Copyright © 2022 Pearson Education, Inc.
36) Is the EU an optimum currency area? Why or why not? Answer: Europe is not an optimum currency area. Labor mobility is highly limited. Economic and political conflicts within the euro zone have been persistent and they continue to result in questions regarding the ability of the euro zone to survive going forward. Difficulty: Moderate AACSB: Application of knowledge
41 Copyright © 2022 Pearson Education, Inc.
International Economics: Theory and Policy, 12e (Krugman) Chapter 22 Developing Countries: Growth, Crisis, and Reform 22.1
Income, Wealth, and Growth in the World Economy
1) Industrialized market economies is characterized by the following EXCEPT A) strict direct government control. B) rich endowment of capital and skilled labor. C) political stability. D) high level of income for their residents. Answer: A Difficulty: Easy AACSB: Application of knowledge 2) Over the decades following World War II, A) trade between developing and industrial nations has expanded. B) trade between developing and industrial nations has decreased. C) trade between developing and industrial nations has been up and down all the time. D) trade between developing and industrial nations has been non-existent. Answer: A Difficulty: Easy AACSB: Application of knowledge 3) Which of the following statements is TRUE? A) Events in developing countries have a significant impact on welfare and policies in more advanced economies. B) Events in developing countries have an insignificant impact on welfare and policies in more advanced economies. C) Events in developing countries have no impact on welfare and policies in more advanced economies. D) Events in developing countries have a global impact on welfare and policies in all countries including closed economies. Answer: A Difficulty: Easy AACSB: Application of knowledge 4) Lower income levels of developing areas can have A) consequences that can threaten political and social cohesion. B) no consequences that can threaten political and social cohesion. C) consequences that can threaten democracy and voting rights in developed areas. D) no consequences that can threaten national security and social equality. Answer: A Difficulty: Easy AACSB: Application of knowledge
1 Copyright © 2022 Pearson Education, Inc.
5) What is the basic problem of developing countries? A) corruption B) murder C) poverty D) stock market E) natural resources Answer: C Difficulty: Easy AACSB: Application of knowledge 6) Compared with industrialized economies, most developing countries are poor in the factors of production essential to modern industry. These factors are A) capital and skilled labor. B) capital and unskilled labor. C) fertile land and unskilled labor. D) fertile land and skilled labor. E) water and capital. Answer: A Difficulty: Easy AACSB: Application of knowledge 7) The relative scarcity of capital and skilled labor contributes to ________ levels of per capita income in developing countries and often prevents them from realizing the ________. A) low; economies of scale B) high; economies of scale C) low; diseconomies of scale D) high; economies of scope Answer: A Difficulty: Easy AACSB: Application of knowledge 8) The main factors that discourage investment in capital and skills in developing countries are A) political stability, secure property rights. B) political instability, insecure property rights, misguided economic policies. C) political stability, well-guided economic policies. D) secure property rights, well-guided economic policies. Answer: B Difficulty: Easy AACSB: Application of knowledge
2 Copyright © 2022 Pearson Education, Inc.
9) The world's economies can be divided into four main categories according to their annual percapita income levels. Which one of the following is NOT one of the categories? A) low-income B) upper middle-income C) high-income D) lower middle-income E) middle-income Answer: E Difficulty: Easy AACSB: Application of knowledge 10) Average national income per-capita in the riches economies is ________ times that of the average in the poorest developing economies. A) 115 B) 95 C) 73 D) 44 E) 57 Answer: E Difficulty: Easy AACSB: Application of knowledge 11) The upper middle-income countries enjoy about ________ of the per capita income of the industrial group. A) one-fifth B) one-third C) one-half D) one-fourth Answer: A Difficulty: Easy AACSB: Application of knowledge 12) The per-capita GNP of the industrial group is about ________ times that of the upper middle-income countries. A) 5 B) 10 C) 15 D) 19 E) 2 Answer: A Difficulty: Easy AACSB: Application of knowledge
3 Copyright © 2022 Pearson Education, Inc.
13) In general, one would expect that life expectancies reflect international differences in income levels. Do the data support such a claim? A) Average life span falls as relative poverty falls. B) Average life span increases as relative poverty falls. C) There is no statistically significant relationship between the two. D) The relation is not very strong. E) The relationship looks more like a U-shape. Answer: B Difficulty: Easy AACSB: Application of knowledge 14) How would you define convergence? A) tendency for gaps between industrial countries' per-capital incomes to narrow B) tendency for gaps between all countries' per-capital incomes to narrow C) the theory that a crisis in a low-income country will spread to all countries, regardless of debt structure D) the theory that a crisis in a low-income country will spread to only those countries which had lent money to the original country E) tendency for the world distribution of income to be persistently unequal Answer: A Difficulty: Easy AACSB: Application of knowledge 15) How would you describe the world distribution of income? A) persistently unequal B) temporarily unequal C) converging D) fairly equal E) completely unpredictable Answer: A Difficulty: Easy AACSB: Application of knowledge 16) When one compares per-capital output growth rates among countries A) one needs to correct the data to account for departures from purchasing power parity. B) corrections are not often necessary. C) corrections are sometimes necessary. D) the evidence whether such corrections are necessary are vague. E) corrections are never necessary. Answer: A Difficulty: Easy AACSB: Application of knowledge
4 Copyright © 2022 Pearson Education, Inc.
17) Over the period 1960-2017, the United States economy grew at roughly A) 2 percent. B) 3 percent. C) 4 percent. D) one percent. E) 3.5 percent. Answer: A Difficulty: Easy AACSB: Application of knowledge 18) Japan had been ________ percent poorer than the United States in 1960, was only around ________ percent poorer in 2017 - thereby having narrowed the earlier income gap significantly. A) 63; 28 B) 28; 63 C) 37; 28 D) 72; 37 Answer: A Difficulty: Easy AACSB: Application of knowledge 19) Spain had been ________ percent poorer than the United States in 1960, was only around ________ percent poorer in 2017 - thereby having narrowed the earlier income gap. A) 58; 38 B) 38; 28 C) 58; 28 D) 62; 38 Answer: A Difficulty: Easy AACSB: Application of knowledge 20) Over the period 1960-2017, in terms of annual average growth rate, France grew ________ than the United States economy. A) 2 % slower B) 2% faster C) more than 0.2% slower D) 0.2% faster E) more than 2% faster Answer: D Difficulty: Easy AACSB: Application of knowledge
5 Copyright © 2022 Pearson Education, Inc.
21) Over the post-war era, the gaps between industrial countries' living standards A) disappeared. B) stayed the same. C) increased. D) decreased. E) fluctuated. Answer: D Difficulty: Easy AACSB: Application of knowledge 22) Between 1960 and 2017, the annual growth rate in percent per year was the highest in A) South Korea. B) United States. C) Brazil. D) Singapore. E) China. Answer: A Difficulty: Easy AACSB: Application of knowledge 23) Which of the following countries had a larger growth rate since 1960? A) U.S. B) Senegal C) South Korea D) Mexico E) Colombia Answer: C Difficulty: Easy AACSB: Application of knowledge 24) Over the post-war era, the gaps between countries' living standards A) disappeared. B) stayed the same. C) increased. D) decreased. E) changed inconsistently. Answer: E Difficulty: Easy AACSB: Application of knowledge
6 Copyright © 2022 Pearson Education, Inc.
25) Over the post-war era, poorer countries grew A) faster. B) slower. C) stayed the same. D) grew faster, then grew slower. E) No general tendency can be found. Answer: E Difficulty: Easy AACSB: Application of knowledge 26) Since 1960, countries in sub-Saharan Africa have grown at rates ________ those of the main industrial countries. A) far below B) far above C) about the same D) slightly below E) slightly above Answer: A Difficulty: Easy AACSB: Application of knowledge 27) Since 1960, South Korea and Singapore enjoyed average per-capita growth rates ________ the average industrialized world. A) far below B) far above C) about the same D) slightly below E) slightly above Answer: B Difficulty: Easy AACSB: Application of knowledge 28) Until recently, per-capita income increased in East Asian countries such as Hong Kong, Singapore, South Korea, and Taiwan by ________-fold every generation A) 2 B) 3 C) 4 D) 5 E) 1 Answer: D Difficulty: Easy AACSB: Application of knowledge
7 Copyright © 2022 Pearson Education, Inc.
29) Three key macroeconomic goals to achieve in developing countries' success stories include A) low inflation, low unemployment, and financial-sector stability. B) high employment, stable exchange rate, and capital inflows. C) low inflation, high economic growth rate, and balanced budget. D) high employment, low taxes, and current account surplus. Answer: A Difficulty: Easy AACSB: Application of knowledge 30) Over the past half century, poorer economies now account for a much ________ share of global income than in the past - more than ________ measured on a PPP basis–and account for a ________ share of global economic growth. A) bigger; half; bigger B) smaller; half; smaller C) bigger; half; smaller D) smaller; half; bigger Answer: A Difficulty: Easy AACSB: Application of knowledge 31) Explain what the four main categories of world economies are and give examples? Answer: The four main categories of the world economies are categorized by annual per-capita income levels. Low-income economies which include India, Pakistan, and much of the subSaharan Africa. Lower middle income includes most Middle Eastern Countries, Latin American, Caribbean countries, the former Soviet countries and most of the African countries. The upper middle income economies which are Latin American countries, Saudi Arabia, Malaysia, South Africa, Poland, Hungary, Czech and the Slovak Republic. The last category is the high-income economies such as Korea, Israel, Kuwait, Singapore and the United states. Difficulty: Easy AACSB: Application of knowledge 32) Explain the theory behind convergence and why it is a "deceptively simple" theory. Answer: Convergence is the tendency for gaps between industrialized countries' living standards (i.e. per capita income) to narrow. If trade is free, if capital can move to countries offering the highest returns, and if knowledge itself moves across political borders so that countries always have access to new production technologies, then there is no reason for international income differences to persist for long. Some differences, however, do exist because of policy differences across industrial countries. Difficulty: Moderate AACSB: Application of knowledge 33) What explains the sharply divergent long-run growth patterns in general? Answer: The answer lies in the economic and political features of developing countries and the way these have changed over time in response to both world events and internal pressures. Difficulty: Moderate AACSB: Application of knowledge 8 Copyright © 2022 Pearson Education, Inc.
34) How have rich countries become less important for global GDP growth? Answer: As many developing economies have grown more quickly and come to account for larger shares of world output, their GDP growth rates have become more important in determining overall world growth. At the same time, growth in the richer economies has tended to slow over time. Difficulty: Easy AACSB: Application of knowledge
9 Copyright © 2022 Pearson Education, Inc.
35) Please consider Table 22-2 below.
Assuming constant Annual Average Growth Rate in the future, calculate the output per capita for the United States and South Korea for the year 2074. 10 Copyright © 2022 Pearson Education, Inc.
Answer: Since 2074 - 2017 = 2017 - 1960, then output per capita for U.S. = 54586 * (54586 / 17319) = 172,044 output per capita for South Korea = 36999 * (36999 / 1573) = 870,264 Difficulty: Moderate AACSB: Application of knowledge
11 Copyright © 2022 Pearson Education, Inc.
36) Please consider Table 22-2 below.
At that Annual Average Growth Rate, how many years does it take for the output per capita to double in both the United States and South Korea. 12 Copyright © 2022 Pearson Education, Inc.
Answer: United States (1 + 0.02 = 2 t = ln(2) / ln(1.02) ≈ 35 years South Korea (1 + 0.06 = 2 t = ln(2) / ln(1.06) = 12 years (short-cut-rule of 69) United States t = 69/2 ≈ 35 years South Korea t = 69/6 ≈ 12 years Difficulty: Moderate AACSB: Application of knowledge 22.2
Structural Features of Developing Countries
1) Which of the following are characteristic of a developing country? A) extensive direct government control of the economy B) low inflation C) high national savings D) a current account surplus and high national savings E) strong credit institutions Answer: A Difficulty: Easy AACSB: Application of knowledge 2) Which of the following is NOT a common characteristic of a developing country? A) extensive direct government control of the economy B) history of low inflation C) many weak credit institutions D) "pegged" exchange rates E) agricultural commodities make up a large share of its exports Answer: B Difficulty: Easy AACSB: Application of knowledge
13 Copyright © 2022 Pearson Education, Inc.
3) Which of the following are characteristic of a developing country? A) extensive embrace of free trade policies B) low inflation C) high national savings D) rampant tax evasion and underground economic activity E) strong credit institutions Answer: D Difficulty: Easy AACSB: Application of knowledge 4) The real resource a government earns when it prints money and spends it on goods and services is called A) seigniorage. B) control of capital movements by limiting foreign exchange transactions. C) pure profits. D) inflation profits. E) greenback. Answer: A Difficulty: Easy AACSB: Application of knowledge 5) Seigniorage refers to A) real resources a government earns when it prints money to use for spending on goods and services. B) nominal resources a government earns when it prints money to use for spending on goods and services. C) real resources a government earns when it prints money. D) nominal resources a government earns when it prints money. E) real resources a government earns when it issues bonds to use for spending on goods and services. Answer: A Difficulty: Easy AACSB: Application of knowledge 6) Which of the following is NOT a common characteristic of a developing country? A) strong credit institutions B) extensive direct government control of the economy C) history of high inflation D) "pegged" exchange rate E) agricultural commodities make up a large share of its exports Answer: A Difficulty: Easy AACSB: Application of knowledge
14 Copyright © 2022 Pearson Education, Inc.
7) In developing countries, exchange rates tend to be A) floating with some government intervention. B) managed heavily by developing-country governments. C) hard to tell from the data. D) run by currency boards. E) flexible. Answer: B Difficulty: Easy AACSB: Application of knowledge 8) How would you define exchange control? A) The government allocates foreign exchange through decree rather than through the market. B) a country NOT pegging its exchange rate C) a country pegging its exchange rate D) a country buying up excess current account so that CA=0 E) a country restricting all foreign exchange Answer: A Difficulty: Easy AACSB: Application of knowledge 9) Most developing countries have tried to A) liberalize capital movement. B) control capital movements. C) Hard to tell from the data. D) in the 1960s and 1970s control, now to liberalize. E) in the 1960s and 1970s liberalize, now to control. Answer: B Difficulty: Easy AACSB: Application of knowledge 10) For many developing countries, natural resources or agricultural commodities make up a ________ share of exports. A) large B) moderate C) nonexistent D) small E) insubstantial Answer: A Difficulty: Easy AACSB: Application of knowledge
15 Copyright © 2022 Pearson Education, Inc.
11) For many developing countries, natural resources or agricultural commodities make up ________ share of exports A) close to no B) an unimportant C) an important D) close a to 5 percent E) close to a 10 percent Answer: C Difficulty: Easy AACSB: Application of knowledge 12) While many developing countries have reformed their economies in order to imitate the success of the successful industrial economies, the process remains incomplete and most developing countries tend to be characterized by all of the following EXCEPT A) seigniorage. B) control of capital movements by limiting foreign exchange transactions connected with trade in assets. C) use of natural resources or agricultural commodities as an important share of exports. D) a worse job of directing savings toward their most efficient investment uses. E) reduced corruption and poverty due to limited underground markets. Answer: E Difficulty: Easy AACSB: Application of knowledge 13) In general, the development of underground economic activity ________ economic efficiency A) hinders B) has no effect C) aides D) hard to tell, sometime hinders, sometimes aides E) spikes Answer: A Difficulty: Easy AACSB: Application of knowledge 14) One should expect ________ relationship between annual per-capita GDP and an inverse index of corruption. A) a weak and negative B) a weak and positive C) a strong and negative D) a strong and positive E) an unpredictable Answer: D Difficulty: Easy AACSB: Application of knowledge
16 Copyright © 2022 Pearson Education, Inc.
15) The relationship between annual real per-capita GDP and corruption across countries has been found to be A) negative. B) positive. C) The relationship was negative in the late 1960s but is now positive. D) The relationship was in the late 1960s but is now negative. E) There is no relationship between these two variables. Answer: A Difficulty: Easy AACSB: Application of knowledge 16) Which of the following does NOT explain why developing countries encouraged new manufacturing industries of their own in the mid 20th century? A) They were cut off from traditional suppliers of manufactures during WWII. B) They wanted to attain the same income levels as their former rulers. C) Leaders of these countries feared that their efforts to escape poverty would be doomed if they continue to specialize in primary commodity exports. D) There was political pressure to protect these industries. E) Developing countries ran out of the natural resources that traditionally made up the majority of their trade. Answer: E Difficulty: Easy AACSB: Application of knowledge 17) Over the past two decades, aggregate commodity prices have experienced a boom-bust cycle, rising by more than ________ percent between 1998 and 2020 with a sharp but brief interruption during the Great Recession that began in 2007 and then falling by more than ________ percent between 2011 and 2016. A) 400; 50 B) 50; 400 C) 30; 40 D) 40; 30 Answer: A Difficulty: Easy AACSB: Application of knowledge 18) Each commodity super cycle since the 19th century usually lasted A) about 30 to 40 years. B) about 20 to 30 years. C) about 10 to 20 years. D) about 5 to 10 years. Answer: A Difficulty: Easy AACSB: Application of knowledge
17 Copyright © 2022 Pearson Education, Inc.
19) Major factors contributing to the commodity price run-up between 1999 and 2011 include A) rapid growth of emerging economies, supply constraints, and low global real interest rates. B) slow growth of emerging economies, supply surplus, and high global real interest rates. C) rapid growth of emerging economies, supply surplus, and high global real interest rates. D) slow growth of emerging economies, supply constraints, and low global real interest rates. Answer: A Difficulty: Easy AACSB: Application of knowledge 20) Describe some of the features hindering developing countries from growing faster. Answer: One of the features that can be holding developing countries from growing faster is corruption. The way governments control the economy by developing restrictions that would not allow international trade among other countries; knowing that by having the doors open for international trading the country can be better off. Moreover, governments also owning or controlling the largest industries, that produce more in the countries, and controlling international transactions, they do not let new opportunities to come into their society. These governments also do tax evasion, which most of the time in some countries it's been out of control. Basically, developing countries have been managed by corrupt and inexperienced people that just want to disrupt instead of encouraging new opportunities for a better future. Difficulty: Moderate AACSB: Application of knowledge 21) Explain the extensive economic role of government within a developing country. Answer: An open question. Students should include two of the following in their answer along with discussion: (1) Restrictions on international trade (2) Government control over large industrial firms (3) High level of government consumption as a share of GNP (4) Strict management of exchange rates; limit exchange rate flexibility Difficulty: Moderate AACSB: Application of knowledge 22) List major factors that contributed to the commodity price run-up between 1999 and 2011. Answer: Rapid growth of emerging economies, supply constraints, and low global real interest rates. Difficulty: Easy AACSB: Application of knowledge
18 Copyright © 2022 Pearson Education, Inc.
23) What caused the Chinese economic growth to slow since 2011 and how it impacted the global commodity prices? Answer: Since 2011, China has attempted to shift its economy away from investment and manufacturing, including construction, in the direction of more domestic consumption, especially of services. This reorientation, however, led the overall pace of Chinese economic growth to slow considerably, from double digits to somewhere around 6 percent per year before the COVID-19 pandemic. As a result, as the world price of oil plummeted in 2014—2015, driven by slowing world growth as well as a price war among oil producers. Difficulty: Easy AACSB: Application of knowledge 24) According to Prebisch and Singer's hypothesis, what would happen to commodity exporters? Answer: Developing commodity exporters were doomed to a secular process of terms of trade decline and therefore would have a hard time converging to advanced-country income levels. They argued that global commodity demand is income inelastic while demand for manufactured goods and services rises strongly with incomes, so that as the world economy grew, the terms of trade would inevitably fall for primary exporters. Difficulty: Easy AACSB: Application of knowledge 22.3
Developing-Country Borrowing and Debt
1) As of 2020, how large is the debt of developing countries to the rest of the world? A) $350 million B) $350 billion C) $11 trillion D) $35 trillion E) $3.5 trillion Answer: C Difficulty: Easy AACSB: Application of knowledge 2) If national saving falls short of domestic investment, the difference equals A) the current account deficit. B) the current account surplus. C) the current account positive. D) the current account shortage. Answer: A Difficulty: Easy AACSB: Application of knowledge
19 Copyright © 2022 Pearson Education, Inc.
3) In developing economies, national saving is often ________ relative to developed economies. A) high B) the same C) hard to tell D) low E) low for the very poor countries and high for the more developed Answer: D Difficulty: Easy AACSB: Application of knowledge 4) Which of the following is a reason that developing countries are running large surpluses? A) They are required to do so by IMF. B) They have defaulted on international loans. C) They have pegged exchange rates and thus the growth of exports must drive surplus up. D) They have to accumulate international reserves to protect against a sudden stop of capital inflows. E) They don't know how to manage their surpluses. Answer: D Difficulty: Easy AACSB: Application of knowledge 5) What does it mean for a loan to be in default? A) when the borrower of the a loan fails to repay on schedule according to a loan contract, without the agreement of the lender B) when the borrower of a loan fails to repay on schedule according to a loan contract, with the agreement of the lender C) when the lender of a loan fails to supply the full amount of a loan to the borrower D) when the lender of a loan supplies the full amount of a loan to a borrower without any promise of being repaid E) when the lender of a loan fails to offer the promised sum Answer: A Difficulty: Easy AACSB: Application of knowledge 6) In the instances where a loan has been issued under certain terms and has to be repaid, what happens when the borrower does not uphold these stipulations? A) call B) option C) payment D) default E) fraud Answer: D Difficulty: Easy AACSB: Application of knowledge
20 Copyright © 2022 Pearson Education, Inc.
7) When a government defaults on its obligations, the event is called a A) sovereign default. B) magisterial default. C) private default. D) sudden stop default. E) national default. Answer: A Difficulty: Easy AACSB: Application of knowledge 8) There are many ways developing countries finance their external deficits EXCEPT A) bank finance. B) portfolio investment in ownership of firms. C) bond finance. D) official lending. E) foreign exchange rates. Answer: E Difficulty: Easy AACSB: Application of knowledge 9) Which of the following are all the forms of debt finance? A) bond, bank, and official finance B) bond and bank finance C) bond, bank, and portfolio finance D) foreign direct and portfolio investment E) direct investment, stock, and dividends Answer: A Difficulty: Easy AACSB: Application of knowledge 10) Why may equity finance be preferred to debt finance for developing countries? A) A fall in domestic income automatically reduces the earnings of foreign shareholders without violating any loan agreement. B) There are laws insuring against any default with equity finance. C) The risk is shared between debtor and creditor with debt finance. D) The tax structure leaves equity finance unconstrained. E) Repayments are unaffected by falls in real income. Answer: A Difficulty: Easy AACSB: Application of knowledge
21 Copyright © 2022 Pearson Education, Inc.
11) A trend that has been reinforced by many developing countries is privatization. Privatization refers to A) purchasing large companies and turning them into state-owned enterprises. B) investing government money in large, privately-owned companies. C) exchanging bonds for shares in state-owned enterprises. D) selling large state-owned enterprises to private owners in the financial sector. E) selling large state-owned enterprises to private owners in key areas such as electricity, telecommunications, or petroleum. Answer: E Difficulty: Easy AACSB: Application of knowledge 12) When developing countries incur debts to foreigners, those debts are often denominated in ________ currencies. In contrast, richer countries almost always borrow in terms of ________ currencies. A) foreign; their own B) their own; foreign C) foreign; foreign D) their own; their own Answer: A Difficulty: Easy AACSB: Application of knowledge 13) A considerable advantage that richer countries have over poorer ones is exemplified by the fact that A) richer countries do not have to denominate their foreign debts in their own currencies. B) richer countries have the ability to denominate their foreign debts in foreign currencies. C) when demand falls for a poorer country's goods, this leads to a significant wealth transfer from foreigners to the poorer country, a kind of international insurance payment. D) richer countries have the ability to denominate their foreign debts in their own currencies. E) richer countries can extract trade advantages by using military power. Answer: D Difficulty: Easy AACSB: Application of knowledge 14) The term Original Sin by two economists Barry Eichengreen and Ricardo Hausmann is used to describe what? A) low-income economy B) developing countries' inability to borrow in their own currencies C) borrows not able to receive loans D) not diversifying economies portfolios Answer: B Difficulty: Easy AACSB: Application of knowledge
22 Copyright © 2022 Pearson Education, Inc.
15) During the time period of 1981-1983 what dramatic world issue happened? A) political instability, insecure property rights B) stock market crashed C) world wide hyperinflation D) the collapse of the U.S. mortgages market E) A world economic recession caused developing countries to not be able to make payments on foreign loans, in turn causing a universal default. Answer: E Difficulty: Easy AACSB: Application of knowledge 16) In 1981-1983, the world economy suffered a steep recession. Naturally, the fall in industrial countries' aggregate demand had a direct negative impact on the developing countries. What other mechanism was an even more important contributor to this event? A) the immediate steep inflation that followed the recession B) the dollar's sharp depreciation in the foreign exchange market C) the increase in primary commodity prices, increasing terms of trade in many poor countries D) the collapse in primary commodity prices and the immediate, large rise in the interest burden that debtors had to pay E) the influx of defaulting credit Answer: D Difficulty: Easy AACSB: Application of knowledge 17) With which country did the Debt Crisis of the early 1980s begin? A) France B) Mexico C) Argentina D) Japan E) Germany Answer: B Difficulty: Easy AACSB: Application of knowledge 18) In 1991, Argentina established a radical institutional reform after experiencing a decade marked by financial instability. This program was called the new Convertibility Law. What did this law do? A) made Argentina's currency fully convertible into Eurocurrency at a fixed rate B) required that the monetary base be backed completely by U.S. dollars C) placed limits on exports of commodities D) made Argentina's currency fully convertible into U.S. dollars at a fixed rate and required that the monetary base be backed completely by gold or foreign currency E) restricted risky international trade activity Answer: D Difficulty: Easy AACSB: Application of knowledge 23 Copyright © 2022 Pearson Education, Inc.
19) The Convertibility Law of April 1991 in Argentina A) pegged the Argentinean currency to the U.S. dollar at a ratio of one to one. B) pegged the Argentinean currency to the U.S. dollar at a ratio of one to two. C) pegged the Argentinean currency to the U.S. dollar at a ratio of one to 0.5. D) represents an era of floating exchange rate in Argentina. E) pegged the Argentinean currency to the British pound at a ratio of one to one. Answer: A Difficulty: Easy AACSB: Application of knowledge 20) How would you define a currency board? A) the process by which non-pegged interest rates are allowed to fluctuate B) the stockpiling of international reserves by developing countries C) using the dollar to carry out all domestic transactions, making the domestic currency a currency in name alone D) a constraint placed on monetary policy E) The monetary base is backed entirely by foreign currency and the central bank holds no domestic assets. Answer: E Difficulty: Easy AACSB: Application of knowledge 21) If a developing country institutes a currency board, it relinquishes control over having A) monetary policy autonomy. B) exchange rate stability. C) freedom of capital movement. D) freedom of labor movement. E) all of its funds. Answer: A Difficulty: Moderate AACSB: Application of knowledge 22) A currency board can ________ a country's ability to act as a lender of last resort. A) aggrandize B) limit C) enhance D) offset E) not affect Answer: B Difficulty: Easy AACSB: Application of knowledge
24 Copyright © 2022 Pearson Education, Inc.
23) Which Latin American country defaulted on its foreign debts in December 2001 and output fell by nearly 11 percent in 2002? A) Argentina B) Brazil C) Chile D) Colombia E) Mexico Answer: A Difficulty: Easy AACSB: Application of knowledge 24) Brazil's 1999 crisis was relatively short lived because A) Brazil's financial institutions had avoided borrowing all together. B) Brazil's financial institutions had avoided heavy borrowing in local currency. C) Brazil's financial institutions had avoided heavy borrowing in dollars. D) Brazil's financial institutions had extended low-interest loans. E) Brazil's financial institutions had extended high-interest loans. Answer: C Difficulty: Easy AACSB: Application of knowledge 25) The $50 billion emergency loan orchestrated by the U.S. Treasury and the IMF to Mexico in 1994 A) was a disastrous policy for Mexico. B) avoided a disaster to the Mexican economy. C) did not affect Mexico in the short run. D) did not affect Mexico in the long run. E) was ineffective both in the short and long runs. Answer: B Difficulty: Easy AACSB: Application of knowledge 26) What factors lie behind capital inflows to the developing world? Answer: Many developing countries have received a lot of capital inflows that lead them to a huge debt to foreigners. These debts are being produced because the economy of the developing world is very small compared to the economy of the industrial world. Since developing countries face a lot of poverty and poor financial institutions, national savings is often low and because of that, they are always facing current account deficit. Even though these countries are very poor in capital, there are opportunities for profitable introduction or expansion of firms and equipment, and these opportunities give good reason for a high level of investment. However, because these countries always have deficits in their current account, a country can obtain resources from abroad to invest even if its domestic savings level is low. This means that the country is going to have to borrow money from a foreign country. These ways of production are the ones that lie behind capital inflows because by helping these countries to grow and expand, the price to be paid is a big debt which they know based on their circumstances is going to be hard to repay. Difficulty: Moderate AACSB: Application of knowledge 25 Copyright © 2022 Pearson Education, Inc.
27) "Sharp contractions in a country's output and employment invariably result from a crisis in which the country suddenly loses access to all foreign sources of funds." Explain how the current account identity necessitates these contractions. Answer: S - I = CA (Current Account Identity) Imagine that a country is running a current account deficit (i.e. borrowing from abroad) a certain amount of its GNP when foreign lenders suddenly become scared of default and cut off all new loans. This action causes the current account balance to be at least 0 due to either a rise in saving or a fall in investment (or a combination of both). The required sharp fall in aggregate demand necessarily depresses the country's output dramatically. Difficulty: Moderate AACSB: Application of knowledge 28) List and explain 3 major channels through which developing countries have financed their external deficits. Answer: Any of the 3 below along with clear explanation will suffice: (1) Bond Finance (2) Bank Finance (3) Official Lending (4) Foreign Direct Investment (5) Portfolio investment in ownership of firms Difficulty: Moderate AACSB: Application of knowledge 29) What are the five major channels, which developing countries use to finance their external deficit? Answer: The five major channels are bonds finance, bank finance, official lending, foreign investment, and portfolio investment. Difficulty: Moderate AACSB: Application of knowledge 30) Describe alternative forms of capital inflow to finance external deficits and explain why these methods were used in different times? Answer: The capital inflows that finance developing countries' deficits are: Bond finance in which developing countries sell bonds to private foreign citizens to finance their deficits. At that time bond finance is a key to get money to solve the deficit of the country. Bank finance, which help developing countries to borrow widely from commercial banks. At that time banks provide more or less a quarter of a developing country's external finance. Official lending, this is used because developing countries sometimes borrow from official foreign agencies such as the World Bank or Inter American Development Bank. They like to take advantage of these banks because they lend at interest rates below market level or on a market basis that allows the lender to earn the market rate of return. Direct foreign investment, which allows a foreign largest firm owned by foreigner's residents, acquires or expands a subsidiary firm or factory domestically. Since WWII, direct investment has been a consistently important source of developing country's capital. Difficulty: Moderate AACSB: Application of knowledge 26 Copyright © 2022 Pearson Education, Inc.
31) Explain why the distinction between debt and equity finance is useful in analyzing the response of developing countries to unforeseen events such as recession or terms of trade change? Answer: When a country's liabilities are in the form of debt, its fixed scheduled payments to creditors do not fall when its real income falls. This makes it difficult to honor the developing country's foreign obligations and may lead to default. Difficulty: Moderate AACSB: Application of knowledge 32) Economists Eichengreen and Hausmann coined the phrase original sin to describe developing countries inability to borrow in their own currencies. Where do they believe that this inability comes from? What are other beliefs on this topic? Answer: Eichengreen and Hausmann believe that this inability is a structural problem of poor countries caused by features of the global capital market such as limited additional diversification potential. For the second part of the question, students' discussion may include but is not limited to the following: (1) Bad history of economic policies (2) Problems arise with debt finance in international markets for developing countries Difficulty: Moderate AACSB: Application of knowledge 33) Explain why despite enormous natural resources, much of Latin America's population remains in poverty and the region has been repeatedly experiencing financial crises. Answer: Most Latin America population remains in poverty because of bad advice and inefficient proliferated about investment decisions having taken. At the same time, the revenues available to those able to exploit limited domestic markets inspired lobbying for imports licenses and expanding the market as well as corruption. Discrimination in the import that alternates financial system and poverty at the lowest income levels grew over time. Government corruption and bad administration of money have been one of the factors that enable Latin America population from growing. Since the 1950s and 1960s many of the Latin America countries in the region were able to attain amazing growth rates by exploiting the initially high returns from moving resources in to industrial uses from inefficient agricultural activities. Instead of using the growth to get rid of debts and decrease the deficit of the country, governments along with corrupt people wasted for getting about the debt that the country was facing. Difficulty: Moderate AACSB: Application of knowledge
27 Copyright © 2022 Pearson Education, Inc.
34) The 1980s are considered as the "lost decade" of Latin American growth. Explain why? Answer: Just as the Great Depression made it hard for developing countries to make payments on their foreign loans, the great recession of the 1980s also sparked a crisis over developing country debt. The fall in the industrial countries' aggregate demand had a direct negative impact on the developing countries. The problem was made worse by the dollar's sharp appreciation in the foreign exchange market, which raised the real value of the dollar debt burden substantially. The crisis began in August 1982 when Mexico announced that its central bank had run out of foreign reserves and that it could no longer meet payments on its $80 billion in foreign debt. Seeing potential similarities between Mexico and other large Latin American debtors such as Argentina, Brazil, and Chile, banks in the industrial countries, the largest private lenders to Latin America scrambled to reduce their risks by cutting off new credits and demanding repayment on earlier loans. The result was a widespread inability of developing countries to meet prior debt obligations, and a rapid move to the edge of a generalized default. Latin America was perhaps hardest hit, but so were Soviet bloc countries like Poland that had borrowed from the European banks. Nonetheless, by the end of 1986 more than 40 countries had encountered severe financing problems. Growth had slowed sharply in much of the developing countries because they had to stop producing in order to pay the debtors. Difficulty: Moderate AACSB: Application of knowledge 35) What is Argentina's Convertibility Law of April 1991? Explain. Answer: The convertibility law is the law that made Argentina currency fully convertible into U.S. dollars at a fixed rate of exactly one peso per dollar. The law also required that the monetary base be backed entirely by gold or foreign currency. Difficulty: Moderate AACSB: Application of knowledge 36) Evaluate the Argentinean Convertibility Law of April, 1991. Answer: Good idea in the short run, catastrophic idea in the long run. The law was abandoned only in January 2002. Difficulty: Moderate AACSB: Application of knowledge 37) During 1991, Argentina's monetary law had a currency board. Explain and give an example. Answer: The currency board is monetary base backed entirely by foreign currency and the central bank therefore held no domestic assets. An example of currency board is 100% foreign exchange backing for monetary base. Difficulty: Moderate AACSB: Application of knowledge
28 Copyright © 2022 Pearson Education, Inc.
38) Does it appear that currency boards make fixed exchange rates credible? Answer: No, because is prohibited by law from acquiring any domestic assets, so all the currency it issues automatically is fully backed by foreign reserves. Also, countries that adopt currency board do it because one of the major advantages, aside from the constraint it places on fiscal policy, is that the central bank can never run out of foreign exchange reserves in the face of a speculative attack on the exchange rate. So the currency board cannot fix exchange rates. Difficulty: Moderate AACSB: Application of knowledge 39) Does it appear that currency boards make low-inflation policies credible? Answer: Currency boards have the power to bring in anti-inflation credibility from the country to which the domestic currency is hooked. Currency boards typically may not acquire government debt, but it can discourage fiscal deficits leading to reduce a major cause of inflation and devaluation. In order for a currency board to be successful is by increasing the banking sector and that can get the government under pressure to abandon the currency board. Moreover, if the markets anticipate that the government is leaving the currency board, the country may not benefit from the potential of a currency board. Difficulty: Moderate AACSB: Application of knowledge 40) Compare currency board to conventional fixed exchange rate. Answer: Currency board may not acquire domestic assets and thus cannot lend currency freely to domestic banks in time of financial crisis. Also, limit government ability to "surprise" the market and have real devaluation. Difficulty: Moderate AACSB: Application of knowledge 41) What do you think about dollarization? Answer: This is an open question. The answer is probably a bad idea unless in the very short run. Students should talk about the loss of seigniorage and the gain in credibility against devaluation, which should lead to lower domestic interest rates. Students should mention the importance of political will to repair the fundamental economic weaknesses of the country. Difficulty: Moderate AACSB: Application of knowledge 42) Explain why Argentina, one of the world's richest countries at the start of the twentieth century, has become progressively poorer relative to the industrial countries. [An alternative question: What explains Argentina's regress from riches to rags?] Answer: As usual, the answer is complex, but the country's inward orientation and macroeconomic instability appear to be the major culprits. Difficulty: Moderate AACSB: Application of knowledge
29 Copyright © 2022 Pearson Education, Inc.
43) The main reason for the crisis in Argentina in 2001 and 2002, has to do with exchange rate policy, i.e., the continued peg of the exchange rate to the dollar. Discuss. Answer: Student should emphasize that the quote is mainly true. Students should compare Argentina in those years with the better experience of Chile and Mexico with flexible exchange rates and emphasize the appreciation of the dollar. Difficulty: Moderate AACSB: Application of knowledge 44) Explain how Brazil was able to reduce the rate of inflation from 2,669 percent in 1994 to less than 10 percent in 1997? Answer: By introducing a new currency and initially pegging it to the dollar. At the cost of widespread bank failures, high interest rates in 1995 and the shift to a fixed upwardly crawling peg and a substantial real appreciation of the local currency. Difficulty: Moderate AACSB: Application of knowledge 45) Write an essay on the importance of a sound banking system in developing countries. Answer: See for example the vivid example of Chile in 1981-1982 experience. Students should explain the phenomena of moral hazard as a part of their answer. Difficulty: Moderate AACSB: Application of knowledge 46) Some economists claim that the Chilean experience during the 1990s was much more successful than its Latin American neighbors. Evaluate the Chilean policies during that decade. Answer: Students should refer to the democratic nature of the regime, and the fact that the policies specifically targeted corruption. Difficulty: Moderate AACSB: Application of knowledge 47) What were Mexico's major achievements to help the country to lower inflation by 1996 and regain access to private capital markets? Answer: Democratic institutions and moving away from the virtual one-party rule are among major achievements of Mexico. Difficulty: Easy AACSB: Application of knowledge
30 Copyright © 2022 Pearson Education, Inc.
22.4
East Asia: Success and Crisis
1) In the early 1960s South Korea was an extremely poor country. However, in 1963, the country began a remarkable economic ascent. What was a direct cause of this? A) a shift in strategy that emphasized exports rather than imports B) an increase in wages C) an increase in the labor force D) an increase in the money supply E) an emphasis on education, leading to a highly productive labor force Answer: A Difficulty: Easy AACSB: Application of knowledge 2) Over 50 years since the country launched a series if sweeping economic reforms in 1963, South Korea increased its real per capita GDP by a factor of about ________, more than the increase that the United States has achieved over the past century. A) 16 B) 10 C) 6 D) 20 Answer: A Difficulty: Easy AACSB: Application of knowledge 3) The high-growth East Asian economies shared something in common EXCEPT A) low rates of saving and investment. B) rapidly moving educational levels among the workforce. C) relatively moderate inflation rates. D) a high degree of openness to and integration with world markets. Answer: A Difficulty: Easy AACSB: Application of knowledge 4) Since foreign credit dries up in crises when it is most needed, developing countries can protect themselves from default by A) cutting off imports of goods. B) allowing the exchange rate to float. C) using equity finance only. D) accumulating high levels of international reserves. E) avoiding the international capital market. Answer: D Difficulty: Moderate AACSB: Application of knowledge
31 Copyright © 2022 Pearson Education, Inc.
5) What weakness in the economic structures of Asian countries contributed to the severe financial crisis that Asian economies experienced in 1997? A) Productivity: There had been relatively little increase in productivity. B) Banking regulation: Banks were excessively regulated, which reduced profits. C) Legal Framework: The system dealt successfully with companies in financial trouble. D) Natural Resources: Countries' lack of natural resources and failure to explore developing industries accumulated and led to the crisis. E) High Taxes: High rates of taxation resulted in a reliance on imports. Answer: A Difficulty: Easy AACSB: Application of knowledge 6) What weakness in the economic structures of Asian countries contributed to the severe financial crisis that Asian economies experienced in 1997? A) Poor state of banking regulation B) There had been much increase in productivity. C) Countries' lack of natural resources and failure to explore developing industries accumulated and led to the crisis. D) High rates of taxation resulted in a reliance on imports. Answer: A Difficulty: Easy AACSB: Application of knowledge 7) The problem of moral hazard has led A) the governments of many developing countries to guarantee repayment of all loans. B) to higher growth rates in Latin America. C) to excessively speculative investment. D) to both privilege and responsibility of creditors. E) to stable investments with small and steady expected gains. Answer: C Difficulty: Easy AACSB: Application of knowledge 8) What weakness in the economic structures of Asian countries contributed to the severe financial crisis that Asian economies experienced in 1997? A) Productivity: It increased rapidly and the countries were victims of their own success. B) Banking regulation: Banks were excessively regulated, which reduced profits. C) Legal Framework: The system dealt unsuccessfully with companies in financial trouble. D) Natural Resources: Countries' lack of natural resources and failure to explore developing industries accumulated and led to the crisis. E) High Taxes: High rates of taxation resulted in a reliance on imports. Answer: C Difficulty: Easy AACSB: Application of knowledge
32 Copyright © 2022 Pearson Education, Inc.
9) Important ingredients for success among the high-performing Asian economies include the following EXCEPT A) high taxes. B) high saving rate. C) strong emphasis on education. D) stable macroeconomic environment. E) high share of trade in GDP. Answer: A Difficulty: Easy AACSB: Application of knowledge 10) What event started the Asian financial crisis in 1997? A) Indonesia's inability to pay its debts B) devaluation of Indonesia's currency C) Thailand's inability to pay its debts D) devaluation of Thailand's currency E) devaluation of Malaysia's currency Answer: D Difficulty: Easy AACSB: Application of knowledge 11) The Asian financial crisis started in Thailand in 1997, and next spread to other countries including A) Malaysia, Indonesia, South Korea. B) India, Singapore, Iran. C) Cambodia, Taiwan, Vietnam. D) China, Vietnam, Brunei. Answer: A Difficulty: Easy AACSB: Application of knowledge 12) By the summer of 1998, the Indonesian rupiah had lost ________ percent of its original value. A) 85 B) 55 C) 35 D) 15 Answer: A Difficulty: Easy AACSB: Application of knowledge 13) Explain why East Asian countries have done so well relative to South American countries. Answer: Mainly, the reasons are: less moral hazard, less government debt to foreigners and smaller budget deficits in East Asian countries. Difficulty: Moderate AACSB: Application of knowledge 33 Copyright © 2022 Pearson Education, Inc.
14) Explain the reasons for the economic "miracle" of the East Asian countries between 1960 and 1997. Is it only because of the common Asian practice of industrial policy and businessgovernment cooperation? Answer: Students should emphasize high rates of savings and investment, rapidly improving educational levels among the work force, and a high degree of openness to and integration with world markets. Difficulty: Moderate AACSB: Application of knowledge 15) Based on the 1997 Crisis and your own experience, what are the main weaknesses of the East Asian economies? Answer: The textbook raised mainly three issues. The first weakness is little productivity increase, most of the growth due to capital and labor inputs increase, eventually leading to diminishing returns. The second weakness is the poor state of banking regulation in most of the Asian economies. The third reason is inadequate legal framework for dealing with companies in trouble. Difficulty: Moderate AACSB: Application of knowledge 16) Describe the Asian financial crisis as it unfolds beginning with the devaluation of the Thai currency in July 1997, followed by the Malaysian, Indonesian and South Korean crises. As part of your answer, elaborate on the Malaysian response to the crisis versus its troubled neighbors' responses. Answer: Students should emphasize the relation to the slowdown in their largest industrial neighbor, Japan, and the reliability on large debts denominated in dollars. Malaysia did not turn to the IMF with its austerity plans. Malaysia imposed extensive foreign exchange controls on capital movements. Difficulty: Moderate AACSB: Application of knowledge 17) How was South Korea able to become one of the East Asian Economic Miracles? Answer: South Korea was able to become one of the East Asian Economic Miracles by launching a series of economic reforms, shifting from an inward-looking, import-substitution development strategy to one that emphasized exports. Difficulty: Moderate AACSB: Application of knowledge
34 Copyright © 2022 Pearson Education, Inc.
22.5
Lessons of Developing-Country Crises
1) Crony capitalism is A) an economic system characterized by excessively cozy relationships between businesspeople and government officials. B) an economic system characterized by unfriendly relationships between businesspeople and government officials. C) an economic system characterized by rival relationships between businesspeople and government officials. D) an economic system characterized by excessively hostile relationships between businesspeople and government officials. Answer: A Difficulty: Easy AACSB: Application of knowledge 2) Which of the following economic lessons should we take from developing country crises in Latin America (and elsewhere)? A) only that it is important to choose the right exchange rate regime B) only that banking is of central importance in any government C) the order in which reform measures are implemented are irrelevant D) It is important to choose the right exchange rate regime and banking is of central importance in any government. E) The order in which reform measures are implemented are irrelevant and banking is of central importance in any government. Answer: D Difficulty: Easy AACSB: Application of knowledge 3) The term contagion refers to A) a government's complete control over it's banking system. B) a drop in interest rates across industrialized countries. C) the vulnerability of healthy economies to crises generated by events elsewhere. D) a directed attack on one market by a foreign market. E) a side effect of international trade. Answer: C Difficulty: Easy AACSB: Application of knowledge
35 Copyright © 2022 Pearson Education, Inc.
4) What are the three main lessons on crisis learned from early developing countries in Latin America? A) choosing the right exchange rate regime, the importance of contagion and the importance of the banking system B) choosing the right real rate, the importance of following exchange rates, and keeping prices high to make the most profit C) pegging exchange rates with Euros, keeping labor cost and wages low D) maintaining money supply, avoiding tariffs, and increasing output E) maintaining money supply, avoiding inflation, and increasing production Answer: A Difficulty: Easy AACSB: Application of knowledge 5) What are the main lessons economists learned from the developing country crisis? Answer: (1) Choosing the right exchange rate regime (2) The central importance of sound banking system (3) The proper sequence of reform measures (4) The importance of contagion Difficulty: Moderate AACSB: Application of knowledge 6) Choosing the right exchange rate regime is important. Give examples. Answer: The experiences of Brazil, Chile, Mexico, and other Latin American countries shows that larger developing countries can manage quite well with a floating exchange rate. Students can give other examples based on the reading and lecture. Difficulty: Moderate AACSB: Application of knowledge 7) What role does banking play in financial crisis? Give examples. Answer: The collapse of many banks disrupted the economy by cutting off channels of credit, which made it difficult for even profitable companies to stay in business. Banking fragility had earlier played roles in the crises of Argentina, Chile, and Uruguay in the 1980s; of Mexico in 1994—1995; and even of industrial countries like Sweden during the 1992 attacks on the EMS. It caused the Asian crisis when governments were faced with the conflict between restricting the money supply to support the currency and the need to print large quantities of money to deal with bank runs. The advanced country banking crises of 2007—2009 were another example. Difficulty: Moderate AACSB: Application of knowledge 8) What is the theory of Second Best? Answer: The principal of the second best tells us that when an economy suffers from multiple distortions, the removal of only a few may make matters worse, not better. Difficulty: Moderate AACSB: Application of knowledge
36 Copyright © 2022 Pearson Education, Inc.
9) "Developing countries should delay opening the capital account until the domestic financial system is strong enough to withstand the sometimes violent ebb and flow of world capital." Discuss. Answer: Probably true. The issue is related to the theory of second best and the proper sequence of reform measures. Of course, students may argue against such step-by-step measures. Difficulty: Moderate AACSB: Application of knowledge 10) "Trade liberalization should precede capital account liberalization." Discuss. Answer: The answer is probably true. The issue is related to the theory of second best and the proper sequence of reform measures. Of course, students may argue against such step-by-step measures. Difficulty: Moderate AACSB: Application of knowledge 11) What is the domino effect or contagion? Answer: The definition is the vulnerability of even seemingly healthy economies to crisis of confidence generated by events elsewhere in the world. Students should provide examples like the Thai crisis, which provoked another crisis in South Korea, a much larger economy some 7,000 miles away. Or the Russian crisis sparking massive speculation against the Brazil's real. Difficulty: Moderate AACSB: Application of knowledge 22.6
Reforming the World's Financial "Architecture"
1) The Asian crisis demonstrated that A) a country can be vulnerable to a currency crisis even if its own position looks healthy by normal measures. B) a country can never be vulnerable to a currency crisis if its own position looks healthy by normal measures. C) a country can avoid a currency crisis if it is closely connected with the world capital market. D) a country can take preventive measures to protect itself from contagious market disturbances. Answer: A Difficulty: Easy AACSB: Application of knowledge 2) The widespread developing-country practice of borrowing in dollars or other major currencies (both externally and internally) means that A) currency depreciations can sharply increase the real burden of debts. B) currency appreciations can sharply increase the real burden of debts. C) currency depreciations can sharply decrease the real burden of debts. D) currency depreciations have no effects on the real burden of debts. E) currency appreciations have no effects on the real burden of debts. Answer: A Difficulty: Easy AACSB: Application of knowledge 37 Copyright © 2022 Pearson Education, Inc.
3) Increased transparency, that is better provision of financial information, hopefully will A) reduce both the tendency of too much money rushing into a country when things are going well, and the rush for the exits when the risks are larger than expected. B) promote too much money rushing into a country. C) promote too much money from rushing out of a country. D) encourage foreign banks and other investors to lend money to enterprises without any clear idea of what the risks are. Answer: A Difficulty: Easy AACSB: Application of knowledge 4) Prophylactic measures to reduce the risk of financial crisis include the following EXCEPT A) less transparency. B) stronger banking systems. C) enhanced credit lines. D) increased equity capital inflows relative to debt inflows. Answer: A Difficulty: Easy AACSB: Application of knowledge 5) ________ served as a forum for developing country dept restructuring. A) The Paris Club B) The IMF C) The WB D) The WTO Answer: A Difficulty: Easy AACSB: Application of knowledge 6) The COVID-19 pandemic sparked massive private capital ________ poorer countries, bringing back into focus the prospect of widespread debt defaults and the absence of dependable orderly restructuring procedures. A) outflows from B) inflows into C) paradoxes among D) accumulation in Answer: A Difficulty: Easy AACSB: Application of knowledge
38 Copyright © 2022 Pearson Education, Inc.
7) Although emerging and developing countries make up a ________ share of the global economy measured by GDP, their financial markets are much ________ than those of the advanced economies. A) large; smaller B) small; larger C) large; larger D) small; smaller Answer: A Difficulty: Easy AACSB: Application of knowledge 8) The GDP growth rate of poorer countries ________ the global financial cycle. A) moves closely in tandem with B) moves in opposite direction with C) has no relation with D) has a negative relationship with Answer: A Difficulty: Easy AACSB: Application of knowledge 9) Contractionary Federal Reserve monetary policy ________ global asset prices, therefore_____ economic activity in developing countries. A) reduces; reduces B) increases; increases C) reduces; increases D) increases; reduces Answer: A Difficulty: Easy AACSB: Application of knowledge 10) To navigate an increasingly complex global financial environment, emerging markets will need to use an array of tools EXCEPT A) trade embargo. B) foreign exchange intervention. C) monetary policy. D) macroprudential policy. E) capital control measures. Answer: A Difficulty: Easy AACSB: Application of knowledge 11) Explain the basic macroeconomic policy trilemma for open economies. Answer: Of three goals most countries share—independence in monetary policy, stability in the exchange rate, and the free movement of capital—only two can be reached simultaneously. Difficulty: Moderate AACSB: Application of knowledge 39 Copyright © 2022 Pearson Education, Inc.
12) Explain the macroprudential motive for emerging markets to impose limited controls on financial inflows. Answer: Limits on financial inflows could limit excessive bank lending during booms and thereby temper the resulting contraction in case of a sudden stop or financial-flow reversal later. Equally (if not more) important as a motivation has been the desire to limit real currency appreciation, and the resulting harm to exports, without resorting to inflationary monetary policies. Difficulty: Easy AACSB: Application of knowledge 13) Discuss the role of more "transparency" in reducing the risk of financial crisis. Answer: Students should discuss the Asian crisis where foreign banks lent money to Asian enterprises without any clear idea of what the risks were, and then pulled their money out equally blindly when it became clear those risks were larger than they anticipated. Difficulty: Moderate AACSB: Application of knowledge 14) List the prophylactic measures to reduce the risk of financial crisis. Answer: More "transparency," stronger banking systems, enhanced credit lines, increased equity capital inflows relative to debt inflows. Difficulty: Easy AACSB: Application of knowledge 15) Should the IMF be abolished? Discuss. Answer: An open answer. Arguments for abolishing the IMF should mention moral hazard, insistence on high interest rates, hasty structural reforms, etc. Arguments against abolishing will stress more coordination, more credit lines, transparency, etc. Difficulty: Moderate AACSB: Application of knowledge 16) How do the booms and busts in global asset prices and capital flows affect the emerging economies? Answer: Booms in global asset prices and capital flows lift emerging economies, whereas busts lead to slowdowns, notwithstanding countercyclical monetary and fiscal responses. Difficulty: Easy AACSB: Application of knowledge 17) Compare the macroeconomic performances in the 1990s of the following countries under the following exchange-rate regimes: floating exchange rates, Mexico and Brazil; capital control, China and Malaysia; and currency boards, Estonia and Hong Kong; dollarization, Argentina. Answer: An open question. Students should use the IMF web site to discuss the issue, and to obtain data. Difficulty: Moderate AACSB: Application of knowledge
40 Copyright © 2022 Pearson Education, Inc.
22.7 Understanding Global Capital Flows and the Global Distribution of Income: Is Geography Destiny? 1) Which of the following statements is TRUE? A) There is no systematic tendency for poorer countries' income levels to converge, even slowly, to those of richer countries. B) There is systematic tendency for poorer countries' income levels to converge to those of richer countries. C) The theory of convergence applies to today's world. D) Poor countries grow more quickly than the rich ones as the standard models predict. Answer: A Difficulty: Easy AACSB: Application of knowledge 2) Which of the following statements is the MOST accurate since the late 1990s? A) Relatively little capital flows to developing countries. B) Relatively little capital flows to developed countries. C) Developing countries suck most of the world's available current account surpluses. D) The scale of capital flows between advanced countries is dwarfed by the capital flow to the developing countries. Answer: A Difficulty: Easy AACSB: Application of knowledge 3) Which theory best explains the wealth inequalities amongst nations? A) weather B) government institutions C) natural selection D) factors outside of any human control E) levels of corruption Answer: B Difficulty: Easy AACSB: Application of knowledge 4) Why did China run huge current account surpluses in the 2000s? A) China's spending failed to keep up with rapid growth and China intervened in the foreign exchange market to keep its currency artificially weak. B) China intervened in the foreign exchange market to keep its currency strong. C) China's imports exceeded exports in many years. D) China imports a lot of oil from OPEC countries. Answer: A Difficulty: Easy AACSB: Application of knowledge
41 Copyright © 2022 Pearson Education, Inc.
5) Two main schools of thought to explain the fact that some countries have grown very rich while some attract little or no foreign investment and remain in extreme poverty include A) geographical features and institutions of government. B) religions and institutions of government. C) colonial history and economic planning. D) culture and institutions of government. Answer: A Difficulty: Easy AACSB: Application of knowledge 6) According to Jared Diamond's Guns Germs, and Steel: The Fates of Human Societies book, what determines a country's long-run economic performance? A) Aspects of a country's physical environment such as climate, soil type, diseases, and geographical accessibility. B) Aspects of a country's political environment such as political regime and stability. C) Aspects of a country's social environment such as race relations, and social inequality. D) Aspects of a country's cultural environment such as religions, family, and educational systems. Answer: A Difficulty: Easy AACSB: Application of knowledge 7) Which of the following statements is NOT true? A) Countries with a high corruption level are associated with higher per capita income for the majority of their population. B) Countries that are landlocked and mountainous trade less with the outside world than those countries blessed with good ocean harbors, navigable internal waterways, and easily traveled roadways. C) Countries that favor the institutions of government in protecting private property rights encourage private enterprise, investment, innovation, and ultimately economic growth. D) Countries with a low corruption level promotes productive economic activity by ensuring investors that the fruits of their labors will not be arbitrarily seized. Answer: A Difficulty: Easy AACSB: Application of knowledge 8) Explain the vicious circle of slow growth of developing countries that attract little or no foreign investment and remain in extreme poverty. Answer: Developing countries' poor economic growth performances are closely related to high risks of investing that limit their attractiveness for investors. When governments are unwilling or unable to protect property rights, investors will be unwilling to invest in either physical or human capital, so growth will be nonexistent or low. The circle continues. Difficulty: Easy AACSB: Application of knowledge
42 Copyright © 2022 Pearson Education, Inc.
9) What are the "capital paradoxes"? Answer: Capital was flowing uphill, from poorer to richer countries, contrary to what simple economic theory would predict. Difficulty: Easy AACSB: Application of knowledge 10) Give Luca's and other scholars' explanation on why the big income disparities between rich and poor countries. Answer: Scarcity in poor countries of human capital–in the form of a highly educated work force and managerial know-how. Other scholars put more weight on the greater fragility of property rights and government stability in poorer countries, a position that was partially borne out by the crises of the 1990s. Difficulty: Easy AACSB: Application of knowledge 11) What is the allocation puzzle? Answer: Countries with lower growth in the productivity of labor and capital actually attract relatively more foreign financial inflows than countries with high productivity growth. Difficulty: Easy AACSB: Application of knowledge 12) Give Gourincha and Jeanne's explanation on the allocation puzzle. Answer: Gourincha and Jeanne suggest that the allocation puzzle is related to accumulation of international reserves by some fast-growing economies (such as China's). These economies often do receive substantial inflows of foreign direct investment, but their saving is so high that they still run overall surpluses in their current accounts. Difficulty: Easy AACSB: Application of knowledge 13) Why do some critics suggest that Acemoglu, Johnson, and Robinson's measures of institutional quality are inadequate? Answer: Acemoglu, Johnson, and Robinson show that the effect of early European-settler mortality rates on current per capita income, operating through the influence of mortality on later institutions, is large. Some argue that their mortality data are faulty or even that historical mortality rates could be related directly to productivity today. Difficulty: Moderate AACSB: Application of knowledge
43 Copyright © 2022 Pearson Education, Inc.